Magnetic Effect of Current
Magnetic Effect of Current
1
Since a magnetic needle can be known as Biot-Savart law.
deflected by a magnetic field only, it follows
Statement. As shown in Fig. 4.3, consider
from the above experiment that a current
a current element 𝑑𝑙 ⃗⃗⃗ of a conductor XY
carrying conductor produces a magnetic
field around it. carrying current I. Let P be the point where
the magnetic field 𝑑𝐵⃗⃗⃗⃗⃗ due to the current
Ampere's swimming rule. This rule ⃗⃗⃗ is to be calculated. Let the
predicts the direction of deflection of the element 𝑑𝑙
magnetic needle in the Oersted's position vector of point P relative to
element 𝑑𝑙⃗⃗⃗ be 𝑟 . Let θ be the angle
experiment, it can be stated as follows :
⃗⃗⃗
between 𝑑𝑙 and 𝑟.
Imagine a man swimming along the wire in
the direction of the flow of the current with
his face always turned towards the
magnetic needle, then the north pole of the
needle will get deflected towards his left
hand, as shown in Fig. 4.2.
The direction can also be remembered
with the help of the word SNOW. It
indicates that if the current flows from
South to North and the wire is held Over
the needle, the north pole is deflected
towards the West. Fig. 4.3 Biot-Savart law.
According to Biot-Savart law, the
⃗⃗⃗⃗⃗ is
magnitude of the field 𝑑𝐵
1. directly proportional to the current I
through the conductor,
𝑑𝐵 ∝ 𝐼
2. directly proportional to the length dl of
the current element,
dB ∝ dl
Fig. 4.2 Ampere's swimming rule.
3. directly proportional to sin θ,
4.3 BIOT-SAVART LAW
dB ∝ sin θ
3. State and explain Biot-Savart lazv for
the magnetic field produced by a current 4. inversely proportional to the square of
element. Define the SI unit of magnetic the distance r of the point P from the
field from this law. current element,
1
Biot-Savart law. Oersted experiment 𝑑𝐵 ∝
showed that a current carrying conductor 𝑟2
produces a magnetic field around it. It is Combining all these four factors, we get
convenient to assume that this field is 𝐼𝑑𝑙sin𝜃
made of contributions from different 𝑑𝐵 ∝
segments of the conductor, called current 𝑟2
elements. A current elementis" denoted by 𝐼𝑑𝑙sin𝜃
or 𝑑𝐵 = 𝐾 ⋅ 𝑟2
⃗⃗⃗ , which has the same direction as that of
𝑑𝑙
current I. From a series of experiments on The proportionality constant K depends
current carrying conductors of simple on the medium between the observation
shapes, two French physicists point P and the current element and the
Jean-Baptiste Biot and Felix Savart, in system of units chosen. For free space and
1820, deduced an expression for the in SI units,
magnetic field of a current element which is
2
𝜇0 4𝜋 × 10−7
𝐾= = 10−7 TmA−1 ( or Wbm−1 A−1 ) = = 10−7 tesla
4𝜋 4𝜋
Here μ0 is a constant called Thus one tesla is 107 times the
permeability of free space. So the magnetic field produced by a conducting
Biot-Savart law in SI units may be wire of length one metre and carrying
expressed as current of one ampere at a distance of one
𝜇0 𝐼𝑑𝑙sin𝜃 metre from it and perpendicular to it.
𝑑𝐵 = ⋅
4𝜋 𝑟2 4.4 BIOT-SAVART LAW VS.
We can write the above equation as 𝑑𝐵 = COULOMB'S LAW
𝜇0𝐼 𝑑𝑙𝑟sin𝜃
4𝜋 𝑟3
4. Give some points of similarities and
differences between Biot-Savart law for the
As the direction of dB is perpendicular magnetic field and Coulomb's law for the
⃗⃗⃗ and 𝑟, so from the above
to the plane of 𝑑𝑙 electrostatic field.
equation, we get the vector form of the
Biot-Savart law as Comparison of Biot-Savart law with
Coulomb's law. According to Coulomb's
⃗⃗⃗ × 𝑟
𝜇0 𝐼 𝑑𝑙 law, the electric field produced by a
⃗⃗⃗⃗⃗ =
𝑑𝐵
4𝜋 𝑟 3 charged element at a distance r is given by
Direction of ⃗⃗⃗⃗⃗
𝑑𝐵. The direction of ⃗⃗⃗⃗⃗𝑑𝐵 is 1 𝑑𝑞
𝑑𝐸 =
⃗⃗⃗ × 𝑟 . It is given
the direction of the vector 𝑑𝑙 4𝜋𝜀0 𝑟 2
by right hand screw ride. If we place a right According to Biot-Savart law, the magnetic
handed screw at point P perpendicular to ⃗⃗⃗ at
field produced by a current element I 𝑑𝑙
the plane of paper and turn its handle from a distance r is given by
dl to 𝑟 , then the direction in which the
𝜇0 𝐼𝑑𝑙sin𝜃
screw advances gives the direction ⃗⃗⃗⃗⃗ 𝑑𝐵 . 𝑑𝐵 =
Thus the direction of dB is perpendicular to 4𝜋 𝑟 2
and into the plane of paper, as has been On comparing the above two
shown by encircled cross 𝑟 at point P in equations, we note the following points of
Fig. 4.3. similarities and differences between the
two laws.
Special Cases
Points of similarity :
1. If θ =0°, sin θ =0, so that dB=0
1. Both fields depend inversely on the
i.e., the magnetic field is zero at points on
square of the distance from the source
the axis of the current element.
to the point of observation.
2. If θ =90°, sin θ =1, so that dB is
2. Both are long range fields.
maximum i.e., the magnetic field due to
a current element is maximum in a 3. The principle of superposition is
plane passing through the element and applicable to both fields. This is
perpendicular to its axis. because the magnetic field is linearly
related to its source, namely, the
SI unit of magnetic field from
current element I dl and the
Biot-Savart law. The SI unit of magnetic
electrostatic field is related linearly to
field is tesla, named after the great
its source, namely, the electric charge.
Yugoslav inventor and scientist Nikola
Tesla. According to Biot-Savart law, Points of difference :
𝜇0 𝐼𝑑𝑙sin𝜃 1. The magnetic field is produced by a
𝑑𝐵 = ⋅ ⃗⃗⃗ .
4𝜋 𝑟2 vector source : the current element I 𝑑𝑙
If I =1 A, dl =lm, r=l m and θ =90° so The electrostatic field is produced by a
that sin𝜃 = 1, then scalar source : the electric charge dq.
𝜇0 2. The direction of the electrostatic field is
𝑑𝐵 = along the displacement vector joining
4𝜋
3
the source and the field point. The 200 cm north-east from the piece.
direction of the magnetic field is
Solution. The problem is illustrated in
perpendicular to the plane containing
Fig. 4.4.
the displacement vector 𝑟 and the
⃗⃗⃗ .
current element I 𝑑𝑙
3. In Bio-Savart law, the magnitude of the
magnetic field is proportional to the
sine of the angle between the current
element I 𝑑𝑙⃗⃗⃗ and displacement vector 𝑟
while there is no such angle
dependence in the Coulomb's law for
the electrostatic field. Along the axial Fig. 4.4
line of the current element θ=0°, sinθ=0
As the distance OP is much larger than
and hence dB = 0.
the length of the wire, we can treat the wire
5. Write a relation between μ0, 𝜀0 and a as a small current element.
Relation between μ0, 𝜀0 and c. We know Here I = 8 A dl = 1cm = 1 × 10−2 m,
that
γ = 200 cm = 2m, 𝜃 = 45o
1
= 9 × 109 Nm2 C−2 𝜇0 𝐼𝑑𝑙sin𝜃
4πε0 𝑑𝐵 = ,
4𝜋 𝑟2
μ
and 4π0 = 10−7 Tm A−1 4π × 10−7 8 × 1 × 10−2 × sin45o
= .
μ 4πε0
4π 22
∴ μ0 ε0 = (4π0 I( )
1 = 1.4 × 10−9 T.
1 1
= 10−7 × 9 = The direction of the magnetic field at point
9 × 10 (3 × 108 )2
P is normally into the plane of paper.
But 3 × 10 ms−1 speed of light in vacuum Example 2. An element 𝛥𝑙 = 𝛥𝑥𝑖̂ is placed
(c) at the origin and carries a large current I
1 =10 A What is the magnetic field on the
∴ 𝜇0 𝜀0 = 𝑐 2
y-axis at a distance of 0.5 m. 𝛥𝑥 =1 cm
1
Or 𝑐 = [NCERT]
√𝜇0𝜀0
Solution. Here dl = 𝛥𝑥 -1 cm - 10-2m, I
Examples Based on
=10 A, r = y = 0.5 m, 𝜃=90°, μ0/4π = 10-7
Boit-savart law Tm A-1
Formula Used
𝜇 𝐼𝑑𝑙sin𝜃
Biot-Savart law, 𝑑𝐵 = 4𝜋0 𝑟2
Units Used
Magnetic field B is in tesla, current I in
ampere and distance r in metre.
Constant Used
Permeability constant, 𝜇0 = 4𝜋 ×
10−7 TmA−1 Fig. 4.5
4
10−7 × 10 × 10−2 × sin90° HUNTS
=
(0.5)2 Proceed as in Example 1.
=4 × 10−8 T Proceed as in Example 2.
⃗⃗⃗⃗⃗ will be the
The direction of the field 𝑑𝐵 Proceed as in Example 2.
⃗⃗⃗
direction of vector 𝑑𝑙 × 𝑟. But We shall now apply Biot-Savart law to
⃗⃗⃗ × 𝑟 = 𝛥𝑥𝑖̂ × 𝑦𝑗̂ = 𝛥𝑥𝑦(𝑖̂ × 𝑗̂) = 𝛥𝑥𝑦𝑘̂ calculate the magnetic field due to (i) a
𝑑𝑙
straight current carrying conductor and (ii)
Hence field ⃗⃗⃗⃗⃗
𝑑𝐵 is in the + z-direction. a circular current loop.
Problems for Practice 4.5 MAGNETIC FIELD DUE TO A LONG
STRAIGHT CURRENT CARRYING
1. A wire placed along east-west direction CONDUCTOR
carries a current of 10 A from west to
east direction. Determine the magnetic 6. Apply Biot-Savart law to derive an
field due to a 1.8 cm piece of wire at a expression for the magnetic field produced
point 300 cm north-east from the piece. at a point due to the current flowing through
a straight wire of infinite length. Also draw
(Ans 1.4 × 10−9 T, normally out of the plane the sketch of the magnetic field. State the
of paper) rules used for finding the direction of this
⃗⃗⃗ , with 𝑑𝑙
2. A small current element I 𝑑𝑙 ⃗⃗⃗ = 2 magnetic field.
k mm and I = 2 A is centred at the origin. Magnetic field due to a long straight
Find magnetic field ⃗⃗⃗⃗⃗
𝑑𝐵 at the following current carrying conductor As shown in
points : Fig. 4.7, consider a straight conductor XY
carrying current I. We wish to find its
(i) On the x-axis at x = 3 m.
magnetic field at the point P whose
(Ans. 4.44 × 10-11 𝑗̂T) perpendicular distance from the wire is a
i.e., PQ = a.
(ii) On the x-axis at x = - 6 m.
(Ans. −1.11 × 10−11 𝑗̂T)
(iii) On the z-axis at z = 3 m.
(Ans. 0)
3. An element ⃗⃗⃗𝛥𝑙 = 𝛥𝑥𝑖̂ is placed at the
origin (as shown in Fig. 4.6) and carries
a current I = 2 A. Find out the magnetic
field at a point Pon the y-axis at a
distance of 1.0 m due to the element Ax
- 1 cm. Give also the direction of the
field produced. [CBSE D 09C]
(Ans. 2 × 10−9 T, in + 𝑧 -direction)
Fig. 4.7 Magnetic field due to a straight
current carrying conductor.
Consider a small current element 𝑑𝑙 ⃗⃗⃗ of
the conductor at O. Its distance from Q is l
i.e., OQ = l. Let 𝑟 be the position vector of
point P relative to the current element and
θ be the angle between dl and r. According
to Biot-Savart law, the magnitude of the
⃗⃗⃗⃗⃗ due to the current element 𝑑𝑙
field 𝑑𝐵 ⃗⃗⃗ will
be
Fig. 4.6
5
𝜇0 𝐼𝑑𝑙sin𝜃 2. If the conductor XY is infinitely long
𝑑𝐵 = ⋅
4𝜋 𝑟2 but the point P lies near the end Y (or X),
From right Δ OQP, then 𝜙1 = 90° and 𝜙2 = 0° .
𝜇 𝐼 𝜇 𝐼
θ + ϕ =90°
0
∴𝐵 = 4𝜋𝑎 [sin90° + sin0° ] = 4𝜋𝑎
0
0 𝜇 𝐼 𝜇0 𝐼 2𝐿
Or 𝑑𝐵 = 4𝜋𝑎 cos𝜙𝑑𝜙 = ⋅
4𝜋𝑎 √4𝑎 2 + 𝐿2
According to right hand rule, the 𝜇0 𝐼𝐿
direction of the magnetic field at the P due 𝐵=
to all such current elements will be in the 2𝜋𝑎√4𝑎 2 + 𝐿2
same direction, namely, normally into the Direction of magnetic field. For an
plane of paper. Hence the total field B at infinitely long conductor,
the point P due to the entire conductor is 𝜇0 𝐼
obtained by integrating the above equation 𝐵=
within the limits −𝜙1 and 𝜙2 2𝜋𝑎
1
𝜙2
𝜇0 𝐼 𝜙2 𝐵∝
𝐵=∫ 𝑑𝐵 = ∫ cos 𝜙𝑑𝜙 𝑎
−𝜙1 4𝜋𝑎 −𝜙1
Clearly, the magnitude of the magnetic
𝜇0 𝐼 𝜙2 field will be same at all points located at the
= [sin𝜙]−𝜙 same distance from the conductor. Hence
4𝜋𝑎 1
the magnetic lines of force of a straight
𝜇0 𝐼
= [sin𝜙2 − sin(−𝜙1 )]
4𝜋𝑎
𝜇0 𝐼
𝐵= [sin𝜙1 + sin𝜙2 ]
4𝜋𝑎
This equation gives magnetic field due
to a finite wire in. terms of the angles
subtended at the observation point by the
ends of the wire.
Special Cages
1. If the conductor XY is infinitely long Fig. 4.8 Magnetic lines of force of a
and the point P lies near the middle of the straight current carrying conductor.
conductor, then 𝜙1 = 𝜙2 = 𝜋/2 current carrying conductor are concentric
𝜇0 𝐼 circles with the wire at the centre and in a
∴𝐵 = 4𝜋𝑎
[sin90° + sin90° ] plane perpendicular to the wire. [A line of
𝜇 𝐼
0 force is a curve, the tangent to which at any
𝐵 = 2𝜋𝑎 point gives the direction of magnetic field at
6
that point]. If the current flows upwards, the
lines of force have anticlockwise sense
[Fig. 4.8(A)] and if the current flows
downwards, then the lines of force have
clockwise sense [Fig. 4.8(b)].
Rules for finding the direction of
magnetic field due to straight current
carrying conductor. Either of the following
two rules can be used for this purpose :
1. Right hand thumb rule. If we hold the
straight conductor in the grip of our right
hand in such a way that the extended
Fig. 4.10 Cork screw rule for field due to
thumb points in the direction of current,
a straight conductor.
then the direction of the curl of the angers
will give the direction of the magnetic field Thus the graph plotted between the
(Fig. 4.9). magnetic field B and the distance a from
the straight conductor is a hyperbola, as
shown in Fig. 4.11.
2. Maxwell's cork screw rule. If a right Magnetic field due to a straight current
handed screw be rotated along the wire so carrying conductor
that it advances in the direction of current, Formulae Used
then the direction in which the thumb
rotates gives the direction of the magnetic 1. Magnetic field due to a straight
field (Fig. 4.10). conductor of finite length,
𝜇0 𝐼
Variation of magnetic field with 𝐵= (sin𝜙 + sin𝜙2 )
distance from -straight current carrying 4𝜋𝑎
conductor. For a straight current carrying 2. Magnetic field due to an infinitely long
conductor, straight conductor,
1 𝜇0 𝐼
𝐵∝ 𝐵=
𝑎 2𝜋𝑎
Units Used
Magnetic field B is in tesla, current I in
ampere and distance α in metre.
Example 3. A current of 10 A is flowing east
to west in a long wire kept horizontally in
the east-west direction. Find magnetic field
in a horizontal plane at a distance of
7
(i) 10 cm north
𝐵 = √𝐵12 + 𝐵22
(ii) 20 cm south from the wire; and in
the vertical plane at a distance of = √(3 × 10−4 )2 + (4.0 × 10−4 )2
(iii) 40 cm downward and = 5 × 10−4 T
(iv) 50 cm upward. Example 5. Figure 4.12 shows two
current-carrying wires 1 and 2. Find the
Solution, (i) Magnetic field in a
magnitudes and directions of the magnetic
horizontal plane at 10 cm north of the wire
field at points P, Q and R.
is
𝜇 𝐼 4𝜋×10−7×10
0
𝐵𝑁 = 2𝜋𝑟 = = 2 × 10−5 T
2𝜋×0.10
8
Fig. 4.13 4 × √2𝜇0 𝐼 √2𝜇0 𝐼
𝐵 = 4𝐵1 = =
Solution. At the required point, the 4𝜋𝑎 𝜋𝑎
resultant magnetic field will be zero when directed normally outwards.
the fields due to the two wires have equal
magnitude and opposite directions. Such Example 8. A current of 1.0 A is flowing in
point should lie either to the left of P or to the sides of an equilateral triangle of side
the right of Q. But the wire Q has a smaller 4.5 × 10-2 m. Find the magnetic field at the
current, the point should lie closer to and to centroid of the triangle.
the right of Q. Let this point be R at [
distance x from Q, as shown in Fig. 4.13. Roorkee 91]
Field due to current f at point R, Solution. The situation is shown in Fig.
𝜇0 𝐼1 4.15. The magnetic field at the centre O
𝐵1 = due to the current through side PQ is given
2𝜋(𝑟 + 𝑥)
by
normally into the plane of paper. Field due
𝜇0 𝐼
to current I2 at point R, 𝐵1 = [sin𝜃1 + sin𝜃2 ]
4𝜋𝑎
𝜇0 𝐼1
𝐵1 = where a is the distance of PQ from O and
2𝜋(𝑟 + 𝑥) θ1, θ2 are the angles as shown. The
normally out of the plane of plane magnetic field due to each of the three
sides is the same in magnitude and
But B1 = B2 direction, therefore, total magnetic field at
𝐼1 𝐼2 O is
∴ =
𝑟+𝑥 𝑥
𝐼2𝑟 3𝜇0 𝐼
Or 𝑥 = 𝐵 = 3𝐵1 = [sin𝜃1 + sin𝜃2 ]
𝐼1 −𝐼2 4𝜋𝑎
2A × 6cm
= = 4cm
5A − 2A
Example 7. Use Biot-Savart law to obtain
an expression for the magnetic field at the
centre of a coil bent in the form of a square
of side 2a carrying current I.
Solution. Refer to Fig. 4.14. Magnetic
field at O due to finite length of wire AB is Fig. 4.15
𝜇0 𝐼 Here / =1.0 A, θ1 = θ2 =60°, 𝜇0 =
𝐵1 = (sin𝛼 + sin𝛽)
4𝜋𝑎 4𝜋 × 10−7 TmA−1
𝜇0 𝐼 √2𝜇0 𝐼 𝑃𝑆 𝑙⁄2
= (sin45° + sin45° ) = = tan𝜃1 or = tan60°
4𝜋𝑎 4𝜋𝑎 𝑂𝑆 𝑎
𝑙 4.5×10−2
∴𝑎 = 2tan60° = m
2√3
9
wire cutting the plane of paper normally as 3. The magnetic induction at a point P
shown at Q. Likewise a similar wire carries which is at a distance of 4 cm from a
an equal current passing normally upwards long current carrying wire is 10 -3 T.
at R. Find the magnitude and direction of What is the magnetic induction at
the magnetic induction B at P. Assume the another point Q which is at a distance
wires to be infinitely long. of 12 cm from this current carrying
wire? (Ans. 3.33 ×10-4 T)
[ISCE 97]
4. What current must flow in an infinitely
long straight wire to give a flux density
of 3 × 10-5 T at 6 cm from the wire?
(Ans. 9 A)
5. A vertical wire in which a current is
flowing produces a neutral point with
Fig. 4.16 the earth's magnetic field at a distance
of 10 cm from the wire. What is the
Solution. Let PQ = QR = r. In right
current if BH = 1.8 × 10-4 T? (Ans. 90 A)
𝛥𝑃𝑄𝑅,
𝑎 6. Fig. 4.17 shows two long, straight wires
𝑎 2 = 𝑟 2 + 𝑟 2 = 2𝑟 2 or 𝑟 = carrying electric currents of 10 A each
√2 in opposite directions. The separation
Magnetic induction at point P due to the between the wires is 5.0 cm. Find the
conductor passing through Q magnetic field at a point P midway
𝜇0 𝐼 √2𝜇0 𝐼 𝜇0 𝐼
between the wires.
𝐵1 = = = acting along PR
2𝜋𝑟 2𝜋𝑎 √2𝜋𝑎 (Ans. 1.6 ×10-6 T)
Magnetic induction at point P due to the
conductor passing through R,
𝜇0 𝐼
𝐵2 = acting along PQ
√2𝜋𝑎
𝐵 = √𝐵12 + 𝐵22
7. Two long parallel wires are placed at a
distance of 16 cm from each other in
𝜇0 𝐼 2 𝜇0 𝐼 2 𝜇0 𝐼 air. Each wire has a current of 4 A.
= √( ) +( ) = √2 ⋅
√2𝜋𝑎 √2𝜋𝑎 √2𝜋𝑎 Calculate the magnetic field at midpoint
between them when the currents in
𝜇0 𝐼 them are (i) in the same direction and
Or 𝐵 =
𝜋𝑎 (ii) in opposite directions. [Ans. (i) Zero
This field acts towards the midpoint of (ii) 2 × 10-5 T]
QR.
8. Two infinitely long insulated wires are
Problems for Practice kept perpendicular to each other. They
carry currents I1 = 2 A and I2 = 1.5 A. (i)
1. A straight wire carries a current of 3 A.
Find the magnitude and direction of the
Calculate the magnitude of the
magnetic field at P. (ii) If the direction of
magnetic field at a point 10 cm away
current is reversed in one of the wires,
from the wire. [CBSE D 96]
what would be the magnitude of the
(Ans. 6 × 10-6 T) field B ?
2. At what distance from a long straight [Ans. (I) 2 × 10 5 T, normally into the plane
wire carrying a current of 12 A will the of paper (ii) zero]
magnetic field be equal to 3 × 10 -5 Wb
m-2. (Ans. 8 × 10-2 m)
10
[IPUEE 13]
√3𝜇0 𝐼
( Ans. 𝐵 = )
𝜋𝑎
13. Two straight long conductors AOB and
COD are perpendicular to each other
and carry currents I1 and I2
respectively. Find the magnitude of the
magnetic field at a point P at a distance
9. A long straight wire carrying a current
a from the point O in a direction
of 200 A, runs through a cubical box,
perpendicular to the plane ABCD.
entering and leaving through holes in
the centres of opposite faces, as shown 𝜇0 2
( Ans. (𝐼 + 𝐼22 )1⁄2 )
in Fig. 4.19. Each side of the box is of 2𝜋𝑎 1
20 cm. 14. Two insulating infinitely long
conductors carrying currents I2 and I2
lie mutually perpendicular to each other
in the same plane, as shown in Fig.
4.20. Find the magnetic field at the
point P (a, b).
𝜇0 𝐼2 𝐼
Ans. ( Ans. ( − 𝑎1 ) , directed inward )
2𝜋 𝑏
Fig. 4.19
Consider an element PQ of the wire 1 cm
long at the centre of the box. Calculate the
magnetic field produced by this element at
the points A, B, C and D. The points A, B
and C are the centres of the faces of the
cube and D is the midpoint of one edge.
(Ans. 20 × 10-6 T, 20 × 10-6 T, 0, 7.07 × 10-6
T)
Fig. 4.20
10. A long straight telephone cable
contains six wires, each carrying a HINTS
current of 0.5 A. The distance between 𝜇 𝐼 4𝜋×10−7×3
0
1. 𝐵 = 2𝜋𝑟 = = 6.0 × 10−6 T
the wires is negligible. What is the 2𝜋×0.10
magnitude of magnetic field at a 𝜇 𝐼 4𝜋×10−7×12
distance of 10 cm from the cable (i) if
0
2. 𝑟 = 2𝜋𝐵 = = 8 × 10−2 m
2𝜋×3×10−5
the currents in all the six wires are in 3. Magnetic field due to a straight current
same direction (ii) if four wires carry carrying conductor,
current in one direction and the other
two in opposite direction. [Ans. (i) 6.0 × 𝜇0 𝐼 1
𝐵= i.e., 𝐵∝
10-6 T, (ii) 2.0 × 10 -6 T] 2𝜋𝑟 𝑟
𝐵 𝑟
11. Calculate the magnetic induction at the ∴𝐵𝑄 = 𝑟𝑃
𝑃 𝑄
centre of a coil bent in the form of a
𝑟𝑃 4
square of side 10 cm carrying a current Or 𝐵𝑄 = ⋅ 𝐵𝑃 = 12 × 10−3 = 3.33 ×
𝑟𝑄
of 10 A. [Punjab 01]
10−4 T
(Ans. 1.13 × 10-4 T)
2𝜋𝑟𝐵 2𝜋×6×10−2 ×3×10−5
12. A closed circuit is in the form of a 4. 𝐼 = = = 9A
𝜇0 4𝜋×10−7
regular hexagon of side a. If the circuit
5. If neutral point is obtained at distance r
carries current I, what is magnetic
from the wire, then
induction at the centre of the hexagon?
11
𝜇0 𝐼 𝑟 = √102 + 102 = 10√2cm = 0.1√2m, 𝜃
= 𝐵𝐻
2𝜋𝑟 = 45°
2𝜋𝑟𝐵𝐻 2𝜋×0.10×1.8×10−4
or 𝐼 = 𝜇0
= 4𝜋×10−7
= 90A 𝜇0 𝐼𝑑𝑙sin45°
𝐵𝐷 = ⋅
4𝜋 𝑟2
6. According to right hand thumb rule, the 10−2 × 200 × 0.01 × 1
direction of magnetic field due to =
current in each wire is perpendicular to (0.1√2)2 × √2
and pointing into the plane of paper. = 7.07 ×10-6 T.
Hence total field at point P is
10. (i) Net current, I = 0.5 × 6 = 3.0 A, r =
𝜇0 𝐼 10 cm = 0.1 m
𝐵 =2×
2𝜋𝑥
𝜇0 𝐼 4𝜋 × 10−7 × 3.0
2 × 4𝜋 × 10−7× 10 𝐵= = = 6.0 × 10−6 T
= = 1.6 × 10−6 T 2𝜋𝑟 2𝜋 × 0.1
2𝜋 × 2.5 × 10−2
(ii) Net current, I = 0.5 × 4 - 0.5 × 2 = 1.0 A
5
[𝑟 = cm = 2.5 × 10−2 m] 𝜇0 𝐼 4𝜋 × 10−7 × 1.0
2 𝐵= = = 2.0 × 10−6 T
2𝜋𝑟 2𝜋 × 0.1
7. (i) When the currents are in same
direction, 11. Refer to Fig. 4.21. Magnetic field at O
due to finite wire AB,
𝐵 = 𝐵1 − 𝐵2
(ii) When the currents are in opposite
directions,
𝐵 = 𝐵1 + 𝐵2
𝜇 𝐼 4𝜋×10−7×2
8.
0 1
𝐵1 = 2𝜋𝑟 = 2𝜋×4×10−2 = 10−5 T𝑟
1
12
Fig. 4.22 loop can be divided into a large number of
small current elements.
Magnetic field at O due to current in AB is
𝜇0 𝐼 Consider a current element 𝑑𝑙 ⃗⃗⃗ of the loop.
𝐵1 = [sin𝛼 + sin𝛽] According to Biot-Savart law, the magnetic
4𝜋𝑎 ′
field at the centre O due to this element is
𝜇0 𝐼 𝜇0 𝐼
= [sin30° + sin30° ] = ⃗⃗⃗ × 𝑟
𝜇0 𝐼 𝑑𝑙
4𝜋√3𝑎/2 2𝜋√3𝑎 ⃗⃗⃗⃗⃗ =
𝑑𝐵 ⋅
4𝜋 𝑟3
√3𝜇0𝐼
Total field at 𝑂 = 6𝐵1 = 𝜋𝑎
Magnetic field at point P due to current I2, magnetic field at the centre O is
𝜇0𝐼2 𝜇0 𝐼 𝑑𝑙 𝜇0 𝐼
𝐵2 = directed normally outward 𝐵 = ∫ 𝑑𝐵 = ∫ ⋅ 2= ∫ 𝑑𝑙
2𝜋𝑏 4𝜋 𝑟 4𝜋𝑟 2
As 𝑏 < 𝑎, so 𝐵2 > 𝐵1 𝜇0 𝐼 𝜇0 𝐼
= 2 ⋅𝑙 = ⋅ 2𝜋𝑟
Hence the net magnetic field at the point P, 4𝜋𝑟 4𝜋𝑟 2
𝜇0 𝐼
𝜇0 𝐼2 𝐼1 or 𝐵=
𝐵 = 𝐵2 − 𝐵1 = ( − ) 2𝑟
2𝜋 𝑏 𝑎 If instead of a single loop, there is a coil
directed normally inward. of N turns, all wound over one another,
4.6 MAGNETIC FIELD AT THE CENTRE then
OF CIRCULAR CURRENT LOOP 𝜇0 𝑁𝐼
𝐵=
7. Apply Biot-Savart law to derive an 2𝑎
expression for the magnetic field at the 4.7 MAGNETIC FIELD ON THE AXIS OF
centre of a current carrying circular loop. A CIRCULAR CURRENT LOOP
Magnetic field at the centre of a circular 8. Apply Biot-Savart law to find the
current loop. As shown in Fig. 4.23, magnetic field due to a circular current
consider a circular loop of wire of radius r carrying loop at a point on the axis of the
carrying current I. We wish to calculate its loop. State the rules used to find the
magnetic field at the centre O. The entire direction of this magnetic field.
13
Magnetic field along the axis of a 𝑎 𝜇 𝐼𝑑𝑙
But sin𝜙 = and 𝑑𝐵 = 4𝜋0 ⋅
𝑠 𝑠2
circular current loop. Consider a circular
loop of wire of radius a and carrying current 𝜇0 𝐼𝑑𝑙 𝑎
∴ 𝐵=∫ , ,
4𝜋 𝑠2 𝑠
I, as shown in Fig. 4.24. Let the plane of the
loop be perpendicular to the plane of Since μQ and I are constant, and s and
paper. We wish to find field B at an axial a are same for all points on the circular
point P at a distance r from the centre C. loop, we have
𝜇0 𝐼𝑎 𝜇0 𝐼𝑎 𝜇0 𝐼𝑎 2
𝐵= ∫ 𝑑𝑙 = ⋅ 2𝜋𝑎 =
4𝜋𝑠 3 4𝜋𝑠 3 2𝑠 3
[∵ ∫ 𝑑𝑙 = circumference = 2𝜋𝑎]
0 𝜇 𝐼𝑎 2
or 𝐵 = 2(𝑟2 +𝑎 2 )3⁄2 [∵ 𝑠 =
(𝑟 2 + 𝑎 2 )1⁄2 ]
𝜇0 𝐼𝑎 2
⃗ =
𝐵 𝑖̂
2(𝑟 2 + 𝑎 2 )3⁄2
Fig. 4.24 Magnetic field on the axis of a As the direction of the field is along +ve
circular current loop. X-direction, so we can write
Consider a current element dl at the top 𝜇0 𝐼𝑎 2
of the loop. It has an outward coming ⃗ =
𝐵 𝑖̂
2(𝑟 2 + 𝑎 2 )3⁄2
current.
If the coil consists of N turns, then 𝐵 =
If s be the position vector of point P 𝜇0𝑁𝐼𝑎 2
relative to the element 𝑑𝑙 ⃗⃗⃗ , then from 2(𝑟 2 +𝑎 2 )3⁄2
Biot-Savart law, the field at point P due to
𝜇 𝐼𝑑𝑙sin𝜃 Special Cases
the current element is 𝑑𝐵 = 4𝜋0 ⋅ 𝑠2
1. At the centre of the current loop,
⃗⃗⃗ ⊥ 𝑠, i.e., θ =90°, therefore
Since 𝑑𝑙 r = 0, therefore
𝜇0 𝐼𝑑𝑙 𝜇0 𝑁𝐼𝑎 2 𝜇0 𝑁𝐼
𝑑𝐵 = ⋅ 𝐵= =
4𝜋 𝑠 2 2𝑎 3 2𝑎
⃗ lies in the plane of paper
The field 𝑑𝐵 𝜇0 𝑁𝐼𝐴
𝐵=
and is perpendicular to s, as shown by ⃗⃗⃗⃗⃗
𝑃𝑄 . 2𝜋𝑎 3
Let Φ be the angle between OP and CP. where A = 𝜋α2 = area of the circular current
Then dB can be resolved into two loop. The field is directed perpendicular to
rectangular components. the plane of the current loop.
1. dB sin Φ along the axis, 2. At the axial points lying far away
2. dB cos Φ perpendicular to the from the coil, r » a, so that
axis. 𝜇0 𝑁𝐼𝑎 2 𝜇0 𝑁𝐼𝐴
𝐵= =
For any two diametrically opposite 2𝑟 3 2𝜋𝑟 3
elements of the loop, the components This field is directed along the axis of
perpendicular to the axis of the loop will be the loop and falls off as the cube of the
equal and opposite and will cancel out. distance from the current loop.
Their axial components will be in the same
direction, 3. At an axial point at a distance
equal to the radius of the coil i.e., r = a,
i.e., along CP and get added up. we have
∴ Total magnetic field at the point P in 𝜇0 𝑁𝐼𝑎 2 𝜇0 𝑁𝐼
the direction CP is 𝐵= 2 2 3 ⁄ 2
= 5 ⁄2
2(𝑎 + 𝑎 ) 2 𝑎
𝐵 = ∫ 𝑑𝐵sin𝜙 Direction of the magnetic field. Fig.
14
4.25 shows the magnetic lines of force of a
circular wire carrying current. The lines of
force near the wire are almost concentric
circles. As we move radially towards the
centre of the loop, the concentric circles
become larger and larger i.e., the lines of
force become less and less curved. If the
plane of the circular loop is held
perpendicular to the magnetic meridian,
the lines at the centre are almost straight, Fig. 4.27 Variation of B along the axis of
parallel and perpendicular to the plane of a circular current loop.
the loop. Thus the magnetic field is uniform the magnetic field along the axis of a
at the centre of the loop. circular loop with distance from its centre.
The value of B is maximum at the centre,
and it decreases as we go away from the
centre, on either side of the loop.
Examples based on
Formulae Used
1. Magnetic field at the centre of a
circular loop,
𝜇0 𝐼
𝐵=
Fig. 4.25 Magnetic lines of force of a 2𝑟
circular current loop. 2. Magnetic field at an axial point of a
Rules for finding the direction of a circular loop,
magnetic field due to a circular current 𝜇0 𝐼𝑎 2
loop. Either of the following two rules can 𝐵=
be used for finding the direction of B. 2(𝑟 2 + 𝑎 2 )3⁄2
15
hydrogen atom, an electron revolves lies in the XY plane with its centre at the
around the nucleus in a circular orbit of origin of co-ordinates. Find the magnetic
radius 5.11 × 10-11 m at a frequency of 6.8 field, at the point (0, 0, 20 √3 cm), when
× 1015 Hz. What is the magnetic field set up this coil carries a current of (𝜋) 𝐴
2
at the centre of the orbit?
[CBSED16C]
Solution. If n is the frequency of
revolution of the electron, then Solution. Here N = 100, a = 20 cm = 0.2
m
𝐼 = 𝑛𝑒 = 6.8 × 1015 × 1.6 × 10−19
2
= 6.8 × 1.6 × 10−4 A 𝑧 = 20√3cm = 0.2√3m, 𝐼 = 𝐴
𝜋
𝜇0 𝐼
𝐵= The coil lies in XY-plane and the field
2𝑟
4𝜋 × 10−7 × 6.8 × 1.6 × 10−4 point (0, 0, 20√3) lies on 2-axis.
= = 13.4T
2 × 5.11 × 10−11 Magnetic field at the axial field point,
Example 12. The radius of the first orbit of 𝜇0 𝑁𝐼𝑎 2
hydrogen atom is 0.5 A. The electron 𝐵=
2(𝑎 2 + 𝑧 2 )3⁄2
moves in an orbit with a uniform speed of 2
2.2 × 106 ms-1. Calculate (i) the equivalent 4𝜋 × 10−7 × 100 × (𝜋) × (0.2)2
current (ii) the equivalent dipole moment = T
2 3 ⁄2
and (iii) the magnetic field produced at the 2 [(0.2)2 + (0.2√3) ]
centre of the nucleus due to the motion of
this electron? Use 𝜇0 /4𝜋 = 10−7 𝑁𝐴−2 and 4 × 10−5 × 0.04 0.16 × 10−5
= T = T
electronic charge = 1.6 × 10−19 C. 2(0.2)3 (1 + 3)3⁄2 2 × 0.008 × 8
[ISCE981 1
-10 = × 10−4 T = 25 × 10−6 T = 25𝜇T
Solution, (i) Here r = 0.5 A =0.5 × 10 8
m, Example 14. The magnetic field due to a
v = 2.2 × 10 ms 6 -1 current-carrying circular loop of radius 12
cm at its centre is 0.50 × 10-4 T. Find the
Period of revolution of electron, magnetic field due to this loop at a point on
2𝜋𝑟 2 × 22 × 0.5 × 10−10 the axis at a distance of 5.0 cm from the
𝑇= = centre.
𝑣 7 × 2.2 × 106
1 𝜇0 𝐼
= × 10−15 s Solution. 𝐵 centre = and 𝐵axial =
7 2𝑎
𝜇0 𝐼𝑎 2
Equivalent current, 2(𝑎 2+𝑟 2 )3⁄2
16
of 0.5 A is passed through both of them (i) Solution. For the net magnetic field at
in the same direction, (ii) in the opposite the point O to be zero, the direction of
directions. Find the magnetic field at the current in loop L2 should be opposite to that
centre of each coil. in loop 𝐿1
Solution. Here a =0.1 m, N =20, r = 0.1 m, I Magnitude of magnetic field due to
=0.5 A current 𝐼1 in 𝐿1 = Magnitude of magnetic
Magnetic field at the centre of each coil field due to current 𝐼2 in 𝐼2
due to its own current is 𝜇0 𝐼1 (0.03)2
or
𝜇0 𝑁𝐼 4𝜋 × 10−7 × 20 × 0.5 2[(0.03)2 + (0.04)2 ]3⁄2
𝐵1 = = 𝜇0 𝐼2 (0.04)2
2𝑎 2 × 0.1 =
= 6.28 × 10−5 T 2[(0.04)2 + (0.03)2⁄2
Magnetic field at the centre of one coil (0.03)2 9
due to the current in the other coil is or 𝐼2 = 2 𝐼1 = × 1A = 0.56A
(0.04) 16
𝜇0 𝑁𝐼𝑎 2 Example 1 7. A long wire having a
𝐵2 =
2(𝑎 2 + 𝑟 2 )3⁄2 semi-circular loop of radius r carries a
current I, as shown in Fig. 4.28. Find the
4𝜋 × 10−7 × 20 × 0.5 × (0.1)2
= magnetic field due to entire wire at the
2[(0.1)2 + (0.1)2 ]3⁄2 point O.
0.628 × 10−7
=
[2 × (0.1)2 ]3⁄2
0.628 × 10−7
= = 2.22 × 10−5 T
2√2 × 10−3
Fig. 4.28
(i) When the currents are in the same
direction, the resultant field at the Solution. Magnetic field due to linear
centre of each coil is portion. Any element 𝑑𝑙 ⃗⃗⃗ of linear portions
like PQ or ST will make angles 0 or π with
𝐵 = 𝐵1 + 𝐵2 = 6.28 × 10−5 + 2.22 × 10−5 =
the position vector r. Therefore, field at O
8.50 × 10-5 T. due to linear portion is
(ii) When the currents are in opposite 𝜇0 𝐼𝑑𝑙sin𝜃
directions, the resultant field is 𝐵= ⋅ =0
4𝜋 𝑟2
𝐵 = 𝐵1 − 𝐵2 = 6.28 × 10−5 − 2.22 × 10−5 = Magnetic field due to semi-circular
4.06 × 10-5 T. ⃗⃗⃗ on this portion will
portion. Any element 𝑑𝑙
Example 16. Two coaxial circular loops be perpendicular to the position vector 𝑟 ,
𝐿1 and 𝐿2 of radii 3 therefore, field due to one such element at
c point will be
m and 4 cm are placed as shown. What
𝜇0 𝐼𝑑𝑙sin𝜋/2 𝜇0 𝐼𝑑𝑙
should be the magnitude and direction of 𝑑𝐵 = ⋅ =
the current in the loop so that the net 4𝜋 𝑟2 4𝜋 𝑟 2
magnetic field at the point O be zero? Magnetic field due to the entire circular
portion is given by
[CBSE SP 08] 𝜇0 𝐼 𝜇0 𝐼 𝜇0 𝐼
𝐵 = ∫ 𝑑𝐵 = 2 ∫ 𝑑𝑙 = 2 ⋅ 𝜋𝑟 =
4𝜋𝑟 4𝜋𝑟 4𝑟
𝜇0 𝐼
∴ Total magnetic field at point 𝑂 = 4𝑟
17
∴ Total field at O is
𝜇0 𝐼 1
𝐵 = 𝐵1 + 𝐵2 = (1 + 𝜋), up the plane of
2𝑟
paper.
Example 20. Figure 4.31 shows a current
Fig. 4.29 loop having two circular segments and
joined by two radial lines. Find the
centre of the arc ? Would your answer magnetic field at the centre O.
change if the wire were bent into a
semicircular arc of the same radius but in
the opposite way as shown in Fig. 4.29(b)?
[NCERT; Pb 91]
Solution, (i) Magnetic field at the centre
𝜇 𝐼
of the arc is 𝐵 = 4𝑟0
Here, I= 12 A r =2.0 cm =0.02 m, μ0 =4π×
10-7 TmA-1
𝜇0 = 4𝜋 × 10−7 TmA−1 Fig. 4.31
4𝜋 × 10−7 × 12 Solution. Since the point O lies on lines
𝐵= = 1.9 × 10−4 T SP and QR, so the magnetic field at O due
4 × 0.02
to these straight portions is zero.
According to right hand rule, the
direction of the field is normally into the The magnetic field at O due to the
plane of paper. circular segment PQ is
(ii) The magnetic field will be of same 𝜇0 𝐼
𝐵1 = 𝑙
magnitude, 𝐵 = 1.9 × 10−4 T 4𝜋 𝑎 2
The direction of the field is normally out Here, l = length of arc PQ = a
of the plane of paper. 𝜇0 𝐼𝛼
𝐵1 = directed normally upward
4𝜋 𝑎
Example 19. A long wire is bent as shown
in Fig. 4.30. What will be the magnitude Similarly, the magnetic field at O due to
and direction of the field at the centre O of the circular segment SR is
the circular portion, if a current I is passed 𝜇 𝐼𝛼
𝐵2 = 4𝜋0 ⋅ , directed normally downward.
through the wire? Assume that the various 𝑏
portions of the wire do not touch at point P. The resultant field at O is
𝜇0 𝐼𝛼 1 1
𝐵 = 𝐵1 − 𝐵2 = [ − ]
4𝜋 𝑎 𝑏
𝜇0 𝐼𝛼(𝑏 − 𝑎)
𝐵=
4𝜋𝑎𝑏
Example 21. The wire shown in Fig. 4.32
carries a current of 10 A. Determine the
magnitude of the magnetic field at the
centre O. Given radius of the bent coil is 3
Solution. The system consists of a cm.
straight conductor and a circular loop. Field
due to straight conductor at point O is [Punjab 01 ; AIIMS 13]
𝜇0 𝐼
𝐵1 = , up the plane of paper
2𝜋𝑟
18
Similarly, magnetic induction at O due
to straight conductor qc is
𝐵 = 𝐵1 + 𝑅2 = 10−5 + 10−5 = 2 × 10−5 T
normally out of the plane of paper.
Total magnetic induction at O is
19
a thin wire shaped as shown in Fig. 4.35. Now suppose the wire B is bent into a
The radius of the curved part of the wire is square of side L. We know that the
equal to R =120 mm, the angle 2 ϕ = 90°. magnetic field due to a wire of finite length
Find the magnetic induction of the field at whose ends make angles α and β with the
the point O. perpendicular dropped on wire from the
given point at distance r from it is given by
𝜇0 𝐼
𝑑𝐵 = (sin𝛼 + sin𝛽)
4𝜋𝑟
Fig. 4.35,
Solution. Magnetic induction at O due
to the line segment AB is
𝜇0 𝐼
𝐵1 = × [sin𝜙 + sin𝜙]
4𝜋 𝑅cos𝜙 Fig. 4.36
𝜇 2𝐼 ∴ Magnetic field at O due to conductor
= 4𝜋0 ⋅ tan𝜙 / acting normally downwards
𝑅
AB is
Magnetic field at O due to the current
through arc segment is 𝜇0 𝐼 2√2𝜇0 𝐼
𝑑𝐵 = (sin45° + sin45° ) =
𝜇 𝐼 4𝜋 ⋅ 𝐿/2 4𝜋𝐿
𝐵2 = 4𝜋0 × 𝑅 (2𝜋 − 2𝜙) / acting normally
[∵ 𝛼 = 𝛽 = 45° , 𝑟 = 𝐿/2]
downwards
By symmetry, magnetic field at O due
Total magnetic induction at O,
to all the four sides of the square will be in
𝜇0 𝐼 the same direction. Hence total field at O
𝐵 = 𝐵1 + 𝐵2 = ⋅ [𝜋 − 𝜙 + tan𝜙]
2𝜋 𝑅 due to the current-carrying square is
4𝜋 × 10−7 × 5 𝜋 𝜋 2√2𝜇0𝐼 8√2𝜇0𝐼
= [𝜋 − + tan ] 𝐵=4× = ...(2)
2𝜋 × 0.120 4 4 4𝜋𝐿 4𝜋𝐿
20
centre. Find the magnitude and direction of 1.57 × 10-5T)
the magnetic field at the common centre of
5. Two identical circular wires P and Q
the two coils, if they carry currents equal to
each of radius R and carrying current T
I and √3𝐼 respectively. are kept in perpendicular planes such
[CBS that they have a common centre as
E F 16 ; OD 17] shown in figure. Find the magnitude
and direction of the net magnetic field
at the common centre of the two coils.
[CBSE D 12]
𝜇0 𝐼
(Ans. at an angle of 45° with either of
√2𝑅
the two fields)
⃗ 𝑝 = 𝜇0 𝐼 vertically upwards,
Solution. 𝐵
2𝑅
𝜇0 √3𝐼
⃗0 =
𝐵
2𝑅
Resultant field at the centre is 6. Two identical loops P and Q each of
radius 5 cm are lying in perpendicular
2 1⁄2
𝜇0 𝐼 2 𝜇0 √3𝐼 planes such that they have a common
𝐵 = √𝐵𝑃2 + 𝐵𝑄2 = [( ) + ( ) ] centre as shown in the figure. Find the
2𝑅 2𝑅
magnitude and direction of the net
𝜇0 𝐼 𝜇0 𝐼 magnetic field at the common centre of
= (1 + 3)1⁄2 = ( ) the two coils, if the carry currents equal
2𝑅 𝑅
to 3 A and 4 A respectively.
𝐵𝑃 1
tan𝜃 = = ⇒ 𝜃 = 30° [CBSE OD 17]
𝐵𝑄 √3
4
Problems for Practice (Ans. 20𝜋𝜇T, 𝜃 = tan−1 3 )
1. Consider a tightly wound 100 turn coil
of radius 10 cm, carrying a current of 1
A. What is the magnitude of the
magnetic field at the centre of the coil?
[NCERT] (Ans. 6.28 × 10-4 T)
2. A circular loop of one turn carries a
current of 5.0 A. If the magnetic field at 𝜋
the centre is 0.20 mT, find the radius of 7. A straight wire, of length 2 metre, is
the loop. (Ans. 1.57 cm) bent into a circular shape. If the wire
3. What current has to be maintained in a were to carry a current of 5 A, calculate
circular coil of wire of 50 turns and 2.54 the magnetic field, due to it, before
cm in radius in order to just cancel the bending, at a point distant 0.01 times
effect of earth's magnetic field at a the radius of the circle formed from it.
place where the horizontal component Also calculate the magnetic field, at the
of earth's field is 1.86 × 10 -5 T ? (Ans. centre of the circular loop formed, for
0.015 A) the same value of current.
21
1.2 A. Find the magnetic field (i) at a
point on the axis of the coil at a
distance of 40 cm from its centre and
(ii) at the centre of the coil. [Ans. (i) 6.5
× 10-7 T (ii) 6.6 × 10-4 T]
9. A thick straight copper wire, carrying a
current of 10 A is bent into a
semicircular arc of radius 7.0 cm as
shown in Fig. 4.38(a). (i) State the
direction and calculate the magnitude
12. A circular segment of radius 10 cm
of magnetic field at the centre of arc. (ii)
subtends an angle of 60° at its centre.
How would your answer change if the
A current of 9 A is flowing through it.
same wire were bent into a semicircular
Find the magnitude and direction of the
arc of the same radius but in opposite
magnetic field produced at the centre
way as shown in Fig. 4.38(b)?
(Fig. 4.41).
[Ans. (i) 4.5 × 10-5 T, outside the plane of
(Ans. 9.42 ×10-6 T)
paper, (ii) 4.5 × 10-5T, into the plane of
paper]
Fig. 4.38
10. A long wire is bent as shown in Fig.
4.39. Find the magnitude and direction
of the magnetic field at the centre O of
the circular part, if a current I is passed 13. A current of I ampere is flowing through
through the wire. the bent wire shown in Fig. 4.42. Find the
magnitude and direction of the magnetic
field at point O.
𝜇 𝐼𝛼
[Ans. 𝐵 = 4𝜋0 directed normally
𝑟
downward]
14. In Fig. 4.43, the curved portion is a
semi-circle and the straight wires are long.
𝜇 1 Find the magnetic field at the point O.
[Ans. 2𝑅0 (1 − 𝜋) normally into the plane of
𝜇0 𝐼 2
paper] Ans. [ Ans. (1 + 𝜋)]
2𝑑
22
find the magnetic field at the point O. 12𝜋 × 10−6 3
tan𝜃 = =
5𝜇0 𝐼 16𝜋 × 10−6 4
Ans. 𝜃,
( 24𝜋𝑟 ) ∴
normally inward 4
𝜃 = tan−1
3
𝜋 1
7. Here 2𝜋𝑟 = 2 metre ⇒ 𝑟 = 4 = 0.25m
23
0 𝜇 𝐼 𝜇0 𝐼 2
𝐵2 = 2𝑅 = normally into the plane of [1 + 𝜋]
2𝑑
paper Resultant field at O is
15. Magnetic field at O due to the straight
𝜇0 𝐼 1 parts of the wire will be zero. Magnetic
𝐵 = 𝑅2 − 𝐵1 = (1 − )
2𝑅 𝜋 fields at O due to the three circular arcs
normally into the plane of paper. of radii r, 2r and 3r are
𝜇0 𝐼 𝜃
𝜇 𝐼 𝜇 𝐼 𝐵1 = ⋅ 𝑟 , acting normally inward
11. 𝐵 = 𝐵1 + 𝐵2 = 4𝑅0 + 4𝑅0 = 4𝜋
1 2
𝜇0 𝐼 1 1 𝜇0 𝐼 𝜃
( + ) 𝐵2 = ⋅ 2𝑟, acting normally outward
4 𝑅1 𝑅2 4𝜋
𝜋 𝜇0 𝐼 𝜃
12. Here 𝜃 = 60° = 3 rad 𝐵3 = ⋅ 3𝑟, acting normally inward
4𝜋
1 𝜋 𝑙 𝜋𝑟 Thus the total magnetic field at the centre
As 𝜃(rad) = ∴ = or 𝑙=
𝑟 3 𝑟 3 O is
According to Biot-Savart law, magnetic 𝜇0 𝐼 𝜃 𝜃 𝜃
field at the centre O is 𝐵 = 𝐵1 − 𝐵2 + 𝐵3 = ( − + )
4𝜋 𝑟 2𝑟 3𝑟
𝜇0 𝐼𝑙 𝜇0 𝐼 𝜋𝑟 𝜇0 𝜋 𝐼 5𝜇 𝐼
0
= 24𝜋𝑟 𝜃, acting normally inward.
𝐵= ⋅ 2= ⋅ ⋅ = ⋅ ⋅
4𝜋 𝑟 4𝜋 𝑟 2 3 4𝜋 3 𝑟
4.8 AMPERE'S CIRCUITAL LAW AND ITS
10−7 × 3.14 × 9
= = 9.42 × 10−6 T APPLICATION TO INFINITELY LONG
3 × 0.10 STRAIGHT WIRE
13. Any element 𝑑𝑙 ⃗⃗⃗ on the arc will be
9. (a) State Ampere's circuital law and
perpendicular to the position vector 𝑟, prove it for the magnetic field produced
so the field due to one such element at by a straight current carrying
the centre O will be conductor.
𝜇0 𝐼𝑑𝑙sin𝜋/2 𝜇0 𝐼𝑑𝑙 Ampere's circuital law. Just as Gauss's
𝑑𝐵 = ⋅ = ⋅
4𝜋 𝑟2 4𝜋 𝑟 2 law is an alternative form of Coulomb's law
Magnetic field due to the entire arc at the in electrostatics, similarly we have
centre O, Ampere's circuital law as an alternative
form of Biot-Savart law in magnetostatics.
𝜇0 𝐼 𝜇0 𝐼 Ampere's circuital law gives a relationship
𝐵 = ∫ 𝑑𝐵 = 2 ∫ 𝑑𝑙 = .𝑙
4𝜋𝑟 4𝜋𝑟 2 between the line integral of a magnetic field
But / = length of arc = car B and the total current I which produces
this field.
𝜇0 𝐼 𝜇0 𝐼𝛼
𝐵= , 𝛼𝑟 = Ampere's circuital law states that the line
4𝜋𝑟 2 4𝜋𝑟
⃗ around
integral of the magnetic field 𝐵
14. Magnetic field at point O due to any
current element is perpendicular to and any closed circuit is equal to μ0
points out of the plane of paper. (permeability constant) times the total
current I threading or passing through
Magnetic field at O due to the upper this closed circuit. Mathematically,
straight wire is
⃗⃗⃗ = 𝜇0 𝐼
⃗ ⋅ 𝑑𝑙
∮𝐵
1 𝜇0 𝐼 𝜇0 𝐼
𝐵1 = × = In a simplified form, Ampere's circuital law
2 2𝜋(𝑑/2) 2𝜋𝑑
⃗ is directed along the
states that if field 𝐵
Similarly, field at O due to lower straight tangent to every point on the perimeter L of
𝜇0 𝐼
wire is 𝐵2 = 2𝜋𝑑 a closed curve and its magnitude is
constant along the curve, then
Field at O due to the semicircle of radius d
1 𝜇0 𝐼
12 is 𝐵3 = 2 × 2(𝑑/2)
𝜇0 𝐼
= 2𝑑 BL=μ0 I
where I is the net current enclosed by the
Resultant field at O, 𝐵 = 𝐵1 + 𝐵2 + 𝐵3 = closed circuit. The closed curve is called
24
Amperean loop which is a geometrical conductor. Fig. 4.46 shows a circular loop
entity and not a real wire loop. of radius r around an infinitely long straight
wire carrying current I. As the field
Proof for a straight current carrying
⃗ at any point of
lines are circular, the field 𝐵
conductor. Consider an infinitely long
straight conductor carrying a current I. the circular loop is directed along the
From Biot-Savart law, the magnitude of the tangent to the
⃗ due to the current carrying
magnetic field 𝐵
conductor at a point, distant r from it is
given by
𝜇0 𝐼 𝜇 𝐼
∮𝐵 = ⃗⃗⃗ = 𝜇0 𝐼𝐵 = 0
⋅ 𝑑𝑙
2𝜋𝑟 2𝜋𝑟
Fig. 4.46
circle at that point. By symmetry, the
magnitude of field 𝐵⃗ is same at every point
of the circular loop. Therefore,
⃗⃗⃗ = ∮ 𝐵𝑑𝑙cos0° = 𝐵∮ 𝑑𝑙 = 𝐵 ⋅ 2𝜋𝑟
⃗ ⋅ 𝑑𝑙
∮𝐵
Fig. 4.45 Ampere's circuital law.
From Ampere's circuital law,
As shown in Fig. 4.45, the field B is
directed along the circumference of the B.2𝜋𝑟 = μQI
circle of radius r with the wire as centre. 0𝜇 𝐼
⃗ is same for all ∴𝐵 = 2𝜋𝑟
The magnitude of the field 𝐵
points on the circle. To evaluate the line For Your Knowledge
integral of the magnetic field 𝐵 ⃗ along the
➢ Ampere's circuital law is not
circle, we consider a small current element
independent of the Biot- Savart law.
⃗⃗⃗ along the circle. At every point on the
𝑑𝑙 It can be derived from the
circle, both 𝐵 ⃗⃗⃗ are tangential to the
⃗ and 𝑑𝑙 Biot-Savart law. Its relationship to
circle so that the angle between them is the Biot-Savart law is similar to the
zero. relationship between Gauss's law
⃗⃗⃗ = 𝐵𝑑𝑙cos0° = 𝐵𝑑𝑙 ⋅ 𝑑𝑙
⃗⃗⃗ = 𝜇0 𝐼 and Coulomb's law.
⃗ ⋅ 𝑑𝑙
∴∮ 𝐵 ➢ Both Ampere's circuital law and
Hence the line integral of the magnetic Biot-Savart law relate magnetic field
field along the circular path is to the electric current.
➢ Ampere's and Gauss's laws relate
𝜇0 𝐼
∮𝐵 ⃗⃗⃗ = ∮ 𝐵𝑑𝑙 = 𝐵∮ 𝑑𝑙 =
⃗ ⋅ 𝑑𝑙 .𝑙 one physical quantity (magnetic or
2𝜋𝑟 electric quantity) on the boundary or
𝜇0 𝐼 periphery to another physical
= ⋅ 2𝜋𝑟
2𝜋𝑟 quantity (current or charge), called
source, in the interior.
⃗⃗⃗ = 𝜇0 𝐼
⃗ ⋅ 𝑑𝑙
∴∮ 𝐵 ➢ Ampere's circuital law holds for
This proves Ampere's law. This law is steady currents which do not
valid for any assembly of current and for change with time.
any arbitrary closed loop. ➢ Although both Ampere's law and
Biot-Savart law are equivalent in
9. (b) Calculate, using Ampere's physical content, yet the Ampere's
circuital theorem, the magnetic field due to law is more useful under certain
an infinitely long wire carrying a current I. symmetrical situations. The
Application of Ampere's law to a straight mathematics of finding the magnetic
25
field of a solenoid and toroid midpoint P is uniform and strong. The field
becomes much simpler if we apply at the exterior midpoint Q is weak and is
Ampere's law. along the axis of the solenoid with no
perpendicular component. Fig. 4.48 shows
4.9 MAGNETIC FIELD INSIDE A
the field pattern of a solenoid of finite
STRAIGHT SOLENOID
length.
10. Give a qualitative discussion of
the magnetic field produced by a straight
solenoid. Apply Ampere's circuital law to
calculate magnetic field inside a straight
solenoid.
Magnetic field of a straight solenoid : A
qualitative discussion. A solenoid means
an insulated copper wire wound closely in
the form of a helix. The word solenoid
comes from a Greek word meaning
channel and was first used by Ampere. By
a long solenoid, we mean that the length of
the solenoid is very large as compared to Fig 4.48 Magnetic field of a finite
its diameter. solenoid.
The polarity of any end of the solenoid
can be determined by using clock rule or
Ampere's right hand rule.
Ampere's right hand rule. Grasp the
solenoid with the right hand so that the
fingers point along the direction of the
current, the extended thumb will then
indicate the face of the solenoid that has
north polarity (Fig. 4.49).
26
threads the loop abed, nl times.
∴ Total current threading the loop abed =
nil
Hence 𝐵𝑙 = 𝜇0 𝑛𝑙𝐼 or 𝐵 = 𝜇0 𝑛𝐼
It can be easily shown that the
magnetic field at the end of the solenoid is
just one half of that at its middle. Thus
1
𝐵end = 2 𝜇0 𝑛𝐼
Fig. 4.50 The magnetic field of a very
long solenoid. Figure 4.51 shows the variation of
magnetic field on the axis of a long straight
it. Fig. 4.50 shows the sectional view of a solenoid with distance x from its centre.
long solenoid. At various turns of the
solenoid, current comes out of the plane of
paper at points marked ⊙ and enters the
plane of paper at points marked ⊗ . To
determine the magnetic field 𝐵 ⃗ at any
inside point, consider a rectangular closed
path abed as the Amperean loop.
According to Ampere's circuital law,
⃗⃗⃗
⃗ ⋅ 𝑑𝑙
∮𝐵
Fig. 4.51 Variation of magnetic field
= μ0 ×Total current through the loop abed b along the axis of solenoid.
Now ∮ 𝐵 ⃗⃗⃗ = ∫𝑏 𝐵
⃗ ⋅ 𝑑𝑙 ⃗⃗⃗
⃗ ⋅ 𝑑𝑙 4.10 MAGNETIC FIELD DUE TO A
𝑎
TOROIDAL SOLENOID
𝑐 𝑑 𝑎
+∫ 𝐵 ⃗⃗⃗ + ∫ 𝐵
⃗ ⋅ 𝑑𝑙 ⃗⃗⃗ + ∫ 𝐵
⃗ ⋅ 𝑑𝑙 ⃗⃗⃗
⃗ ⋅ 𝑑𝑙 11. Apply Ampere's circuital law to find
𝑏 𝑐 𝑑 the magnetic field both inside and outside
𝑐
⃗⃗⃗ = ∫𝑐 𝐵𝑑𝑙cos90° = 0
⃗ ⋅ 𝑑𝑙
But ∫𝑏 𝐵 of a toroidal solenoid.
𝑏
𝑎 𝑎 Magnetic field due to a toroidal
∫ 𝐵 ⃗⃗⃗ = ∫ 𝐵𝑑𝑙cos90° = 0
⃗ ⋅ 𝑑𝑙 solenoid. A solenoid bent into the form of a
𝑑 𝑑 closed ring is called a toroidal solenoid.
𝑑 Alternatively, it is an anchor ring (torous)
∫ 𝐵 ⃗⃗⃗ = 0
⃗ ⋅ 𝑑𝑙 around which a large number of turns of a
𝑐 metallic wire are wound, as shown in Fig.
as B = 0 for points outside the solenoid. 4.52. We shall see that the magnetic field B
𝑏 has a constant magnitude everywhere
⃗⃗⃗ = ∫ 𝐵
⃗ ⋅ 𝑑𝑙
∮𝐵 ⃗⃗⃗
⃗ ⋅ 𝑑𝑙 inside the toroid while it is zero in the open
𝑎 space interior (point P) and exterior (point
𝑏 𝑏 Q) to the toroid.
= ∫ 𝐵𝑑𝑙cos0° = 𝐵 ∫ 𝑑𝑙 = 𝐵𝑙
𝑎 𝑎
where,
l = length of the side ab of the rectangular
loop abed.
Let number of turns per unit length of
the solenoid = n
Then number of turns in length l of the
solenoid = nl
Thus the current I of the solenoid Fig. 4.52 A toroidal solenoid.
27
Figure 4.53 shows a sectional view of 𝜇0 𝑁𝐼
𝐵=
the toroidal solenoid. The direction of the 2𝜋𝑟
magnetic field inside is clockwise as per If r be the average radius of the toroid
the right-hand thumb rule for circular loops.
and n the number of turns per unit length,
Three circular Amperean loops are shown then
by
N = 2 πrn
𝐵 = 𝜇0 𝑛𝐼
3. For points in the open space
exterior to the toroid. Each turn of the toroid
passes twice through the area enclosed by
the Amperean loop 3. But for each turn, the
current coming out of the plane of paper is
cancelled by the current going into the
plane of paper. Thus, I =0 and hence 𝐵3
=0.
28
length. wind a 400 turn solenoid of radius 1.0
cm and length 20 cm. Find the emf
3. Magnetic field inside a toroidal
of a battery which when connected across
solenoid, 𝐵 = 𝜇0 𝑛𝐼
the solenoid would produce a magnetic
4. Magnetic field is zero outside the
field of 10-2 T near the centre of the
toroid.
solenoid.
Units Used
Solution. Length of wire used = 2𝜋r× No. of
B is in tesla, current I in ampere and nin turns
m-1.
= 2𝜋× 1.0 × 10-2 × 400 m
Example 27. A solenoid coil of 300 turns/m
Resistance per unit length = 0.01 Ω m -1
is carrying a current of 5 A. The length of
the solenoid is 0.5 m and has a radius of 1 Total resistance of wire,
cm. Find the magnitude of the magnetic
R = 2𝜋× 1.0 ×10-2 × 400×0.01 = 8𝜋 × 10-2 Ω
field inside the solenoid.
No. of turns per unit length,
[CBSE F 04]
400
Solution. Here n =300 tums/m, 1 = 5 A 𝑛= = 2000m−1
20 × 10−2
∴ B = μQnI = 4𝜋× 10-7 × 300 × 5 = 1.9 × 𝜀
As 𝐵 = 𝜇0 𝑛𝐼 = 𝜇0 𝑛 𝑅
10-3 T.
Example 28. A solenoid of length 0.5 m has 𝐵𝑅 10−2 × 8𝜋 × 10−2
𝐸= = = 1V
a radius of 1 cm and is made up of 500 𝜇0 𝑛 4𝜋 × 10−7 × 2000
turns. It carries a current of 5 A. What is the
Example 31. A solenoid 50 cm long has 4
magnitude of the magnetic field inside the
layers of windings of350 turns each. The
solenoid?
radius of the lowest layer is 1.4 cm. If the
[NCERT] current carried is 6.0 A, estimate the
magnitude of 𝐵 ⃗ (a) near the centre of the
Solution. Number of turns per unit length,
solenoid on its axis and off its axis, (b) near
𝑁 500 its ends on its axis, (c) outside the solenoid
𝑛= = = 1000turns/m
𝑙 0.5m near its centre.
Here I = 0.5 m and r = 0.01 m i.e., I » a. Solution, (a) The magnitude of the
So we can use formula for magnetic field magnetic field at or near the centre of the
inside a long solenoid. solenoid is given by B = μ0 n I
𝐵 = 𝜇0 𝑛𝐼 = 4𝜋 × 10-7 × 1000 × 5 = 6.28 × where n is the number of turns per unit
10-3 T. length. This expression for B can also be
used if the solenoid has more than one
Example 29. A 0.5 m long solenoid has 500 layer of windings because the radius of the
turns and has a flux density of 2.52 × 10-3 T wire does not enter this equation.
at its centre. Find the current in the Therefore,
solenoid. Given 𝜇0 = 4𝜋 × 10−7 Hm−1 .
[ 𝑛
No. of turns per layer × No. of layers
ISCE 95] =
Length of the solenoid
Solution. Number of turns per unit length,
350 × 4
𝑁 500 = = 2800m−1
𝑛= = = 1000 turns /m 0.50
𝑙 0.5m
As 𝐵 = 𝜇0 𝑛𝐼
𝐵 2.52×10−3
Now 𝐼 = 6.0A, 𝜇0 = 4𝜋 × 10−7 TmA−1 , 𝑛 =
∴𝐼 = = = 2.0A 2800m−1
𝜇0 𝑛 4𝜋×10−7×1000
29
This value of B is for both on and off the
axis, since for an infinitely long solenoid,
the internal field near the centre is uniform
over the entire cross-section.
(b) Magnetic field at the ends of the
solenoid is
𝜇0 𝑛𝐼
𝐵 end = = 1.05 × 10−2 T
2
(c) The outside field near the centre of Fig. 4.54 A steady current I distributed
a long solenoid is negligible compared to uniformly across a wire of radius α.
the internal field.
Solution, (i) Application of Ampere's
Example 32. A coil wrapped around a law to a long straight cylindrical wire.
toroid has inner radius of 20.0 cm and an By symmetry, the magnetic lines of force
outer radius of 25.0 cm. If the wire will be circles, with their centres on the axis
wrapping makes 800 turns and carries a of the cylinder and in planes perpendicular
current of 12.0 A, what are the maximum to the axis of the cylinder. So we consider
and minimum values of the magnetic field Amperean loop as a circle of radius r.
within the toroid?
Field at outside points. The Amperean
Solution. Let a and b denote the inner loop is a circle labelled 2 having radius r >
and outer radii of the toroid. Then a.
𝑁 Length of the loop, L = 2 𝜋𝑟
𝐵max = 𝜇0 𝑛𝐼 = 𝜇0 𝐼
2𝜋𝑎
4𝜋 × 10−7 × 800 × 12.0 Net current enclosed by the loop = I
=
2𝜋 × 20.0 × 10−2 By Ampere's circuital law,
= 9.6 × 10−3 T = 9.6mT BL=μo I
𝑁 or 𝐵 × 2𝜋𝑟 = 𝜇0 𝐼
𝐵min = 𝜇0 𝑛𝐼 = 𝜇0 𝐼
2𝜋𝑏 0 𝜇 𝐼
4𝜋 × 10−7 × 800 × 12.0 or 𝐵 = 2𝜋𝑟
= [For r>α]
2𝜋 × 25.0 × 10−2
1
= 7.68 × 10−3 T = 7.68mT i.e., 𝐵∝𝑟 [For
Example 33. (i) A straight thick long wire of outside points]
uniform cross- section of radius 'a' is Field at inside points. The Amperean
carrying a steady current I. Use Ampere's loop is a circle labelled 1 with r < a.
circuital law to obtain a relation showing
the variation of the magnetic field (B) inside Length of the loop, L = 2 𝜋𝑟
and outside the wire zvith distance r, (r < a) Clearly, the current enclosed by loop 1
and (r > a) of the field point from the centre is less than I. As the current distribution is
of its cross-section. Plot a graph showing uniform, the fraction of I enclosed is
the variation of field B with distance r.
𝐼 𝐼𝑟 2
(ii) Calculate the ratio of magnetic field 𝐼′ = × 𝜋𝑟 2 =
30
[For r< a] We use the results of the above example.
i.e., B∝r [For inside 𝜇 𝐼
0
(i) 𝐵 inside = 2𝜋𝑅 2 ⋅ 𝑟
points]
4𝜋 × 10−7 × 100 × 0.1 × 10−2
Thus the field B is proportional to r as =
we move from the axis of the cylinder 2𝜋 × (0.5 × 10−2 )2
towards its surface and then it decreases = 8.0 × 10-4 T.
1
as 𝑟 . The variation of B with distance r from 𝜇 𝐼 4𝜋×10−7×100
0
(ii) 𝐵 SURFACE = 2𝜋𝑅 =
the centre of the wire is shown in Fig. 2𝜋×0.5×10−2
4.55(a). = 4.0 × 10-3 T.
(iii) Here r = 0.5 + 0.2 = 0.7 cm = 0.7 × 10 -2
m
𝜇0 𝐼 4𝜋 × 10−7 × 100
𝐵 outside = =
2𝜋𝑟 2𝜋 × 0.7 × 10−2
= 2.86 × 10-5 T.
Problems For Practice
Fig. 4.55 (a) Sketch of the magnitude of 1. A long solenoid consists of 20 turns per
the magnetic field for the long cm. What current is necessary to
conductor of radius a. produce a magnetic field of 20 mT
inside the solenoid? (Ans. 8.0 A)
(ii) Suppose the point P lies at distance
a 12 above the surface of the wire and 2. A long solenoid is made by closely
point Q lies at distance a / 2 below the winding a wire of radius 0.5 mm over a
surface. [Fig. 4.55(b)] cylindrical non-magnetic frame so that
successive turns nearly touch each
Magnetic field at point P at distance r other. What will be the magnetic field at
=3α/2 from the axis of the wire is the centre of the solenoid if a current of
𝜇0 𝐼 𝜇0 𝐼 𝜇0 𝐼 5 A flows through it?
𝐵𝑃 = = =
2𝜋𝑟 2𝜋(3/2)𝑎 3𝜋𝑎 (Ans. 2𝜋 × 10-3 T)
Magnetic field at point Q at distance r = 3. The magnetic field at the centre of a 50
α / 2 from the axis of the wire is cm long solenoid is 4.0 × 10-2 T when a
𝜇0 𝐼𝑟 𝜇0 𝐼 𝑎 𝜇0 𝐼 current of 8.0 A flows through it. What
𝐵𝑄 = 2 = 2 ( )= is the number of turns in the solenoid?
2𝜋𝑎 2𝜋𝑎 2 4𝜋𝑎
Take 𝜋 = 3.14 (Ans. 1990)
𝐵𝑃 𝜇0 𝐼 4𝜋𝑎
= × = 4: 3 4. A solenoid is 1.0 m long and 3.0 cm in
𝐵𝑄 3𝜋𝑎 𝜇0 𝐼
diameter. It has five layers of windings
Clearly, B is maximum on the surface of 850 turns each and carries a current
of the wire i.e., at r = α. Hence, of 5.0 A. (i) What is B at its centre? (ii)
𝜇0 𝐼 What is the magnetic flux Φg for a
𝐵MAX = cross-section of the solenoid at the
2𝜋𝑎
centre?
Example 34. A wire of radius 0.5 cm carries
a current of 100 A, which is uniformly [Ans. (0 2.67 × 10-2 T, (ii) 1.9 × 10-5 Wb]
distributed over its cross-section. Find the 5. A solenoid is 2.0 m long and 3.0 cm in
magnetic field (i) at 0.1 cm from the axis of diameter. It has 5 layers of winding of
the wire, (ii) at the surface of the wire and 1000 turns each and carries a current
(iii) at a point outside the wire 0.2 cm from of 5.0 A. What is the magnetic field at
the surface of the wire. the centre? Use the standard value of
Solution. Here R =0.5 cm =0.5 × 10-2 m, I μ0. [Punjab 97C1
=100 A (Ans. 1.57 ×10-2 T)
31
6. A toroid has a core of inner radius 20 6. Mean radius of toroid,
cm and outer radius 22 cm around 20 + 22
which 4200 turns of a wire are wound. If 𝑟= = 21cm = 0.21m
the current in the wire is 10 A, what is 2
the magnetic field (i) inside the core of Number of turns per unit length
toroid (ii) outside the toroid and (iii) in 4200 4200 1000 −1
the empty space surrounded by the = = = m
2𝜋𝑟 2𝜋 × 0.21 𝜋
toroid. [Ans. (i) 0.04 T (ii) Zero (iii)
Zero] (i) Field inside the core of the toroid,
7. A long straight solid conductor of radius 1000
𝐵 = 𝜇0 𝑛𝐼 = 4𝜋 × 10−7 × × 10
4 cm carries a current of 2 A, which is 𝜋
uniformly distributed over its circular = 0.04T
cross-section. Find the magnetic field (ii) Magnetic field outside the toroid is
at a distance of 3 cm from the axis of zero.
the conductor. (Ans. 7.5 × 10-6 T)
(iii) Magnetic field in the empty space
HINTS surrounded by toroid is zero.
1. Here n = 20 cm-1 = 20 × 102 m-1, 7. Current enclosed by the loop of radius
B = 20 mT = 20 × 10 T -3 r,
𝐵 20×10−3 𝐼 𝐼𝑟 2
Current, 𝐼 = 𝜇 = 4𝜋×10−7×20×102 = 8.0A 𝐼′ = × 𝜋𝑟 2 =
0𝑛 𝜋𝑅2 𝑅2
2. Diameter of the wire = 2 × 0.5 = 1.0 mm Using Ampere's circuital law,
= 10-3 m
𝐵𝐿 = 𝜇0 𝐼 ′
∴ Number of turns per unit length,
𝐼𝑟 2 𝜇0 𝐼𝑟
1 𝐵 × 2𝜋𝑟 = 𝜇0 2 or 𝐵=
𝑛= = 103 m−1 𝑅 2𝜋𝑅2
10−3 m
4𝜋 × 10−7 × 2 × 3 × 10−2
Also, 𝐼 = 5A, 𝜇0 = 4𝜋 × 10−7 TmA−1 = = 7.5 × 10−6 T
2𝜋 × (4 × 10−2 )2
𝐵 = 𝜇0 𝑛𝐼 = 4𝜋 × 10−7 × 103 × 5 4.11 FORCE ON A MOVING CHARGE
= 2𝜋 × 10−3 T IN A MAGNETIC FIELD
𝑁
3. 𝐵 = 𝜇0 𝑛𝐼 = 𝜇0 𝑙 𝐼 12. State the factors on which the force
acting on a charge moving in a magnetic
𝐵𝑙 4.0 × 10−2 × 0.50 field depends. Write the expression for this
𝑁= = = 1990
𝜇0 𝐼 4 × 3.14 × 10−7 × 8 force. When is this force minimum and
maximum? Define magnetic field. Also
4. Number of turns per unit length,
define the SI unit of magnetic field.
5 × 850
𝑛= = 4250m−1 Magnetic force on a moving charge.
1.0 The electric charges moving in a magnetic
(i) 𝐵 = 𝜇0 𝑛𝐼 = 4𝜋 × 10−7 × 4250 × 5.0 = field experience a force, while there is no
2.67 × 10−2 T such force on static charges. This fact was
first recognized by Hendrik Antoon
(ii) 𝜙𝐵 = 𝐵𝐴 = 𝐵 × 𝜋𝑟 2
Lorentz, a great Dutch physicist, nearly a
= 2.67 × 1(T2 × 3.14 × (1.5 × 10-2)2 century ago.
= 1.9 ×10-5 Wb. Suppose a positive charge q moves
⃗ and 𝑣
with velocity 𝑣 in a magnetic field 𝐵
5. Number of turns per unit length,
makes an angle θ with 𝐵 ⃗ , as shown in Fig.
5 × 1000
𝑛= = 2500m−1 4.56. It is found from experiments that the
2.0 charge q moving in the magnetic field B
𝐵 = 𝜇0 𝑛𝐼 = 4𝜋 × 10−7 × 2500 × 5.0 experiences a force 𝐹 such that
= 1.57 × 10−2 T
32
1. the force is proportional to the Case 2. If θ =0° or 180°, then F =0
magnitude of the magnetic field, i.e., F
Thus a charged particle moving parallel
∝B
or antiparallel to a magnetic field does not
2. the force is proportional to the charge experience any force in the magnetic field.
q, i.e., F ∝ q
Case 3. If θ = 90°, then F - qvB sin 90° =
3. the force is proportional to the qvB
component of the velocity v in the Thus a charged particle experiences
perpendicular direction of the field B, the maximum force when it moves
i.e., perpendicular to the magnetic field.
F ∝ v sin θ Rules for finding the direction of force on a
charged particle moving perpendicular to a
magnetic field. The direction of magnetic
Lorentz force 𝐹 can be determined by
using either of the following two rules :
1. Fleming's left hand rule. Stretch the
thumb and the first two fingers of the
left hand mutually perpendicular to
each other. If the forefinger points in
the direction of the magnetic field,
central finger in the direction of current,
Fig. 4.56 Magnetic Lorentz force. then the thumb gives the direction of
Combining the above factors, we get the force on the charged particle. (Fig.
4.57)
F ∝ Bqv sin θ or
F = kqvB sin θ
The unit of magnetic field is so defined
that the proportionality constant k becomes
unity in the above equation. Thus
F = qvB sin θ
This force deflects the charged particle
sideways and is called magnetic Lorentz
force. As the direction of 𝐹 is perpendicular
Fig. 4.57 Fleming's left hand rule.
to both 𝑣 and 𝐵 ⃗ , so we can express 𝐹 in
terms of the vector product of 𝑣 and 𝐵⃗ as 2. Right hand (palm) rule. Open the right
hand and place it so that tips of the
⃗)
𝐹 = 𝑞(𝑣 × 𝐵 fingers point in the direction of the field
⃗ and thumb in the direction of velocity
𝐵
Figure 4.56 shows the relationship
𝑣 of the positive charge, then the palm
among the direc- tions of vectors 𝐹 , 𝑣 and
⃗ . Vectors v and B lie in the XY-plane. The faces towards the force 𝐹 , as shown in
𝐵
Fig. 4.58.
direction of 𝐹 is perpendicular to this plane
and points along + Z-axis i.e., 𝐹 acts in the
direction of 𝑣 × 𝐵⃗.
Special Cases
Case 1. If v = 0, then F = 0
Thus a stationary charged particle does
not experience any force in a magnetic
field.
33
Field near a bar magnet 10−2 T
34
a force on another moving charge. magnitude 1.0 T exists in the direction
➢ A stationary source does not south to north. Find that magnetic force
produce any magnetic field to that acts on the particle.
interact with an external magnetic
Solution. Charge on α-particle,
field. Hence no force is exerted on
stationary charge in a magnetic q = + 2e -2 × 1.6 × 10-19C
field. Here v =3 × 104 km s-1 =3 × 107 ms-1, B
➢ An electric charge always = 1.0 T, θ =90°.
experiences a force in an electric
field, whether the charge is
stationary or in motion.
➢ A charge moving parallel or
antiparallel to the direction of the
magnetic field does not experience
any magnetic Lorentz force.
➢ If in a field, the force experienced by
a moving charge depends on the Fig. 4.59
strength of the field and not on the
velocity of the charge, then the field Magnetic force on the α-particle is F = qvB
must be an electric field. sin θ
➢ If in a field, the force experienced by = 2 × 1.6 × 10-19 × 3 × 107 × 1.0 × sin 90°
a moving charge depends not only
on the strength of the field but also = 9.6 × 10-12 N
on the velocity of the charge, then According to Fleming's left hand rule,
the field must be a magnetic field. the magnetic force on the α-particle acts
Examples Based on towards west.
Force moving charges in a magnetic field Example 37. An electron is moving
northwards with a velocity of 3.0 × 107ms-1
Formulae Used in a uniform magnetic field of 10 T directed
Force on a charge q moving with velocity v eastwards. Find the magnitude and the
in a magnetic field at an angle θ with it is direction of the force on the electron.
F = qvB sin θ Solution. q= e = 1.6 × 10-19C, v =3.0 ×
10 ms-1,
7
The direction of the force is given by
Fleming's left hand rule. B = 10 T, θ =90°.
Units Used
Force F is in newton, charge q in coulomb,
velocity v in ms-1 and B in tesla.
Example 35. A proton enters α magnetic
field of flux density 2.5 T with a velocity of
1.5 × 107ms-1 at an angle of 30° with the
field. Find the force on the proton.
Fig. 4.60
-19
Solution. Here q = e = 1.6 × 10 C, The magnitude of magnetic force on the
7
u =1.5 × 10 ms -1
, B=2.5X θ =30° electron is F = qvB sin θ
Force, F = qvB sin θ = 1.6 × 10-19 × 3 × 107 × 10 × sin 90°
= 1.6 × 10-19 × 1.5 × 107 × 2.5 × sin 30° = 4.8 × 10-11 N
= 3 × 10-12 N. As the electron moves northwards,
direction of current is eastwards. According
Example 36. An alpha particle is projected
to Fleming's left hand rule, the magnetic
vertically upward with a speed of 3 × 104
force on the electron acts vertically
kms-1 in a region where a magnetic field of
35
upwards.
Example 38. A positive charge of 1.5 μC is
moving with a speed of 2 × 106 ms-1 along
the positive X-axis. A magnetic field, 𝐵 ⃗ =
̂
(0.2𝑗̂ + 0.4𝑘 ) tesla acts in space. Find the
magnetic force acting on the charge.
Solution. Here q = 1.5 μC =1.5 × 10-6
C,
⃗ = (0.2𝑗̂ + 0.4𝑘̂ )T
𝑣 = 2 × 106 𝑖̂ms−1 , 𝐵
Solution. Magnetic field at point P due
Magnetic force on the positive charge is to the current in wire AB,
⃗)
𝐹 = 𝑞(𝑣 × 𝐵
𝜇0 𝐼 4𝜋 × 10−7 × 4
= 1.5 × 10−6 [2 × 106 𝑖̂ × (0.2𝑗̂ + 0.4𝑘̂)] 𝐵= = = 4 × 10−6 T
2𝜋𝑟 2𝜋 × 0.2
= 3.0[0.2𝑖̂ × 𝑗̂ + 0.4𝑖̂ × 𝑘̂ ] This field acts on the proton normally
into the plane of paper. According to
= (0.6𝑘̂ − 1.2𝑗̂)N. [∵ 𝑖 × 𝑗̂ = 𝑘̂, 𝑖̂ × 𝑘̂ = −𝑗̂]
Fleming's left hand rule, a magnetic force
Example 39. A 5.0 MeV proton is falling acts on the proton towards right in the
vertically downward through a region of plane of paper. The magnitude of this force
magnetic field 1.5 T acting horizontally is
from south to north. Find the magnitude
F = qvB sin 90°
and the direction of the magnetic force
exerted on the proton. Take mass of the = 1.6 × 10-19 × 4 × 106 × 4 × 10-6 × 1
proton as 1.6 × 10-27 kg. = 2.56 × 10-18 N.
Solution. Kinetic energy of the proton is Example 41. Copper has 8.0 × 1028
1 electrons per cubic metre. A copper wire of
𝑚𝑣 2 = 5.0MeV = 5 × 1.6 × 10−13 J length 1 m and cross-sectional area 8.0
2
× 10-6 m2 carrying a current and
2 × 5 × 1.6 × 10−13
𝑣2 = J lying at right angle to a magnetic field of
𝑚 strength 5 × 10-3 T experiences a force of
10×1.6×10−13 8.0 × 10 N. Calculate the drift velocity of
or = = 10 × 1014
1.6×10−27 free electrons in the wire.
𝑣 = 3.16 × 107 ms−1 Solution, n =8 × 1028m-3, l =1 m
Force on the proton is A = 8 × 10-6m2, e = 1.6 × 10-19 C
F = qv B sin 90° Total charge contained in the wire,
-19 7
= 1.6 × 10 × 3.16 ×10 × 1.5 × q = Volume of wire × ne= Alne
-12
= 7.58 × 10 N = 8× 10-6 × 1 × 8 × 1028 × 1.6 × 10-19 C
According to Fleming's left hand rule, = 102.4 × 103 C
the magnetic force on the proton acts
eastwards. If vd is the drift speed of electrons, then
Example 40. A long straight wire AB carries F = qvd B sin 90° -qvd B
a current of 4 A. A proton P travels at 4 × 𝐹 8.0 × 10−2
106 ml s, parallel to the wire, 0.2 m from it 𝑣𝑑 = = ms−1
𝑞𝐵 102.4 × 103 × 5 × 10−3
and in a direction opposite to the current as
shown in Fig. 4.61. Calculate the force = 1.56 × 10-4ms-1.
which the magnetic field of current exerts
Problems For Practice
on the proton. Also specify the direction of
the force. 1. An electron moving with a velocity of
5.0 × 10' ms-1 enters a magnetic field of
[CBSE OD 02]
36
1.0 Wb m-2 at an angle of 30°. Calculate = 1.6 × 10-19 × 5.0 × 107 × 1.0 × sin 30°
the force on the electron.
= 4.0 ×10-12 N.
-12
(Ans. 4.0 × 10 N)
2. (i) Here m = 6.65 × 1027 kg,
2. An α-particle of mass 6.65 × 10 -27 kg
q = + 2e = 2×1.6× 10-16C, B = 0.2 T,
and charge twice that of an electron but
of positive sign travels at right angles to v= 6×105 ms-1, 0 = 90°
a magnetic field with a speed of 6 × 10 5 F = qvB sin 90°
ms-1. The strength of the magnetic field
is 0.2 T. (i) Calculate the force on the = 2 x 1.6 x 10-19 × 6 x 105 × 0.2 × 1N
α-particle. (ii) Also calculate its = 3.84 ×10-14 N
acceleration.
𝐹 3.84 × 10−14
[Ans. (0 3.84 × 10-14 N (ii) 5.77 × 1012 ms-2] 𝑎= = = 5.77 × 1012 ms−2
𝑚 6.65 × 10−27
3. An electron is moving northwards with
3. F = qvB sin 90° = 1.6 × 10-19 × 107 × 3 ×
a velocity of 107 ms-1 in a magnetic field
1
of 3 T, directed downwards. Calculate
the instantaneous force on the = 4.8 ×10-12 N
electron. According to Fleming's left hand rule, the
(Ans. 4.8 × 10-12 N, vertically upwards) force acts vertically upwards.
4. A solenoid, of length 1.5 m, has a 𝜇0 𝑁𝐼 4𝜋×10−7×1500×3
4. 𝐵 = = T = 0.38T
radius of 1.5 cm and has a total of 1500 𝑙 1.5×10−2
37
𝑑𝑣 on a particle perpendicular to its velocity, it
𝐹 = 𝑚𝑎 = 𝑚 does not do any work on the particle. It
𝑑𝑡
⃗
𝑑𝑣
does not change the kinetic energy or
∴𝑚 ⋅𝑣 =0 speed of the particle.
𝑑𝑡
⃗
𝑚 𝑑𝑣 ⃗
𝑑𝑣 Figure 4.62 shows a magnetic field 𝐵 ⃗
Or [ ,𝑣 + 𝑣 + 𝑣 ⋅ 𝑑𝑡 ] = 0
2 𝑑𝑡 directed normally into the plane of paper,
𝑚 𝑑 as shown by small crosses. A charge + q is
Or (𝑣 ⋅ 𝑣) = 0 [𝑣 ⋅ 𝑣 = 𝑣 2 ]
2 𝑑𝑡 projected with a speed v in the plane of the
𝑑𝐾 paper. The velocity is perpendicular to the
Or =0
𝑑𝑡
Or K = constant
Thus a magnetic force does not change
the kinetic energy of the charged particle.
This indicates that the speed of the particle
does not change. According to the
work-energy theorem, the change in kinetic
energy is equal to the work done on the
particle by the net force. Hence the work
done on the charged particle by the
magnetic force is zero. Fig. 4.62 A positively charged particle
4.14 MOTION OF A CHARGED moving in a magnetic field directed into the
PARTICLE IN A UNIFORM plane of paper.
MAGNETIC FIELD
magnetic field. A force F = qvB acts on the
15. Discuss the motion of a charged ⃗ . This
particle perpendicular to both 𝑣 and 𝐵
particle in a uniform magnetic field with force continuously deflects the particle
initial velocity (i) parallel to the field, (ii) sideways without changing its speed and
perpendicular to the magnetic field and (iii) the particle will move along a circle
at an arbitrary angle with the field direction. perpendicular to the field. Thus the
Motion of a charged particle in a magnetic force provides the centripetal
uniform magnetic field. When a charged force. Let r be the radius of the circular
particle having charge q and velocity 𝑣 path. Now
enters a magnetic field B, it experiences a 𝑚𝑣 2
Centripetal force, = Magnetic force,
force 𝑟
qvB
⃗)
𝐹 = 𝑞(𝑣 × 𝐵 𝑚𝑣
𝑟=
The direction of this force is 𝑞𝐵
perpendicular to both 𝑣 and 𝐵 ⃗ . The
Thus the radius of the circular orbit is
magnitude of this force is F = qv B sin θ inversely proportional to the specific
Following three cases are possible : charge (charge to mass ratio q / m) and to
the magnetic field.
1. When the initial velocity is parallel to
Circumference
the magnetic field. Here θ =0°, so F = qvB Period of revolution = Speed
sin 0° =0.
2𝜋𝑟 2𝜋 𝑚𝑣 2𝜋𝑚
Thus the parallel magnetic field does 𝑇= = ⋅ =
not exert any force on the moving charged 𝑣 𝑣 𝑞𝐵 𝑞𝐵
particle. The charged particle will continue Clearly, the time period is independent
to move along the line of force. of v and r. If the particle moves faster, the
2. When the initial velocity is radius is larger, it has to move along a
perpendicular to the magnetic field. Here θ larger circle so that the time taken is the
= 90°, so F = qvB sin 90° = qvB = a same.
maximum force. As the magnetic force acts The frequency of revolution is
38
1 𝑞𝐵 Thus a charged particle moving in a
𝑓𝑐 = =
𝑇 2𝜋𝑚 uniform magnetic field has two concurrent
motions : a linear motion in the direction of
This frequency is called cyclotron
⃗ (along X-axis) and a circular motion in a
𝐵
frequency.
plane perpendicular to B (in YZ-plane).
3. When the initial velocity makes an Hence the resultant path of the charged
arbitrary angle with the field direction. A particle will be a helix, with its axis along
uniform magnetic field B is set up along ⃗.
the direction of 𝐵
+ve X-axis. A particle of charge q and mass
⃗ with velocity 𝑣 inclined
m enters the field 𝐵 The linear distance travelled by the
⃗,
at angle θ with the direction of the field 𝐵 charged particle in the direction of the
magnetic field during its period of
as shown in Fig. 4.63.
revolution is called pitch of the helical path.
2𝜋𝑚
pitch = 𝑣‖ × 𝑇 = 𝑣cos𝜃 ×
𝑞𝐵
2𝜋𝑚𝑣cos𝜃
=
𝑞𝐵
4.15 MOTION OF A CHARGE IN
PERPENDICULAR MAGNETIC AND
ELECTRIC FIELDS
16. Electric and magnetic fields are
Fig. 4.63 Helical motion of charged applied mutually perpendicular to each
particle in a magnetic field. other. Show that a charged particle will
follow a straight line path perpendicular to
The velocity 𝑣 can be resolved into two
both of these fields, if its velocity is 𝐸⃗ Bin
rectangular components :
magnitude.
1. The component along the
Velocity selector. Suppose a beam of
direction of the field i.e., along X-axis.
charged particles, say electrons,
Clearly
possessing a range of speeds passes
𝑣‖ = 𝑣cos𝜃 through a slit S1 and then enters a region in
which crossed (perpendicular) electric and
The parallel component remains magnetic fields exist. As shown in Fig.
unaffected by the magnetic field and so the
4.64, the electric field 𝐸⃗ acts in the
charged particle continues to move along
downward direction and deflects the
the field with a speed of v cos θ.
electrons in the upward direction. The
2. The component 𝑣⊥ perpendicular magnetic field 𝐵 ⃗ acts normally into the
to the direction of the field i.e., in the plane of paper and deflects the electrons in
YZ-plane. Clearly the downward direction.
𝑣⊥ = 𝑣sin𝜃
Due to this component of velocity, the
charged particle experiences a force F =
qv± B which acts perpendicular to both v±
and B. This force makes the particle move
along a circular path in the YZ-plane. The
radius of the circular path is
𝑚𝑣⊥ 𝑚𝑣sin𝜃
𝑟= =
𝑞𝐵 𝑞𝐵
The period of revolution is Fig. 4.64 Motion of an electron in a
2𝜋𝑟 2𝜋 𝑚𝑣sin𝜃 2𝜋𝑚 region of crossed magnetic and electric
𝑇= = ⋅ = fields.
𝑣⊥ 𝑣sin𝜃 𝑞𝐵 𝑞𝐵
39
Only those electrons will pass undeflected enters a region of uniform magnetic field of
through the slit S2 on which the electric and 10-5 T acting vertically downward as shown
magnetic forces are equal and opposite. in Fig. 4.65(a). Draw its trajectory and find
The velocity v of the undeflected electrons out the time it takes to come out of the
is given by region of magnetic field.
𝐸 [CBSE F 15]
𝑒𝐸 = 𝑒𝑣𝐵 or 𝑣=
𝐵
Such an arrangement can be used to
select charged particles of a particular
velocity out of a beam in which the particles
are moving with different speeds. This
arrangement is called velocity selector or
velocity filter. This method was used by J.J. Fig. 4.65 (a)
Thomson to determine the charge to mass
ratio (e / m) of an electron. Solution. The electron moves along
semicircular trajectory inside the magnetic
Examples based on field and comes out, as shown in Fig.
Motion of charges in electric and magnetic 4.65(b). Radius r of the path is given by
fields 𝑚𝑣 2
= 𝑞𝑣𝐵
Formulae Used 𝑟
1. Electric force on a charge, Fe = qE 𝑚𝑣 9.1 × 10−31 × 4 × 104
𝑟= = m
2. Magnetic force on a charge, Fm=qvB 𝑞𝐵 1.6 × 10−19 × 10−5
sin θ 9.1 × 4
= × 10−3 m = 22.75 × 10−3 m
3. In a perpendicular magnetic field, the 1.6
charge follows a circular path.
𝑚𝑣 2 𝑚𝑣
𝑞𝑣𝐵 = or 𝑟=
𝑟 𝑞𝐵
2𝜋𝑚 𝑞𝐵
𝑇= and 𝑓=
𝑞𝐵 2𝜋𝑚
4. When v makes angle θ with B, the Fig. 4.65 (b)
charge follows helical path. Time taken to come out of the region of
𝑚𝑣⊥ 𝑚𝑣sin𝜃 2𝜋𝑟 2𝜋𝑚 magnetic field,
𝑟= = ;𝑇 = =
𝑞𝐵 𝑞𝐵 𝑣⊥ 𝑞𝐵 𝜋𝑟 22 × 22.75 × 10−3
𝑡= = s
Pitch of helix, ℎ = 𝑣‖ 𝑇 = 𝑣cos𝜃. 𝑇 = 𝑣 7 × 4 × 104
2𝜋𝑚𝑣cos𝜃 = 17.875 × 10−7 s ≃ 1.8 × 10−6 s
𝑞𝐵
Example 43. An electron travels in a
5. K.E. gained by an electron when circular path of radius 20 cm in a magnetic
accelerated through a potential field 2 × 10-3 T. (i) Calculate the speed of
difference V, the electron, (ii) What is the potential
difference through which the electron must
1 2𝑒𝑉 be accelerated to acquire this speed ?
𝑚𝑣 2 = 𝑒𝑉 ∴ 𝑣=√
2 𝑚
Solution. Here r =20 cm =20 × 10-2m,
Units Used B = 2 × 10-3 T, o = 1.6 × 10-19 Q m =
E is in Vm-1 or NC-1, B in tesla, v in ms-1, r in 9.1× 10-31 kg
metre. (i) Magnetic force on the electron =
Example 42. An electron moving Centripetal force on electron
horizontally with a velocity of 4 ×104m/s
40
𝑚𝑣 2 A beam of proton passes undeflected
𝑒𝑣𝐵 = with a horizontal velocity v, through a
𝑟
𝑒𝐵𝑟 region of electric and magnetic fields,
∴ Speed, 𝑣 = mutually perpendicular to each other and
𝑚
normal to the direction of the beam. If the
1.6 × 10−19 × 2 × 10−3 × 20 × 10−2
= magnitudes of the electric and magnetic
9.1 × 10−31 fields are 100 kV/m and 50 mT
= 7.0 × 107 ms-1. respectively, calculate : (i) velocity v of the
beam, (ii) force with which it strikes a target
(ii) If V is the p.d. required to give
on a screen, if the proton beam current is
speed v to the electron, then
equal to 0.80 mA.
1
𝑒𝑉 = 𝑚𝑣 2 [CBSE OD 08]
2
⃗ ) = 𝑞(𝑣𝑗̂ × 𝐵𝑘̂ )
Solution: 𝐹 = 𝑞(𝑣 × 𝐵
𝑚𝑣 2 9.1 × 10−31 × (7.0 × 107 )2
𝑉= =
2𝑒 2 × 1.6 × 10−19 = 𝑞𝑣𝐵𝑗̂ × 𝑘̂ = 𝑞𝑣𝐵𝑖̂
= 13.9 × 103 V -14 kV. Thus the force F acts on the charge q
Example 44. An electron after being along the +ve x-direction.
accelerated through a potential difference (i) For undeflected proton beam,
of 104 V enters a uniform magnetic field of
0.04 T perpendicular to its direction of qvB - qE
motion. Calculate the radius of curvature of 𝐸 100kVm−1 100 × 103 Vm−1
its trajectory. 𝑟= = =
𝑖 50mT 50 × 10−3 T
[CBSE F 17] = 2 × 106 ms-1.
Solution. Here V = 104 V, B = 0.04 T, (ii) Current carried by proton beam,
e = 1.6 × 10-19 C m = 9.1 × 10-31 kg I =0.8 mA=8 × 10-4 A
An electron accelerated through a p.d. Number of protons striking the screen per
V acquires a velocity v given by second,
1 2𝑒𝑉 𝐼 8 × 10−4
𝑚𝑣 2 = 𝑒𝑉 or
𝑣=√ 𝑛= = = 5 × 1015 s−1
2 𝑚 𝑒 1.6 × 10−19
mp = 1.675 × 10-27kg
As the electron describes a circular
path of radius of r in the perpendicular Force with which a proton beam strikes
magnetic field B, therefore, a target on the screen,
𝑚𝑣 2 𝑑𝑝
= 𝑒𝑣𝐵 𝐹= = 𝑚𝑝 𝑛𝑣
𝑟 𝑑𝑡
= 1.675 × 10-27 × 5 × 1015 × 2 × 106 N
𝑚𝑣 𝑚 2𝑒𝑉 √2𝑚𝑒𝑉
or 𝑟 = = √ =
𝑒𝐵 𝑒𝐵 𝑚 𝑒𝐵 = 1.675 × 10-5 N.
√2 × 9.1 × 10−31 × 1.6 × 10−19 × 104 Example 46. An electron beam passes
= through a magnetic field of 2 × 10-3 Wb m1
1.6 × 10−19 × 0.04
and an electric field of 3.4 × 10 4 Vm-1 both
5.4 × 10−23 acting simultaneously. If the path of the
=
1.6 × 10−19 × 0.04 electron remains undeviated, calculate the
= 8.43 × 10−3 m = 8.43mm speed of the electrons. If the electric field is
removed, what will be the radius of the
Example 45. If a particle of charge q is circular path? Mass of an electron = 9.1 ×
moving with velocity v along the z-axis and 10-31 kg.
the magnetic field B is acting along the
⃗ ) to Solution. Here B = 2 × 10-3 Wb m-2,
x-axis, use the expression 𝐹 = 𝑞(𝑣 × 𝐵
find the direction of the force F acting on it. E = 3.4 × 104 Vm-1
41
Magnetic force on the electron = 𝑒𝐵 1.6 × 10−19 × 8.0 × 10−4
Electric force on the electron 𝑓= =
2𝜋𝑚 2𝜋 × 9.1 × 10−31
or evB = eE = 0.22 ×108 Hz = 22 MHz.
∴ Velocity of electrons, (c) In successive collisions, electron
loses its speed progressively. If after
𝐸 3.4 × 104
𝑣= = ms−1 = 1.7 × 107 ms−1 collision its velocity vector remains in the
𝐵 2 × 10−3 same plane of the initial circular orbit, the
When electric field has been removed, radius of the circular orbit will decrease in
Force exerted by the magnetic field on proportion to the decreasing speed.
an electron = Centripetal force on an Otherwise, the path of the electron will be
electron helical between two collisions.
𝑚𝑣 2 Example 48. A monoenergetic electron
i.e., 𝑒𝑣𝐵 = 𝑅 beam of initial energy 18 keV moving
𝑚𝑣 9.1×10−31×1.7×107 horizontally is subjected to a horizontal
or 𝑅 = = 1.6×10−19 ×2×10−3
𝑒𝐵 magnetic field of 0.4 G normal to its initial
= 4.8 × 10-2 m = 4.8 cm. direction. Calculate the vertical deflection
of the beam over a distance of 30 cm.
Example 47. In a chamber a uniform
magnetic field of 8.0 G (1 G = 10-4 T) is
maintained. An electron with a speed of 4.0 [CBSE Sample Paper 98]
× 10 6 ms-1 enters the chamber in a Solution. Under the action of the
direction normal to the field. magnetic field, the electrons will move
(i) Describe the path of the electron. along a circular path.
= 11.32 m
e = 1.6x10-19 C, m = 9.1x10-31 kg
9.1×10−31×4.0×106 Thus the electron moves in a circle of
∴𝑟 = radius 11.32 m, as shown in Fig. 4.66. As it
1.6×10−19 ×8.0×10−4
covers a distance PQ =30 cm, it goes down
= 2.8 × 10-2 m = 2.8 cm.
through a vertical distance
The sense of rotation of the electron in
its orbit can be ascertained from the
direction of the centripetal force 𝐹 =
−𝑒(𝑣 × 𝐵 ⃗ ) . Thus if we look along the
direction of 𝐵 ⃗ , the electron revolves
clockwise.
(b) The frequency of revolution of the
electron in its circular orbit is Fig. 4.66
42
equal to PA. If θ is the angle subtended by 𝑝 = 𝑣‖ × 𝑇 = 2 × 105 × 21.75 × 10−8
arc PQ at the centre O, then
= 43.5 ×10-3m = 4.35 cm.
PA - OP - OA -r-r cos θ = r (1 - cos θ)
Example 50. A proton projected in a
Arc 30×10−2
Now 𝜃= = = magnetic field of
Radius 11.32
0.02650rad 0.02 T travels along a helical path of radius
0.02650 × 180 5.0 cm and pitch 20 cm. Find the
= = 1.52° components of the velocity of the proton
𝜋 along and perpendicular to the magnetic
∴ cos θ = cos 1.52 ° = 0.99965 field. Take the mass of the proton =1.6 ×
10-27 kg.
Hence PA = 11.32 (1 -0.99965)
𝑚𝑣⊥
= 3.9744 × 10-3 m - 4 mm. Solution. Radius of helical path, 𝑟 = 𝑞𝐵
43
cm in a magnetic field. Find the
magnetic field and the number of
revolutions made by the electron per
second.
(Ans. 3.4 × 10-4 T, 9.4 × 106 rps)
5. An electron beam passes through a
magnetic field of 2×10_3Wbm-2 and an
electric field of 1.0 × 104 V m-1, both
acting simultaneously. If the path of the
electrons remains undeviated,
calculate the speed of the electrons. If Fig. 4.67
the electric field is removed, what will dicular) electric and magnetic fields. The
be the radius of the circular path? electric field has magnitude 100Vrn-1. We
(Ans. 5 × 106 ms-1, 1.43 cm) want the particles emerging from slit into
6. An electron moving perpendicular to a region II to have a fixed velocity of 1000
ms-1. What should be the value of the
uniform magnetic field completes a
circular orbit in 10-6 s. Calculate the uniform magnetic field in region I?
value of the magnetic field. Mass of (Ans. 0.1 T)
electron = 9 × 10-31 kg. (Ans. 3.5 ×
11. A proton, a deutron and an alpha
10-3 T)
particle, after being accelerated
7. Find the flux density of the magnetic through the same potential difference,
field to cause 62.5 eV electron to move enter a region of uniform magnetic field
in a circular path of radius 5 cm. Given ⃗ , in a direction perpendicular to 𝐵
𝐵 ⃗.
me = 9.1 × 10-31 kg and e - 1.6 × 10-19 C. Compare their kinetic energies. If the
(Ans. 5.335 ×10-4 T) radius of proton's circular path is 5 cm,
what will be the radii of the paths of
8. An electron of energy 2000 eV deutron and alpha particle?
describes a circular path in a magnetic
field of 0.2 T. What is the radius of (Ans. 1:1:2, rd = 7.07 cm, ra = 10 cm)
path? Take me = 9 × 10-31 kg, e = 1.6 × 12. A particle having a charge of 5.0 μC
10-19 C. and a mass of 5.0 × 10-12 kg is projected
(Ans. 0.75 mm) with a velocity of 1.0 km s-1 in a magnetic
field of magnitude 5.0 mT. The angle
9. What should be the minimum between the magnetic field and the velocity
magnitude and direction of the is sin-1 (0.90). Show that the path of the
magnetic field that must be produced at particle will be a helix. Find the diameter of
the equator of earth so that a proton the helix and its pitch. (Ans. 36 cm, 55 cm)
may go round the earth with a speed of
1.0 × 107ms-1? Earth's radius is 6.4 × HINTS
106 m. 𝑒
1. Use 𝑚 = 𝑟𝐵
𝑣
44
follow circular path.
𝑚𝑣 𝑚 2𝐾 √2𝑚𝐾
𝑚𝑣 2 𝑟= = ⋅√ =
∴ 𝑟 = 𝑒𝑣𝐵 𝑞𝐵 𝑞𝐵 𝑚 𝑞𝐵
𝑚𝑣 9.1×10−31 ×5×106 √2𝑚𝜇 𝐾𝑝
or 𝑟 = = 1.6×10−19 ×2×10−3 ∴ For proton, 𝑟𝑝 = = 5cm
𝑒𝐵 𝑒𝐵
= 1.43 × 10-2m =1.43 cm. √2𝑚𝑑 𝐾𝑑
2𝜋𝑚 2𝜋𝑚 For deutron, 𝑟𝑑 =
6. As 𝑇 = ∴ 𝐵= 𝑒𝐵
𝑒𝐵 𝑒𝑇
√2 × 2𝑚𝑝 × 𝐾𝑝
1 =
7. Here 2 𝑚𝑣 2 = 62.5eV = 62.5 × 1.6 × 𝑒𝐵
10−19 J = √2𝑇𝑝 = 1.414 × 5cm = 7.07cm
-17
= 10 J
For
√2𝑚𝛼 𝐾𝑙
𝛼 -particle, 𝑟𝛼 =
2 × 10−17 2𝑒𝐵
𝑣=√ √2 × 4 × 𝑚𝑝 × 2𝐾𝑝
𝑚 =
2𝑒𝐵
2 × 10−17 = 2rp =10 cm.
=√ = 4.69 × 106 ms−1
9.1 × 10−31 12. Here q = 5.0 μC = 5 × 10 -6 C, m = 5 ×
10-12 kg,
𝑚𝑣 9.1 × 10−31 × 4.69 × 106
𝐵= = v = 1.0 km s-1 = 103 ms-1, B = 5.0 mT = 5 ×
𝑒𝑟 1.6 × 10−19 × 5 × 10−2 10-3 T
= 5.335 × 10−4 T.
As θ = sin-1 (0.90), so sin θ = 0.90
1 2𝑒𝑉
8. 𝑚𝑣 2 = 𝑒𝑉 or 𝑣=√ cos𝜃 = √1 − sin2 𝜃 = √1 − 0.81
2 𝑚
= √0.19 ≃ 0.436
𝑚𝑣 𝑚 2𝑒𝑉 √2𝑚𝑒𝑉 𝑣⊥ = 𝑣sin𝜃 = 103 × 0.90 = 0.9 × 103 ms−1
𝑟= = √ =
𝑒𝐵 𝑒𝐵 𝑚 𝑒𝐵
𝑣‖ = 𝑣cos𝜃 = 103 × 0.436
√2 × 9 × 10−31 × 1.6 × 10−19 × 2000 = 4.36 × 102 ms−1
=
1.6 × 10−19 × 0.2 Velocity component moves the electron
= 7.5 × 10-4 m = 0.75 mm. along the field and v∑ along circular path.
Hence the motion is helical.
𝑚𝑣 1.67×10−27 ×107
9. 𝐵= = = 2𝑚𝑣⊥ 2×5×10−12×0.9×103
𝑞𝑟 1.6×10−19 ×6.4×106
−8 Diameter = 2𝑟 = =
1.63 × 10 T 𝑞𝐵 5×10−6×5×10−3
11. For a given p.d., the kinetic energy of a 17. What is a cyclotron? Discuss the
charged particle is proportional to its principle, construction, theory and working
charge. of a cyclotron. What is the maximum kinetic
energy acquired by the accelerated
∴ Kp:Kd:Ka=e:e:2e = 1:1:2 charged particles? Give the limitations and
Radius of the circular path of any particle of uses of a cyclotron.
kinetic energy K, Cyclotron. It is a device used to
45
accelerate charged particles like protons, 𝑚𝑣 2 𝑚𝑣
or 𝑞𝑣𝐵sin90° = or 𝑟=
deutrons, a-particles, etc., to very high 𝑟 𝑞𝐵
energies. It was invented by E.O. Period of revolution of the charged
Lawrence and M.S. particle is given by
- Livingston in 1934 at Berkeley, California 2𝜋𝑟 2𝜋 𝑚𝑣 2𝜋𝑚
University. 𝑇= = ⋅ =
𝑣 𝑣 𝑞𝐵 𝑞𝐵
Principle. A charged particle can be
accelerated to very high energies by
making it pass through a moderate
electric field a number of times. This
can be done with the help of a
perpendicular magnetic field which
throws the charged particle into
agircular motion, the frequency of
which does not depend on the speed of
the particle and the radius of the
circular orbit.
Construction. As shown in Fig. 4.68, a
cyclotron consists of the following main
parts :
1. It consists of two small, hollow, metallic
half-cylinders D1 and D2, called dees as
they are in the shape of D.
2. They are mounted inside a vacuum
chamber between the poles of a
powerful electromagnet.
3. The dees are connected to the source
of high frequency alternating voltage of
few hundred kilovolts.
4. The beam of charged particles to be
accelerated is injected into the dees
near their centre, in a plane
perpendicular to the magnetic field.
5. The charged particles are pulled out of
the dees by a deflecting plate (which is
negatively charged) through a window
W.
6. The whole device is in high vacuum
(pressure ~ 10-6 mm of Hg) so that the
air molecules may not collide with the Fig. 4.68 Cyclotron (σ) Front view (b)
charged particles. Section diagram.
Theory: Let a particle of charge q and mass Hence frequency of revolution of the
m enter a region of magnetic field 𝐵⃗ with a particle will be
velocity 𝑣 , normal to the field 𝐵 ⃗ . The 1 𝑞𝐵
particle follows a circular path, the 𝑓𝑐 = =
𝑇 2𝜋𝑚
necessary centripetal force being provided
by the magnetic field. Therefore, Clearly, this frequency is independent
of both the velocity of the particle and the
Magnetic force on charge q = radius of the orbit and is called cyclotron
Centripetal force on charge q frequency or magnetic resonance
46
frequency. This is the key fact which is mass. This will throw the particles out of
made use of in the operation of a cyclotron. resonance with the oscillating field. That is,
as the ions reach the gap between the
Working. Suppose a positive ion, say a
dees, the polarity of the dees is not
proton, enters the gap between the two
reversed at that instant. Consequently the
dees and finds dee D1 to be negative. It
ions are not accelerated further.
gets accelerated towards dee D 1 As it
enters the dee D2 it does not experience The above drawback is overcome
any electric field due to shielding effect of either by increasing magnetic field as in a
the metallic dee. The perpendicular synchrotron or by decreasing the
magnetic field throws it into a circular path. frequency of the alternating electric field as
At the instant the proton comes out of dee in a synchro-cyclotron.
D3 it finds dee D↑ positive and dee D2
2. Electrons cannot be accelerated in
negative. It now gets accelerated towards
a cyclotron. A large increase in their energy
dee D2. It moves faster through D?
increases their velocity to a very large
describing a larger semicircle than before.
extent. This throws the electrons out of
Thus if the frequency of the applied voltage
step with the oscillating field.
is kept exactly the same as the frequency
of revolution of the proton, then every time 3. Neutrons, being electrically neutral,
the proton reaches the gap between the cannot be accelerated in a cyclotron.
two dees, the electric field is reversed and Uses of cyclotron :
proton receives a push and finally it
acquires very high energy. This is called 1. The high energy particles produced in a
the cyclotron's resonance condition. The cyclotron are used to bombard nuclei
proton follows a spiral path. The and study the resulting nuclear
accelerated proton is ejected through a reactions and hence investigate
window by a deflecting voltage and hits the nuclear structure.
target. 2. The high energy particles are used to
Maximum K.E. of the accelerated ions. produce other high energy particles,
The ions will attain maximum velocity near such as neutrons, by collisions. These
the periphery of the dees. If V0 is the fast neutrons are used in atomic
maximum velocity acquired by the ions and reactors.
r0 is the radius of the dees, then 3. It is used to implant ions into solids and
𝑚𝑣02 𝑞𝐵𝑟0 modify their properties or even
= 𝑞𝑣0 𝐵 or 𝑣0 = synthesise new materials.
𝑟0 𝑚
The maximum kinetic energy of the 4. It is used to produce radioactive
ions will be isotopes which are used in hospitals for
diagnosis and treatment.
1 2
1 𝑞𝐵𝑟0 2
𝐾0 = 𝑚𝑣0 = 𝑚 ( ) For Your Knowledge
2 2 𝑚
➢ As the magnetic force on a charged
𝑞2 𝐵 2𝛾02
Or 𝐾0 = 2𝑚
particle acts perpendicular to the
velocity, it does not do any work on
Limitations of cyclotron : the particle. As a result, the kinetic
1. According to the Einstein's special energy or the speed of the particle
theory of relativity, the mass of a particle does not change due to the
increases with the increase in its velocity magnetic force.
as ➢ When a charged particle is
𝑚0 projected into a uniform magnetic
𝑚= field with its initial velocity
√1 − 𝑣 2 /𝑐 2 perpendicular to the field, the
where is the rest mass of the particle. At magnetic force acts on the charged
high velocities, the cyclotron frequency (f = particle perpendicular to both the
qB/2 πm) will decrease due to increase in magnetic field and its direction of
47
motion. This force produces 𝑞 2 𝐵 2𝑅 2
centripetal force to make the 2𝑚
particle move in a circle in a plane where R is the radius of the dees.
perpendicular to the magnetic field.
➢ When a charged particle moves Units Used
perpendicular to a uniform B is in tesla, v in ms-1, r in metre, T in
magnetic field : (i) its path is circular second and fc in Hz.
in a plane perpendicular to the
Example 51. Deutrons are accelerated in a
magnetic field and its direction of
cyclotron that has an oscillatory frequency
motion, (ii) the radius of the circular
of 107 Hz and a dee radius of 50 cm. (i)
path is proportional to its
What is the strength of the magnetic field
momentum, (iii) the kinetic energy
needed to accelerate the deutrons? (ii)
and speed of the particle do not
What is the energy of deutrons emerging
change, the force acting on the
from the cyclotron. Mass of a deutron=3.34
particle is independent of the radius
× 10-27 kg and charge of a deutron =1.6 ×
of the circular orbit but is
10-19 C.
proportional to its speed i.e., F cc r°
and F cc v and (v) the period of Solution, v = 107 Hz, R = 50 cm = 0.50
revolution of the charged particle is m,
independent of its speed and the
m=3.34× 10-27 kg, q=1.6×10-19C
radius of its circular orbit.
𝑞𝐵
➢ When a charged particle is (i) Cyclotron frequency, 𝑓𝑐 = 2𝜋𝑚
projected into a uniform magnetic
field at an arbitrary angle with the 2𝜋𝑚𝑓𝑐 2×3.14×3.34×10−27 ×107
∴ 𝐵= =
field, the component of the initial 𝑞 1.6×10−19
48
2𝜋𝑚𝑝 𝑓𝑐 revolutions before emerging from the dees.
𝐵= The gain in its kinetic energy will be
𝑒
2 × 3.14 × 1.67 × 10−27 × 107
= 1 𝑚𝑣 2
1.6 × 10−19 𝑚𝑣 2 = 2 neV or 𝑛=
2 4𝑒𝑉
= 0.66 T. 𝑐 3×108
Given 𝑣 = 5 = = 0.6 × 108 ms−1
Kinetic energy of the emerging beam 5
will be m = 1.67× 10-27 kg
𝑒 2 𝐵2 𝑅2 1.67 × 10−27 × (0.6 × 108 )2
𝐾max = ∴ 𝑛=
2𝑚𝑝 4 × 1.6 × 10−19 × 20 × 103
(1.6 × 10−19 )2 × (0.66)2 × (0.6)2 = 470 revolutions.
=
2 × 1.67 × 10−27 Problems for Practice
1.2× 10−12 1. An electron of energy 10,000 eV
= 1.2 × 10−12 J = MeV
1.602 × 10−13 describes a circular path in a plane at
= 7.4 MeV. right angles to a uniform magnetic field
of 0.01 T. (a) What is the radius of the
Example 53. In a cyclotron, a magnetic circular orbit? (b) What is the cyclotron
induction of 1.4 T is used to accelerate frequency? (c) What is the period of its
protons. How rapidly should the electric revolution? (d) What is the direction of
field between the dees be reversed? The revolution as viewed by an observer
mass and charge of proton are 1.67 × looking in the direction of the field?
10-27kg and 1.6 × 10-19 C respectively.
(Ans. 3.4×10-2m, 2.8×108s-1, 3.6× 10-9s,
Solution. Here B = 1.4 T, m = 1.67 × clockwise sense).
10-27 kg,
2. The protons are accelerated by a
e = 1.6x 10-19 C cyclotron, when a magnetic field of 2.0
The time required by a charged particle T is applied perpendicular to the plane
to complete a semicircle in a dee is of the dees. Calculate the energy of the
proton in MeV, if the circular path of the
𝜋𝑚 3.14 × 1.67 × 10−27 protons has a radius of 40 cm before
𝑡= =
𝑒𝐵 1.6 × 10−19 × 1.4 the protons leave the cyclotron. Given
= 2.34 × 10−8 s mass of a proton = 1.67 × 10 -27kg.
Thus the direction of electric field (Ans. 30.6 MeV)
should reverse after every 2.34 × 10 -8 s.
3. A cyclotron has an oscillatory
The frequency of the applied electric frequency of 12 MHz and a dee radius
field should be of 50 cm. Calculate the magnetic field
1 1 required to accelerate deutrons of
𝑓𝑐 = = = 2.14 × 107 Hz mass 3.3 × 10-2 kg and charge 1.6 × 10
2𝑡 2 × 2.34 × 10−8 -19
C. What is the energy of the
Example 54. If the maximum value of deutrons emerging from the cyclotron?
accelerating potential provided by a radio
frequency oscillator be 20 kV, find the (Ans. 1.56 T, 14.7 MeV)
number of revolutions made by a proton in 4. Alpha particles of mass 6.68 × 10-27 kg
a cyclotron to achieve one fifth of the speed and charge 3.2 × 10 -19 C are
of light. Mass of a proton = 1.67 × 10-27 kg. accelerated in a cyclotron in which a
Solution. In a cyclotron, a proton gains magnetic field of 1.25 T is applied
energy eV, when it crosses a region of perpendicular to the dees. How rapidly
potential difference V. In one revolution, should the electric field between the
the particle crosses the gap twice. So the dees be reversed? What are the
energy gained in each revolution = 2 eV. velocity and kinetic energy of an alpha
particle when it moves in a circular orbit
Suppose the particle makes n of radius 25 cm?
49
(Ans. 9.5 × 106 Hz, 1.5 × 107 ms-1, 7.5 × What is the cause of this force?
10-13 J)
Force on a current carrying conductor
HINTS in a magnetic field. When a conductor
𝑞 2 𝐵 2𝑅 2
carrying a current is placed in an external
4. 𝐾max = = magnetic field, it experiences a mechanical
2𝑚
(1.6×10−19)2 ×(2.0)2×(0.40)2 force. To demonstrate this force, take a
J
2×1.67×10−27 small aluminium rod AB. Suspend it
(1.6 × 10−19 )2 × (2.0)2 × (0.40)2 horizontally by means of connecting wires
= MeV from a stand, as shown in Fig. 4.69.
2 × 1.67 × 10−27 × 1.6 × 10−13
= 30.6 MeV.
𝑞𝐵
3. As 𝑓𝑐 = 2𝜋𝑚
2𝜋𝑚𝑓𝑐 2×3.142×3.3×10−27 ×12×106
∴𝐵 = =
𝑞 1.6×10−19
= 1.56 T.
𝑞 2 𝐵2 𝑅2
𝐾max = =
2𝑚
(1.6 × 10−19 )2 × (1.56)2 × (0.50)2
J
2 × 3.3 × 10−27
(1.6 × 10−19 )2 × (1.56)2 × (0.50)2 Fig. 4.69 Force on a current in a
= MeV magnetic field.
2 × 3.3 × 10−27 × 1.6 × 10−13
= 14.7 MeV. Place a strong horse-shoe magnet in such
a way that the rod is between the two poles
𝜋𝑚 3.14×6.68×10−27
4. 𝑡 = = = 5.25 × 10−8 s with the field directed upwards. Now, if a
𝑞𝐵 3.2×10−19 ×1.25
current is passed through the rod from A to
qB 3.2 × 10-19 × 1.25 B, the rod gets deflected to the right. If we
reverse the direction of current or
Direction of electric field should be
interchange the poles of the magnet, the
reversed after every 5.25 × 10 -8s.
deflection of the rod is also reversed. The
Applied frequency, direction of force is perpendicular to both
1 1 the current and the magnetic field and is
𝑓𝑐 = = = 9.5 × 106 Hz given by Fleming's left hand rule.
2𝑡 2 × 5.25 × 10−8
𝑚𝑣 Cause of the force on a current carrying
As 𝑟= 𝑞𝐵 conductor in a magnetic field. A current is
𝑟𝑞𝐵 0.25 × 3.2 × 10−19 × 1.25 an assembly of moving charges and a
𝑣= = magnetic field exerts a force on a moving
𝑚 6.68 × 10−27 charge. That is why a current carrying
= 1.5 × 107 ms-1 conductor when placed in a magnetic field
1 experiences a sideways force as the force
𝐾= 𝑚𝑣 2 experienced by the moving charges (free
2
electrons) is transmitted to the conductor
1
= 2 × 6.68 × 10“27 × (1.5 × 107)2 2 as a whole.
50
direction of this force? Special Cases (i) If θ=0°or 180°, then
F=IIB (0)=0
Expression for the force on a current
carrying conductor in a magnetic field. As Thus a current carrying conductor
shown in Fig. 4.70, consider a conductor placed parallel to the direction of the
PQ of length l, area of cross- section A, magnetic field does not experience any
carrying current I along +ve Y-direction. force, (ii) If θ =90°, then
⃗ acts along +ve Z-direction. The
The field 𝐵 F = FBsin90°= IIB or Fmax = IIB
electrons drift towards left with velocity vd.
Each electron experiences a magnetic Thus a current carrying conductor
Lorentz force along +ve X-axis, which is placed perpendicular to the direction of a
given by magnetic field experiences a maximum
force.
𝑓 = −𝑒(𝑣 ⃗)
⃗⃗⃗⃗𝑑 × 𝐵
Direction of force. The direction of force
If n is the number of free electrons per unit on a current carrying conductor placed in a
perpendicular magnetic field is given by
Fleming's left hand rule.
Stretch the thumb and the first two fingers
of the left hand in mutually perpendicular
directions. If the forefinger points in the
direction of the magnetic field, central
finger in the direction of current, then the
thumb gives the direction of force on the
conductor. In Fig. 4.70, the field B is along
+ Z-direction, the current I along +
Fig. 4.70 Force on a current in a Y-direction and so the force 𝐹 acts along +
magnetic field. X-direction.
volume, then total number of electrons in Examples based on
the conductor is
Force on a current carrying conductor in a
N - n× volume = nAl magnetic field
Total force on the conductor is Formulae Used
𝐹 = 𝑁𝑓 = 𝑛𝐴𝑙[−𝑒(𝑣 ⃗ )]
⃗⃗⃗⃗𝑑 × 𝐵 ⃗)
1. 𝐹 = 𝐼(𝑙 × 𝐵 2. F = II B sin
⃗] θ
= 𝑒𝑛𝐴[−𝑙𝑣𝑑 × 𝐵
3. 𝐹max = 𝐼𝑙𝐵
If 𝐼𝑙 represents a current element
vector in the direction of current, then Units Used
vectors 𝑙 and 𝑣𝑑 will have opposite Force F is in newton, current I in ampere,
directions and we can take length I in metre and field B in tesla.
−𝑙𝑣
⃗⃗⃗⃗𝑑 = 𝑣𝑑 𝑙 Example 55. A wire of length l carries a
current I along the X-axis. A magnetic field
⃗)
∴ 𝐹 = 𝑒𝑛𝐴𝑣𝑑 (𝐼 × 𝐵 ⃗ = 𝐵0 (𝑖̂ + 𝑗̂ + 𝑘̂ ) tesla exists in space.
𝐵
But enAvd = current, I Find the magnitude of the magnetic force
on the wire.
⃗)
Hence 𝐹 = 𝐼(𝑙 × 𝐵
Solution. As the wire carries current I
Magnitude of force. The magnitude of along the X-axis, so l =1 i
the force on the current carrying conductor
⃗) ⃗ = 𝐵0 (𝑖̂ + 𝑗̂ + 𝑘̂ ) tesla
Also, 𝐵
is given by 𝐹 = 𝐼(𝑙 × 𝐵
where θ is the angle between the direction Magnetic force on the wire is
of the magnetic field and the direction of ⃗ ) = 𝐼[𝑙𝑖̂ × 𝐵0 (𝑖̂ + 𝑗̂ + 𝑘̂ )]
𝐹 = 𝐼(𝑙 × 𝐵
flow of current.
51
= 𝐵0 𝐼𝑙[𝑖̂ × (𝑖̂ + 𝑗̂ + 𝑘̂ )] forces on the four sides of the frame.
= 𝐵0 𝐼𝑙[𝑖̂ × 𝑖̂ + 𝑖̂ × 𝑗̂ + 𝑖̂ × 𝑘̂ ]
= 𝐵0 𝐼𝑙[𝑖̂ × 𝑖̂ + 𝑖̂ × 𝑗̂ + 𝑖̂ × 𝑘̂ ]
⃗ + 𝑘̂ − 𝑗̂) = (𝑘̂ − 𝑗̂)𝐵0 𝐼𝑙
= 𝐵0 𝐼𝑙(0
Magnitude of the magnetic force is
𝐹 = √12 + (−1)2 𝐵0 𝐼𝑙 = √2𝐵0 𝐼𝑙 newton.
Example 56. The horizontal component of Fig. 4.71
the earth's magnetic field at a certain place Solution. By symmetry, the current
is 3.0 × 10-5 T and the direction of the field through each of the four sides will be 1 A.
is from the geographic south to the Also,
geographic north. A very long straight
conductor is carrying a steady current of l l =20 cm =0.20 m, B = 0.25T
A. What is the force per unit length on it Magnitude of force on each side is F =
when it is placed on a horizontal table and IIBsin90°
the direction of the current is (a) east to
west, (b) south to north? = 1 × 0.20 × 0.25×1 =0.05 N
[NCERT] By Fleming's left hand rule, forces on
ab and dc will be towards left and on ad
Solution. The force on a conductor of and be downward.
length / placed in a magnetic field B, and
carrying current I, is Example 58. A magnetic field of 1.0 T is
produced by an electromagnet in a
F = IIB sin θ cylindrical region of radius 4.0 cm, as
The force per unit length will be shown in Fig. 4.72. A wire, carrying current
of 2.0 A, is placed peiγen- dicular to and
𝐹
𝑓= = 𝐼𝐵sin𝜃 intersecting the axis of the cylindrical
𝑙 region. Find the magnetic force acting on
where θ is the angle that the conductor the wire.
makes with the direction of B . Solution. Clearly, the magnetic field
(a) When the current flows east to west, θ acts vertically downwards while the current
=90°. flows horizontally, so
∴ 𝑓 = 𝐼𝐵sin90° = 1 × 3.0 × 10−5 × 1 θ =90°.
= 3.0 × 10-5 Nm-1
According to Fleming's left hand rule,
this force acts vertically downwards.
(b) When the current flows from south
to north, θ =0°
𝑓 = 𝛱sin0° = 0
Thus the force per unit length of the Fig. 4.72
conductor is zero. Length of the wire in the cylindrical
Example 57. A current of 2 A enters at the region
corner 'a' of a square frame of side 20 cm = 2 r = 2 × 4.0 cm = 0.08 m
and leaves at opposite corner 'c'. A
magnetic field of B- 0.25 T acts in a ∴ F= IIBsin90° =2.0x0.08 × 1.0 × 1 =
direction perpendicular to the plane of 0.16 N
paper, as shown in Fig. 4.71. Find the
This force acts on the wire normally into
magnitude and direction of the magnetic
the plane of paper.
52
Example 59. A straight wire of mass 200 g ∴ F =12 × 0.04 × 0.25 sin 60° = 0.10 N.
and length 1.5 m carries a current of 2 A. It
is suspended in mid-air by a uniform Example 61. On a smooth plane inclined at
horizontal magnetic field B. What is the 30° with the horizontal, a thin
magnitude of the magnetic field? current-carrying metallic rod is placed
[NCERT; CBSE F 15] parallel to the horizontal ground. The plane
is located in a uniform magnetic field of
Solution. Suppose that a wire AB 0.15 T in the vertical direction. For what
carries a current of 2 A in the direction as value of current can the rod remain
shown in Fig. 4.73. The weight mg of the stationary? The mass per unit length of the
wire acts vertically downwards. Therefore, rod is 0.03 kg m-1. [NCERT] Solution.
according to Fleming's left hand rule, the Suppose a rod PQ is placed horizontally on
magnetic field B must act perpendicularly an inclined plane as shown in Fig. 4.74.
into the plane of paper so that the magnetic Various forces acting on the current
force F on the wire acts vertically upwards. carrying rod PQ are
(i) its weight Mg acting vertically
downwards; and
(ii) horizontal force BIl due to the
magnetic field B .
In order that the rod remains stationary,
the component of the weight of the rod
along the inclined plane must be balanced
by the component of the force BIl along the
Fig. 4.73 inclined plane, i.e.,
For mid-air suspension, Mg sin θ = BIl cos θ
Magnetic force on the wire = Weight of the
wire IIB sin 90° = mg
𝑚𝑔
Or 𝐵 =
𝑙𝐼
53
the total force experienced by the long 𝐹 = 𝐼 ′ 𝑙𝐵 = 6 × (2 × 10−2 ) × 6𝜋
conductor when they are 3.0 cm apart? × 10−4 𝐼 newton
[NCERT]
The current I would support the wire if
Solution. As the two conductors have the above force equals the weight of the
different lengths, the longer conductor may wire,
be considered to be of infinite length.
i.e., 6 × 2 × 10−2 × 6𝜋 × 10−4 𝐼 =
Therefore, magnetic field produced by it at
a distance of 3 cm (0.03 m) is given by 2.5 × 10−3 × 9.8
2.5×10−3×9.8
𝜇0 𝐼2 4𝜋 × 10−7 × 3 or 𝐼 = 72×3.14×10−6 A = 108.3A
𝐵= = T
2𝜋𝑟 2𝜋 × 0.03
−5
= 2 × 10 T Example 64. Figure 4.75 shows a
triangular loop PQR carrying current I. The
Force on the short conductor due to triangle is equilateral with side equal to l. If
this magnetic field will be a uniform magnetic field B exists parallel to
𝐹 = 𝐼1 𝑙1 𝐵 = 4 × 5 × 10−2 × 2 × 10−5 N = 4 PQ, then find the forces acting on the three
× 10-6 N wires separately.
54
(Ans. 0.05 N, 0) Fig. 4.76
2. A current of 5.0 A is flowing upward in a 7. A horizontal wire 0.1 m long carries a
long vertical wire placed in a uniform current of 5 A. Find the magnitude and
horizontal northward magnetic field of direction of the magnetic field which
0.02 T. How much force and in what can support the weight of wire
direction will the field exert on 0.06 m assuming that its mass is 3 ×10 -3 kg
length of the wire? m~h
(Ans. 6 × 10-3 N, towards west) (Ans. 5.88 × 10-3 T)
3. What is the magnitude of force on a 8. A conductor of length 10 cm is placed
wire of length 0.04 m placed inside a perpendicular to a uniform magnetic
solenoid near its centre, making an field of strength 100 oersted. If a
angle of 30° with its axis? The wire charge of 5 C passes through it in 5 s,
carries a current of 12 A and the find the force experienced by the
magnetic field due to the solenoid is of conductor.
magnitude 0.25 T.
(Ans. 10-3 N)
[CBSE OD 90 C] (Ans. 0.06 N)
9. A conductor of length 20 cm is placed
4. A long straight conductor P carrying a (i) parallel (ii) perpendicular (iii) inclined
current of 2 A is placed parallel to a at an angle 30° to a uniform magnetic
short conductor Q of length 0.05 m field of 2 T. If a charge of 10 C passes
carrying a current of 3 A. The two through it in 5 s, find the force
conductors are 0.10 m apart. experienced by the conductor. [Ans. (i)
zero (ii) 0.8 N (iii) 0.4 N]
Calculate
10. A current of 5.0 A exists in the loop
(a) the magnetic field due to P at Q
shown in Fig. 4.77. The wire AB has a
(b) the approxim ate force on Q. length of 50 cm and lies in a magnetic
field of 0.20 T. What is the magnetic
(Ans. 4 × 10-6 T, 6 × 10-7 N)
force acting on the wire?
5. A straight wire 1 m long carries a
current of 10 A at right angles to a (Ans. 0.50 N, towards the inside of the
uniform magnetic field of 1 Wb m -2. loop)
Find the mechanical force on the wire
and the power required to move it at 15
ms-1 in a plane at right angles to the
field. (Ans. 10 N, 150 W)
6. A wire AB making an angle of 30° with
a horizontal is supported by a magnetic
field of 0.65 T, directed normally into Fig. 4.77
the plane of paper. If the wire carries a 11. A horizontal wire 0.1 m long having
current of 5 A, determine its mass per mass 3 g carries a current of 5 A. Find
unit length. the magnitude of the magnetic field
(Ans. 0.2872 kg m-1) which must act at 30° to the length of
the wire in order to support its weight ?
(Ans. 0.1176 T)
12. Find the magnitude of the magnetic
force on the segment PQ placed in a
magnetic field of 0.25 T, if a current of 5
A flows through it, as shown in Fig.
4.78. (Ans. 0.32 N)
55
𝑚𝑔 3×10−3×9.8
∴ 𝐵 = 𝐼𝑙sin𝜃 = 0.1×5×sin30° = 0.1176T
= 0.2872 kg m-1.
7. In equilibrium,
Magnetic force on wire = Weight of wire
or IIB sin 90° = mg
𝑚 𝑔 3×10−3×9.8
or 𝐵 = 𝑙
⋅𝐼= 5
= 5.88 × 10−3 T
𝑞
8. F = IIB sin 𝜃 = 𝑡 . IB sin 90° Fig. 4.79 Attractive force between parallel
currents.
5 × 0.10 × 100 × 10−1 × 1
= = 10−3 N Experiment 2. As shown in Fig. 4.80,
5
two wires are connected to a battery
9. Proceed as in Problem 8 above. through a mercury bath in such a way that
10. F = IIB sin θ = 5.0 × 0.50 × 0.20 × current flows in them in succession. When
sin 90° = 0.50 N. the circuit is closed, the currents in the two
wires flow in opposite directions. The two
11. F = IIB sin θ = mg
wires move away from each other,
indicating a force of repulsion between
56
them. exerts a force on current carrying wire CD.
The force acting on length l of the wire CD
will be
𝜇0 𝐼1 𝜇0 𝐼1 𝐼2
𝐹2 = 𝐼2 𝑙𝐵1 sin90° = 𝐼2 𝑙 ⋅ = .𝑙
2𝜋𝑟 2𝜋𝑟
Force per unit length,
𝐹2 𝜇0 𝐼1 𝐼2
𝑓= =
𝑙 2𝜋𝑟
According to Fleming's left hand rule,
this force acts at right angles to CD,
towards AB in the plane of the paper.
Fig. 4.80 Repulsive force between Similarly, an equal force is exerted on the
antiparallel currents. wire AB by the field of wire CD. Thus when
21. Derive an expression for the force the currents in the two wires are in the
per unit length between two infinitely long same direction, the forces between them
straight parallel current carrying wires. are attractive. It can be easily seen that
Hence define one ampere. Also define 𝑟1 = 𝑟2
coulomb in terms of ampere.
As shown in Fig. 4.81(b), when the
Expression for the force between two currents in the two parallel wires flow in
parallel current-carrying wires. As shown in opposite directions (antiparallel), the forces
Fig. 4.81(a), consider two long parallel between the two wires are repulsive. Thus,
wires AB and CD carrying currents f and f.
Let r be the separation between them. Parallel currents attract and antiparallel
currents repel.
The magnetic field produced by current
f at any point on wire CD is Definition of ampere.
𝜇0 𝐼1 When f = I2 = 1 A and r = 1 m, we get
𝐵1 =
2𝜋𝑟 𝜇0
𝑓= = 2 × 10−7 Nm−1
2𝜋
One ampere is that value of steady
current, which on flowing in each of the two
parallel infinitely long conductors of
negligible cross-section placed in vacuum
at a distance of lm from each other,
produces between them a force of 2 × 10~7
newton per metre of their length.
Definition of coulomb in terms of
ampere. If a steady current of 1 ampere is
set up in a conductor, then the quantity of
charge that flows through its cross-section
in 1 second is called one coulomb.
Examples based on
Forces between parallel current carrying
wires
Formulae Used
Fig. 4.81 (a) Parallel currents attract, 𝜇0𝐼1 𝐼2
1. Force per unit length, 𝑓 =
(b) Antiparallel currents repel. 2𝜋𝑟
This field acts perpendicular to the wire 2. Force on length / of one of the
𝜇 𝐼1𝐼2 𝑙
CD and points into the plane of paper. It wires, 𝐹 = 02𝜋𝑟
57
Units Used opposite directions, only then Q will
experience a repulsive force which would
Force is in newton, currents in ampere,
balance the weight of Q.
distance r in metre and field B in tesla.
𝜇0𝐼1 𝐼2
Constant Used ∴𝐹 = =𝑊
2𝜋𝑟
𝜇0 = 4𝜋 × 10−7 TmA−1 𝜇
Or 𝑟 = 2𝜋0 ⋅
𝐼1𝐼2
=
2×10−7 ×50×25
𝑊 0.075
Example 65. A current of 5.0 A flows
= 3.33 × 10-3 m = 3.33 mm.
through each of two parallel long wires.
The wires are 2.5 cm apart. Calculate the Example 67. A current balance (or ampere
force acting per unit length of each wire. balance) is a device for measuring
Use the standard value of constant currents. The current to be measured is
required. What will be the nature of the arranged to go through two long parallel
force, if both currents flow in the same wires of equal length in opposite directions
direction? one of which is linked to the pivot of the
balance. The resulting repulsive force on
[Punjab 99]
the wire is balanced by putting a suitable
Solution. Here I1 = I2 = 5 A, mass in the scale pan hanging on the other
side of the pivot. In one measurement, the
r =2.5 cm =2.5 ×10-2m, μ0=4 𝜋× 10-7 TmA-1
mass in the scale pan is 30.0 g, the length
Force acting per unit length of each wire, of the wires is 50.0 cm each, and the
separation between them is 10.0 mm.
𝜇0 𝐼1 𝐼2 4𝜋 × 10−7 × 5 × 5
𝑓= = What is the value of the current being
2𝜋𝑟 2𝜋 × 2.5 × 10−2 measured? Take g =9.80 ms-2 and assume
= 2 × 10−4 Nm−1 that the arms of the balance are equal.
[
As the currents in both the wires flow in NCERT]
the same direction, the force will be
attractive. Solution, w = 30.0 g = 0.03 kg, l = 50 cm
= 0.50 m, r = 10.0 mm = 0.01 m, g = 9.8
Example 66. A long horizontal wire P ms-2
carries a current of 50 A. It is rigidly fixed.
Another fine wire Q is placed directly above Force per unit length between two parallel
and parallel to P. The weight of the wire Q conductors,
is 0.075 Nm-1 and it carries a current of 25 𝜇0 𝐼1 𝐼2
A. Find the position of the wire Q from the 𝑓= ⋅
2𝜋 𝑟
wire P so that Q remains suspended due to
the magnetic repulsion. Also indicate the ∴ Force on a conductor of length l,
direction of current in Q with respect to P. 𝜇0 𝐼1 𝐼2 𝑙
[Roorkee 96] 𝐹= ⋅
2𝜋 𝑟
Solution. The magnetic force per unit When the pan is balanced,
length on the wire Q due to the current in
wire P is Weight in scale pan = Balancing force
𝜇 𝐼×𝐼
𝜇0 𝐼1 𝐼2 i.e., 𝑚𝑔 = 2𝜋0 ⋅ .𝑙
𝐹= ⋅ 𝑟
2𝜋 𝑟 2𝜋𝑚𝑔𝑟 2𝜋×0.03×9.8×0.01
or 𝐼 2 = =
𝜇0 𝑙 4𝜋×10−7 ×0.50
= 29400
I = √29400 = 171.46 A.
Example 68. A rectangular loop of sides 25
cm and 10 cm carrying a current of 15 A is
Fig. 4.82 placed with its longer side parallel to a long
straight conductor 2.0 cm apart carrying a
The currents in P and Q must have current of 25 A. What is the net force on the
58
loop? resistances. The separation between the
neighbouring wires is 1.0 cm. The wires AE
[CBSE OD 05]
and BF have negligible resistance and the
Solution. Consider the rectangular loop ammeter reads 15 A. Calculate the
ABCD placed near a long straight magnetic force per unit length of AB and
conductor XY, as shown in Fig. 4.83. The CD.
arm AB will get attracted, while CD will get
Solution. By symmetry, current through
repelled. Forces on arms BC and AD,
each of the wires AB, CD and EF is 5 A.
being equal, opposite and collinear, will
cancel each other.
Fig. 4.84
Force per unit length of AB due to current
in CD is
Fig. 4.83
𝜇0 𝐼1 𝐼2 4𝜋 × 10−7 × 5 × 5
Current through the rectangular loop, 𝐼1 = 𝑓1 = =
2𝜋𝑟 2𝜋 × 1.0 × 10−2
15 A
= 5.0 × 10-4 Nm-1, directed downward Force
Current through the long wire XY, 𝐼2 =25 A per unit length of AB due to current in EF is
Force on AB, 4𝜋 × 10−7 × 5 × 5
𝑓2 =
𝜇
𝐹1 = 4𝜋0 ⋅
2𝐼1𝐼2
× length of conductor AB 2𝜋 × 2.0 × 10−2
𝑟1
= 2.5 × 10-4 Nm-1, directed downward
10−7 × 2 × 15 × 25 Total force per unit length of AB is
= × 25 × 10−2
2.0 × 10−2 f=𝑓1 + 𝑓2
-4
= 9.375 × 10 N
= 7.5 × 10-4 Nm-1, directed downward Force
(Att
per unit length of CD due to current in AB is
ractive)
4𝜋 × 10−7 × 5 × 5
Force on CD, 𝑓1 =
𝜇 2𝐼1𝐼2
2𝜋 × 1.0 × 10−2
𝐹2 = 4𝜋0 ⋅ × length of conductor CD = 5.0 × 10-4 Nm-1, directed upward
𝑟2
Force per unit length of CD due to current
10−7 × 2 × 15 × 25
= × 25 × 10−2 in EF is
12.0 × 10−2
4𝜋 × 10−7 × 5 × 5
= 1.5625 × 10-4 𝑓2 =
(Re 2𝜋 × 1.0 × 10−2
pulsive) = 5.0 × 10-4 Nm-1 directed downward ∴
∴ Net force on the loop, Total force per unit length of CD = zero.
Problems For Practice
F=Fl-F2 =9.375 × 10-4 -1.5625 × 10-4
1. A long horizontal rigidly supported wire
= 7.8125×10-4 N- 7.8 × 10-4 N (Attractive)
carries a current of 100 A. Directly
Thus the force on the loop will act above it and parallel to it is a fine wire
towards the long conductor (attractive) if that carries a current of 200 A and
the current in its closer side is in the same weighs 0.05 Nm-1. How far above the
direction as the current in the long wire should the second wire be kept to
conductor, otherwise it will be repulsive. support it by magnetic repulsion? (Ans.
Example 69. In Fig. 4.84, the wires AB, CD 8 cm)
and EF are long and have identical 2. A wire AB is carrying a steady current
59
of 12 A and is lying on the table.
Another wire CD carrying 5 A is held
directly above AB at a height of 1 mm.
Find the mass per unit length of the
wire CD so that it remains suspended
at its position when left free. Give the
direction of the current flowing in CD
with respect to that in AB.
[Take the value of g = 10 ms-2].
[CBSE OD 13]
[Ans. (i) x = 0 (ii) F = 2.67 μN, towards the
-3 -1
(Ans. 1.2 × 10 kg m , in the opposite straight wire]
direction)
6. A square loop of side 20 cm carrying
3. Two very long, straight, parallel wires A current of 1 A is kept near an infinite
and B carry currents of 10 A and 20 A long straight wire carrying a current of 2
respectively, and are at a distance 20 A in the same plane as shown in Fig.
cm apart, as shown in Fig. 4.85. If a 4.88. Calculate the magnitude and
third wire C (length 15 cm) having a direction of the net force exerted on the
current of 10 A is placed between them, loop due to the current carrying
how much force will act on C? The conductor.
direction of current in all the three wires
is same. (Ans. 3.0 × 10-5 N, towards B) [CBSE OD 15C]
(Ans. 5.33 ×10-7N)
60
force between the two wires is repulsive.
3. Force on C due to A,
4𝜋 × 10−7 × 10 × 10 × 0.15
𝐹1 =
2𝜋 × 0.10
= 3.0 × 10-5 N, towards A
Force on C due to B,
4𝜋 × 10−7 × 20 × 10 × 0.15
𝐹2 =
2𝜋 × 0.10
= 6.0 × 10-3 N, towards B
Net force on C
= F2 – F1 = 3.0 × 10-5 N, towards B.
𝜇0 𝐼1 𝐼2 4𝜋 × 10−7 × 5 × 5
4. 𝑓 = =
2𝜋𝑟 2𝜋 × 0.01
= 5 ×10-4 Nm-1, attractive
5. (i) As direction of the magnetic field due
to the straight conductor is parallel to
the area vector (both normal to the
plane of the loop), so torque x = 0.
(ii) Proceed as in Example 68 on page
4.51 Fig. 4.89 (a) A rectangular loop PQRS in a
𝜇0𝐼1 𝐼2 uniform magnetic field B. (b) Top view of
6. 𝐹 = 𝑙
2𝜋𝑟 the loop, magnetic dipole moment m is
Net force on sides ab and cd shown.
𝜇0 𝐼1 𝐼2 1 1 Let I = current flowing through the coil
= 𝑙[ − ] PQRS
2𝜋 𝑟1 𝑟2
4𝜋 × 10−7 × 2 × 1 × 20 × 10−2 1 a,b = sides of the coil PQRS
= [
2𝜋 10 × 10−2 A = ab = area of the coil
1
− ]N θ = angle between the direction of B and
30 × 10−2
normal to the plane of the coil.
20
= 4 × 10−7 × 20 [ ]N According to Fleming's left hand rule, the
10 × 30
= 5.33 × 10−7 N magnetic forces on sides PS and QR are
equal, opposite and collinear (along the
This force is directed towards the axis of the loop), so their resultant is zero.
infinitely long straight wire.
The side PQ experiences a normal
4.19 TORQUE EXPERIENCED BY A inward force equal to IbB while the side RS
CURRENT LOOP IN A UNIFORM experiences an equal normal outward
MAGNETIC FIELD force. These two forces form a couple
22. Derive an expression for the torque which exerts a torque given by
acting on a current carrying loop x = Force × perpendicular distance
suspended in a uniform magnetic field.
= IbB × a sin θ = IB A sin θ
Torque on a current loop in a uniform
magnetic field. As shown in Fig. 4.89(a), If the rectangular loop has N turns, the
consider a rectangular coil PQRS torque increases N times i.e.,
suspended in a uniform magnetic field B, 𝜏 = 𝑁𝐼𝐵𝐴sin𝜃
with its axis perpendicular to the field.
61
But NIA = m, the magnetic moment of the ➢ In a uniform magnetic field, the net
loop, so 𝜏 = mB sin θ magnetic force on a current loop is
zero but torque acting on it may be
In vector notation, the torque x is given by
zero or non-zero.
𝜏=𝑚 ⃗
⃗⃗ × 𝐵 ➢ In a non-uniform magnetic field, the
net magnetic force on a current is
The direction of the torque x is such non-zero but torque acting on it
that it rotates the loop clockwise about the may be zero or non-zero.
axis of suspension.
Examples based on
Special Cases
Torques on current loops
(i) When θ =0°, x =0, i.e., the torque is
minimum when the plane of the loop is Formulae Used
perpendicular to the magnetic field. Torque on a current loop in a magnetic
(ii) When θ =90°, x = NIB A, i.e., the torque field, 𝜏 = NIB A sin θ = mB sin θ
is maximum when the plane of the loop where m = NIA = magnetic dipole moment
is parallel to the magnetic field. Thus of the current loop.
𝜏max = 𝑁𝐼𝐵𝐴 ⃗
In vector form, 𝜏 = 𝑚
⃗⃗ × 𝐵
For Your Knowledge
Units Used
➢ The expression for torque ( 𝜏 =
Current I is in ampere, area A in m2, field B
NIBA sin θ) holds for a planar loop
in tesla, torque x in Nm and magnetic
of any shape. Thus the torque on a
moment m in Am2.
planar current loop depends on
current, strength of magnetic field Example 70. The maximum torque acting
and area of the loop. It is on a coil of effective area 0.04 m2 is 4× 10-8
independent of the shape of the Nm when the current in it is 100 μA Find
loop. the magnetic induction in which it is kept.
➢ For a planar current loop of a given
Solution. A =0.04 m2, 𝜏 max = 4×10-8 Nm,
perimeter suspended in a magnetic
field, the torque is maximum when I = 100 μA =10-4 A, N =1
the loop is circular in shape. This is As 𝜏max = 𝑁𝐼𝐵𝐴
because for a given perimeter, a
circle has maximum area. ∴Magnetic induction,
➢ The expression 𝜏 = 𝑚 ⃗ for the
⃗⃗ × 𝐵 𝜏max
torque on a current loop in a 𝐵=
𝑁𝐼𝐴
magnetic field is analogous to the
4 × 10−8
expression 𝜏 = 𝑝𝑒 × 𝐸⃗ for the =
torque on an electric dipole in 1 × 10−4 × 0.04
anelectric field. This supports the = 10−2 Wbm−2
fact that a current loop is a
magnetic dipole. Example 71. Calculate the torque on a 100
➢ The torque on a current loop in a turn rectangular coil of length 40 cm and
magnetic field is the operating breadth 20 cm, carrying a current of 10 A,
principle of the electric motor and when placed making an angle of 60° with a
most electric meters used for magnetic field of 3 T.
measuring currents and voltages, Solution. Here I =10 A, N =100, B =3 T,
called galvanometers.
A = 40 cm × 20 cm = 800 cm2 = 8 × 10-2 m2
➢ If the direction of the magnetic field
θ = 90°-60° =30°
makes an angle α with the plane of
the current loop, then = Angle between B and the normal to the
θ + α = 90° or θ = 90° - α plane of the coil
∴ 𝜏 = NIBA sin (90° - α) = NIB A cos
∴ Torque,
α.
62
𝜏 = NIBA sin θ magnetic field is zero.
= 100 × 10 × 3 × 8 × 10-2 × sin 30° Fig. 4.90
= 120 Nm. In Fig. 4.90, n is a unit vector normal to the
plane of the loop, directed outward. The
Example 72. Given a uniform magnetic
angle between n and 8 is 90°. The
field of 100 G in an east to west direction
magnitude of the torque acting on the loop
and a 44 cm long wire with a current
is
carrying capacity of almost 10 A. What is
the shape and orientation of the loop made 𝜏 = NIB A sin θ
of this wire which yields maximum turning
= 25 × 10 × 1.2 × π (0.06)2 × sin 90°
effect on the loop? What is the magnitude
of the maximum torque? = 3.4 Nm
Solution. B = 100 G = 100 × 10-4 T, I = This torque acts in the vertically upward
10 A direction producing turning effect in the
direction of curved arrow. To prevent the
The torque on the planar loop will be
coil from turning, a balancing torque 𝜏 ′ = 𝜏
maximum if its area is maximum. Since for
must be applied.
a given perimeter, a circle encloses
maximum area, therefore, the wire should Example 74. A rectangular coil of sides 8
be bent into a circular loop of radius r given cm and 6 cm having 2000 turns and
by carrying a current of 200 mA is placed in a
uniform magnetic field of 0.2 T directed
2πr = 44
along the + ve X-axis.
44 44 × 7
𝑟= = = 7cm = 0.07m (i) What is the maximum torque the coil
2𝜋 2 × 22 can experience? In which orientation
∴ Area of the circular loop, does it experience the maximum
22 torque?
𝐴 = 𝜋𝑟 2 = × (0.07)2 = 0.0154m2 (ii) For which orientations of the coil is the
7
torque zero?
Again, for maximum torque, the loop
must be oriented with its plane in N-S When is this equilibrium stable and when is
direction. it unstable?
Then
[NCERT]
𝜏max = 𝐼𝐵𝐴 = 10 × 100 × 10−4
Solution. I =8 cm =0.08 m, b = 6 cm
× 0.0154Nm
=0.06 m N =2000, I =200 mA =0.2 A, B =
= 1.54 × 10-3Nm. 0.2 T
Example 73. A circular coil of 25 turns and The magnitude of the magnetic dipole
radius 6.0 cm, carrying a current of 10 A, is moment is given by
suspended vertically in a uniform magnetic m= NIA =2000 × 0.2 × (0.08 × 0.06) =1.92
field of magnitude 1.2 T. The field lines run
Am2
horizontally in the plane of the coil.
Calculate the force and the torque on the The direction of 𝑚
⃗⃗ is normal to area 𝐴
coil due to the magnetic field. In which of the coil from S-pole to N-pole.
direction should a balancing torque be Magnitude of torque on the coil is
applied to prevent the coil from turning?
𝜏 = mB sin θ
Solution. Consider any element dl of
⃗⃗⃗ × 𝐵
⃗. For maximum torque, m must be
the wire. Force on this element is 𝐼𝑑𝑙
perpendicular to B.
For each element dl, there is another
element -dl on the current loop. Forces on Therefore,
each pair of such elements cancel out. 𝜏 max = mB = 1.92 × 0.2 = 0.384 Nm
Hence net force on the coil in a uniform
63
Thus the torque on the coil is maximum
whenever the X-axis lies in the plane of the
coil.
The torque on the coil is zero when 𝑚 ⃗⃗
is parallel or antiparallel to 8, i.e., when it
lies in the YZ-plane. The coil will be in
stable equilibrium when 𝑚 ⃗⃗ is parallel to 𝐵⃗
and in unstable equilibrium when 𝑚 ⃗⃗ is
Fig. 4.92
⃗.
antiparallel to 𝐵
Solution. As the magnetic field B acts
Example 75. A 100-turns coil kept in a parallel to sides PS and QR, no forces act
magnetic field B =0.05 Wb m, carries a on these sides.
current of 1 A, as shown in Fig. 4.91. Find
the torque acting on the coil. [MNREC 97] Force on side PQ is
F = IIB sin θ = 1.5 × 0.7 × 40 × sin 60°
= 1.5 × 0.7 × 40 × 0.866 = 36.37 N
According to Fleming's left hand rule,
the force F will act normally upward.
Similarly, force on side SR will also be
36.37 N, but it will be directed normally
Fig. 4.91 inward.
Solution. Here the angle between the (ii) As the forces on the sides PQ and
axis of rotation of the coil and the magnetic SR are equal, opposite and parallel, they
⃗ is 90°.
field 𝐵 form a couple which exerts a torque.
∴ N = 100, 7 = 1 A, 𝜏 = Force × ⊥ distance between the two
forces
A-15 cm × 15 cm =225 × 10-4m2,
= F × PS sin 60° = 36.37 × 0.5 × 0.866
8 = 0.5 Wbm-2, θ =90°
= 15.75 Nm.
Torque,
Example 77. A 100 turn closely wound
𝜏 = NIB A sin θ circular coil of radius 10 cm carries a
= 100 × 1 × 0.5 × 225 × 10-4 × sin 90° current of 3.2 A. (i) What is the field at the
center of the coil? (ii) What is the magnetic
= 1.125 Nm moment of this arrangement?
As the force on the arm PQ acts The coil is placed in a vertical plane
upwards and that on SR downwards, so and is free to rotate about a horizontal axis
the torque acts anticlockwise. which coincides with its diameter. A
Example 76. A parallelogram-shaped coil uniform magnetic field of 2T in the
PQRS of sides 0.7 m and 0.5 m carries a horizontal direction exists such that initially
current of 1.5 A, as shown in Fig. 4.92. the axis of the coil is in the direction of the
field. The coil rotates through an angle of
It is placed in a magnetic field B =40 T 90° under the influence of the magnetic
parallel to PS. Find (i) forces on the sides field. (iii) What are the magnitudes of the
of the coil and (ii) torque on the coil. torques on the coil in the initial and final
position? (iv) What is the angular speed
acquired by the coil when it has rotated by
90°? The M.I. of the coil is 0.1 kg m2.
[NCERT]
Solution, (i) Here N =100, I =3.2 A, r = 10
cm =0.1 m
64
Magnetic field at the centre of the coil, current of 3 A. A thin coil having 10 turns of
wire and of radius 0.01 m carries a current
𝜇0 𝑁𝐼 4𝜋 × 10−7 × 100 × 3.2
𝐵= = of 0.4 A. Calculate the torque required to
2𝑟 2 × 0.1 hold the coil in the middle of the solenoid
[∵ 𝜋 × 3.2 = 10] with its axis perpendicular to the axis of the
solenoid (μ0 = 4π × 10-7 V-s/ A- m).
4 × 10−5 × 10
= = 2 × 10−3 T [Roorkee 90]
2 × 0.1
The direction of the field is given by Solution. For solenoid, l = 0.4 m, N1 =
right hand thumb rule. 500, I1 = 3 A
(ii) Magnetic moment associated with For coil, N2 =10, r = 0.01m, I2= 0.4 A
the coil, Field inside the solenoid,
m = NIA = Nl× 𝜋𝑟 2 = 100 × 3.2 × 3.14 × 𝜇0 𝑁1 𝐼1
𝐵= along the axis of solenoid.
(0.1)2 𝑙
65
orientation will the coil be in stable 4. 𝜏 = IB A sin θ = 2.0 × 2.0 × 10 -2 × 0.40 ×
equilibrium? 0.25 × sin 90°
[CBSE OD 15C] (Ans. 1 Nm) = 4.0 ×10-3 Nm.
4. A rectangular coil PQRS is placed in a 𝑡
5 𝐵 = 𝑁𝐿𝐴sin𝜃
uniform magnetic field B, as shown in 0.2
Fig. 4.93. Find the torque on the coil =
when it carries a current of 2.0 A. The 100 × 2 × 0.05 × 0.04 × sin90°
magnitude of the field B is 2.0 × 10-2 T. = 0.5T.
(Ans. 4.0 × 10-3 Nm) 6. 𝜏 = NIB (πr2) sin θ
5. A rectangular coil of 100 turns has = 500 × 1.0 × 0.40 × 3.14 × (0.02) 2 × sin 30°
length 5 cm and width 4 cm. It is placed = 0.1256 ≈ 0.13 Nm.
with its plane parallel to a uniform
magnetic field and a current of 2 A is 7. Proceed as in Exercise 4.25 on page
passed through the coil. If the torque 4.104.
acting on the coil is 0.2 Nm, find the 4.20 MOVING COIL GALVANOMETER
magnitude of the magnetic field.
23. Describe the principle, construction
(Ans. 0.5 T) and working of a pivoted-type moving coil
6. A circular coil of radius 2.0 cm has 500 galvanometer. Define its figure of merit.
turns and carries a current of 1.0 A. Its Moving coil galvanometer. A
axis makes an angle of 30° with the galvanometer is a device to detect current
uniform magnetic field of 0.40 T that in a circuit. The commonly used moving
exists in the space. Find the torque coil galvanometer is named so because it
acting on the coil. (Ans. 0.13 Nm) uses a current-carrying coil that rotates (or
7. A circular coil of 200 turns and radius moves) in a magnetic field due to the
10 cm is placed in a uniform magnetic torque acting on it.
field of 0.5 T, normal to the plane of the In a D'Arsonval galvanometer, the coil
coil. If the current in the coil is 3.0 A, is suspended on a phosphor-bronze wire. It
calculate the (a) total torque on the coil, is highly sensitive and requires careful
(b) total force on the coil, (c) average handling. In Weston galvanometer, the coil
force on each electron in the coil, due is pivoted between two jewellel bearings. It
to the magnetic field. is rugged and portable though less
Assume the area of cross-section of the sensitive, and is generally used in
wire to be 10-5 m2 and the free electron laboratories. The basic principle of both
density is 1029/m3. types of galvanometers is same.
[CBSE OD 08] Principle. A current carrying coil placed in a
magnetic field experiences a current
[Ans. (a) zero (b) zero (c) 1.5 ×10-24 N] dependent torque, which tends to rotate
HINTS the coil and produces angular deflection.
1. A = 5cm × 12cm = 60×10 4m2 Construction. As shown in Fig. 4.94, a
Weston (pivoted-type) galvanometer
𝜏max = NIB A
consists of a rectangular coil of fine
= 600 ×10-5 × 0.10×60 ×10-4 = 3.6×10-6Nm. insulated copper wire wound on a light
non-magnetic metallic (aluminium) frame.
2. 𝜏 = NIBA sin θ = 40 × 10 × 0.2 × 100 ×
The two ends of the axle of this frame are
10-4 sin 90°
pivoted between two jewelled bearings.
= 0.8 Nm. The motion of the coil is controlled by a pair
of hair springs of phosphor-bronze. The
3. 𝜏 = NIBA sin θ = 200×5×0.2×
inner ends of the springs are soldered to
100×10-4sin(90°-60°)
the two ends of the coil and the outer ends
= 1 Nm. are connected to the binding screws. The
66
springs provide the restoring torque and have
serve as current leads. A light aluminium
I = current flowing through the coil PQRS
pointer attached to the coil measures its
deflection on a suitable scale. a, b = sides of the rectangular coil PQRS
A = ab = area of the coil
N = number of turns in the coil.
67
𝑁𝐵𝐴 moving coil galvanometer depends :
or 𝛼= ⋅𝐼
𝑘
1. Number of turns N in its coil.
or α∝I
2. Magnetic field B.
Thus the deflection produced in the
galvanometer coil is proportional to the 3. Area A of the coil.
current flowing through it. Consequently, 4. Torsion constant k of the spring and
the instrument can be provided with a scale
suspension wire.
with equal divisions along a circular scale
to indicate equal steps in current. Such a Factors by which the sensitivity of a
scale is called linear scale. moving coil galvanometer can be
𝑘 increased :
Also, 𝐼 = 𝑁𝐵𝐴 ⋅ 𝛼 = 𝐺𝛼
1. By increasing the number of turns N of
The factor G-k/ NBA is constant for a the coil. But the value of N cannot be
galvanometer and is called galvanometer increased beyond a certain limit
constant or current reduction factor of the because that will make the
galvanometer. galvanometer bulky and increase its
resistance R.
Figure of merit of a galvanometer. It is
defined as the current which produces a 2. By increasing the magnetic field B. This
deflection of one scale division in the can be done by using a strong
galvanometer and is given by horse-shoe magnet and placing a soft
𝐼 𝑘 iron core within the coil.
𝐺= = 3. By increasing the area A of the coil.
𝛼 𝑁𝐵𝐴
However, increasing A beyond a
4.21 SENSITIVITY OF A
certain limit will make the galvanometer
GALVANOMETER
bulky and unmanageable.
24. When is a galvanometer said to be
4. By decreasing the value of torsion
sensitive? Define current sensitivity and
constant k. The torsion constant k is
voltage sensitivity of a galvanometer. State
made small by using suspension wire
the factors on which the sensitivity of a
and springs of phosphor bronze.
moving coil galvanometer depends. How
can we increase the sensitivity of a 25. Give the advantages and
galvanometer? disadvantages of using a moving coil
galvanometer.
Sensitivity of a galvanometer. A
galvanometer is said to be sensitive if it Advantages of a moving coil galvanometer
shows large scale deflection even when a :
small current is passed through it or a small
1. As the deflection of the coil is
voltage is applied across it. proportional to the current passed
Current sensitivity. It is defined as the through it, so a linear scale can be used
deflection produced in the galvanometer to measure the deflection.
when a unit current flows through it. 2. A moving coil galvanometer can be
𝛼 𝑁𝐵𝐴 made highly sensitive by increasing N,
Current sensitivity, 𝐼𝑆 = =
𝐼 𝑘 B, A and decreasing k.
Voltage sensitivity. It is defined as the
3. As the coil is placed in a strong
deflection produced in the galvanometer
magnetic field of a powerful magnet, its
when a unit potential difference is applied
deflection is not affected by external
across its ends.
magnetic fields. This enables us to use
𝛼 𝛼 𝑁𝐵𝐴
Voltage sensitivity, 𝑉𝑆 = 𝑉 = 𝐼𝑅 = the galvanometer in any position.
𝑘𝑅
Current sensitivity 4. As the coil is wound over a metallic
Clearly, voltage sensitivity = 𝑅 frame, the eddy currents produced in
the frame bring the coil to rest quickly.
Factors on which the sensitivity of a
68
Disadvantages of a moving coil in tesla, angle α in degrees, torque t in Nm,
galvanometer : resistance R in ohm, potential difference V
in volt, torsion constant k in Nm deg-1.
1. The main disadvantage is that its
sensitiveness cannot be changed at Example 79. A rectangular coil of area 5.0
will.
× 10 -4 m2 and 60 turns is pivoted about one
2. All types of moving coil galvanometers
of its vertical sides. The coil is in a radial
are easily damaged by overloading. A
horizontal field of 90 G ('radial' here means
current greater than that which the
the field lines are in the plane of the coil for
instrument is intended to measure will
any orientation). What is the torsional
burn out its hair-springs or suspension.
constant of the hairsprings connected to
For Your Knowledge the coil if a current of 0.20 mA produces an
➢ If the radial field were not present in angular deflection of 18°?
a moving coil galvanometer, for
example, if the soft iron cylinder [NCERT]
were removed, then the torque Solution. B = 90 G = 90 × 10-4 T,
would be NBAI sin θ and 1 would be
A = 5.0 × 10-4m2, I = 0.20 mA = 0.20 × 10-3
proportional α / sin θ. The scale
A,
would then be non-linear and difficult
to calibrate or to read accurately. N =60, a =18°
➢ Phosphor-bronze is used for
Torsional constant of the hair spring is
suspension or hair springs because
given by
of several reasons :
𝑁𝐼𝐵𝐴
1. It is a good conductor of electricity. 𝑘=
𝛼
2. It does not oxidise.
60 × 0.2 × 10−3 × 90 × 10−4 × 5 × 10−4
3. It is perfectly elastic. =
18
4. It has very little elastic after effect. Nm𝑑𝑒𝑔 −1
5. It is non-magnetic. = 3.0 × 10-9 Nm deg
6. Of all materials, it has the minimum Example 80. A rectangular coil having
value for restoring torque per unit twist each turn of length 5 cm and breadth 2 cm
i.e., smallest torsion constant k. is suspended freely in a radial magnetic
Examples based on field of induction 2.5 × 10-2 Wb m-2,
torsional constant of the suspension fibre is
Moving coil galvanometer and its 1.5 × 10-8 Nm rad-1. The coil deflects
Sensitivity through an angle of 0.2 radian when a
Formulae Used current of 2 μA is passed through it. Find
the number of turns of the coil.
1. In a moving coil galvanometer,
Solution. A = 5 cm × 2 cm =10 × 10 -4m2
𝑘
Current, 𝐼 = ⋅𝛼 = 10-3m2
𝑁𝐵𝐴
𝑁𝐵𝐴 B = 2.5×10-2 Wb m-2, k =1.5 × 10-8Nm
Deflection produced, 𝛼 = ⋅𝐼 -1
𝑘 rad
𝐼 𝑘
2. Figure of merit, 𝐺 = = θ =0.2 rad, I =2 μ A =2 × 10-6A
𝛼 𝑁𝐵𝐴
𝛼 𝑁𝐵𝐴 𝑘
3. Current sensitivity, 𝐼𝑆 = = As 𝐼 = 𝑁𝐵𝐴 ⋅ 𝛼
𝐼 𝑘
𝛼 𝛼 𝑁𝐵𝐴 𝑘
4. Voltage sensitivity, 𝑉𝑆 = = 𝐼𝑅 = 𝑁= ⋅𝛼
𝑉 𝑘𝑅 𝐼𝐵𝐴
Units Used 1.5 × 10−8 × 0.2
= = 60
Current I is in ampere, area A in m2, field B 2 × 10−6 × 2.5 × 10−2 × 10−3
69
Example 81. The coil of a moving coil 50 divisions
= = 103 divV−1
galvanometer has an effective area of 5 × 50 × 10−3 V
10-2 m2 It is suspended in a magnetic field
Resistance of galvanometer,
of 2 × 10-2 Wb m-1. If the torsional constant
of the suspension fibre is 4 × 10-9 Nm deg-1 𝐼𝑆 1𝑑𝑖𝑣𝜇A−1
find its current sensitivity in degree 𝑅𝑔 = = 3
𝑉𝑆 10 𝑑𝑖𝑣𝑉 −1
per-microampere.
106 𝑑𝑖𝑣A−1
Solution. Here N = 1, A = 5 × 10-2 m2, = = 1000𝛺
-2 -2 -9 -1
103 𝑑𝑖𝑣V−1
B = 2 × 10 Wb m , k = 4 × 10 Nm deg
Example 84. A moving coil meter has the
Current sensitivity following particulars : Number of turns, N =
𝑁𝐵𝐴 1 × 2 × 10−2 × 5 × 10−2 24 ; Area of the coil, A =20 × 10-3m2
= = Magnetic field strength, B =0.20 T;
𝑘 4 × 10−9
Resistance of the coil, R = 14 Ω.
= 0.25 × 106 deg A-1
(i) Indicate a simple way to increase the
= 0.25 deg μ A-1 current sensitivity of the meter by 25%.
[∵ 1 A = 106 μA] (It is not easy to change A or B).
(ii) If in doing so, the resistance of the coil
Example 82. A current of 200 μA deflects
increases to 21Ω, is the voltage
the coil of a moving coil galvanometer
sensitivity of the modified meter greater
through 30°. What should be the current to
or less than the original meter?
cause the rotation through π/10 radian?
[NCERT]
What is the sensitivity of the
galvanometer? Solution, (i) Current sensitivity,
Solution. Here 𝐼1 =200 μA, θ1 =30°, 𝛼 𝑁𝐵𝐴
𝐼𝑆 = =
𝜋 𝐼 𝑘
θ2 =10 rad = 18° , 𝐼2 =?
Since it is easier to change N than K, A
𝑘 or B, so the current sensitivity can be
𝐼1 = ⋅𝛼
𝑁𝐵𝐴 1 increased by increasing N. To increase it
𝑘 by 25%, N should be increased from 24 to
and 𝐼2 = ⋅𝛼 30.
𝑁𝐵𝐴 2
∴𝐼2 𝛼 (ii) Voltage sensitivity,
= 𝛼2
𝐼1 1 𝛼 𝛼 𝑁𝐵𝐴
𝛼2 18 𝑉𝑆 = = =
𝐼2 = ⋅ 𝐼1 = × 200 = 120𝜇A 𝑉 𝑅𝐼 𝑘𝑅
𝛼1 30 As k, A, B are same in the two cases,
2
a2 1
30 we need to compare only N/R ratio.
𝑁 24
Current sensitivity For original meter, = = 1.7
𝑅 14
𝛼2 18𝑑𝑒𝑔 𝑁 30
= = = 0.15𝑑𝑒𝑔𝜇𝐴−1 For modified meter, = = 1.4
𝐼2 120𝜇A 𝑅 21
70
sensitivity change? [CBSE OD 01] 1. A rectangular coil of area 100 cm2 and
consisting of 100 turns is suspended in
Solution. Current sensitivity,
a magnetic field of 5 × 10 -2 T. What
𝛼 current should be made to pass
𝐼𝑆 =
𝐼 through it in order to keep equilibrium at
Voltage sensitivity, an angle of 45° with the field? Given
that torsion constant of the suspension
𝛼 𝛼 𝐼𝑆
𝑉𝑆 = = = fibre is 10-8Nm deg-1.
𝑉 𝐼𝑅 𝑅
(Ans. 9 × 10-6 A)
New current sensitivity,
2. The coil of a galvanometer consists of
50 3
𝐼𝑆′ = 𝐼𝑆 + 𝐼𝑆 = 𝐼𝑆 250 turns of fine wire wound on a 2.0
100 2 cm × 1.0 cm rectangular frame. It is
New voltage sensitivity, suspended in a uniform radial magnetic
field of strength 2,000 G. A current of
3
𝐼𝑆′ 𝐼𝑆 3 10“4 A produces an angular deflection
𝑉𝑆′ = = 2 = 𝑉𝑆 = 0.75𝑉𝑆 of 30° in the coil. Find the torsional
2𝑅 2𝑅 4
constant of its suspension.
Thus new voltage sensitivity becomes
75% of its initial value i.e., it decreases by (Ans. 1.9 ×10-6 Nm rad-1)
25%. 3. A moving coil galvanometer is placed in
Example 86. The coil of a galvanometer is a radial magnetic field of 0.2 T. The
0.02 m× 0.08 m It consists of200 turns of galvanometer coil has 200 turns and
fine wire and is in a magnetic field of 0.2 area of 1.6×10-4m2. The torsion
tesla. The restoring torque constant of the constant of the suspension fibre is 10 -6
suspension fibre is 10-6 Nm deg-1. Nm deg-3 Determine the maximum
Assuming the magnetic field to be radial, current that can be measured by this
(a) what is the maximum current that can galvanometer if its scale can
be measured by this galvanometer, if the accommodate a deflection of 45°. (Ans.
scale can accommodate 30° deflection? 7 × 10-J A)
(b) What is the smallest current that can be 4. The coil of moving coil galvanometer is
detected, if the minimum observable 40 mm long and 25 mm wide. It has
deflection is 0.1 degree? 100 turns and is suspended in a radial
[CBSE OD 13C] magnetic field of 10-2 T. If the
suspension fibre has a torsional
Solution. Here A =0.02 × 0.08 m2 =1.6 constant of 10-8 Nm deg-1, find the
× 10-3m2 current sensitivity of the moving coil
N = 200, B = 0.2 T, k = 10-6 Nm deg-1 galvanometer. (Ans. 0.1 deg μA_1)
(a) The maximum current (Imax) that can 5. A coil of a moving coil galvanometer
be measured is given by twists through 45° when a current of 1
micro-ampere is passed through it. If
𝑁𝐵𝐴𝐼max = 𝑘𝛼max the area of the coil is 10 -5m2 and it has
𝑘𝛼max 10−6 × 30 1000 turns, find the magnetic field of
𝐼max = = A the magnet of the galvanometer. The
𝑁𝐵𝐴 200 × 0.2 × 1.6 × 10−3
restoring torque per unit twist of the
= 4.69 × 10-4 A. galvanometer coil is 10 -4Nm deg-1.
(b) The smallest current (Imin) that can (Ans. 45 T)
be detected is given by 6. The coil of a pivoted-type galvanometer
𝐼min =
𝑘𝛼min has 50 turns and encloses an area of 6
𝑁𝐵𝐴 m2. The magnetic field in the region in
10−6 × 0.1 A
which the coil swings is 0.01 T and is
= = 1.56 × 10−6 A radial. The torsional constant of the hair
200 × 0.2 × 1.6 × 10−3
spring is 1.0×10-8Nm deg-1. Find the
Problems for Practice
71
angular deflection of the coil for a 𝑘𝛼MAX
3. 𝐼MAX = 𝑁𝐵𝐴
current of 1 mA. (Ans. 30°)
10−6 × 45
7. A rectangular coil of area 8×10 -4m2 is = = 7 × 10−3 A
suspended freely in a radial magnetic 200 × 0.2 × 1.6 × 10−4
field of induction 2 × 10-2 Wb m-2. When 4. Current sensitivity =
𝑁𝐵𝐴
a current of 5 μA is passed through the 𝑘
72
galvanometer into ammeter or voltmeter : gives full scale deflection
1. Ammeter or voltmeter should be 0-1 = the required current range of the
accurate, reliable and sensitive. ammeter
2. The use of these devices in a circuit S = shunt resistance
must not alter the current in the circuit
I - Ig = current through the shunt.
or the potential difference across any
element in the circuit. As galvanometer and shunt are
connected in parallel, so
4.23 CONVERSION OF A
GALVANOMETER INTO AN P.D. across the galvanometer = P.D.
AMMETER across the shunt
26. Explain how can we convert a 𝐼𝑔 𝐺 = (𝐼 − 𝐼𝑔 )𝑆
galvanometer into an ammeter of given 𝐼𝑔
range. 𝑆= ×𝐺
𝐼 − 𝐼𝑔
Conversion of a galvanometer into an
ammeter. An ammeter is a device used to So by connecting a shunt of resistance
measure current through a circuit element. S across the given galvanometer, we get
To measure current through a circuit an ammeter of desired range. Moreover,
element, an ammeter is connected in 𝑆
series with that element so that the current 𝐼𝑔 = ×𝐼
𝐺 +𝑆
which is to be measured actually passes
through it. In order to ensure that its The deflection in the galvanometer is
insertion in the circuit does not change the proportional to Ig and hence to I. So the
current, an ammeter should have zero scale can be graduated to read the value of
resistance. So ammeter is designed to current I directly.
have very small effective resistance. In Hence an ammeter is a shunted or low
fact, an ideal ammeter should have zero resistance galvanometer. Its effective
resistance. resistance is
An ordinary galvanometer is a sensitive 27. What is a shunt? Mention its important
instrument. It gives full scale deflection with uses. Shunt. A shunt is a low
a small current of few microamperes. To resistance which is connected in
measure large currents with it, a small parallel with a galvanometer (or
resistance is connected in parallel with the ammeter) to protect it from strong
galvanometer coil. The resistance currents.
connected in this way is called a shunt.
Only a small part of the total current passes Uses of shunt:
through the galvanometer and remaining 1. To prevent a galvanometer from being
current passes through the shunt. The damaged due to large current.
value of shunt resistance depends on the
range of the current required to be 2. To convert a galvanometer into
measured. ammeter.
3. To increase the range of an
ammeter.
For Your Knowledge
➢ Since an ammeter is a parallel
combination of the galvanometer
and the shunt resistance, so its
Fig. 4.96
resistance is even less than that of
Let G = resistance of the galvanometer
the shunt resistance. Moreover, RA
Ig = the current with which galvanometer
73
<<G.
➢ Because of its very small
resistance, an ammeter placed in a
series circuit does not practically
change the current in the circuit to Fig. 4.97
be measured. A galvanometer can be converted into
a voltmeter by connecting a high
➢ The resistance of an ideal ammeter resistance in series with it. The value of this
is zero. resistance is so adjusted that only current /
which produces full scale deflection in the
➢ Higher the range of ammeter to be galvanometer, passes through the
prepared from a given galvanometer.
galvanometer, lower is the value of Let
the shunt resistance required for the G = resistance of the galvanometer
purpose. lg = the current with which galvanometer
gives full scale deflection
➢ The ammeter of lower range has a
0 - V = required range of the voltmeter,
higher resistance than the ammeter and
of higher range. R = the high series resistance which
➢ The range of an ammeter can be restricts the current to safe limit I.
74
voltage is to be measured. Because of put in a circuit, does it read (slightly) less or
its high resistance, it draws a very small more than the actual current in the original
current and hence the potential circuit? When a voltmeter is put across a
difference across the element remains part of the circuit, does it read (slightly) less
practically unaffected. or more than the original voltage drop?
➢ Higher the range of voltmeter to be Explain.
prepared from a given galvanometer,
[NCERT ; CBSE D 05]
higher is the value of series high
resistance required for the purpose. Solution, (i) For conversion into
➢ The voltmeter of lower range has a ammeter :
lower resistance than the voltmeter of Rg = 12 Ω, I = 2.5 mA= 0.0025 A, 1 =7.5 A
higher range.
➢ The range of voltmeter can be both 𝐼𝑔 0.0025
𝑅𝑠 = × 𝑅𝑔 = × 12
increased or decreased. 𝐼 − 𝐼𝑔 7.5 − 0.0025
Examples based on 2.5 × 12 × 10−3
= = 4.0 × 10−3 𝛺
Conversion of galvanometer into (i) 7.4975
ammeter and (ii) voltmeter and So by connecting a shunt resistance of
measurement of current and voltage 4.0 × 10-3 Ω in parallel with the
Formulae Used galvanometer, we get an ammeter of range
0 to 7.5 A.
1. For conversion of a galvanometer into
ammeter, the shunt resistance, Net resistance RA is given by
𝐼 𝑅𝑠 1 1 1 3001
𝑅5 = 𝐼−𝐼𝑔 × 𝑅𝑔 ; Here 𝐼𝑔 = 𝑅 ×𝐼 = + −3 =
𝑔 𝑔 +𝑅𝑠 𝑅𝐴 12 4 × 10 12
𝑅 𝑅 12
2. Resistance of an ammeter, 𝑅𝐴 = 𝑅 𝑔+𝑅𝑆 or 𝑅𝐴 = 3001 𝛺 = 4 × 10−3 𝛺
𝑔 𝑠
75
[IIT] 𝐼𝑔 1.0
𝑅𝑠 = × 𝑅𝑔 = × 0.80
Solution. As the current sensitivity is 𝐼 − 𝐼𝑔 5.0 − 1.0
10 div per mA and there are 100 divisions = 0.20𝛺
on the scale, so current required for full (ii) The combined resistance RA of the
scale deflection is ammeter and the shunt is given by
1
𝐼𝑔 = 10 × 100 mA = 10 mA = 10× 10-3A 1 1 1 1 1 1 + 4 25
= + = + = =
𝑅𝐴 𝑅𝑔 𝑅𝑠 0.8 0.2 0.8 4
=0.01 A
As voltage sensitivity is 2 div per mV, or RA =4/25 = 0.16 Ω.
so voltage required for full scale deflection Example 90. In the circuit (Fig. 4.98), the
is current is to be measured. What is the
1 value of the current if the ammeter shown
𝑉𝑔 = 2 × 100 mV = 50 mV = 50 × 10-3V (a) is a galvanometer with a resistance R =
Galvanometer resistance, 60.00 Ω;
𝑉𝑔 50 × 10−3
𝑅𝑔 = = = 5𝛺
𝐼𝑔 10 × 10−3
(i) For conversion into an ammeter. I =
5A
𝐼𝑔 0.01 5 Fig. 4.98
𝑅𝑠 = × 𝑅𝑔 = ×5= 𝛺
𝐼 − 𝐼𝑔 5 − .01 499 (b) is a galvanometer described in (a) but
converted to an ammeter by a shunt
So a shunt of 5 / 499 Ω should be resistance Rs =0.02 Ω ; and (c) is an ideal
connected across the galvanometer to ammeter with zero resistance ?
read 5 A for full scale deflection.
[NCERT]
(ii) For conversion into voltmeter. For
reading 1 div per volt, the voltage range Solution, (a) Total resistance in the circuit
should be 100 V because there are 100 = Rg + 3Ω = 60 + 3 = 63 Ω
divisions. 3V
Current, 𝐼 = 63𝛺 = 0.048A
𝑉 100
∴ 𝑅= − 𝑅𝑔 = − 5 = 10000 − 5 =
𝐼𝑔 0.01 (b) Resistance of the galvanometer
9995𝛺 converted to an ammeter is
So a resistance of 9995 Ω should be 𝑅𝑔 𝑅𝑠 60 × 0.02
connected in series with the given 𝑅𝐴 = = ≃ 0.02𝛺
𝑅𝑔 + 𝑅𝑠 60 + 0.02
galvanometer to read 1 div per volt.
Rg + Rs 60 + 0.02
Example 89. An ammeter of resistance
0.80 Ω can measure currents upto 1.0 A (i) Total resistance in the circuit
What must be the shunt resistance to = 𝑅𝐴 + 3𝛺 = 0.02 + 3 = 3.02𝛺
enable the ammeter to measure current
3V
upto 5.0 A? (ii) What is the combined Current, 𝐼 = 3.02𝛺 = 0.99A
resistance of the ammeter and the shunt?
(c) As the ideal ammeter has zero
resistance, so
[NCERT ; CBSE D 13]
3.00V
Solution. The given ammeter can be Current, 𝐼 = 3.00𝛺 = 1.00A
regarded as the galvanometer. Example 91. In a galvanometer there is a
∴ 𝐼𝑔 = 1.0A Rg = 0.80 Ω deflection of 10 divisions per mA. The
internal resistance of the galvanometer is
(I) Total current in the circuit, I = 5.0 A 60 Ω. If a shunt of 2.5 Ω is connected to the
The required shunt resistance, galvanometer and there are 50 divisions in
76
all, on the scale of galvanometer, what to produce full scale deflection is given by
maximum current can this galvanometer 𝑅𝑠
read? 𝐼𝑔 = .𝐼
𝑅𝑔 + 𝑅𝑠
[CBSE D 01C]
5×250×10−3
Solution. As the galvanometer has 50 ∴ 𝐼𝑔 = …(1)
𝑅𝑔 +5
divisions, current required to produce full
When a resistance R is connected in
scale deflection is
series,
1
𝐼𝑔 = 10 x 50 mA = 5 mA =0.005 A 𝑉 25
𝐼𝑔 = 𝑅 =𝑅 ...(2)
𝑔 +𝑅 𝑔 +1050
𝑅𝑔 = 60𝛺 and 𝑅s = 2.5𝛺
From equations (1) and (2),
Let I be the maximum current that the
galvanometer can read. 1.25 25
=
𝑅𝑔 + 5 𝑅𝑔 + 1050
𝑅𝑠
𝐼𝑔 = ×𝐼
𝑅𝑔 + 𝑅𝑠 Rg + 5 “ Rg + 1050
(𝑅𝑔 +𝑅𝑠 )𝐼𝑔 (60+2.5)5 or 1.25 R𝑔 + 1312.5 = 25 R𝑔 + 125
or 𝐼 = = = 125mA
𝑅𝑠 2.5
or 23.75 Rg = 1187.5
Example 92. A galvanometer having a 1187.5
resistance of 50 Ω gives a full deflection for or 𝑅𝑔 = = 50𝛺
23.75
a current of 0.05 A Calculate the length of
Example 94. When a galvanometer having
the shunt wire of 2 mm diameter required
30 divisions scale and 100 Ω resistance is
to convert the galvanometer to an ammeter
connected in series to a battery of emf 3 V
reading current upto 5 A Specific
through a resistance of 200 Ω, shows full
resistance for the material of the wire is 5 ×
scale deflection. Find the figure of merit of
10 7Ω m
the galvanometer in μA
[Punjab 96]
Solution. Here n = 30, Rg =100Ω,
Solution. 𝑅𝑔 = 50 Ω, I𝑔 = 0.05 A, I = 5 A S=3V,
𝐼𝑔 0.05 50 R =200 Ω, k =?
𝑅𝑠 = × 𝑅𝑔 = × 50 =
𝐼 − 𝐼𝑔 5 − 0.05 99 The current required to produce full
50 scale deflection in the galvanometer is
Now 𝑅= 𝛺, 𝜌 = 5 × 10−7 𝛺m, 𝑟 =
99 𝜉 3 1
1mm = 10−3 m 𝐼𝑔 = = = A = 104 𝜇A
𝑅𝑔 + 𝑅 100 + 200 100
𝑙
As 𝑅 = 𝜌 𝐴, so length of required shunt is
As Ig = nk, therefore, the figure of merit is
𝑅𝐴 50 × 𝜋 × (10−3 )2 𝐼𝑔 104 𝜇A
𝑙= = m 𝑘= = = 333.3𝜇A𝑑𝑖𝑣 −1
𝜌 99 × 5 × 10−7 𝑛 30 divisions
10 × 3.142 × 10 Example 95. The deflection produced in a
= m = 3.17m
99 galvanometer is reduced to 45 divisions
Example 93. A moving coil galvanometer from 55 when a shunt of 8 Ω is used.
when shunted with a resistance of 5 Ω Calculate the resistance of the
gives a full scale deflection for 250 mA and galvanometer.
when a resistance of 1050 Ω is connected Solution. Without shunt, 𝐼𝑔 = I = 55k
in series, it gives a full scale deflection for
25 volt. Find the resistance of the where k is the figure of merit of the
galvanometer and the current required to galvanometer. With shunt,
produce afldl scale deflection when it is 𝐼𝑔′ = (55 − 45)𝑘 = 10𝑘
used alone.
𝐼𝑔′ 10𝑘 2
Solution. With shunt, current required = =
𝐼𝑔 55𝑘 11
77
2 2 or Rg = 40 Ω.
𝐼𝑔′ = 𝐼𝑔 = 𝐼
11 11 Example 98. A voltmeter reads 5.0 V at full
When shunt of 8 Ω is used, scale deflection and is graded according to
𝑅𝑠 its resistance per volt at full scale deflection
𝐼𝑔′ = ×𝐼 as 5000 Ω V-1. How will you convert it into a
𝑅𝑔 + 𝑅𝑠
voltmeter that reads 20 V at full scale
8 2 deflection? Will it still be graded as 5000
or ×𝐼 = 11 𝐼 or 88 = 2 R𝑔 + 16
𝑅𝑔 +8 ΩV? Will you prefer this voltmeter to one
72 that is graded as 2000 ΩV-1?
or 𝑅𝑔 = = 36𝛺
2
[NCERT ; CBSE D 01C]
Example 96. A galvanometer of resistance
'G' can be converted into a voltmeter of Solution. Resistance per volt is another
range (0-V) volts by connecting a way of specifying the current at full scale
resistance 'R' in series with it. How much deflection. The grading of 5000 ΩV~ 1 at full
resistance will be required to change its scale deflection means that the current
range from 0 to V/2? required for full scale deflection is
1
[CBSEOD15C] 𝐼𝑔 = A = 0.2mA
𝑉
5000
Solution. In first case, 𝑅 = 𝐼 − 𝐺 In order to convert it into a voltmeter of
𝑔
range 0 to 20 V, a resistance R has to be
𝑉
𝐼𝑔 = connected in series with it. Then on
𝑅+𝐺 applying an extra P.D. of 15 V (20 V - 5 V),
Let R’ be the required resistance to the current through it is again 0.2 mA at full
change the range from 0 to V12. So in scale deflection.
second case,
R × 0.2 × 10-3 = 15
𝑉 ⁄2 15
𝐼𝑔 = or 𝑅 = 0.2×10−3 𝛺 = 75,000𝛺
𝑅′ + 𝐺
𝑉 𝑉 ⁄2 Thus (i) to convert the given voltmeter
= ′ (0 - 5 V range) into a voltmeter of range 0 to
𝑅+𝐺 𝑅 +𝐺
20 V, a resistance of 75,000 Ω should be
2𝑅′ + 2𝐺 = 𝑅 + 𝐺 connected in series with the given meter.
𝑅−𝐺
Hence, 𝑅′ = (ii) Original resistance of voltmeter
2
78
ammeter measuring upto 1 ampere? How Fig. 4.99
will you convert this ammeter into voltmeter
Solution, (i) Resistance of the parallel
reading upto 1 volt?
combination of voltmeter V (400 Ω) and
[Roorkee 87] 100 Ω resistance
Solution. Here n = 30, Rg= 20 Ω 400 × 100
𝑅′ = = 80𝛺
Current sensitivity, k = 20 μA / div 400 + 100
Total resistance in the circuit,
∴ Current required for full-scale deflection
is 𝑅 = 𝑅′ + 200 = 80 + 200 = 280𝛺
Ig = nk = 30 × 20 = 600 μA = 6 × 10 -4 A = Current in the circuit,
0.0006 A 𝜀 84 3
𝐼= = = A
(i) For conversion into ammeter . I = 1A 𝑅 280 10
𝐼𝑔 0.0006 × 20 Reading on the voltmeter =P.D. across
𝑅s = × 𝑅𝑔 = R'
𝐼 − 𝐼𝑔 1 − 0.0006
3
25 × 6 = × 80 = 24V
= = 0.15𝛺 10
9994
(ii) Total resistance before the voltmeter is
i.e., a shunt of 0.15 Ω should be connected
connected = 100 + 200 =300 Ω
across the galvanometer.
84V 7
(ii) For conversion of resulting ammeter 𝐼= = A
into voltmeter. The resistance of the 300𝛺 25
ammeter formed is 7
P.D. across 100 Ω resistor = 25 × 100 =
𝑅𝑔 𝑅𝑠 20 × 0.15 28V
𝑅𝑔′ = = = 0.015𝛺
𝑅𝑔 + 𝑅𝑠 20 + 0.15 Example 101. A d.c. supply of 120 V is
Current for full scale deflection, 𝐼𝑔′ = 1A connected to a large resistance X. A
voltmeter of resistance 10 k Ω placed in
Voltage range, V = 1 V series in the circuit reads 4 V. What is the
.'. Required series resistance, value of X? What do you think is the
purpose in using a voltmeter, instead of an
𝑉 1 ammeter, to determine the large resistance
𝑅= ′ − 𝑅𝑔′ = − 0.015 = 0.985𝛺
𝐼𝑔 1 X?
Example 100. A voltmeter V of resistance [NCERT]
400 Ω is used to measure the potential Solution. Current through voltmeter,
difference across a 100 Ω resistor in the
circuit shown in Fig. 4.99. 𝑉 4V
𝐼= = = 4 × 10−4 A
𝑅 10 × 103 𝛺
(i) What will be the reading on the
voltmeter?
(ii) Calculate the potential difference
across 100 Ω resistor before the
voltmeter is connected.
[CBSE D 98]
total e.m.f.
Also 𝐼 = total resistance
120
∴ 4 × 10−4 = 𝑋+104
or 4× 10-4X + 4 = 120
116
or 𝑋 = 4×10−4 𝛺 = 29 × 104 𝛺 = 290k𝛺
79
As the current in the circuit is very
small, the ammeter's reading will be too
small to be measured accurately. This is an
unusual use of voltmeter for measuring
very high resistance.
Example 102. A multirange voltmeter can
be constructed by using a galvanometer
circuit as shown in the figure 4.100. We
want to construct a voltmeter that can Solution. Here I = 1 mA, G = 10Ω
measure 2 V, 20 V and 200 V using a 𝐺
galvanometer of resistance 10Ω and that 𝑆=
𝐼
𝐼𝑔 − 1
produces maximum deflection for current
of l mA. Find the value of R1, R2 and R3 that
have to be used. 10𝛺 10
𝑆1 = = 𝛺 ≃ 1𝛺
[Example Problem ; CBSE SP 18] 10mA 9
1mA − 1
10𝛺 10
𝑆2 = = 𝛺 = 0.1𝛺
100mA 99
−1
1mA
10𝛺 10
𝑆3 = = A ≃ 0.01𝛺
1A 999
−1
1mA
Fig. 4.100 Example 104. (a) A battery of emf 9 V and
negligible internal resistance is connected
Solution. Here Ig =lmA =10-3 A, G = 10Ω to a 3 k Ω resistor. The potential drop
𝑉 across a part of the resistor (between
But 𝑅 + 𝐺 = 𝐼
𝑔 points A and B in Fig. 4.101) is measured
2V by (i) a 20 kΩ voltmeter; (ii) a 1 kΩ
𝑅1 + 10𝛺 = = 2000𝛺 ⇒ 𝑅1 voltmeter. In (iii) both the voltmeters are
10−3 A connected across AB. In which case would
= 1990𝛺
you get the (1) highest, (2) lowest reading?
20V
𝑅1 + 𝑅2 + 10𝛺 = = 20000𝛺
10−3 A
𝑅2 = 20000 − 2000 = 18000𝛺 = 18k𝛺
200V
𝑅1 + 𝑅2 + 𝑅3 + 10𝛺 = = 200,000𝛺
10−3 A
k
= 200 𝛺
𝑅3 = 200 − 18 = 2k𝛺
Example 103. A multirange currentmeter Fig. 4.101
can be constructed by using a
(b) Do your answers to this problem
galvanometer circuit as shown in the
alter if the potential drop across the entire
figure. We want a currentmeter that can
resistor is measured? What if the battery
measure 10 mA, 100 mA and 1A using a
has non-negligible resistance?
galvanometer of resistance 10Ω and that
produces maximum deflection for current [NCERT]
of l mA. Find S1, S2 and S3 that have to be
Solution, (a) The voltmeter, which has
used. [Example Problem]
maximum resistance, will draw minimum
current and allow maximum current to flow
through resistor AB. Consequently, there
will be maximum potential difference
across AB.
80
In case (i), resistance of voltmeter, the circuit (b) is suitable only for the
measurement of the small resistance X.
Rv =20 kΩ
We can justify the above arguments
In case (ii), Ry = 1 k Ω
quantitatively as follows :
1 1 1 1+20
In case (iii), = 20 + 1 = or 𝑅𝑉 = (i) Measurement of X. Let X = 5 Ω. In
𝑅𝑉 20
20 circuit (a), the ammeter reading is I, and
k𝛺
21 the voltmeter reading is I, (X + 0.5).
(1) As the resistance of voltmeter is Voltmeter reading 𝐼1 (𝑋 + 0.5)
maximum in case (i), it will show =
maximum reading. Ammeter reading 𝐼1
𝐼1 (5 + 0.5)
(2) As the resistance of voltmeter is = = 5.5𝛺
𝐼1
minimum in case (iii), it will show lowest
reading.
(b) In all cases, the voltmeter reading
will be same if the battery has negligible
internal resistance. But if the internal
resistance is non-negligible, then the
answers will be similar to those in (a).
Example 105. You are given two resistors Fig. 4.103
X and Y whose resistances are to be
determined using an ammeter of With circuit (a), the error in the
resistance 0.5 Ω and a voltmeter of measurement of X is 0.5 Ω.
resistance 20 kΩ. It is known that X is in the In circuit (b), the ammeter reading is I
range of a few ohms, while Y is the range and the voltmeter reading is XI1, (=20,000
of several thousand ohms. In each case, I2).
which of the two connections shown in Fig.
Clearly,
4.102 would you choose for resistance
measurement? Justify your answer 𝑋𝐼1 5
𝐼2 = = 𝐼
quantitatively. [NCERT] 20,000 20,000 1
Voltmeter reading
𝑋𝐼1 5𝐼1
Ammeter reading = =
𝐼 𝐼1 + 𝐼2
5𝐼1
=
5
𝐼1 + 𝐼
20,000 1
Solution. In circuit (a), the voltmeter V 5 × 20,000
= = 4.9987𝛺
will measure the sum of the potential drops 20,005
across the resistance and the ammeter. With circuit (b), the error in the
The value of the resistance determined measurement of X is 0.0013 Ω. This error
from these calculations will include the is much smaller than that obtained by using
resistance of the ammeter. This will not be circuit (a). Hence for measuring a
desirable if resistance is very small. So the resistance of few ohms, the circuit (b)
circuit (a) is suitable only for measurement should be used.
of large resistance Y.
(ii) Measurement of Y. Let Y = 20,000
In circuit (b), the ammeter will read the Ω. In circuit (a), we get
sum of currents flowing through the
resistance and the voltmeter V. The value Voltmeter reading 20,005𝐼1
=
of the resistance obtained by these Ammeter reading 𝐼1
calculations will be less than the actual = 20,005𝛺
value. The difference will increase with the
The error in the measurement of Y is 5 Ω.
increase in the value of the resistance. So
81
In circuit (b), we get across its terminals. Calculate the
Voltmeter reading 20,000𝐼2 resistance of the shunt if the current of
= 10 A is to be measured by it. The
Ammeter reading 𝐼 galvanometer has 25 divisions.
20,000𝐼2
= (Ans. 0.01 Ω)
𝐼1 + 𝐼2
20,000𝐼2 6. A galvanometer of resistance 40 Ω
= gives a deflection of 5 divisions per mA.
𝐼2 + 𝐼2
There are 50 divisions on the scale.
[∵ 20,000𝐼1 = 20,000𝐼2 ] Calculate the maximum current that
= 10,000 Ω can pass through it when a shunt
resistance of 2 Ω is connected. [IIT]
The error in the measurement of Y is
10/100 Ω, which is much larger than error (Ans. 210 mA)
obtained by using circuit (a). Hence for 7. It is intended to measure a maximum
measuring large resistance of several current of 25 A with an ammeter of
thousand ohms, the circuit (a) should be range 2.5 A and resistance 0.9 Ω. How
used. will you do it? What will be the
Problems for Practice combined resistance?
82
area of cross-section 0.2 mm2 required reading on the voltmeter? (Ans. 11.52
to make a shunt for this purpose. V)
HINTS
(Ans. 1.25 m)
13. A galvanometer gives a full scale 𝑔 𝑔𝐼 𝑅 0.04𝐼×96
1. 𝑅𝑆 = 𝐼−𝑅 = = 4𝛺
deflection with a current of 1 A. It is 𝑔 𝐼−0.04𝐼
83
𝑉 1 Current through the resistance
8. 𝐼𝑔 = 𝑅𝑔 = 100 = 0.01A
𝑔
R = 0.13-0.013 = 0.117 A
𝐼𝑔 0.01 × 100
𝑅𝑠 = × 𝑅𝑔 = = 0.5𝛺 Resistance,
𝐼 − 𝐼𝑔 2 − 0.01
𝑉𝑔 1
117V
9. 𝑅 = − 𝑅𝑔 or 900 = 𝐼 − 100 𝑅= = 1000𝛺
𝐼𝑔 𝑔
0.117A
1
Or 𝐼𝑔 =
1
A = 10−3 A 16. 𝐼𝑔 = 10 × 150mA = 15 × 10−3 A
1000
1
Current sensitivity 𝑉𝑔 = × 150mV = 75 × 10−3 V
2
𝐼𝑔 10−3 A
= = = 10−5 Adiv−1 𝑉𝑔 75 × 10−3
𝑛 100div 𝑅𝑔 = = = 5𝛺
𝐼𝑔 15 × 10−3
10. Here I ∞ 60 and I ∝ 10
(i) Required current range, 7 = 6 × 150 =
𝐼𝑔 10 1
= = and 𝑅𝑔 = 20𝛺 900 A
𝐼 60 6
𝐼𝑔 𝑅𝑔 15 × 10−3 × 5
𝐼𝑔 𝑅𝑠 𝑅5 = =
= 𝐼 − 𝐼𝑔 900 − 15 × 10−3
𝐼 𝑅𝑔 + 𝑅𝑆
= 8.33 × 10−5 𝛺
6𝑅S = 20 + 𝑅 or 𝑅s = 4𝛺
(ii) Required voltage range, V = 1 × 150 =
𝐼𝑔 𝑅𝑠 6 1
11. = = = or 2% of the 150 V.
𝐼 𝑅𝑔 +𝑅𝑠 294+6 50
total current passes through the 𝑉 150
𝑅= − 𝑅𝑔 = − 5 = 9995𝛺
galvanometer. 𝐼𝑔 15 × 10−3
𝐼
𝐼𝑔 𝑅𝑔 (10)×27 17. Proceed as in Example 100 on page
12. 𝑅S = 𝐼−𝐼 = 𝐼 = 3𝛺 4.68.
𝑔 𝐼−10
84
field an analogue to the source of 𝜇0 𝑁 × 2𝐼
𝐵′ = = 4𝐵
electric field ? [Haryana 92] 2 ⋅ (𝑅/2)
Solution. No. The source of electric i.e., magnetic field becomes four times the
field is an electric charge. The source of original field.
magnetic field is not a magnetic charge.
Instead, moving electric charges produce Problem 6. Consider the circuit
magnetic field. shown where APB and AQB are
semicircles. What will be the magnetic
Problem 3. How can it be shown that field at the centre C of the circular loop?
an electric current in a wire produces a
magnetic field around it?
[CBSE D 95]
Solution. Bring a magnetic needle near
the current carrying wire. The magnetic
field produced by the electric current will
deflect the magnetic needle from
equilibrium position in the north-south
direction.
Problem 4. State Biot-Savart law and
express this law in vector form. Fig. 4.106
[CBSE OD 17] Solution. Magnetic field at the centre C
due to the semicircular part APB,
Solution. According to Biot-Savart law,
𝜇0 𝐼
the magnetic field due to a current element 𝐵1 = directed normally inward
4𝜋𝑎
I dl at the observation point whose position
vector is r depends (i) directly on current Magnetic field at the centre C due to
the semicircular part AQB,
I, (ii) directly on length dl of the element, (iii)
𝜇 𝐼
directly on the sine of the angle between dl 0
𝐵2 = 4𝜋𝑎 , directed normally outward 4πα
and r and (iv) inversely on the square of the
distance r. As the two fields are equal and
opposite, the net magnetic field at the
𝜇0 𝐼 𝑑𝑙sin𝜃
Its magnitude is 𝑑𝐵 = centre C is zero.
4𝜋 𝑟 2
𝜇0 𝐼 ⃗⃗⃗⃗
𝑑𝑙 ×𝑟 Problem 7. Figure 4.107 shows a
⃗⃗⃗⃗⃗ =
In vector form, dB =𝑑𝐵 ⋅ 𝑟3
4𝜋 square loop made from a uniform wire.
If a battery is connected between the
where μ 0 is the permeability of free space.
points A and C, what will be the
The direction of dB is perpendicular to the
magnetic field at the centre of the
plane of dl and r and is given by right hand
square?
screw rule.
Problem 5. How will the magnetic
field intensity at the centre of a circular
coil carrying current change, if the
current through the coil is doubled and
the radius of the coil is halved?
[CBSE OD 01]
Solution. Magnetic field at the centre of
a circular coil is given by Fig. 4.107
𝜇0 𝑁𝐼 Solution. Current I divides equally at A.
𝐵= Magnetic fields due to the currents in wires
2𝑅
AD and BC will be equal and opposite.
When current 1 is doubled and radius R Also, the fields due to currents in wires AB
is halved, and DC will be equal and opposite. Hence
the resultant field at the centre will be zero.
85
Problem 8. Looking at the circular stationary charge when placed in a
coil, the current is found to be flowing magnetic field. Why?
in anticlockwise direction. Predict the
[Haryana Sample Paper 91 ; Punjab 01]
direction of magnetic field produced at
a point on the axis of the coil on the Solution. A stationary charge does not
same side as the observer. produce any magnetic field and so it does
not suffer any interaction against the
Solution. By clock rule, the magnetic
external magnetic field. Moreover,
field has N polarity at the given point. Thus
the direction of the magnetic field is F = qvB sin θ = q (0) B sin θ = 0.
perpendicular to the plane of the coil and is Problem 14. Write the expression for
directed towards the observer. Lorentz magnetic force on the particle
Problem 9. A current is set up in a of charge 'q' moving with velocity 𝑣 in a
long copper pipe. Is there a magnetic magnetic field 𝐵⃗ . Show that no work is
field (i) inside (ii) outside the pipe? done by this force on the charged
[CBSE D 95C] particle. [CBSE OD 11]
Solution, (i) The magnetic field is zero Solution. Lorentz magnetic force, 𝐹 =
inside the pipe. (ii) A finite magnetic field ⃗)
𝑞(𝑣 × 𝐵
exists outside the pipe.
As the magnetic force 𝐹 acts in a
Problem 10. What is the basic direction perpendicular to the direction of
difference between magnetic field and the velocity 𝑣 or the direction of motion of
electric field ? [Punjab 02] the charged particle, so work done is zero.
Solution. Whether a charged particle is W = 𝐹 ⋅ ⃗⃗⃗⃗
𝑑𝑠 = Fds cos 90°= 0
at rest or in motion, an electric field always
exerts a force on it and changes its speed Problem 15. A charged particle in a
and hence its kinetic energy. A magnetic plasma trapped in a 'magnetic bottle'
field exerts a force on a charged particle leaks out after a millisecond. What is
only when it is in motion. In a magnetic the total work done by the magnetic
field, there is no change in the speed or the field during the time the particle is
kinetic energy of the charged particle. trapped?
Problem 11. The force F experienced Solution. Work done is zero. Since a
by a particle of charge q moving with magnetic field exerts force perpendicular to
⃗ is given
velocity 𝑣 in magnetic field 𝐵 the direction of motion of the charged
⃗ particle, no work is done by it on the
by 𝐹 = 𝑞(𝑣 × 𝐵). Which pairs of vectors
charged particle.
are always at right angles to each
other? [CBSE D 93C] Problem 16. A particle of mass 'm'
and charge 'q' moving with velocity ‘v’
Solution. The vector 𝐹 is perpendicular enters the region of uniform magnetic
⃗.
to the plane of 𝑣 and 𝐵 Hence the field at right angle to the direction of its
⃗
pairs 𝐹 and 𝑣 ; and 𝐹 and 𝐵 are always at motion. How does its kinetic energy get
right angles to each other. affected? [CBSE D 15C]
Problem 12. In a chamber of uniform Solution. K.E. of the particle is not
magnetic field 𝐵 ⃗ , an electron beam affected because no work is done by the
enters with velocity 𝑣 . Write the perpendicular magnetic force acting on it.
expression for the force experienced by Problem 17. An electron moving with
the electron. a velocity of 107 ms-1 enters a uniform
[CBSE D 93C ; Haryana 02] magnetic field of 1T, along a direction
parallel to the field. What would be its
Solution. 𝐹 = −𝑒(𝑣 × 𝐵 ⃗ ), because the trajectory in this field?
charge on an electron is - e.
[CBSE Sample Paper 98]
Problem 13. No force is exerted by a
86
Solution. The electron will continue to particle affected?
follow its straight path because a parallel
[CBSE F 94]
magnetic field does not exert any force on
the electron. Solution. Magnetic force deflects the
charged particle continuously from its path,
Problem 18. In a certain
so its momentum changes due to the
arrangement, a proton (or an electron)
change in its direction of motion.
does not get deflected while moving
through a magnetic field region. Under Problem 23. Which one of the
what condition is it possible? following will experience maximum
force, when projected with the same
[CBSE OD 09]
velocity v perpendicular to the magnetic
Solution. Magnetic force on an electron or field : (i) α-particle, and (ii) β-particle ?
a proton is Fm = evB sin θ
When θ = 0° or 180°, F = 0 [CBSE D 02C]
Thus a proton or an electron moving Solution. Force, F = qvB sin 90° = qvB
parallel or antiparallel to a magnetic field
For α-particle, q = 2e, Fa = 2 evB
does not experience any force.
For β-particle, q = e, Fβ = evB
Problem 19. Under what condition is
the force acting on a charge moving Thus the α-particle will experience
through a uniform magnetic field maximum force.
maximum? Problem 24. Which one of the
[Himachal 02 ; CBSE D 07] following will describe the smallest
circle when projected with the same
Solution. Fm = qvB sin θ
velocity v perpendicular to the magnetic
When θ = 90°, F is maximum. Thus a field B : (i) α-particle, and (π) β-particle
charge experiences maximum force when ?
it moves perpendicular to the direction of
[CBSE D 02C]
the magnetic field.
𝑚𝑣
Problem 20. Under what condition is Solution. Radius, 𝑟 = 𝑞𝐵
the force acting on a charge moving 𝑟𝛼 𝑚𝛼 𝑣 𝑞𝛽 𝐵 𝑚𝛼 𝑞𝛽 4𝑚𝑝 ⋅ 𝑒
through a uniform magnetic field = ⋅ = =
minimum? 𝑟𝛽 𝑞𝛼 𝐵 𝑚𝛽 𝑣 𝑚𝛽 𝑞𝛼 𝑚𝑒 ⋅ 2𝑒
87
𝑚𝑣
able to tell which of the two fields Solution. Radius, 𝑟 = 𝑞𝐵
(electric or magnetic) has caused the
deflection? For same v and B,
Solution. If the path of the proton beam 𝑚𝛼 𝑣
𝑟𝛼 𝑞 𝐵 𝑚𝛼 𝑞𝑝 4𝑚𝑝 𝑞𝑝
is parabolic, the deflection is due to electric = 𝑚𝛼 𝑣 = × = × = 2: 1
field. If the path is circular or helical, the 𝑟𝑝 𝑝 𝑚𝑝 𝑞𝛼 𝑚𝑝 2𝑞𝑝
deflection is due to magnetic field. 𝑞𝑝 𝐵
Problem 27. As shown in Fig. 4.108, a Problem 30. An electron and a proton
charged particle enters a uniform are moving along the same direction
magnetic field and experiences an with the same kinetic energy. When
upward force as indicated. What is the they pass through a uniform magnetic
charge sign on the particle? field perpendicular to the direction of
their motion, they describe paths of the
[CBSE D 03C, 05] same radius. Is this statement true or
Solution. Applying Fleming's left hand false?
rule, we find that the particle has a positive [IIT 85]
charge.
Solution. False. Radius,
𝑚𝑣 𝑚 2𝐾 √2𝑚𝐾
𝑟= = √ =
𝑒𝐵 𝑒𝐵 𝑚 𝑒𝐵
88
are often twisted together to reduce
magnetic effect. What is the basis for
[Himachal 98C]
this procedure?
Solution. No, neutrons are electrically
Solution. Two equal and opposite
neutral. They cannot be accelerated by
currents produce equal and opposite
electric fields or deflected by magnetic
magnetic fields, so their magnetic effects
fields.
get cancelled.
Problem 34. Why is a cyclotron not
Problem 39. Two parallel wires
suitable for accelerating electrons?
carrying currents in the same direction
attract each other, while the two beams
[Punjab 02 ; Haryana 97] of electrons travelling in the same
direction repel each other. Give reason.
Solution. Electrons are very light
particles. Even a small increase in the
energy of the electron increases its speed [Punjab 95]
to a very large value. Due to the high
Solution. In case of parallel wires, only
speed, the electrons get quickly out of step
attractive magnetic interaction acts. In
with the oscillating electric field. Moreover,
case of electron beams, the electrostatic
these light particles require unusually high
repulsion is stronger than the attractive
frequencies (GHz).
magnetic interaction.
Problem 35. The frequency of
Problem 40. A stream of protons is
revolution of a charged particle in a
moving parallel to a stream of
cyclotron does not depend on the
electrons. Do the two streams tend to
speed of the particle. Why?
come closer or move apart?
Solution. The radius of the circular path
Solution. The behaviour of the two
of the charged particle increases in direct
streams depends on their speeds. If they
proportion to its speed. Consequently, both
have large speeds, they will move apart
its time period and frequency of revolution
because the repulsive magnetic interaction
are independent of its speed.
is stronger than the attractive electrostatic
Problem 36. What are the main interaction. If they have small speeds, they
functions of electric and magnetic will come closer because repulsive
fields in a cyclotron? magnetic interaction is weaker than the
Solution, (i) The electric field attractive electrostatic interaction.
accelerates the charged particle whenever Problem 41. An electron beam
it crosses the gap between the two dees. moving with uniform velocity is
(ii) The magnetic field makes the gradually diverging. As it is accelerated
to a high velocity, it starts converging.
charged particle to cross the gap between
the two dees again and again by providing Why does it happen so?
it a circular motion. Solution. When the electron beam has
a low velocity, the electrostatic repulsion
Problem 37. Why does a current
carrying conductor experience a force between the electrons is stronger than the
magnetic attraction between them. So the
in a magnetic field?
beam diverges. When the velocity
Solution. When a current flows, free becomes high, the magnetic attraction
electrons drift in the conductor in a definite between the electrons becomes stronger
direction. These moving electrons than electrostatic repulsion and the beam
experience a force in the magnetic field converges.
which gets transmitted to the conductor as
a whole. Problem 42. Why does a solenoid
contract when a current is passed
Problem 38. In electronics, two wires through it?
carrying equal and opposite currents
Solution. The current in adjacent turns
89
of the solenoid flows in the same direction. Solution. A galvanometer in which coil
So different turns attract one another and comes to rest at once, without suffering
the solenoid contracts. oscillations, on passing current through it is
called a dead beat galvanometer.
Problem 43. Does the torque on a
planar current loop in a magnetic field Problem 49. A neutron, an electron and
change, when its shape is changed an alpha particle moving with equal
without changing its area? velocities, enter a uniform magnetic field
going into the plane of the paper as shown
Solution. No. The torque on a planar
in Fig. 4.109(a). Trace their paths in the
current loop remains same as long as its
field and justify your answer.
area remains unchanged.
[CBSE D 16]
Problem 44. Why do we prefer
phosphor-bronze alloy for the
suspension wire of a moving coil
galvanometer?
Solution. The phosphor bronze alloy is
used for the suspension wire of a moving
coil galvanometer because of its following
properties :
(i) Small restoring torque per unit twist. Solution. Neutrons, being neutral, are
This makes galvanometer highly not deflected by the magnetic field. Alpha
sensitive. particles and electrons will tend to move
along circular paths in opposite directions
(ii) High tensile strength so that even thin as shown in Fig. 4.109(b), in accordance
fibre does not break under the weight of with Fleming's left hand rule.
the suspended coil.
Problem 50. Why is the coil wrapped on
(iii) Rust resisting. a conducting frame in a galvanometer?
Problem 45. What is the main Solution. Eddy currents set up in the
function of soft iron cylinder between conducting frame help in bringing the coil
the poles of a galvanometer? to rest at once i.e., help in making the
Solution. Due to the high permeability galvanometer dead beat.
of soft iron, the magnetic lines of force Problem 51 Why the earth's magnetic
crowd through the soft iron core. This field does not affect the working of a
increases the magnetic field and hence moving coil galvanometer?
sensitivity of the galvanometer.
Solution. This is because the earth's
Problem 46. Why are pole pieces of magnetic field is much weaker than the
galvanometer made concave? [Haryana strong radial magnetic field used in the
93, 94] galvanometer.
Solution. Concave poles produce Problem 52. Two wires of equal lengths
strong, uniform and radial magnetic field. are bent in the form of two loops. One of
Problem 47. What is the importance the loops is square shaped whereas the
of radial magnetic field in a moving coil other loop is circular. These are suspended
galvanometer? in a uniform magnetic field and the same
current is passed through them. Which
Solution. Radial magnetic field makes loop will experience greater torque? Give
the arm of the couple fixed and hence the reasons.
torque on the coil is always same in all
positions of the coil in the magnetic field. [CBSE OD 05]
This provides a linear current scale. Solution. For a wire of given length,
Problem 48. What is a dead beat area of the circular loop > area of the
galvanometer? square loop
90
So the circular loop will experience Solution. An ammeter is connected in
greater torque in the magnetic field series in a circuit so that whole of the
because, torque ∝ area of the loop. current, which it is required to measure,
passes through it. In order that its insertion
Problem 53. What is a shunt? State its
in the circuit does not affect the current in
SI unit.
the circuit, the ammeter must have least
[Haryana 96 ; Punjab 99] possible resistance.
Solution. A shunt means a small Problem 58. Why should the resistance
resistance connected in parallel with any of an ideal voltmeter be infinite?
component of an electrical circuit.
Solution. A voltmeter is connected in
However, it commonly refers to a low
parallel with the component across which
resistance connected in parallel to a
the p.d. is required to be measured. The
galvanometer. Its SI unit is ohm (Ω).
voltmeter must have very high resistance
• Problem 54. How can a moving coil so that a very small current passes through
galvanometer be converted into an it and the p.d. across that component is not
ammeter? affected. To measure the exact value of
p.d., an ideal voltmeter has to have infinite
[Himachal 98 ; Punjab 2000 ; CBSE OD 01]
resistance.
Solution. A galvanometer is converted
Problem 59. Why is an ammeter
into an ammeter by connecting a low
connected in series in a circuit?
resistance R$ (shunt) in parallel with the
galvanometer. If the full scale deflection of [CBSE OD 16]
the galvanometer occurs for current 𝐼𝑔 and
Solution. An ammeter is connected in
ammeter is to have range 0 -V, then IR series in a circuit so that whole of the
𝑉 current, which it is required to measure,
𝑅= − 𝑅𝑔 passes through it. Moreover, an ammeter
𝐼𝑔
has a low resistance, so its insertion in the
Problem 55. How can a moving coil series circuit does not practically change
galvanometer be converted into a the main current.
voltmeter?
Problem 60. Why is a voltmeter always
[Himachal 01 ; Punjab 99 ; CBSE OD 01] connected in parallel with a circuit element
Solution. A galvanometer is converted across which voltage is to be measured?
into a voltmeter by connecting a high
resistance R in series with it. If the full scale [CBSE OD 16]
deflection of galvanometer occurs for a
current Ig and the voltmeter is to have a Solution. A voltmeter is a high
range 0 - V, then resistance galvanometer. When it is
connected in parallel across any element
Problem 56. What information would of a circuit, it draws a very small current
you wish to know about the galvanometer from the main circuit. Most of the current
before converting it into an ammeter or passes through that element. Hence
voltmeter? potential difference across that component
Solution. For converting galvanometer is not affected materially.
into ammeter or voltmeter, we must know : Problem 61. What happens when an
(i) Resistance of the galvanometer (𝑅𝑔 ). ammeter is placed in parallel with a circuit?
Solution. An ammeter is a low
(ii) Current (𝐼𝑔 ) required to produce full
resistance device. When it is placed in
scale deflection in the galvanometer.
parallel with a circuit, the resistance
Problem 57. Why should an ammeter decreases and current in the circuit
have a low resistance? increases to a large extent. Moreover, it
measures the current flowing through it
[Haryana 02]
only and not the current in the circuit.
91
Problem 62. What happens when a resistance is smaller than that of the
voltmeter is connected in series in a galvanometer. The voltmeter is obtained
circuit? by connecting a high resistance in series
with the galvanometer, so its resistance is
Solution. A voltmeter is a high
higher than that of the galvanometer.
resistance device, when it is connected in
series with a circuit, the resistance of the Problem 67. A galvanometer is first
circuit becomes very high and the current converted into a voltmeter of range 0 - 3 V
in the circuit reduces to a very small value. and then into a voltmeter of range 0 - 6 V.
So voltmeter will not measure the actual In which case the resistance would be
value of potential difference. Moreover, the higher one?
voltmeter connected this way will not read
Solution. The voltmeter with range 0 - 6
the p.d. that we wish to measure across
V will have a higher resistance. This is
any particular component.
because a higher range voltmeter requires
Problem 63. A galvanometer gives full a higher resistance to be put in series with
scale deflection with the current 𝐼𝑔 . Can it the galvanometer.
be converted into an ammeter of range I < Problem 68. An ammeter and a
𝐼𝑔 ? milliammeter are converted from the same
Solution. The required value of shunt is galvanometer. Out of the two, which
𝐼 ×𝑅 current measuring instrument has a higher
given by 𝑅g = 𝑔𝐼−𝐼 𝑔
𝑔 resistance?
For I < 𝐼𝑔 , the above formula gives a [CBSE OD 02]
negative value for Rs. Solution. Milliammeter has higher
Hence it cannot be converted into an resistance because an ammeter requires a
ammeter of range I < Ig. shunt of smaller resistance.
Problem 64. Can we Problem 69. A voltmeter and a
increase/decrease the range of an millivoltmeter are converted from the same
ammeter? galvanometer. Out of the two, which
potential difference measuring device has
Solution. We can only increase the a higher resistance?
range of ammeter by connecting a suitable
resistance in parallel with the ammeter. We Solution. Voltmeter has a higher
cannot decrease the range of the ammeter resistance because it requires a larger
because for I < 𝐼𝑔 , the value of shunt resistance to be put in series with the
resistance becomes negative. galvanometer.
92
less than the actual current. It has a small ammeter or a milliammeter, has a higher
resistance. When it is connected in a resistance and why?
circuit, it decreases the current slightly.
[CBSE OD 06]
Problem 72. When a voltmeter is put 𝐼𝑔 ×𝑅𝑔
across a part of the circuit, does it read Solution. Shunt resistance, 𝑅s = 𝐼−𝐼𝑔
slightly less or more than the actual voltage
drop across that part? Give reason. Clearly, the shunt needed to convert
galvanometer into a milliammeter has a
Solution. Voltmeter reads slightly less larger value than that required to convert
than the actual voltage drop. When it is into an ammeter. As the shunt resistance is
connected across a part of the circuit, it connected in parallel with the
draws some current from that part. As a galvanometer, so the milliammeter will
result, potential drop across that part have a higher resistance than the
slightly decreases. ammeter.
Problem 73. Which one of the two, an
93
𝐵2 clockwise and in Q is anti-clockwise as
= 4: 1
𝐵1 seen from O which is equidistant from
the loops P and Q. Find the magnitude
Problem 4. Two identical circular loops, of the net magnetic field at O.
P and Q, each of radius r and carrying
currents I and 21 respectively are lying in Solution.
parallel planes such that they have a
common axis. The direction of current in
both the loops is clockwise as seen from O
which is equidistant from the both loops.
Find the magnitude of the net magnetic
field at point O. [CBSE D 12]
Fig. 4.113
𝜇0 𝐼𝑟 2 𝜇0 𝐼
⃗ 𝑃 | = |𝐵
|𝐵 ⃗ 𝑄| = =
2
2(𝑟 + 𝑟 ) 2 3 ⁄ 2
4√2𝑟
The net magnetic field at O is
Fig. 4.110 𝜇0 𝐼 𝜇0 𝐼
⃗ | = |𝐵
|𝐵 ⃗ 𝑃 | + |𝐵
⃗ 𝑄| = 2 =
Solution. 4√2𝑟 2√2𝑟
(Pointing towards P) Problem 6. Two
small identical circular loops, marked (1)
and (2), carrying equal currents, are placed
with the geometrical axes perpendicular to
each other as shown in Fig. 4.114. Find the
magnitude and direction of the net
magnetic field produced at the point O.
Fig. 4.111 [CBSE D 05, 08]
𝐼𝑟 2
⃗ 𝑃 | = 𝜇2 0 2 3⁄2 = 𝜇0𝐼 (Pointing towards
|𝐵 2(𝑟 +𝑟 ) 4√2𝑟
P)
2
⃗ 𝑄 | = 𝜇02(2𝐼)𝑟
|𝐵
𝜇0 𝐼
= 2√2𝑟 (Pointing
2(𝑟 +𝑟 2 )3⁄2
towards Q)
Net field at O is
⃗ | = |𝐵
|𝐵 ⃗ 𝑃 | = 𝜇0 𝐼
⃗ 𝑄 | − |𝐵 (Pointing
4√2𝑟
towards Q) Fig. 4.114
Problem 5. Two identical circular loops, Solution. The fields produced by the
P and Q, each of radius r and carrying two coils are shown in Fig. 4.115.
currents are kept in the parallel planes
having a common axis passing through
Fig. 4.112
O. The direction of current in P is
Fig. 4.115
94
Magnetic field at O due to each loop is 𝑒 𝑒 𝑒𝜔
𝐼= = =
𝑇 2𝜋/𝜔 2𝜋
𝜇0 𝐼𝑅2
⃗ 1 | = |𝐵
|𝐵 ⃗ 2| =
2(𝑥 2 + 𝑅2 )3⁄2 Hence the magnetic field at the centre
of the electron orbit is
⃗1 ⊥ 𝐵
As 𝐵 ⃗ 2 , so the net field at O is 𝜇0 𝑒𝜔 𝜇0 𝑒𝜔
𝐵= ⋅ =
𝜇0 𝐼𝑅2 √2 2𝑟 2𝜋 4𝜋𝑟
𝐵 = √𝐵12 + 𝐵22 =
2(𝑥 2 + 𝑅2 )3⁄2 Problem 9. Describe qualitatively the
path of a charged particle moving in :
√2𝜇0𝐼𝑅 2
For 𝑅 << 𝑥, 𝐵 = 2𝑥 3 (a) a uniform electrostatic field, with initial
Problem 7. Figure 4.116 shows a cube velocity
made from a uniform wire. Find the (i) parallel to the field,
magnetic field at the centre of the cube, if a
battery is connected between the points A (ii) perpendicular to the field,
and G. (iii) at an arbitrary angle with the field
direction.
(b) a uniform magnetic field, with initial
velocity
(i) parallel to (or along) the field, [CBSE
OD 95]
(ii) perpendicular to the field,
[CBSE D 93]
(iii) at an arbitrary angle with the field.
Fig. 4.116 [CBSE F 93]
Solution. Consider the conducting pairs (c) a region with uniform electrostatic and
AB and HG, AE and CG, AD and EG, BF magnetic fields parallel to each other,
and DH, EF and CD, EH and BC. The with initial velocity
currents in the two conductors of each pair
produce equal and opposite magnetic (i) parallel,
fields at the centre. Hence the resultant (ii) perpendicular,
magnetic field at the centrh will be zero.
(iii) at an arbitrary angle with the common
Problem 8. An electron revolves in a direction of the fields.
circular orbit of radius r with angular speed
Solution, (a) (i) The particle will move along
cα Using the expression for the magnetic
the line of force of the electrostatic field
field due to a circular current loop, deduce
and hence its path will be a straight
an expression for the magnetic field at the
line.
centre of the electron orbit.
(ii) When the initial velocity is
Solution. Magnetic field at the centre of
perpendicular to the electrostatic field,
a circular current loop is
the charged particle will follow a
𝜇0 𝐼 parabolic path.
𝐵=
2𝛾 (iii) Again the path of charged particle will
2𝜋 be parabolic. The initial velocity of the
Period of revolution of the electron, 𝑇 = 𝜔 charged particle has two components.
The component parallel to the field will
make it move along the field while the
The motion of the electron in a circular
perpendicular component will make it
orbit is equivalent to a current loop carrying
move along a parabolic path.
current,
(b) (i) Parallel magnetic field does not exert
95
any force on the moving charged kinetic energy of the particle in
particle. So the particle will follow a both cases.
straight path. (ii) [CBSE OD 07C]
(ii) Magnetic field exerts a force on the Solution, (i) The parallel magnetic field
charged particle perpendicular to both does not exert any force on the moving
the magnetic field and its direction of charged particle. Hence there is no change
motion. This force provides centripetal in the velocity or the kinetic energy of the
force to make the particle move in a charged particle.
circle in a plane normal to the magnetic
(ii) The magnetic force acts on the
field.
charged particle perpendicular to its
(iii) The charged particle will follow a helical direction of motion. Hence no work is done
path with its axis parallel to the field. and there is no change in the kinetic
The component of initial velocity energy of the particle.
parallel to the magnetic field will make
Problem 11. A charge 'q’ moving in a
the particle move along the direction of
straight line is accelerated by a potential
the field while the perpendicular
difference W. It enters a uniform magnetic
component will compel it to follow a
field 'B' perpendicular to its path. Deduce in
circular path. The net effect is the
terms of V an expression for the radius of
helical motion.
the circular path in which it travels. [CBSE
The speed is constant in each case. OD 2000]
(c) The total Lorentz force on a Solution. Gain in K.E. of the charge q
charged particle moving with velocity 𝑣 in a when accelerated through V volts is
region of electric field 𝐸⃗ and magnetic field 1 2𝑞𝑉
⃗ is given by
𝐵 𝑚𝑣 2 = 𝑞𝑉 ∴ 𝑣2 =
2 𝑚
𝐹 = 𝐹𝑒 + 𝐹𝑚 = 𝑞𝐸⃗ + 𝑞(𝑣 × 𝐵
⃗) As the perpendicular magnetic field
provides the centripetal force to the charge
(i) Here the charged particle is moving q, therefore
parallel to both electric and magnetic fields.
So the force due to the magnetic field (𝐹𝑚 ) 𝑚𝑣 2
𝑞𝑣𝐵sin90° =
is zero. Under the electric force 𝐹 ,, the 𝑟
charged particle continues to move along
its original straight line path. 𝑚𝑣 𝑚 2𝑞𝑉 2𝑚𝑉
𝑟= = ⋅√ =√ 2
𝑞𝐵 𝑞𝐵 𝑚 𝑞𝐵
(ii) Due to the perpendicular electric
field, the charged particle tends to move Problem 12. A charged particle of mass
along a parabolic path. The perpendicular 'm', charge ‘q’ moving at uniform velocity v,
magnetic field throws the particle into enters a uniform magnetic field 'B' acting
circular motion. The particle follows the normal to the plane of the paper. Deduce
resultant path. expression for the (i) radius of the circular
(iii) Resolve velocity 𝑣 along 𝐵 ⃗ and path in which it travels, (ii) kinetic energy of
⃗ . The transverse the particle (assuming v « c).
perpendicular to 𝐵
component throws the charged particle into [CBSE OD 01; D 17C]
⃗ (or 𝐸⃗ )
circular motion. The motion along 𝐵 Solution, (i) Force exerted by the
gets accelerated or retarded due to 𝐸⃗ . magnetic field = Centripetal force on the
Problem 10. A charged particle enters charge
a region of uniform magnetic field with its 𝑚𝑣 2
or 𝑞𝑣𝐵sin90° =
initial velocity directed 𝑟
𝑚𝑣
(i) parallel to the field, (ii) ∴ Radius, 𝑟 = 𝑞𝐵
perpendicular to the field. Show 𝑞𝐵𝑟
that there is no change in the (ii) Velocity, 𝑣 = 𝑚
96
1 1 𝑞𝐵𝑟 2 𝐵 2 𝑞2 𝑟2
∴ K.E. = 2 𝑚𝑣 2 = 2 𝑚( ) =
𝑚 2𝑚
97
may describe. current is passed through it. Why?
[CB
Solution. In the absence of electric
SE F 10]
field, the free electrons in a conductor have
random motion in all directions. The
velocity of free electrons is zero in any
direction. So the magnetic field does not
exert any force on them. When a current
flows, the free electrons acquire a drift
velocity in a definite direction, hence
magnetic force acts on them.
Fig. 4.120 (a) (b) Problem 19. As shown in Fig. 4.121, a
Solution. rectangular loop of width l is suspended
𝑚𝑣 𝑟𝛼 𝑚𝛼 𝑞𝑝 4 1 2 from the insulated hook of a spring
As 𝑟 = ∴ = ⋅ = ⋅ = balance. A current I flows in the
𝑞𝐵 𝑟𝑝 𝑚𝑝 𝑞𝛼 1 2 1
anticlockwise direction in the loop. A
Both particles move anticlockwise in magnetic field B exists in the lower region.
circular orbits with 𝑟𝛼 = 2𝑟 , as shown in What will be the change in the tension of
Fig. 4.120(b). the spring if the current in the loop is
reversed?
Problem 17. A proton and an alpha
particle enter at right angles into a uniform
magnetic field of intensity B . Calculate the
ratio of the radii of their paths, when they
enter the field with the (i) same momentum,
(ii) same kinetic energy.
[CBSE Sample Paper 03]
Solution, (i) Centripetal force on a charged
particle = Magnetic force on the particle
𝑚𝑣 2 𝑚𝑣 𝑝
Or = 𝑞𝑣𝐵sin90° or 𝑟= = 𝑞𝐵
𝑟 𝑞𝐵
Fig. 4.121
For same momentum p and field B,
Solution. When the current flows
1 𝑟𝑝 𝑞𝛼 2𝑒 anticlockwise, force on side PQ = IIB,
𝑟∝ ∴ = = = 2: 1
𝑞 𝑟𝛼 𝑞𝑝 𝑒 acting upwards.
1 When the current flows clockwise,
(ii) Kinetic energy, 𝐾 = 2 𝑚𝑣 2 or 𝑣=
force on side PQ = IIB, acting downwards.
2𝐾
√ ∴ Change in the tension of the spring = IIB
𝑚
+ IIB = 2 IIB.
𝑚𝑣 𝑚 2𝐾 √2𝑚𝐾
∴Radius, 𝑟 = 𝑞𝐵
= 𝑞𝐵 √ 𝑚 = 𝑞𝐵 Problem 20. Explain, why two wires
carrying currents in opposite directions
For same kinetic energy K and field B, repel each other.
98
Fig. 4.124
Solution. The windings of a helix carry
currents in the same direction, they exert
attractive forces pulling the lower end out
of mercury. Consequently, the circuit
breaks and the current and hence the force
of attraction disappears. The helix regains
its original condition with its lower end
dipping in mercury. The process repeats
Fig. 4.122 and oscillatory motion is set up.
Problem 21. Two parallel conducting Problem 23. Suppose a helical spring is
wires carrying currents in same direction suspended from the roof of a room and
attract each other. Why? very small weight is attached to its lower
end. What will happen to the spring when a
Solution. Each current carrying current is passed through it? Give reason
conductor produces a magnetic field to support your answer.
around it. So each conductor experiences
a force due to the magnetic field of the [CBSE Sample Paper 1998]
other. According to Fleming's left hand Solution. When a current is passed
rule, the direction of the force is through the spring, it will flow in the
perpendicular to the two wires and is adjacent turns of the spring in the same
inwards. Hence the two wires attract each direction. So there will be magnetic
other, as shown in Fig. 4.123. attraction between adjacent turns. The
spring will contract and the weight will be
lifted up.
Problem 24. A horizontal wire AB of
length T' and mass 'm' carries a steady
current Iv free to move in vertical plane is in
equilibrium at a height of 'h' over another
parallel long wire CD carrying a steady
current I2, which is fixed in a horizontal
plane as shown. Derive the expression for
Fig. 4.123 the force acting per unit length on the wire
AB and write the condition for which wire
Problem 22. A loosely wound helix AB is in equilibrium. [CBSE SP 18]
made of stiff wire is mounted vertically with
its lower end just touching a dish of
mercury, as shown in Fig. 4.124. When a
current from a battery is started in the coil
through the mercury, the wire executes
oscillatory motion with the lower end Fig. 4.125
jumping out of and into the mercury. Give
Solution. Magnetic field produced by
reason.
current I2 at any point on the wire AB is
𝜇0𝐼2
𝐵2 = , normally into the plane of paper
2𝜋ℎ
99
For equilibrium of wire AB, Weight per 𝜇0 12𝐼 2 6𝐼 2
unit = Magnetic force per length of AB 𝐹2 = 𝐹23 − 𝐹21 = ( − )
4𝜋 𝑟 𝑟
unit length 𝜇0 6𝐼 2
= ⋅
𝑚𝑔 𝜇0 𝐼1 𝐼2 4𝜋 𝑟
=
𝑙 2𝜋ℎ towards conductor 1.
Problem 25. Figure 4.126 shows three Problem 26. In Fig. 4.127, the straight
infinitely long straight parallel current wire AB is fixed while the loop PQRS is free
carrying conductors. Find the (i) magnitude to move under the influence of the electric
and direction of the net magnetic field at currents flowing in them. In which direction
point A lying on conductor 1, (ii) magnetic does the loop begin to move? Give reason
force on conductor 2. [CBSE F 17] for your answer.
[CBSE D 97 ; OD 99]
Fig. 4.126
Solution, (i) Magnetic field at point A
due to current 31 in conductor 2,
𝜇 2(3𝐼) 𝜇 6𝐼 Fig. 4.127
𝐵2 = 0 ⋅ = 0⋅ acting into the plane
4𝜋 8 4𝜋 𝑟 Solution. The currents in QR and PS
of the paper have no effect on AB. There is a force of
Magnetic field at point A due to current attraction between AB and PQ and a force
4I in conductor 3, of repulsion between AB and SR But PQ is
𝜇 2(4𝐼) 𝜇 8𝐼
closer to AB than SR, so force of attraction
𝐵2 = 4𝜋0 ⋅ 3𝑟 = 4𝜋0 ⋅ 3𝑟 acting out of the is stronger than the force of repulsion. So
plane of the paper the loop begins to move towards AB.
Net field at point A, Problem 27. Write the expression for
the magnetic moment (𝑚 ⃗⃗ ) due to a planar
𝜇0 6𝐼 8𝐼 𝜇0 10𝐼
𝐵𝐴 = 𝐵2 − 𝐵3 = ( − )= ⋅ square loop of side carrying a steady
4𝜋 𝑟 3𝑟 4𝜋 3𝑟 current I in a vector form.
acting into the plane of paper
As shown in Fig. 4.128, this loop is
(ii) Force exerted by conductor 1 on placed in a horizontal plane near a long
conductor 2, straight conductor carrying a steady
𝜇0 2(𝐼)(3𝐼) current J1 at a distance l. Give reasons to
𝐹21 = ⋅ explain that the loop will experience a net
4𝜋 𝑟 force but no torque. Write the expression
away from conductor 1/towards conductor for this force acting on the loop.
3. [CBSE D 10]
Force exerted by conductor 3 on conductor
2, μ0 2(37)(47)
𝜇0 2(3𝐼)(4𝐼)
𝐹31 = ⋅
4𝜋 2𝑟
away from conductor 3/towards conductor
1.
Net magnetic force on conductor 2, Fig. 4.128
Solution. Magnetic moment due to a
100
planar square loop of side I and carrying carrying current I, when placed in a
current I is given by 𝑚 = 𝐼𝐴 = 𝐼𝑙2 ⃗ , is given by 𝜏 =
magnetic field 𝐵
⃗⃗ = 𝐼𝑙2 𝑛̂
In vector form, 𝑚 𝐼(𝐴 × 𝐵⃗)
where 𝑛̂ is a unit vector normal to the plane Obviously, torque 𝜏 will become zero if
of the loop in the sense given by right hand the area vector 𝐴 is in the direction of
thumb rule. ⃗ . Hence in position of stable
external field 𝐵
Force of attraction on conductor AB, equilibrium, the current loop will orient itself
with its plane perpendicular to the direction
𝜇0 𝐼1 𝐼 𝜇0 𝐼1 𝐼 ⃗ (because the direction of 𝐴 is normal
𝐹𝑎 = ×𝑙= of 𝐵
2𝜋𝑙 2𝜋 to the plane of current loop).
Force of repulsion on wire CD, In this orientation, the magnetic field
𝜇0 𝐼1 𝐼 𝜇0 𝐼1 𝐼 produced by the loop is in the same
𝐹𝑟 = ×𝑙 = direction as external field, both normal to
2𝜋 × 2𝑙 4𝜋
𝜇0 𝐼1 𝐼 1 𝜇0 𝐼1 𝐼 the plane of the loop, thus giving rise to
𝐹net = 𝐹𝑎 − 𝐹𝑟 = (1 − ) = maximum flux of the total field.
2𝜋 2 4𝜋
(c) The loop will assume circular shape
As Fa > Fr, the net force on loop ABCD
with its plane perpendicular to the
is attractive.
magnetic field so as to maximise magnetic
As the area vector is parallel to the flux. This is because for a given perimeter,
magnetic field, θ = 0°, so torque on the a circle encloses greater area than any
square loop, other shape.
𝜏 = 𝐼𝐴𝐵SIN𝜃 = 𝐼𝐴𝐵SIN0° = 0 Thus the Problem 29. Why is the magnetic field
torque acting on the loop is zero. radial in a moving coil galvanometer?
Explain how it is achieved.
Problem 28. (a) A current carrying
circular loop lies on a smooth horizontal [CBSE OD 15C]
plane. Can a uniform magnetic field be set
Solution. Radial magnetic field. By
up in such a manner that the loop turns
using pole pieces of a magnet and placing
around itself (i.e., turns about the vertical
a soft iron cylindrical core between the
axis)?
concave poles, we get a magnetic field with
(b) A current carrying circular loop is its lines of force pointing along the radii of a
located in a uniform external magnetic circle. Such a field is called a radial field.
field. If the loop is free to turn, what is its The plane of a coil rotating in such a field is
orientation of stable equilibrium? Show that always parallel to the field, as shown in Fig.
in this orientation the flux of the total field 4.129.
(external field + field produced by the loop)
is maximum.
[CBSE F 13]
(c) A loop of irregular shape carrying
current is located in an external magnetic
field. If the wire is flexible, why does it
change to a circular shape?
Fig. 4.129
[NCERT]
Need of radial field. The current I
Solution, (a) No, that would require 𝜏 to through a galvanometer coil is given by
act in the vertical direction. But 𝜏 = 𝑘
⃗ ) and for the horizontal loop 𝐴 acts
𝐼(𝐴 × 𝐵 𝐼= 𝛼
𝑁𝐵𝐴sin𝜃
in the vertical direction, so 𝜏 acts in the
plane of the horizontal loop. Because of the presence of factor sin θ,
the deflection α of the galvanometer coil is
(b) The torque on a loop of area 𝐴 and not quite proportional to the current I, so
101
that the instrument is not a linear one. To the component across which the p.d. is
make its scale linear, the field is made measured. It must have very high
radial. Then θ = 90°, so that resistance so that a very small current
𝑘 passes through it and the p.d. across that
𝐼= 𝛼 or 𝐼 ∝ 𝛼 component is not affected. So a
𝑁𝐵𝐴 galvanometer is converted into a voltmeter
Problem 30. Describe the working by connecting a high resistance in series
principle of a moving coil galvanometer. with it.
Why is it necessary to use (i) a radial
Problem 32. Compare a voltmeter and
magnetic field and (ii) a cylindrical soft iron
an ammeter.
core in a galvanometer?
Solution.
Can a galvanometer as such be used
for measuring the current? Explain.
Ammeter Voltmeter
[CBSE D 17]
Solution. Principle of a moving coil 1. It is a low It is a high
galvanometer: A current-carrying coil resistance device resistance device
placed in a magnetic field experiences a used to measure used to measure
current dependent torque, which tends to electric current. potential difference.
rotate the coil and produces angular
deflection. 2. It is obtained by It is obtained by
connecting a low connecting a high
(a) Radial magnetic field maximizes the resistance in resistance in series
torque on the coil and provides linear parallel with a with a
current scale. galvanometer. galvanometer.
(b) A cylindrical soft iron core makes the
field radial. This increases the strength 3. It is always It is always
of the magnetic field and hence connected in connected in
increases the sensitivity of the series. parallel.
galvanometer.
4. It is not possible to It is possible to
A galvanometer as such cannot be decrease the range decrease the range
used to measure current in a circuit due to of a given of a given
two reasons: ammeter. voltmeter.
(i) A galvanometer can detect only small
currents and its coil is likely to be Problem 33. Define current sensitivity
damaged by currents of (mA/A) range. and voltage sensitivity of a galvanometer.
Increasing the current sensitivity may not
(ii) For measuring currents, it has to be necessarily increase the voltage sensitivity
first calibrated. of a galvanometer. Justify.
Problem 31 Why is it that while using a [CBSE OD 09,11]
moving coil galvanometer as a voltmeter a
high resistance in series is required Solution. Current sensitivity of a
whereas in an ammeter a shunt is used? galvanometer is the deflection produced
per unit flow of current while voltage
[CBSE OD 10, 12 ; D 13C] sensitivity is the deflection produced per
Solution. In order that the insertion of unit applied potential difference.
an ammeter in series in a circuit does not 𝛼 𝑁𝐵𝐴
Current sensitivity, 𝐼𝑠 = =
affect the current in the circuit, an ammeter 𝐼 𝑘
must have least possible resistance. So a 𝛼 𝛼 𝑁𝐵𝐴
Voltage sensitivity, 𝑉𝑠 = = =
galvanometer is converted into ammeter 𝑉 𝐼𝑅 𝑘𝑅
by connecting a small resistance in parallel If the current sensitivity is increased by
with it. increasing the number of turns N, the
A voltmeter is connected in parallel with resistance R will also increase. So, the
102
voltage sensitivity might not increase on [CBSE SP 2015]
increasing the current sensitivity.
Problem 34. The magnitude F of the
force between two straight parallel current
carrying conductors kept at a distance d
𝜇 𝐼 𝐼
apart in air is given by 𝐹 = 2𝜋0 1𝑑2 where I1
and I2 are the currents flowing through the
two wires. Use this expression, and the
sign convention that the : "Force of
attraction is assigned a negative sign and
Fig. 4.131
force of repulsion is assigned a positive
sign". Draw graphs showing dependence Solution. With key K2 open, the current
of F on I in the galvanometer will be
(i) I1 I2 when d is kept constant. 𝜀
𝐼=
𝑅 + 𝑅𝑔
(ii) d when the product I2I2 is maintained at
a constant positive value. When K2 is closed, the equivalent
(iii) d when the product I1 I2 is maintained at resistance of the parallel combination of S
𝑆𝑅
a constant negative value. and R is given 𝑅′ = 𝑆+𝑅𝑔
𝑔
[CBSE SP 15] The total current /' drawn from the battery
Solution. When I1 and I2 are like would now be
currents, the product I, I2 is +ve and F is an 𝜀
attractive (-ve) force. 𝐼′ =
𝑅 + 𝑅′
When I1 and I2 are unlike currents, the This current gets divided through S and
product I1 I2 is -ve and F is a repulsive (+ G in the inverse ratio of S and Rg.
ve) force.
Hence, the current I" through the
Now F ∝ I1 I2, for constant d galvanometer will be
1
and 𝐹 ∝ 𝑑, for constant I1 I2 𝑆 𝑆 𝜀
𝐼 ′′ = 𝐼′ = ⋅
𝑆 + 𝑅𝑔 𝑆 + 𝑅𝑔 (𝑅 + 𝑅′ )
Hence, the required graphs are of the
forms as shown below (Fig. 4.130). 𝑆. 𝐸 1 𝑆𝜀
= ⋅ =
𝑆 + 𝑅𝑔 𝑆𝑅𝑔 𝑅𝑆 + 𝑅𝑅𝑔 + 𝑆𝑅𝑔
[𝑅 + 𝑆 + 𝑅 ]
𝑔
𝐼 1 𝐸
𝐼 ′′ = = ⋅
𝑛 𝑛 𝑅 + 𝑅𝑔
𝐸 𝑆𝐸
(i) d = constant (ii) I2 = +ve constant (iii) =
𝑛(𝑅 + 𝑅𝑔 ) 𝑅𝑆 + 𝑅𝑅𝑔 + 𝑆𝑅𝑔
I↑I2 = -ve constant
or 𝑛𝑅𝑆 + 𝑛𝑆𝑅𝑔 = 𝑅𝑆 + 𝑅𝑅𝑔 + 𝑆𝑅𝑔
Fig. 4.130
or (𝑛 − 1)𝑅𝑆 = 𝑅𝑅𝑔 − (𝑛 − 1)𝑆𝑅𝑔
Problem 35. The current flowing in the
galvarn meter G when the key K2 is kept or (𝑛 − 1)𝑅𝑆 = 𝑅𝑔 [𝑅 − (𝑛 − 1)𝑆]8
open is I. On closing the key FC2, the
current in the galvanometer becomes I / n, Hence, the resistance of the galvanometer
(𝑛−1)𝑅𝑆
where n is an integer. Obtain an is 𝑅𝑔 = 𝑅−(𝑛−1)𝑆
expression for resistance Rg of the
(𝑛−1)𝑅𝑆
galvanometer in terms of R, S and n. To When R » S, we have, 𝑅𝑔 ≃ = (𝑛 −
𝑅
what form does this expression reduce 1)𝑆
when the value of R is very large as
compared to S?
103
HOTS Problems on Higher Order Thinking Skills
Problem 1. State Biot-Savart law. the X-axis with a velocity 𝑣 is subjected to
a uniform magnetic field B acting along the
A current 1 flows in a conductor placed
Z-axis as it crosses the origin O.
perpendicular to the plane of the paper.
Indicate the direction of the magnetic field
due to a small demerit 𝑑𝑙 at point P
situated at a distance r from the element as
shown in Fig. 4.132.
[CBSE D 09]
Fig. 4.134
(i) Trace its trajectory.
(ii) Does the charge gain kinetic energy as
it enters the magnetic field? Justify your
answer. [CBSE D 09]
Solution. (i) Charge c begins to move in
Fig. 4.132 a circular orbit in the XY-plane as shown in
Fig. 4.135.
Solution. For the statement of
Biot-Savart law, refer to point 2 of (ii) As the magnetic force acts
Glimpses. perpendicular to the velocity 𝑣 of the
charge q, it does no work and hence there
𝜇0 𝐼(𝑑𝑙 × 𝑟) is no gain in the kinetic energy of the
⃗ =
𝑑𝐵
4𝜋 𝑟3 charge q.
Fig. 4.135
Fig. 4.133 Problem 3. A long wire is bent into a
Current element 𝑑𝑙 is along circular coil of one turn and then into a
Z-direction and position vector 𝑟 is along circular coil of smaller radius having n
Y-direction, so turns. If the same current passes in both
the cases, find the ratio of the magnetic
𝜇0 𝐼(𝑑𝑙𝑘̂ × 𝑟𝑗̂) fields produced at the centres in the two
⃗ =
𝑑𝐵
4𝜋 𝑟3 cases.
𝜇0 𝐼𝑑𝑙 Solution. Let l be the length of the wire.
= (−𝑖̂)
4𝜋 𝑟 2 When the wire is first bent in the form of
one turn circular coil,
[𝑘̂ × 𝑗̂ = −𝑖̂]
𝑙
Hence the magnetic field at point P is 𝑙 = 2𝜋𝑟1 , or 𝑟1 =
,𝑁 = 1
directed along negative X-direction as 2𝜋
shown in Fig. 4.133. 𝜇0 𝑁𝐼 𝜇0 × 1 × 𝐼 𝜇0 𝜋𝐼
𝐵1 = = =
2𝑟1 2 × (𝑙/2𝜋) 𝑙
Problem 2. A charge ‘q' moving along
104
When the wire is bent in the form of n-turn
coil,
𝑙
𝑙 = 𝑛 × 2𝜋𝑟2 or 𝑟2 = ,𝑁 = 𝑛
2𝑛𝜋
𝜇0 𝑛𝐼 𝜇0 𝜋𝑛2 𝐼
𝐵2 = =
2(𝑙/2𝑛𝜋) 𝑙
𝐵1 1
= 2 = 1: 𝑛2
𝐵2 𝑛 Fig. 4.136
Problem 4. Show that the magnetic Clearly,
field along the axis of a current carrying 𝜇0 𝐼
circular coil of radius r at a distance x from 𝐵1 = 𝐵3 = ⋅ (sin𝛼 + sin𝛼)
the centre of the coil is smaller by the 4𝜋 𝑏 ⁄2
fraction 3x2/2r2 than the field at the centre But sin𝛼 = 𝑂𝐴 =
𝐴𝑃 𝑎 ⁄2
=
𝑎
of the coil carrying current. (√𝑎 2 +𝑏2 )⁄2 √𝑎 2+𝑏2
𝜇0 𝑁𝐼 𝜇 𝐼 2𝑎
Solution. 𝐵 centre = and 𝐵 axial = ∴𝐵1 = 𝐵3 = 4𝜋0 ⋅ 𝑏⁄2 ⋅
2𝑟 √𝑎 2+𝑏2
𝜇0𝑁𝐼𝑟 2
𝜇0 4𝐼 𝑎
2(𝑟 2 +𝑥 2)3⁄2 = ⋅ ⋅
4𝜋 𝑏 √𝑎 + 𝑏2
2
𝐵 axial 𝑟3 𝑟3 𝑥 2 −3/2
= 2 = [1 + ] Similarly,
𝐵 centre (𝑟 + 𝑥 2 )3⁄2 𝑟 3 𝑟2
𝜇0 4𝐼 𝑏
3 𝑥2 3 𝑥2 𝐵2 = 𝐵4 = ⋅ ⋅
= [1 − + ⋯ ] ≃ 1 − 4𝜋 𝑎 √𝑎 + 𝑏2
2
2 𝑟2 2 𝑟2
𝜇0 8𝐼 𝑎 𝑏
The fractional decrease in magnetic field is = ⋅ [ + ]
4𝜋 √𝑎 2 + 𝑏2 𝑏 𝑎
𝐵 centre − 𝐵 axial 𝐵 centre
=1− 𝜇0 8𝐼 𝑎2 + 𝑏2
𝐵 centre 𝐵 axial = ⋅ ⋅
4𝜋 √𝑎 2 + 𝑏 2 𝑎𝑏
3 𝑥2 3 𝑥2
= 1 − (1 − ) = 𝜇 8𝐼√𝑎 2+𝑏2
2 𝑟2 2 𝑟2 Or 𝐵 = 4𝜋0 ⋅ 𝑎𝑏
Problem 5. A rectangular loop of
Problem 6. Two semi-infinitely long
metallic wire is of length a and breadth b
straight current carrying conductors are
and carries current I. Find the magnitude of
held at right angles to each other so that
the magnetic field at the centre O of the
their common end lies at the origin, as
loop.
shown in Fig. 4.137. If both the conductors
Solution. Figure 4.136 shows the carry the same current I as shown, find the
rectangular loop ABCD carrying current I. magnetic field at the point P (a, b).
The magnetic field due to all the four sides
at the centre O will be directed normally
outward. Hence total magnetic field at O
will be
B=B1 + B2 + B3 + B4
105
𝜇0 𝐼 𝑎 𝐵2 =
𝜇0𝐼2
(directed outward)
= [ + 1] 2𝜋𝑦
4𝜋𝑏 √𝑎 + 𝑏2
2
As the net magnetic field at P is zero, so 𝐵1
Magnetic field at point P due to the
= 𝐵2 ,
conductor along y-axis is
𝜇0 𝐼1 𝜇0 𝐼2 𝐼2
𝜇0 𝐼 𝜋 = or 𝑦= ⋅𝑥
𝐵2 = [sin𝛽 + sin ] 2𝜋𝑥 2𝜋𝑦 𝐼1
4𝜋𝑎 2
𝜇0 𝐼 𝑏 Problem 8. As shown in Fig. 4.140, a
= [ + 1] cell is connected across two points A and B
4𝜋𝑎 √𝑎 + 𝑏2
2
of a uniform circular conductor. Prove that
According to right hand thumb rule, the magnetic field at its centre O will be
both B1 and B2, act normally into the plane zero.
of paper. Hence the resultant magnetic
[Punjab 99C]
field at point P is
B= B1 + B2
𝜇0 𝐼 1 𝑎 𝑏 1 1
= [ ( + ) + ( + )]
4𝜋 √𝑎 + 𝑏 𝑏 𝑎
2 2 𝑎 𝑏
𝜇0 𝐼 √𝑎 2 + 𝑏2 𝑎 + 𝑏
= [ + ]
4𝜋 𝑎𝑏 𝑎𝑏
𝜇0 𝐼
= [√𝑎 2 + 𝑏2 + (𝑎 + 𝑏)] Fig. 4.140
4𝜋𝑎𝑏
Problem 7. Two insulating infinitely Solution. Let the lengths of the two
long conductors carrying currents 1 ↑ and l2 circular segments ACB and ADB be l1 and
lie mutually perpendicular to each other in l2, and p be the resistance per unit length.
the same plane, as shown in Fig. 4.138. Then
Find the locus of the point at which the net Resistance of segment ACB, 𝑅1 = 𝑙1 𝜌
magnetic field is zero.
Resistance of segment ADB, = 𝑅2 =
Fig. 4.138
𝑙2 𝜌
Solution. According to right hand thumb
rule, the magnetic fields due to the two Suppose f and I2 are the currents in
conductors can vanish only in regions 1 segments ACB and ADB respectively. As
and III. the two segments are connected in
parallel, so the potential differences across
Let the magnetic field be zero at point P them must be equal.
(x, y).
∴ 𝐼1 𝑅1 = 𝐼2 𝑅2
or
or 𝐼1 𝑙1 𝜌 = 𝐼2 𝐿2 𝜌 𝐼1 𝐿1 = 𝐼2 𝑙2
Magnetic field at the centre of an arc of
length l carrying current I is
𝜇0 𝐼𝑙
𝐵=
4𝜋𝑟 2
Therefore, the magnetic fields at the
centre O due to the currents I1 and I2 are
Fig. 4.139
𝜇0 𝐼1 𝑙1 𝜇0 𝐼2 𝑙2
Magnetic field at P due to current 𝐼1 , 𝐵1 = and 𝐵2 =
4𝜋𝑟 2 4𝜋𝑟 2
𝜇0𝐼1
𝐵1 = (directed inward) where r is the radius of the circular
2𝜋𝑥
conductor. As 𝐼1 𝑙1 = 𝐼2 𝑙2 ,so
Magnetic field at P due to current I2,
B1 = B2
106
As the currents and I2 are oppositely 𝜙1 = 𝜋/2, 𝜙2 = 0, 𝑎 = 𝑟/2
directed, their magnetic fields B1 and B2 will
The magnetic field at point C due to this
be opposite to each other. Hence the
part is
resultant field at the centre O is zero.
𝜇0 𝐼 𝜋
Problem 9. An infinitely long conductor 𝐵3 = ⋅ (sin + sin0)
is bent into the shape as shown in Fig. 4𝜋 (𝑟/2) 2
4.141. The semicircular part lies in the 𝜇 2𝐼
= 4𝜋0 ⋅ acting along positive X-axis 4πr
YZ-plane; while of the two straight parts, 𝑟
one is along X-axis and the other parallel to 𝜇0 2𝐼
⃗3 =
𝐵 ⋅ 𝑖̂
Y-axis at a distance r from it. If the 4𝜋 𝑟
conductor carries a current I, as shown in
Total magnetic field at point C,
the figure, find the magnetic field at the
centre C of the semicircular loop. ⃗ =𝐵
𝐵 ⃗1 +𝐵
⃗2+𝐵
⃗3
𝜇0 2𝐼 𝜇0 2𝜋𝐼 𝜇0 2𝐼
=− ⋅ 𝑗̂ + ⋅ 𝑖̂ + ⋅ 𝑖̂
4𝜋 𝑟 4𝜋 𝑟 4𝜋 𝑟
𝜇 2𝐼
𝐵⃗ = 0 ⋅ [(𝜋 + 1)𝑖̂ − 𝑗̂]
4𝜋 𝑟
Problem 10. The wire loop PQRSP
formed by joining two semicircular wires of
radii R} and R, carries a current I, as shown
Fig. 4.141 in Fig. 4.142. Find the magnitude and
direction of the magnetic field at the centre
Solution. Magnetic field due to a O.
straight current carrying conductor,
[IIT 88]
𝜇0 𝐼
𝐵= (sin𝜙1 + sin𝜙2 )
4𝜋𝑎
For the straight part lying along X-axis,
we have
𝜙1 = 𝜋/2, 𝜙2 = 0, 𝑎 = 𝑟/2
:. The magnetic field at point C due to
the straight part along X-axis, Fig. 4.142
𝜇0 𝐼 𝜋
𝐵1 = (sin + sin0) Solution. The magnetic field at C due to
4𝜋(𝑟/2) 2 each of the straight parts PQ and RS is
𝜇 2𝐼 zero.
= 4𝜋0 ⋅ acting along negative Y-axis 4πr
𝑟
Magnetic field at C due to the semicircular
or ⃗ 1 = − 𝜇0 ⋅ 2𝐼 𝑗̂
𝐵 loop of radius R1 is directed normally
4𝜋 𝑟
upward.
The semicircular part subtends an
angle π at the centre C and its radius is r/2. 1 𝜇0 𝐼 𝜇0 𝐼
𝐵1 = ⋅ =
Magnetic field due to this part at point C, 2 2𝑅1 4𝑅1
𝜇0 𝐼 Magnetic field at C due to the
𝐵2 = 𝜋 semicircular loop of radius R2 is
4𝜋 (𝑟/2)
𝜇 2𝜋𝐼 1 𝜇0 𝐼 𝜇0 𝐼
= 4𝜋0 ⋅ acting along positive X-axis 4π r 𝐵2 = ⋅ =
𝑟 2 2𝑅2 4𝑅2
𝜇0 2𝜋𝐼
⃗2 =
𝐵 ⋅ 𝑖̂ directed normally downward.
4𝜋 𝑟
Hence the resultant field at the point C
For the straight part lying parallel to is
Y-axis, we have
107
𝜇0 𝐼 1 1
𝐵 = 𝐵1 − 𝐵2 = [ − ]
4 𝑅1 𝑅2
directed normally upward.
Problem 11. A current I is flowing in an
infinitely long conductor bent into the
shape shown in Fig. 4.143. If the radius of
the curved part is R, find the magnetic field
at the centre O.
Fig. 4.144
⃗ =𝐵
∴𝐵 ⃗ 𝐴𝐵 + 𝐵
⃗ 𝐶𝐷
𝜇 𝐼 𝜇 𝐼
0
or 𝐵 = 4𝜋𝑅 (sin90° + sin0° ) + 4𝜋𝑅
0
(sin90° +
Fig. 4.143
sin0° )
Solution. As the point O lies on the 𝜇 𝐼
0
straight part AB, so = 2𝜋𝑅 normally outward.
BAB =0 Problem 13. If the current density in a
0𝜇 𝐼 3𝜋 linear conductor of radius a varies with r
𝐵𝐵𝐶𝐷 = 4𝜋𝑅 ⋅ acting normally outward according to the relation : J = kr2, where k is
2
𝜇0 𝐼 𝜇0 𝐼 a constant and r is the distance of a point
𝐵𝐷𝐸 = (sin90° + sin0° ) = from the axis of the conductor. Find the
4𝜋𝑅 4𝜋𝑅
magnetic field at a point distance r from the
acting normally outward axis when (i) r < a and (ii) r > a.
Total magnetic field at the centre O,
𝐵 = 𝐵𝐴𝐵 + 𝐵𝐵𝐶𝐷 + 𝐵𝐷𝐸
𝜇0 𝐼 3𝜋 𝜇0 𝐼
=0+ ⋅ +
4𝜋𝑅 2 4𝜋𝑅
0𝜇 𝐼 3𝜋
or 𝐵 = 4𝜋𝑅 ( 2 + 1) , acting normally
outward. Fig. 4.145
Problem 12. Two infinitely long wires Solution, (j) As shown in Fig. 4.145,
carry equal current I. Each wire follows a consider an elementary ring of radius r and
90° arc along the circumference of the thickness dr with its centre on the axis of
same circle of radius R, as shown in Fig. the conductor.
4.144. Find the magnetic field at the centre Area of the elementary ring, 𝑑𝐴 = 2𝜋𝑟𝑑𝑟
O.
Current passing through the elementary
⃗ =𝐵
Solution, 𝐵 ⃗ 𝐴𝐵 + 𝐵
⃗ 𝐵𝐶 + 𝐵
⃗ 𝑂𝐷 + 𝐵
⃗ 𝐸𝐹 + ring,
⃗ 𝐹𝐺 + 𝐵
𝐵 ⃗ 𝐺𝐻
𝑑𝐼 = 𝐽𝑑𝐴 ≐ 𝑘𝑟 2 . 2𝜋𝑟𝑑𝑟 = 2𝜋𝑘𝑟 3 𝑑𝑟
⃗ 𝐵𝐶 = −𝐵
But 𝐵 ⃗ 𝐹𝐺 𝑎𝑛𝑑 ⃗ 𝐸𝐹 = 𝐵
𝐵 ⃗ 𝐺𝐻 =
Total current enclosed by the closed loop
0 of radius r,
𝑟
𝜋𝑘𝑟 4
𝐼 = ∫ 𝑑𝐼 = ∫ 2𝜋𝑘𝑟 3 𝑑𝑟 =
0 2
Using Ampere's circuital law,
𝐵𝐿 = 𝜇0 𝐼
108
𝜋𝑘𝑟 4 of 1.0 × 10-9 C and going in the X-Y plane.
𝐵 × 2𝜋𝑟 = 𝜇0 × Find the velocity of the particle.
2
𝜇0 𝑘𝑟 3 Solution. Here 𝐵⃗ = 4.0 × 10−3 𝑘̂T,
𝐵=
4 𝐹 = (4.0𝑖̂ + 3.0𝑗̂) × 10−10 N, 𝑞 = 10−9 C
(ii) For r > a, the total current enclosed Let 𝑣 = 𝑣𝑥 𝑖̂ + 𝑣𝑦 𝑗̂. Then
by the loop of radius r,
𝑎
⃗)
𝐹 = 𝑞(𝑣 × 𝐵
𝜋𝑘𝑎 4
𝐼=∫ 2𝜋𝑘𝑟 3 𝑑𝑟 = (4.0𝑖̂ + 3.0𝑗̂) × 10−10
0 2
𝜋𝑘𝑎 4 = 10−9 [(𝑣𝑥 𝑖̂ + 𝑣𝑦 𝑗̂) × (4.0 × 10−3 𝑘̂)]
𝐵 × 2𝜋𝑟 = 𝜇0
2 = 4 × 10−12 [𝑣𝑥 𝑖̂ × 𝑘̂ + 𝑣𝑦 𝑗̂ × 𝑘̂ ]
𝜇0 𝑘𝑎 4 4.0𝚤̂ +3.0𝑗̂
𝐵= Or r = [−𝑣𝑥 𝑗̂ + 𝑣𝑦 𝑖̂]
4𝑟 4×10−2
109
any two) that these observations are in
good agreement with the expected
theoretical variation of B with x.
Fig. 4.147
Also, 𝐵𝑌 = 𝐵𝑋
0 𝜇 𝐼′ 𝜇0 𝐼
Or 2(𝑅/2) = Fig. 4.149
2𝑅
110
are released from the centre of the 2𝐾
cyclotron and made to accelerate. ∴ Velocity, 𝑣 = √ 𝑚
(i) Can both be accelerated at the same Radius of the circular path of a charged
cyclotron frequency? Give reason to particle in a magnetic field is given by
justify your answer.
(ii) When they are accelerated in turn, 𝑚𝑣 𝑚 2𝐾 √2𝑚𝐾
𝑟= = ⋅√ =
which of the two will have higher 𝑞𝐵 𝑞𝐵 𝑚 𝑞𝐵
velocity at the exist slit of the dees?
√2𝑚𝑝 𝐾 √2𝑚𝑑 𝐾
[CBSE OD 13] ∴𝑟𝑝 = , 𝑟𝑑 =
𝑞𝑝 𝐵 𝑞𝑑 𝐵
𝑞𝐵
Solution. Cyclotron frequency, 𝑓𝑐 = 2𝜋𝑚 √2𝑚𝛼 𝐾
and 𝑟𝛼 =
𝑒𝐵 𝑞𝛼 𝐵
𝑓𝑐 ( protons ) =
2𝜋𝑚𝑝 𝑟𝑝 𝑞𝑑 𝑚𝑝 𝑞𝑝 𝑚𝑝 1
= √ = √ =
2𝑒𝐵 𝑒𝐵 𝑟𝑑 𝑞𝑝 𝑚𝑑 𝑞𝑝 2𝑚𝑝 √2
𝑓𝑐 (𝛼 − particles ) = =
2𝜋 × 4𝑚𝑝 4𝜋𝑚𝑝
Hence 𝑟𝑝 : 𝑟𝑑 : 𝑟𝛼 = 1: √2: 1
Hence, α-particles and protons will not
accelerate at the same cyclotron Problem 20. A neutron, a proton, an
frequency. In fact, cyclotron frequency for electron and an a-particle enter a region of
protons is twice the cyclotron frequency of constant magnetic field with equal
α-particles. velocities. The magnetic field is along the
inward
Also, maximum velocity acquired by
accelerated particles is given by
𝑚𝑣02 𝑞𝐵𝑟0
= 𝑞𝑣0 𝐵 or 𝑣0 =
𝑟 𝑚
𝑒𝐵𝑟0
𝑣0 ( proton ) =
𝑚𝑝
2𝑒𝐵𝑟0 𝑒𝐵𝑟0
𝑣0 (𝛼 − particle ) = =
4𝑚𝑝 2𝑚𝑝
Fig. 4.150
Hence, the exit velocity of protons is
twice the exit velocity of α-particles. normal to the plane of the paper. The
tracks of the particles are labelled in Fig.
Problem 19. A proton, a deutron and 4.150. Relate the tracks to the four
an alpha particle having the same kinetic particles.
energy are allowed to pass through a [
uniform magnetic field perpendicular to IIT 84]
their direction of motion. Compare the radii
of their circular paths. Solution, (i) For a neutron, q = 0, so F =
qvB sin θ = 0. That is, a neutron goes
[AIEEE 12] undeflected through the magnetic field. So
Solution. If mp, md and mα are the the track C corresponds to a neutron.
masses of proton, deutron and α-particle (ii) According to Fleming's left hand
respectively, then rule, a negative charged particle such as
md = 2 mp and ma = 4 mp an electron will be deflected towards right.
and So the track D corresponds to an electron.
𝑞𝑑 = 𝑞𝑝 𝑞𝛼 = 2𝑞𝑝
(iii) According to Fleming's left hand
Kinetic energy, rule, a positively charged particle such as
1 an α-particle or a proton will be deflected
𝐾= 𝑚𝑣 2 towards left. Its radius of curvature is given
2
111
by 𝑚𝑣 2
= 𝑞𝑣𝐵
𝑚𝑣 𝑚 𝑟
𝑟= i.e., 𝑟∝
𝑞𝐵 𝑞 𝑚𝑣 𝑚𝐸 𝐸
𝑟= = 2
[∵ 𝑣 = ]
𝑟𝛼 𝑚𝛼 𝑞𝑝 4𝑚𝑝 𝑒 2 𝑞𝐵 𝑞𝐵 𝐵
= ⋅ = ⋅ =
𝑟𝑝 𝑚𝑝 𝑞𝛼 𝑚𝑝 2𝑒 1 𝑚𝑣 𝑚𝐸 𝑟
𝑟′ = ′ = 2 =
𝑟𝛼 = 2𝑟𝑝 i.e., 𝑟𝛼 > 𝑟𝑝 𝑞𝐵 𝑞𝑛𝐵 𝑛
Let B' = nB, be the new magnetic field
Thus the track B corresponds to an
in region II. If 1 is the radius of the new
a-particle and track A to a proton.
circular path, then
Problem 21. A fine pencil of β 𝑚𝑣 𝑚𝐸 𝑟
-particles, moving with a speed v, enters a 𝑟′ = ′ = 2 =
region (region I), where a uniform electric 𝑞𝐵 𝑞𝑛𝐵 𝑛
and a uniform magnetic field are both Hence, the radius of the circular path
present. These β -particles then move into would decrease by a factor n.
region II where only the magnetic field (out
of the two fields present in region I), exists. Problem 22. Two long wires carrying
The path of the β -particles, in the two currents L and I2 are arranged as shown in
regions, is as shown in Fig. 4.151. Fig. 4.152. The one carrying current f is
along the x-axis. The other carrying current
f is along a line parallel to the y-axis given
by x = 0 and z-d. Find the force exerted at
02 because of the wire along the X-axis.
[Exemplar Problem]
Fig. 4.151
(i) State the direction of the magnetic field.
(ii) State the relation between 'E' and 'B' in
region 1.
(iii) Drive the expression for the radius of
the circular path of the β -particle in
region II.
Fig. 4.152
If the magnitude of magnetic field, in
region II, is changed to n times its earlier Solution. At 02, the magnetic field due
value, (without changing the magnetic field to f is along the y-axis.
in region I) find the factor by which the The second wire also lies along y-axis
radius of this circular path would change. and hence the force is zero.
[CBSE Sample Paper 13]
𝐹 = 𝐼2 𝑑𝐵1 sin0° = 0
Solution, (i) As per Fleming's left hand
rule, the magnetic field acts perpendicular Problem 23. Find the magnitude of the
to the plane of paper and is directed force on each segment of the wire as
inwards. shown in Fig. 4.153, if a magnetic
112
field of 0.30 T, is applied parallel to AB and 𝐼1 = 𝐼2 = 20A
DE. Take the value of the current, flowing
2 × 10−7 × 20 × 20 × 0.10
in the wire as 1 ampere. 𝐹𝑚 =
𝑟
[CBSE Sample Paper 2011]
8.0 × 10−6 N
Solution. As segments AB and DE are =
𝑟
parallel to the magnetic field B,
As the currents in AB and CD are in
𝐹𝐴𝐵 = 𝐹𝐷𝐸 = 0 [∵ 𝐹 = 𝐼𝑙𝐵SIN𝜃] opposite directions, so magnetic force on
𝐹𝐵𝐶 = 𝐼𝑙𝐵𝐶 𝐵sin90° CD will be repulsive and hence upward.
The gravitational force on CD acting
= 1× 0.08×0.30×1 = 0.024 N vertically downward is
FCD = IlCD B sin(90 °- 60 0) Fs = mg= 0.5 × 10-3 × 9.8 = 4.9 × 10-3 N
= 1× 0.10×0.30×0.5 =0.015 N. In the state of equilibrium,
Problem 24. A straight wire, of length L, 8.0 × 10−6
carrying a current I stays suspended = 4.9 × 10−3
𝑟
horizontally in mid air in a region where
there is a uniform magnetic field 𝐵 ⃗ . The 8.0 × 10−6
𝑟= = 1.63 × 10−3 m
linear mass density of the wire is 𝜆 Obtain 4.9 × 10−3
the magnitude and direction of this = 1.63 mm.
magnetic field. [CBSE OD 04C]
Problem 26. A metal zoire of mass m
Solution. The field B must act slides without friction on two horizontal rails
horizontally and perpendicular to the length spaced at a distance d apart as shown in
of the wire. Fig. 4.155. The rails are situated in a
Weight of wire = Force exerted by uniform magnetic field B, directed vertically
magnetic field mg = ILB sin 90° upward, and a battery is sending a current I
through them. Find the velocity of the wire
𝜆𝐿𝑔 = 𝐼𝐿𝐵 as a function of time, assuming it to be at
𝜆𝑔 rest initially.
𝐵=
𝐼 [Roorkee 83]
Problem 25. In Fig. 4.154, a long zvire
AB is placed on a table. A wire CD, 10 cm
long, is arranged just above AB so that it
can slide up and dozvn on two vertical
wires. The wires carry a current 20 A. At
what distance above AB, will the wire CD
settle? The mass of the wire CD is 0.50 g.
Fig. 4.155
Solution. The magnitude of the
magnetic force on the wire of length d is
F = IBd sin 90° = IBd
This force acts perpendicular to the
Fig. 4.154 wire (in the plane of paper), towards left.
The wire moves towards left under the
Solution. Suppose the wire CD settles acceleration,
at distance r above the wire AB: ‘The
magnetic force on CD of length 10 cm due 𝐹 𝐼𝐵𝑑
𝑎= =
to current in AB is 𝑚 𝑚
𝜇0 𝐼1 𝐼2 𝑙 Velocity of the wire at any time t will be
𝐹𝑚 = ⋅
4𝜋 𝑟
Here l = 10 cm = 0.10 m,
113
𝐼𝑑𝐵
𝑣 = 𝑢 + 𝑎𝑡̅ = 0 + ⋅𝑡
𝑚
𝐼𝐵𝑑𝑡
𝑣= , towards left.
𝑚
Problem 27. A given rectangular coil
OLMN of area A, carrying a given current J,
is placed in a uniform magnetic 𝐵 ⃗ = 𝐵𝑘̂
Fig. 4.157
field in two different orientations (a) and (b)
as shown. What is the magnitude of torque Magnetic dipole moment of the coil,
experienced by this coil in the two cases?
𝑚 = 𝐼𝐴 = 0.1 × √3 × 10−4
= √3 × 10−5 Am2
[CBSE Sample Paper 2011]
Magnetic moment m acts
perpendicular to the plane of the loop and
the field B. Therefore, torque acting on the
coil is
𝜏 = 𝑚𝐵sin𝜃 = √3 × 10−5 × 5 × 10−2
× sin90°
5√3 × 10−7 Nm
Problem 29. A length L of a wire carries
a current I. Show that if the wire is formed
Fig. 4.156
into a circular coil, the maximum torque in a
Solution. Here given magnetic field is developed when the
𝑚 = 𝐼𝐴 and 𝜏=𝑚 ⃗
⃗⃗ × 𝐵 coil has one turn only and the maximum
torque has the value, 𝜏 = 𝐿2 𝐼𝐵/4𝜋
(a) 𝜏 = 𝐼𝐴𝑖̂ × 𝐵𝑘̂ = −𝐵𝐼𝐴𝑗̂
Solution. Torque on a current loop, 𝜏 =
∴ Magnitude of torque = BIA 𝑁𝐼𝐵𝐴SIN𝜃
⃗ ) × 𝐵𝑘̂ = ⃗0
(b) 𝜏 = 𝐼𝐴(−𝑘 If the circular coil has N turns, each of
radius r, then L = 2πr N L
∴ Magnitude of torque = 0.
𝐿
Problem 28. A coil in the shape of an 𝑟=
2𝜋𝑁
equilateral triangle of side 0.02 m is
suspended from a vertex such that it is Area of the coil,
hanging in a vertical plane between the 𝜋𝐿2 𝐿2
pole pieces of permanent magnet 𝐴 = 𝜋𝑟 2 = =
4𝜋 2 𝑁 2 4𝜋𝑁 2
producing a horizontal magnetic field 5 ×
10“ 2 tesla. Find the couple acting on the 𝐿2 𝐿2𝐼𝐵sin𝜃
Hence 𝜏 = 𝑁𝐼𝐵 ⋅ 4𝜋𝑁2 ⋅ sin𝜃 = 4𝜋𝑁
coil when a current of 0.1 A is passed
through it and the magnetic field is parallel Clearly, torque will be maximum when
to its plane. sin θ is maximum and N is minimum i.e.,
sin θ =1 and N =1. Then
[Roorkee 91]
𝐿2 𝐼𝐵
Solution. Area of the triangle, 𝜏max =
1 1
4𝜋
A = 2 base × height = 2 a × a sin 60° Problem 30. In the circuit shown in Fig.
1 √3 4.158, a voltmeter reads 30 V, when it is
= × 0.02 × 0.02 × = = √3 × 10-4m2 2 connected across 400 Ω resistance.
2 2
Calculate what the same voltmeter will
read, when connected across the 300 Ω
resistance. [IIT 80; Punjab 95]
114
Fig. 4.158
Solution. Let Ry be the resistance of the
voltmeter. Then the resistance R of the
parallel combination of RV and R1 will be
𝑅𝑉 × 𝑅1 𝑅𝑉 × 400 400𝑅𝑉
𝑅= = =
𝑅𝑉 + 𝑅1 𝑅𝑉 + 400 𝑅𝑉 + 400
∴ Current in the circuit, [CBSE Sample Paper 03]
𝜀 60 Fig. 4.159
𝐼= =
𝑅2 + 𝑅 300 + 400𝑅𝑉 Solution. Apparent resistance,
𝑅𝑉 + 400
Voltmeter reading 𝑉
60(𝑅𝑉 + 400) 𝑅′ = =
= Ammeter reading 𝐼
700𝑅𝑉 + 120000
𝐸 1
As the potential difference across R is = 𝑅𝐴 +
𝐼 1 1
measured to be 30 V, so +
𝑅 𝑅𝑉
IR =30 Also, 𝐸 = 𝐼𝑅𝐴 + 𝑉
60(𝑅𝑉 + 400) 400𝑅𝑉 𝐸 𝑉
× = 30 or = 𝑅𝐴 + = 𝑅𝐴 + 𝑅′
700𝑅𝑉 + 120000 𝑅𝑉 + 100 𝐼 𝐼
1
On solving, or 𝑅𝐴 + 1 1 = 𝑅𝐴 + 𝑅′
+
𝑅 𝑅 𝑉
RV =1200 Ω 1 1 1
or = +𝑅
When this voltmeter is connected 𝑅′ 𝑅 𝑉
across R2, the combined resistance will be 1 1 1
or = −
𝑅 𝑅′ 𝑅𝑉
𝑅𝑉 × 𝑅2 1200 × 300
𝑅′ = = = 240𝛺
𝑅𝑉 + 𝑅2 1200 + 300 Problem 32. A galvanometer is
shunted by 1/nth of its resistance. Find the
Current in the circuit, fraction of the total current passing through
𝐸 60 3 the galvanometer.
𝐼′ = = = A
𝑅′ + 𝑅2 240 + 400 32 𝐼𝑔 𝑅𝑠
1
𝑅 1
𝑛 𝑔
Solution, = = 1 = 𝑛+1
Reading of the voltmeter = P.D. across 𝐼 𝑅𝑔 +𝑅𝑠 𝑅𝑔 +𝑛𝑅𝑔
R' = I' R'
Problem 33. If a galvanometer is
3 connected in series with a high resistance
= × 240 = 22.5V
32 so that potential drop across the
Problem 31. A voltmeter of resistance galvanometer is 1/nth of the total applied
RV and an ammeter of resistance RA are voltage, then show that the combined
connected in a circuit to measure, a resistance of the galvanometer and the
resistance R as shown in Fig. 4.159. The series resistor is n times the resistance of
ratio of the meter readings gives an the galvanometer i.e., RV = nRg.
apparent resistance R'. Show that R and R' 1
Solution. Here 𝑉𝑔 = 𝑛 𝑉 or 𝑉 = 𝑛𝑉𝑔
are related by the relation
1 1 1 Required Resistance,
= ′− 𝑛𝑉𝑔
𝑅 𝑅 𝑅𝑉 𝑉
𝑅′ = − 𝑅𝑔 = − 𝑅𝑔 = 𝑛𝑅𝑔 − 𝑅𝑔
𝐼𝑔 𝐼𝑔
𝑅𝑉 = 𝑅′ + 𝑅𝑔 = 𝑛𝑅𝑔
Combined resistance of the
galvanometer and the series resistor,
115
R
V = R' + Rg = nRg 𝑅2 ) …(ii)
Problem 34. A galvanometer of. On dividing (i) by (ii)
resistance G is converted into a voltmeter 𝐺 + 𝑅1
to measure upto V volts by connecting a 2= or 2𝐺 + 𝑅2 = 𝐺 + 𝑅1
resistance R1 in series with the coil. If a 𝐺 + 𝑅2
resistance R2 is connected in series with it, ∴ 𝐺 = 𝑅1 − 2𝑅2
then it can measure upto V/2 volts. Find the
resistance, in terms of R1 and R2, required Let R3 be the resistance required for
to be connected to convert it into a conversion into voltmeter of range 2 V.
voltmeter that can read upto 2 V. Also find Then,
the resistance G of the galvanometer in 2𝑉 = 𝐼𝑔 (𝐺 + 𝑅3 ) ...(iii)
terms of R1 and R2. [CBSE D 15]
On dividing (iii) by (i), we get
Solution.
𝐺 + 𝑅3
For voltmeter of range V, 𝑉 = 2= or 2𝐺 + 2𝑅1 = 𝐺 + 𝑅3
𝐺 + 𝑅1
𝐼𝑔 𝐺 + 𝑅1 ) … (𝑖)
(
∴𝑅3 = 𝐺 + 2𝑅1 = (𝑅1 − 2𝑅2 ) + 2𝑅1
𝑉
For voltmeter of range 𝑉/2, 2 = 𝐼𝑔 (𝐺 + = 3R1-2R2
116
4.6. A 3.0 cm wire carrying a current of 10 magnetic field of magnitude
A is placed inside a solenoid
0.80 T. What is the magnitude of torque
perpendicular to its axis. The magnetic
experienced by the coil ?
field inside the solenoid is given to be
0.27 T. What is the magnetic force on Ans. Given A= 0.10 m × 0.10 m = 0.01
the wire? m2, N = 20,
Arts. Given / = 3.0 cm = 0.03 m, / = 10 A, θ I = 12 A, θ = 30°, B = 0.80 T
= 90°, B = 0.27 T Magnitude of torque is t = NIB A sin θ
F = IIB sin θ = 10 × 0.03 × 0.27 × sin 90° = 20 × 12 × 0.80 × 0.01 ×sin 30°= 0.96 Nm.
= 8.1 ×10-2 N 4.10. Two moving coil galvanometers
The direction of the force is given by 𝑀1 and 𝑀2 have the following
Fleming's left hand rule. particulars :
4.7. Two long and parallel straight wires A R1=10Ω, N1 = 30, A1 = 3.6 × 10-3 m2, B1
and B carrying currents of 8.0 A and 5.0 = 0.25 T
A in the same direction are separated
R2 = 14 Ω, N2 = 42, A2 = 1.8 × 10-3m2, B2
by a distance of 4.0 cm. Estimate the = 0.50 T
force on a 10 cm section of wire A.
The spring constants are identical for
Ans. Force per unit length of each wire the two springs. Determine the ratio of (i)
is current sensitivity and (ii) voltage
𝜇0𝐼1 𝐼2 4𝜋×10−7×8×5 sensitivity of M2 and M1
𝑓= = = 2 × 10−4 Nm−1
2𝜋𝑟 2𝜋×4×10−2
Ans. Let torsion constant for each
Force on 10 cm section of wire A is meter = k
𝐹 = 𝑓𝑙 = 2 × 10−4 × 10 × 10−2 For a galvanometer, we have
= 2 × 10−5 N
NIB A = ka
4.8. A closely wound solenoid 80 cm
long has 5 layers of windings of400 Its current sensitivity is defined as the
turns each. The diameter of the deflection produced per unit current, i.e.,
solenoid is 1.8 cm. 𝛼 𝑁𝐵𝐴
=
𝐼 𝑘
If the current carried is 8.0 A, estimate the
Current sensitivity of 𝑀
magnitude of 𝐵 ⃗ inside the solenoid near its ∴ Current sensitivity of 𝑀2
1
centre.
𝑁2 𝐵2 𝐴2 /𝑘 𝑁2 𝐵2 𝐴2
Ans. Number of turns per unit length of = =
𝑁1 𝐵1 𝐴1 /𝑘 𝑁1 𝐵1 𝐴1
the solenoid is
42 × 0.50 × 1.8 × 10−3 7
n= = = = 1.4
Number of turns per layer × Number of layers 30 × 0.25 × 3.6 × 10−3 5
Length of solenoid Voltage sensitivity of a galvanometer is
400 × 5 defined as the deflection produced per unit
= = 2500m−1 𝛼 𝛼 𝑁𝐵𝐴
0.80 voltage, i.e., = =
𝑉 𝐼𝑅 𝑘𝑅
Magnetic field inside the solenoid is Voltage sensitivity of 𝑀2 𝑁2 𝐵2 𝐴2 /𝑘𝑅2
=
𝐵 = 𝜇0 𝑛𝐼 = 4𝜋 × 10−7 × 2500 × 8 = Voltage sensitivity of 𝑀1 𝑁1 𝐵1 𝐴1 /𝑘𝑅1
8𝜋 × 10−3 T = 2.5 ×10-2 T. 𝑁2 𝐵2 𝐴2 𝑅1 7 10
4.9. A square coil of the side 10 cm = × = × =1
𝑁1 𝐵1 𝐴1 𝑅2 5 14
consists of 20 turns and carries a
current of 12 A. The coil is suspended 4.11. In a chamber, a uniform magnetic
vertically and normal to the plane of the field of 6.5 G(1 G = 10-4T) is
coil and makes an angle of 30° with the maintained. An electron is shot into the
direction of a uniform horizontal field with a speed of 4.8 × 106 ms-1
117
normal to the field. Explain why the Ans. (a) N = 30, r = 8.0 cm = 0.08 m, I =
path of the electron is a circle. 6.0 A,
Determine the radius of the circular
B = 1T, 0 = 60°
orbit. Given that e = 1.6 × 10-19 C, me =
9.1 × 10-31 kg. Magnitude of counter torque
Ans. The perpendicular magnetic field = Magnitude of deflecting torque = NIB A
exerts a force on the electron sin θ
perpendicular to its path. This force = 30 × 6 × 1 × (3.14 × 0.08 × 0.08) sin 60°
continuously deflects the electron from its
path and makes it move along a circular = 30 × 6 × 3.14 × 64 × 10 -4 × 0.866 = 3.1
path. Nm.
∴ Magnetic force on the electron = (b) No, the answer would not change
Centripetal force because the above formula for the torque
is true for a planar loop of any shape.
𝑚𝑒 𝑣 2
𝑒𝑣𝐵sin90° = 4.14. Two concentric circular coils X
𝑟
and Y of radii 16 cm and 10 cm
𝑚𝑒 𝑣
𝑟= respectively lie in the same vertical plane
𝑒𝐵 containing the north-south direction. Coil X
Now 𝐵 = 6.5G = 6.5 × 10−4 T, 𝑣 = has 20 turns and carries a current of 16 A;
4.8 × 106 ms−1 coil Y has 25 turns and carries a current of
18 A. The sense of the current in X is
9.1 × 10−31 × 4.8 × 106 anticlockwise, and in Y clockwise, for an
𝑟=
1.6 × 10−19 × 6.5 × 10−4 observer looking at the coils facing west.
= 4.2 × 10−2 m = 4.2cm Give the magnitude and direction of the net
magnetic field due to the coils at their
4.12. In Exercise 4.11, obtain the centre.
frequency of revolution of the electron
in its circular orbit. Does the answer
depend on the speed of the electron?
Explain.
Ans. Frequency of revolution of the
electron in its circular orbit,
𝑒𝐵 1.6 × 10−19 × 6.5 × 10−4
𝑓= =
2𝜋𝑚 2 × 3.14 × 9.1 × 10−31
= 18.18 ×106 Hz. = 18 MHz.
No, the frequency f does not depend on
the speed v of the electron.
4.13. (a) A circular coil of 30 turns and Fig. 4.160
radius 8.0 cm carrying a current of 6.0 Ans. For coil X:rx = 16 cm = 0.16 m, Nx =
A is suspended vertically in a uniform 20, Ix = 16 A
horizontal magnetic field of magnitude
1.0 T. The field lines make an angle 60° ∴ Magnetic field at the centre of coil X is
with the normal to the coil. Calculate 𝜇0 𝐼𝑥 𝑁𝑥 4𝜋 × 10−7 16 × 20
the magnitude of the counter torque 𝐵𝑥 = = × T
2𝑟𝑥 2 0.16
that must be applied to prevent the coil
from turning. = 4π ×10-4T
(b) Would your answer change if the As the current in the coil X is
circular coil in (a) were replaced by a anticlockwise, the field is directed towards
planar coil of some irregular shape that east.
encloses the same area? For coil V: r = 10 cm = 0.10 m, N = 25,1
[CBSE OD 98C]
118
= 18 A distance R. Show that the field on the axis
around the mid-point between the coils is
∴ Magnetic field at the centre of coil Y
uniform over a distance that is small as
is compared to R, and is given by
𝜇0 𝐼𝑦 𝑁𝑦 4𝜋 × 10−7 18 × 25 𝜇0 𝑁𝐼
𝐵𝑦 = = × T 𝐵 = 0.72 , approximately
2 ⋅ 𝑟𝑦 2 0.10 𝑅
= 9𝜋 × 10−4 T Such an arrangement used to produce
As the current in the coil Y is clockwise, a nearly uniform magnetic field over a
the field By is directed towards west. Since small region is known as Helmholtz coils.
By > Bx, therefore, the net field is directed Ans. (a) Given
towards west and its magnitude is
𝜇0 𝐼𝑅2 𝑁
𝐵 = 𝐵𝑦 − 𝐵𝑥 = 5𝜋 × 10−4 = 1.6 × 10−3 T 𝐵=
2(𝑥 2 + 𝑅2 )3⁄2
4.15. A magnetic field of 100 G (1 G =
10-4T) is required which is uniform in a At the centre of the coil, x = 0, so
region of near dimension about 10 cm 𝜇0 𝐼𝑅2 𝑁 𝜇0 𝐼𝑁
and area of cross-section about 10-3m2. 𝐵= =
2𝑅3 2𝑅
The maximum current-carrying
capacity of a given coil of wire is 15 A This is the standard result for the field
and the number of turns per unit length at the centre of the coil.
that can be wound round a core is at (b) As shown in Fig. 4.161, consider a
most 1000 turns m-1. Suggest some small region of length 2d about the
appropriate design particulars of a midpoint O between the two coils.
solenoid for the required purpose.
Assume the core is not ferromagnetic.
Ans. Here B = 100 G = 10-2 T, I = 15 A,
n = 1000 turns m-1
Magnetic field inside a solenoid,
𝐵 = 𝜇0 𝑛𝐼
𝐵 10−2
∴𝑛𝐼 = 𝜇 = 4𝜋×10−7 = 7955 ≈ 8000 Fig. 4.161
0
𝜇 𝐼𝑁𝑅 2
We may take I = 10 A, then n = 800 Given 𝐵 = 2(𝑅20+𝑥 2 )3⁄2
The solenoid may have length 50 cm Therefore, the magnetic field at the point P
and area of cross- section 5 × 10_2m2 (five due to coil 1, μ∩m2N
times the given value) so as to avoid edge
effects, etc. 𝐵1
𝜇0 𝐼𝑅2 𝑁
4.16. For a circular coil of radius R and = , acting along 𝑃𝑂2
𝑅
N turns carrying current I, the magnitude of 2[𝑅2 + ( + 𝑑)2 ]3⁄2
2
the magnetic field at a its axis at a distance
x from its centre is given by, Magnetic field at the point P due to coil 2,
𝜇 𝐼𝑅 2 𝑁 𝐵2
𝐵 = 2(𝑥 20+𝑅2)3⁄2 𝜇0 𝐼𝑅2 𝑁
= , acting along PO2
(a) Show that this reduces to the 2 𝑅 2 3 ⁄ 2
2[𝑅 + ( 2 − 𝑑) ]
familiar result for field at the centre of the
coil. Total magnetic field at the point P will be B
=
(b) Consider two parallel co-axial
circular coils of equal radius R, and number 𝜇0 𝐼𝑅2 𝑁 1
= [ 2
of turns N, carrying equal currents in the 2 𝑅
(𝑅2 + 4 + 𝑑 2 + 𝑅𝑑)3⁄2
same direction, and separated by a
119
1 ∴ Number of turns per unit length,
+ ]
𝑅2 3500
(𝑅2 + 4 + 𝑑 2 − 𝑅𝑑)3⁄2
𝑛=
51.0 × 10−2 𝜋
𝜇0 𝐼𝑅2 𝑁 1 (a) The field outside the toroid is zero.
= 3 ⁄2
2 5𝑅2 (b) The field inside the core of the
[ 4 + 𝑅𝑑)
( toroid,
3500
1 𝐵 = 𝜇0 𝑛𝐼 = 4𝜋 × 10−7 × × 11
+ 51.0 × 10−2 𝜋
3 ⁄2
5𝑅2 = 3.02 × 10−2 T
( 4 − 𝑅𝑑) ]
(c) The field in the empty space
[Neglecting d2, as d << R]
surrounded by the toroid is also zero.
𝜇0 𝐼𝑅2 𝑁 1 4.18. Answer the following questions :
= 2 [
2(
5𝑅 3⁄2
) 4 𝑑 3 ⁄2 (a) A magnetic field that varies in
4 (1 + ) magnitude from point to point but has a
5𝑅
constant direction (east to west) is set
1 up in a chamber. A charged particle
+ ]
4 𝑑 3 ⁄2 enters the chamber and travels
(1 − 𝑅) undeflected along a straight path with
5
3 3 constant speed. What can you say
𝜇0 𝐼𝑁 4 3⁄2 4𝑑 −2 4𝑑 −2 about the initial velocity of the particle?
= ( ) [(1 + ) + (1 − ) ]
2𝑅 5 5𝑅 5𝑅 (b) A charged particle enters an
environment of a strong and
𝜇0 𝐼𝑁 4 3⁄2 6𝑑 6𝑑 non-uniform magnetic field varying
= ( ) [(1 − ) + (1 + )]
2𝑅 5 5𝑅 5𝑅 from point to point both in magnitude
[Expanding by binomial theorem and and direction and comes out of it
neglecting higher powers of d / R] following a complicated trajectory.
Would its final speed equal the initial
𝜇0 𝐼𝑁
or 𝐵 ≃ 0.72 speed if it suffered no collisions with the
2𝑅
environment ?
Magnetic field will also be same at the
point Q. In fact, it will be uniform over the (c) An electron travelling west to east
small region of length 2 d around the enters a chamber having a uniform
midpoint O. electrostatic field in a north to south
direction. Specify the direction in which
4.17. A toroid has a core a uniform magnetic field should be set
(non-ferromagnetic) of inner radius 25 up to prevent the electron from
cm and outer radius 26 cm around deflecting from its straight line path.
which 3500 turns of a wire are wound. If
the current in the wire is 11 A, what is Ans. (a) The force on a charged particle
the magnetic field (a) outside the toroid moving in a magnetic field is given by
(b) inside the core of the toroid (c) in the F = qvB sin θ
empty space surrounded by the toroid?
The force on a charged particle will be
Ans. Here, I = 11 A, total number of zero or the particle will remain undeflected
turns = 3500 if sin θ = 0 or θ = 0°, 180°
Mean radius of toroid, i.e., initial velocity v is either parallel or
25 + 26 antiparallel to B .
𝑟= = 25.5cm = 25.5 × 10−2 m
2 (b) Yes, a magnetic field exerts force
Total length (circumference) of the toroid on a charged particle in a direction
= 2𝜋 × 25.5 × 10−2 = 51.0 × 10−2 𝜋m perpendicular to its direction of motion and
120
hence does no work on it. So the charged 1
𝑣⊥ = 𝑣sin30° = 2.65 × 107 ×
particle will have its final speed equal to its 2
initial speed. = 1.33 × 107 ms−1
(c)The electron travelling west to east 𝑣‖ = 𝑣cos30° = 2.65 × 107 × 0.866
experiences a force towards north due to = 2.3 × 107 ms−1
the electrostatic field. It will remain
undeflected if it experiences an equal force The radius of the helical path is given
towards south due to the magnetic field. by
According to Fleming's left hand rule, the 𝑚𝑣⊥ 𝑚𝑣sin30°
magnetic field must act in the vertically 𝑟= =
𝑒𝐵 𝑒𝐵
downward direction. 9.1 × 10−31 × 1.33 × 107
=
4.19. An electron emitted by a heated 1.6 × 10−19 × 0.15
cathode and accelerated through a = 50.4 × 10-5 m = 0.50 mm.
potential difference of 2.0 kV, enters a
region with uniform magnetic field of 4.20. A magnetic field set up using
0.15 T. Determine the trajectory of the Helmholtz coils is uniform in a small
electron if the field (i) is transverse to its region and has a magnitude of 0.75 T.
initial velocity, (ii) makes an angle of In the same region, a uniform
30° with the initial velocity. electrostatic field is maintained in a
direction normal to the common axis of
Ans. V = 2.0 kV = 2 × 103 V, B = 0.15 T, the coils. A narrow beam of
e = 1.6 × 10-19 C, m = 9.1 × 10-31 kg (single-species) charged particles all
Potential difference V imparts kinetic accelerated through 15 kV enters this
energy to the electron given by region in a direction perpendicular to
both the axis of the coils and the
1
𝑚𝑣 2 = 𝑒𝑉 electrostatic field. If the beam remains
2 undeflected when the electrostatic field
or, Velocity gained by electron, is 9.0 × 10 5 Vm-1, make a simple guess
as to what the beam contains. Why is
2𝑒𝑉 the answer not unique?
𝑣=√
𝑚 Ans B = 0.75 T, E = 9.0 × 105 Vm-1,
2 × 1.6 × 10−19 × 2 × 103 V'= 15 kV = 15 × 103 V
=√ ms−1
9.1 × 10−31 For undeflected beam, velocity of
charged particles must be
= 2.65 × 107 ms-1
𝐸 9.0 × 105
(i) When field B is transverse to the 𝑣= = ms−1 = 12 × 105 ms−1
𝐵 0.75
initial velocity 𝑣,
But the kinetic energy of the charged
𝑚𝑣 2 particles is given by
𝑒𝑣𝐵sin90°
=
𝑟
1
𝑚𝑣 9.1 × 10 −31 × 2.65 × 107 𝑚𝑣 2 = 𝑞𝑉
𝑟= = m 2
𝑒𝐵 1.6 × 10−19 × 0.15
𝑞 1 𝑣 2 1 (12 × 105 )2
≃ 10−3 m = 1mm = ⋅ = × Ckg−1
𝑚 2 𝑉2 2 15 × 103
Thus the electron follows a circular = 4.8 × 107 C kg-1
trajectory of radius 1 mm normal to the field
B. Now for deutrons,
121
proton and one neutron. The answer is not long and 1.5 cm apart? Is the force
unique because we have determined only attractive or repulsive?
the ratio of charge to mass. Other possible
[Haryana 01]
answers are He2+ and Li3+, etc.
Ans. 𝐼1 = 𝐼2 = 300A, 𝑟 = 1.5cm =
4.21 A straight horizontal conducting
1.5 × 10−2 m l = 70 cm = 0.70 m
rod of length 0.45 m and mass 60 g is
suspended by two vertical wires at its ends. The force per unit length between the
A current of 5.0 A is set up in the rod wires is
through the wires.
𝜇0 𝐼1 𝐼2 4𝜋 × 10−7 × 300 × 300
(a) What magnetic field should be set 𝑓= = Nm−1
2𝜋𝑟 2𝜋 × 1.5 × 10−2
up normal to the conductor in order that the = 1.2Nm−1
tension in the wires is zero?
Total force between the wires,
[CBSE D 15C]
F= f×l = 1.2 × 0.70 = 0.84 N
(b) What will be the total tension in the
wires if the direction of current is reversed, As the currents in the two wires are in
keeping the magnetic field same as opposite directions, the force is repulsive.
before? (Ignore the mass of the wires) g = 4.23. A uniform magnetic field of 1.5 T
9.8 ms-2. exists in a cylindrical region of radius 10.0
Ans Here l = 0.45 m, m - 60 g = 0.06 kg, cm, its direction being parallel to the axis
along east to west. A wire carrying current
7 = 5.0 A, g = 9.8 ms-2 of 7.0 A in the north to south direction
(a) Tension in the supporting wires will passes through this region. What is the
be zero when the weight of the rod is magnitude and direction of the force on the
balanced by the upward force IIB of the wire if
magnetic field. (i) the wire intersects the axis,
i.e., I IB = mg (ii) the wire is turned from N-S to north
𝑚𝑔 0.06 × 9.8 east or north west direction,
𝐵= = T = 0.26T
𝐼𝑙 5 × 0.45 (iii) the wire in the N-S direction is lowered
from the axis by a distance of 6.0 cm ?
Ans. Here B = 1.5 T, I = 7.0 A (i) As shown
in Fig. 4.163, length of wire in cylindrical
region
= diameter AB of cylindrical region = 20 cm
According to Fleming's left hand rule, - 0.20 m
the magnetic field should be applied
normally into the plane of paper so as to
exert an upward magnetic force on the rod.
(b) If the direction of current is
reversed, the magnetic force will act in the
downward direction. Hence the total
tension in the wires will be
T = 2 × the weight of the rod
Fig. 4.163
= 2 × 0.06 × 9.8 N = 1.176 N.
As the wire lies in N-S direction and
4.22. The wires which connect the battery
field acts along E-W direction, so θ = 90°
of an automobile to its starting motor
carry a current of300 A (for a short ∴ Force on wire,
time). What is the force per unit length
F = IBl sin θ = 7 × 1.5 × 0.20 × 1 = 2.1 N
between the wires if they are 70 cm
122
By Fleming's left hand rule, this force
acts in the vertically downward direction.
(ii) When the wire turns from N-S to N-E
or N-W direction, suppose it makes angle θ
with field B, as shown in Fig. 4.164. Then
length of wire in magnetic field, A'B' = T
say.
𝑙 𝑙
Clearly 𝑙′ = sin𝜃 or 𝑙′ = sin𝜃
Fig. 4.164
Force on wire,
𝑙
𝐹 = 𝐼𝑙′ sin𝜃 = 𝐼. ⋅ 𝐵sin𝜃 = 𝐼𝐵
sin𝜃
= 2.1N
Fig. 4.165
This force acts in the vertically downward
and 5 cm carries a current of 12 A. What is
direction.
the torque on the loop in the different cases
(iii) As shown in Fig. 4.159, when the shown in Fig. 4.165? What is the force on
wire is lowered by 6.0 cm, length of the each case? Which case corresponds to
wire in the magnetic field = 2x stable equilibrium?
But 𝑥 = √102 − 62 = 8cm = Ans. Here B = 3000 G = 3000 × 10 -4 =
0.08m 0.3 T,
∴ 2x = 0.16, θ = 90° A = 10 × 5 = 50cm2 = 50 × 10-4m / = 12
A
Force on wire, F = IIB = 7 × 0.16 × 1.5 =
1.68 N Magnetic moment,
123
⃗⃗ = −0.06𝑗̂Am2 , 𝐵
(c) Here 𝑚 ⃗ = 0.3𝑘̂ T (c) Force on each electron is
𝐵𝐼
𝜏=𝑚 ⃗
⃗⃗ × 𝐵 𝐹 = 𝑒𝑣𝐵 = 𝑛𝐴 [∵
= −0.06𝑖̂ × 0.3𝑘̂ = −1.8 × 10−2 𝑖̂Nm 𝐼 = 𝑒𝑛𝐴𝑣]
124
resistance of 12 Ω and meter shows full 4.28. A galvanometer has a resistance
scale deflection for a current of 3 mA. How of 15 Ω and the meter shows full scale
will you convert the meter into a voltmeter deflection for a current of 4 mA. How will
of range 0 to 18 V? you convert the meter into an ammeter of
range 0 to 6 A?
Ans. Here R = 12Ω, I = 3 mA = 3 × 10-3
A, V = 18 V Ans. Here Rg = 15 Ω, Ig = 4 mA = 0.004
𝑉 18 A, I = 6 A
𝑅= − 𝑅𝑔 = − 12 𝐼 0.004
𝐼𝑔 3 × 10−3 𝑅𝑠 = 𝐼−𝐼𝑔 × 𝑅𝑔 = 6−0.004 × 15= 0.010Ω =10
𝑔
= 6000- 12 = 5988 Ω mΩ
By connecting a resistance of 5988 in By connecting a shunt of resistance 10
series with the given galvanometer, we get mΩ across the given galvanometer, we get
a voltmeter of range 0 to 18 V. an ammeter of range 0 to 6 A.
TEXT BASED EXERCISES
TYPE A : VERY SHORT ANSWER QUESTIONS 1 mark each)
1. State Oersted's observation. element (i) minimum and (ii)
maximum?
2. State Biot-Savart's law.
12. Figure 4.166 shows a circular loop
[ISCE 94]
carrying a current I.
3. Mathematically, Biot-Savart law may
Show the direction of the magnetic field
be expressed as
with the help of lines of force. [CBSE D 04]
𝐼𝑑𝑙sin𝜃
𝑑𝐵 = 𝐾 ⋅
𝑟2
Write the value of K in SI units.
4. What is the SI unit of μ 0 ?
5. What is the value of 4π / μ Q ? Fig. 4.166
6. State the rule that is used to find the 13. Which physical quantity has the unit
direction of magnetic field acting at a Wb m-2 ? Is it a scalar or a vector
point near a current carrying straight quantity?
conductor.
[CBSE D 04]
7. Write an expression for the magnetic
14. An electric current is flowing due south
field produced by an infinitely long
along a power line. What is the
straight wire carrying a current I, at a
direction of the magnetic field at a point
short distance a from itself.
(a) above it and (b) below it?
[ISCE 98]
15. How does a current carrying coil
8. Show the magnetic lines of force behave like a bar magnet?
around a straight
[CBSE D 11]
current carrying conductor.
16. Draw the magnetic field lines due to a
[Punjab 97C] current carrying loop.
9. What is the nature of the magnetic field [CBSE D 13C ; OD 16]
associated with the current in a straight
17. How much is the flux density B at the
conductor ?
centre of a long solenoid ?
10. Where is the magnetic field due to
[ISCE 95, 97]
current through circular loop uniform ?
18. What is a toroid ?
11. Where is the magnetic field of a current
19. What is magnetic Lorentz force ?
125
[Punjab 01] Would the particle gain any energy ?
20. Write the expressionn, in a vector form, [CBSE F 13]
for the Lorentz magnetic force 𝐹 due to 32. An electron with speed v enters at right
a charge moving with velocity 𝑣 in a angle in a region of uniform magnetic
magnetic field 𝐵⃗ . What is the direction field B. Write the expression for the
of the magnetic force ? [CBSE D 14, radius of the path it follows.
16]
[CBSE F 1995]
21. A particle of charge q moves with a
velocity v at an angle θ to a magnetic 33. An electron beam projected along +
field B. What is the force experienced X-axis, experiences a force due to a
by the particle ? magnetic field along the + Y-axis. What
is the direction of the magnetic field ?
[ISCE 93 ; CBSE F 91]
22. What is the force experienced by a [CBSE D 05]
stationary charge in a magnetic field ?
34. An electron and a proton moving with
[Himachal 02]
the same speed enter the same
23. What is the work done by magnetic field magnetic field region at right angles to
on a moving charge ? [Haryana 94] the direction of the field. For which of
24. Write down the expression for the the two particles will the radius of
Lorentz force on a charged particle. circular path be smaller ?
[Himachal 99 ; Punjab 99, 99C] [CBSE OD 98]
25. State the condition under which a 35. A charged particle moving in a uniform
charged particle moving with velocity v magnetic field penetrates a layer of
goes undeflected in a magnetic field B. lead and thereby loses ' one-half of its
[CBSE F 17] kinetic energy. How does the radius of
26. State Fleming's left hand rule. curvature of its path change ?
27. An electron beam is moving vertically 36. Write the condition under which an
downwards. If it passes through a electron will move undeflected in the
magnetic field which is directed from presence of crossed electric and
south to north in a horizontal plane, magnetic fields. [CBSE F 13 ; OD 14C]
then in which direction the beam would 37. A straight conductor AB of a circuit lies
be deflected ? [CBSE D 96C] along the X-axis from x = - a / 2 to x = +
28. What will be the path of a charged a / 2 and carries a current 1. What is the
particle moving perpendicular to a magnetic field due to this conductor AB
uniform magnetic field ? at a point x = + a ?
126
magnitude of the field and does not 53. What is the value of net force acting on
depend upon the velocity. What is the a current carrying (z) rectangular and
nature of the field? (ii) circular loop, placed in a uniform
magnetic field ? What do you expect
⃗⃗⃗ ,
42. What is the force that a conductor 𝑑𝑙 about the torque in each case ?
carrying a current I experiences when
placed in a magnetic field 𝐵⃗ . What is 54. Write an expression for the torque
the direction of the force? acting on a current carrying coil located
in a uniform magnetic field.
[CBSE OD 90]
55. Write an expression for the magnitude
43. An electroii beam is moving of the torque acting on a current
horizontally in a tube. The vertical carrying coil placed in a uniform radial
component of earth's magnetic field is magnetic field.
directed downwards. In which direction
will the electron beam be deflected? 56. Under what circumstances will a
current carrying loop not rotate in the
44. A charged particle moves in a uniform magnetic field ?
magnetic field at right angles to the
direction of the field. Which of the 57. State the principle of working of a
following quantities will change : speed, moving coil galvanometer.
velocity, momentum, kinetic energy, [CBSE D 15, 16]
displacement ? 58. What do you mean by the figure of
45. In which orientation is the force merit of a galvanometer ?
experienced by a current-carrying 59. Define the current sensitivity of a
conductor placed in a magnetic field moving coil galvanometer and state its
maximum ? SI unit.
46. A current carrying conductor does not [CBSE D 17 ; OD 13, 13C, 16, 17C]
tend to deflect in a magnetic field. What
60. Write two factors by which the current
conclusion can be drawn from it ?
sensitivity of a moving coil
47. Name the rule that gives the direction galvanometer can be increased.
of force on a current-carrying conductor
placed perpendicular to the magnetic [CBSE D 01 ; F 08 ; OD 16]
field. 61. Define voltage sensitivity of a moving
48. Write an expression for the force coil galvanometer. Give its SI unit.
between two parallel short wires [Punjab 91]
carrying currents. 62. Write two factors by which voltage
49. Two current elements are placed a sensitivity of a moving coil
certain distance apart but not parallel to galvanometer can be increased.
each other. Do they exert equal and [CBSE D 01 ; F 08]
opposite forces on each other ?
63. What is the nature of the magnetic field
50. What is the direction of force between in a moving coil galvanometer ?
two parallel wires carrying currents in [CBSE D 96 ; OD 96]
opposite directions ?
64. State two properties of the material of
51. The force existing between two parallel the wire used for suspension of the coil
current carrying conductors is F. If the in a moving coil galvanometer.
current in each conductor is doubled, [CBSE OD 01, 06C]
what is the value of the force between
65. The current sensitivity of a moving coil
them ?
galvanometer is 5 division/mA and
52. Is the force between two parallel voltage sensitivity is 20 division/volt.
current-carrying wires affected by the Find the resistance of the
nature of the dielectric medium galvanometer.
between them ?
127
66. An electron and a proton, having equal enters normal to the field direction and
momenta, enter a uniform magnetic the other enters along a direction at 30°
field at right angles to the field lines. with the field, what would be the ratio of
What will be the ratio of curvature of their angular frequencies ?
their trajectories ?
[CBSE Sample Paper 08]
[CBSE Sample Paper 05]
79. An α-particle and a proton are moving
67. An electron is moving with a velocity v, in the plane of the paper in a region
along the axis of a long straight where there is a uniform magnetic field
solenoid, carrying a current I. What will (𝐵 ⃗ ) directed normal to the plane of the
be the force acting on the electron due paper. If the two particles have equal
to the magnetic field of the solenoid ? linear momenta, what will be the ratio of
[CBSE Sample Paper 05] the radii of their trajectories in the field
? [CBSE Sample Paper 08]
68. Among alpha, beta and gamma
radiations, which get deflected by the 80. Why should the spring/suspension wire
magnetic field ? [CBSE F 04] in a moving coil galvanometer have low
torsional constant ?
69. A solenoid coil of 300 turns/m is
carrying a current of 5 A. The length of [CBSE OD 08]
the solenoid is 0.5 m and has a radius 81. The coils, in certain galvanometers,
of 1 cm. Find the magnitude of the have a fixed core made of a
magnetic field inside the solenoid. non-magnetic metallic material. Why
[CBSE F 04] does the oscillating coil come to rest so
70. What is the resistance of an ideal quickly in such a core ?
ammeter ? [CBSE D 08C]
71. What is the resistance of an ideal 82. A long straight wire carries a current I
voltmeter ? along the positive y-direction. A particle
72. Why should an ammeter have a high of charge +Q is moving with a velocity
current carrying capacity ? 𝑣 along the .t-axis. In which direction
will the particle experience a force ?
73. Why should a voltmeter have a low
current carrying capacity ? [CBSE F 13]
74. What is the effective resistance of an 83. Two particles A and B of masses m and
ammeter if a shunt of resistance Rs is 2m have charges q and 2q
used across the terminals of a respectively. Both these particles
galvanometer of resistance R ? moving with velocities v1 and v2
respectively in the same direction enter
75. Suppose a shunt of resistance 0.01 Ω the same magnetic field B acting
is connected across a galvanometer, normally to their direction of motion. If
what can be said about the resistance the two forces FA and FB acting on them
of the resulting ammeter ? are in the ratio 1 : 2, find the ratio of
76. A student wants to increase the range their velocities.
of an ammeter from 1 mA to 5 mA. [CBSE D 11 C]
What should be done to the shunt
resistance ? 84. A beam of α particles projected along +
x-axis, experiences a force due to
77. What is the direction of the force acting magnetic field along the + y-axis. What
on a charged particle q, moving with a is the direction of the magnetic field ?
velocity 𝑣 in a uniform magnetic field 𝐵⃗ (Fig. 4.167) 4 [CBSE OD 10]
?
78. Two identical charged particles moving
with same speed enter a region of
uniform magnetic field. If one of these
128
deuterons, having the same
momentum values, enter a region of a
uniform magnetic field directed
perpendicular to their common
direction of motion. What would be the
ratio of the radii of the circular paths,
described by the protons and
deuterons ?
Fig. 4.167 [CBSE F 11]
85 A beam of electrons projected along + 87. A square coil, OPQR, of side a,
x-axis, experiences a force due to a carrying a current 1, is placed in the
magnetic field along the + y-axis. What Y-Z plane as shown in Fig. 4.169. Find
is the direction of the magnetic field ? the magnetic moment associated with
(Fig. 4.168) this coil.
[CBSE OD 10] [CBSE Sample Paper 13]
Fig. 4.168
86. A narrow stream, of protons and Fig. 4.169
Answers
1. A magnetic needle brought close to a According to this rule if we grasp the
straight current-carrying wire aligns itself conductor in the right hand so that the
perpendicular to the wire, reversing the thumb points in the direction of the
direction of current reverses the direction current, then the magnetic field will be
of deflection. in the direction of the curl of the fingers.
2. According to Biot-Savart law, the 𝜇 𝐼
0
7. 𝐵 = 2𝜋𝑎 8. See Fig. 4.8.
magnetic field due to a current element
⃗⃗⃗ at the observation point whose
I 𝑑𝑙 9. The magnetic field consists of concentric
position vector is 𝑟 is given by circular lines of force with the conductor at
their centre and in a plane perpendicular to
⃗⃗⃗ × 𝑟
𝜇0 𝐼 𝑑𝑙 the conductor.
⃗ =
𝑑𝐵 ⋅
4𝜋 𝑟3 10 At the centre of the current loop.
where μ 0 is the permeability of free space.
11. (i) Magnetic field is minimum (zero)
𝜇0
3. 𝐾 = = 10−7 TmA−1 along the axis of a current element.
4𝜋
129
14. According to right hand rule, the on the charged particle.
direction of the field is (a) towards west
27. Towards west. 28. Circular
above the wire and (b) towards east
path.
below the wire.
29. When a charged particle moves
15. A current carrying loop behaves as a
inclined (θ is neither 0 nor π / 2) to the
bar magnet because (i) it possesses a
magnetic field, its motion is helical.
magnetic dipole moment (m=IA), and
(ii) it experiences a torque in an 30. The charged particle will move along a
external magnetic field. This torque straight line path.
tends to align the axis of the loop along 31. The magnetic force acts perpendicular
the direction of the field. to the direction of motion of the charged
16. See Fig. 4.25. particle. No work is done by the
magnetic force on it. The particle does
17. The magnetic field well inside a long
not gain any energy.
solenoid having n turns per unit length
and carrying current I is 𝐵 = 𝜇0 𝑛𝐼 32. Magnetic force on electron =
Centripetal force
18. An anchor ring around which a large
number of turns of a metallic wire are 𝑚𝑣 2 𝑚𝑣
wound is called a toroid. 𝑒𝑣𝐵SIN90° = ∴ RADIUS, 𝑟 =
𝑟 𝑒𝐵
19. The force experienced by a charged 33. According to Fleming's left hand rule,
particle while moving through a region the magnetic field acts in the
of magnetic field is called magnetic +Z-direction.
Lorentz force. 𝑚𝑣
34. Radius, 𝑟 = i.e., 𝑟 ∝ 𝑚
𝑒𝐵
⃗)
It is given by 𝐹 = 𝑞(𝑣 × 𝐵
As electron has smaller mass than proton,
20. 𝐹 = 𝑞(𝑣 × 𝐵⃗ ) The direction of the force so it will circulate in a circular path of
is perpendicular to the plane containing smaller radius.
vectors 𝑣 and 𝐵 ⃗ .
35. Radius of curvature,
21. F = qvB sin θ.
𝑚𝑣 𝑚 2𝐾 √2𝑚𝐾
22. For a stationary charge, v = 0. 𝑟= = √ = i.e., 𝑟 ∝ √𝐾
𝑞𝐵 𝑞𝐵 𝑚 𝑞𝐵
Therefore, F = qvB sin θ = q (0) B sin θ = 0.
∴ If the kinetic energy is halved, radius
23. Zero, because a magnetic force acts
perpendicular to the direction of of curvature is reduced to 1⁄√2 times its
velocity or the direction of motion of the initial value.
charged particle. 36. For undeflected beam,
24.
𝐹 = 𝑞(𝐸⃗ + 𝑣 × 𝐵
⃗) 𝐸
𝐹𝑚 = 𝐹𝑒 or 𝑒𝑣𝐵sin90° = 𝑒𝐸 or 𝑣 =
25. Force on a charge moving parallel or 𝐵
antiparallel to the direction of the 37. Zero, because the observation point
magnetic field is zero F = qvB sin (0° or lies on the axis of the straight
180°) = 0. conductor.
26. Fleming's left hand rule gives the 38. Refer answer to Q. 16 on page 4.34.
direction of force on a charged particle
39. Time spent by a proton inside the dees
moving in a magnetic field : Stretch the
of a cyclotron is independent of both its
thumb and the first two fingers of the
speed and radius of its circular path.
left hand so that they are perpendicular
to each other. If the forefinger points in 40. The field is magnetic in nature.
the direction of magnetic field, central
41. The field is electric in nature.
finger in the direction of current, then
the thumb gives the direction of force ⃗⃗⃗ × 𝐵
42. 𝑑𝐹 = 𝐼(𝑑𝑙 ⃗)
130
The direction of force 𝑑𝐹 is perpendicular 55. 𝜏 = NIBA.
to the plane of 𝑑𝑙⃗⃗⃗ and 𝐵
⃗ and will point in 56. If the current carrying loop is placed in
the same direction in which a a magnetic field, with its plane
right-handed screw, when rotated from perpendicular to the field, then it will not
⃗⃗⃗ and 𝐵
𝑑𝑙 ⃗ , will advance. rotate.
43. Towards west. 57. Refer to point 26 of Glimpses.
44. Only velocity, momentum and 58. The figure of merit of a galvanometer is
displacement will change as they are defined as the amount of current
all vectors. required to produce one scale
deflection in the galvanometer.
45. When the conductor is held
𝐼 𝑘
perpendicular to the magnetic field, it It is given by 𝐺 = 𝛼 = 𝑁𝐵𝐴
experiences a maximum force.
59. The current sensitivity of a
46. This means that no force is acting on galvanometer is defined as the
the current carrying wire due to the deflection produced in the
magnetic field. This is possible when galvanometer on passing unit current
the conductor is parallel to the direction through it.
of the magnetic field.
𝛼 𝑁𝐵𝐴
47. Fleming's left hand rule. Current sensitivity = 𝐼 = 𝑘
48. The force between two parallel short The SI unit of current sensitivity is radian
wires of lengths 𝑑𝑙1 and 𝑑𝑙2 , separated ampere-1.
by distance r and carrying currents I1 60. The current sensitivity of a moving coil
and I2 respectively, is given by galvanometer can be increased by (i)
𝜇0 𝐼1 𝐼2 𝑑𝑙1 𝑑𝑙2 increasing the number of turns in the
𝑑𝐹 = ⋅ galvanometer coil, (ii) decreasing the
4𝜋 𝑟2
torsion constant of its suspension fibre.
49. Like other forces, these forces also
obey Newton's third law of action and 61. The voltage sensitivity of a moving coil
reaction and are, therefore, equal and galvanometer is defined as the
opposite. deflection produced in the
galvanometer when a unit voltage is
50. The direction of force is perpendicular
applied across its coil.
to the two wires and is outwards, so
𝛼 𝛼 𝑁𝐵𝐴
that the two wires repel each other. Voltage sensitivity = 𝑉 = 𝐼𝑅 = 𝑘𝑅
51. The value of force is 4F. This is The SI unit of voltage sensitivity is radian
because force between two parallel volt-1.
current carrying conductors is
proportional to the product of the 62. The voltage sensitivity of a moving coil
currents through them. galvanometer can be increased by
52. No. This interaction is between the (i) increasing the number of turns of the
magnetic fields produced by the two galvanometer coil
wires which does not depend on the (ii) decreasing the torsion constant of the
nature of the dielectric medium. suspension fibre.
53. In each case the net force is zero but 63. Radial magnetic field is used in a
torque is non-zero. moving coil galvanometer.
54. If a coil of area A, turns N and carrying 64. The material used for the suspension
current 1 is held in a uniform magnetic wire of a moving coil galvanometer
⃗ , it experiences a torque given by
field 𝐵 should have following properties :
𝜏 = NIB A sin θ, where θ is the angle
⃗ and the normal to the plane (i) Small torsion constant k which makes
between 𝐵
the galvanometer highly sensitive.
of the loop.
131
(ii) High tensile strength so that even thin ⃗.
plane of the vectors 𝑣𝑎𝑛𝑑 𝐵
wire does not break under the weight of 𝑞𝐵
the suspension coil. 78. Angular frequency, 𝜔 = . It is
𝑚
65. Here Is - 5 div mA-3 = 5 × 103div A-1, independent of m angle θ.
132
loop of radius r carrying a steady (i) inside on the axis and (ii) outside the
current I. Draw the field lines due to the combined system. [CBSE D 14]
current loop.
[ISCE 96 ; CBSE D 01C ; OD 14 C]
5. State Biot-Savart law. Deduce the
expression for the magnetic field at a
point on the axis of a current carrying
circular loop of radius ' R', distant V
from the centre. Hence write the
magnetic field at the centre of a loop.
[CBSE D 05 ; OD 05, 15]
6. State Ampere's circuital law and prove
this law for a circular path around a
long current carrying conductor.
[Himachal 98 ; Haryana 98C, 01] Fig. 4.170
7. State Ampere's circuital law. Use this 11. A long straight wire of a circular
law to find magnetic field due to straight cross-section of radius V carries a steady
infinite current. current '/'. The current is uniformly
[CBSE OD 16] distributed across the cross-section. Apply
Ampere's circuital law to calculate the
8. A long solenoid with closely wound magnetic field at a point ‘r’ in the region for
turns has n turns, per unit of its length. (i) r <a and (ii) r > a.
A steady current 7 flows through this
solenoid. Use Ampere's circuital law to [CBSE D 10]
obtain an expression, for the magnetic
12. Write an expression for force 𝐹 acting
field, at a point on its axis and close to
on a charge q moving with a velocity 𝑣 in
its mid point. Draw its field lines.
the region, where magnetic induction 𝐵⃗ is
[CBSE D 04. C, 14C]
uniform. How does the speed change, as
9. (a) How is a toroid different from a the charge moves? Under what
solenoid ? circumstances the force 𝐹 shall be zero?
(b) Use Ampere's circuital law to obtain the [ISCE 96]
magnetic field inside a toroid.
13. (a) Write the expression for the force 𝐹
(c) Show that in an ideal toroid, the acting on a particle of mass m and charge
magnetic field (i) inside the toroid and q moving with velocity 𝑣 in a magnetic field
(ii) outside the toroid at any point in the ⃗ . Under what conditions will it move in (i) a
𝐵
open space is zero. circular path and (ii) a helical path?
[CBSE OD 08, 14C] [CBSE D 17]
10. ( a) State Ampere's circuital law, (b) Show that the kinetic energy of the
expressing it in the integral form. particle moving in a magnetic field
remains constant.
(b) Two long coaxial insulated solenoids,
s1 and s2 of equal lengths are wound one 14. Consider themotion of a charged
over the other as shown in the figure. A particle of mass 'm’ and charge 'q'
steady current "I" flows through the inner moving with velocity 𝑣 in a magnetic
solenoid ^ to the other end B, which is ⃗.
field 𝐵
connected to the outer solenoid S, through ⃗ , show that it
(a) If 𝑣 is perpendicular to 𝐵
which the same current "I" flows in the
opposite direction so as to come out at end describes a circular path having angular
A. If n1 and n2 are the number of turns per frequency ω = qB / m
unit length, find the magnitude and (b) If the velocity 𝑣 has a component
direction of the net magnetic field at a point ⃗ , trace the
parallel to the magnetic field 𝐵
133
path described by the particle. Justify your acquired by the particles.
answer. [CBSE D 14C]
[CBSE OD 13]
15. A uniform magnetic field 𝐵 ⃗ is set up
21. State the principle of a cyclotron. Show
along the positive x-axis. A particle of that the time period of revolution of
charge ‘q' and mass 'm' moving with a particles in a cyclotron is independent
velocity 𝑣 enters the field at the origin in of their speeds. Why is this property
X - V plane such that it has velocity necessary for the operation of a
components both along and cyclotron ?
perpendicular to the magnetic field 𝐵 ⃗.
Trace, giving reason, the trajectory [CBSE OD 16]
followed by the particle. Find out the 22. Derive an expression for the maximum
expression for the distance moved by force experienced by a straight
the particle along the magnetic field in conductor of length I, carrying current I
one rotation. [CBSE OD 15] and kept in a uniform magnetic field, B.
16. Find the condition under which the [CBSE D 06C]
charged particles moving with different 23. Two long straight parallel conductors
speeds in the presence of electric and carry steady currents I↑ and I2
magnetic field vectors can be used to separated by a distance ci. If the
select charged particles of a particular currents are flowing in the same
speed. [CBSE OD 17] direction, show how the magnetic field
17. (i) Obtain the expression for the set-up hr one produces an attractive
cyclotron frequency. force on the other. Obtain the
expression for this force. Hence define
(ii) A deuteron and a proton are one ampere. [CBSE D 09,16]
accelerated by the cyclotron. Can both
be accelerated with the same oscillator 24. Derive a formula for the force between
frequency? Give reason to justify your two parallel straight conductors
carrying current in opposite directions
answer.
and write the nature of the force.
[CBSE D 17] Hence, define an ampere.
18. A hydrogen ion of mass 'm’ and charge [CBSE OD 98]
'q' travels with a speed V along a circle
of radius V in a uniform magnetic field 25. Derive an expression for the torque on
of flux density ‘B. Obtain the expression a rectangular coil of area A, carrying a
for the magnetic force on the ion and current I and placed in a magnetic field
determine its time period. [CBSE D B. The angle between the direction of B
03C ; OD 04] and vector perpendicular to the plane
of the coil is θ. Indicate the direction of
19. Show that the frequency of revolution, the torque acting on the loop.
of a charged particle (in the X-Y plane),
⃗ = 𝐵𝑘̂),
⃗ (𝐵 [CBSE F 09 ; D 17C]
in a uniform magnetic field 𝐵
is independent of its speed. 26. A rectangular coil of sides and 'b'
carrying a current I is subjected to a
Which practical machine makes use of this uniform magnetic field B acting
fact? What is the frequency of the perpendicular to its plane. Obtain the
alternating electric field, used in this expression for the torque acting on it.
machine? [CBSE D 14C]
[CBSE D 09C] 27. A rectangular loop of area A, having N
20. Draw a schematic sketch of a turns and
cyclotron. Explain clearly the role of carrying a current of I ampere is held in a
crossed electric and magnetic fields in uniform magnetic field B. (i) Write the
accelerating the charge. Hence derive expression for the maximum torque
the expression for the kinetic energy experienced by the loop, (ii) In which
134
orientation, will the loop be in stable magnetic force 𝐹 acting on a charged
equilibrium ? particle q moving with velocity 𝑣 in the
[CBSE OD 98C] presence of the magnetic field 𝐵 ⃗ in a
vector form. [CBSE OD 16C]
28. State the principle of a moving coil
galvanometer. Show that the current (ii) Show that no work is done and no
passing through the coil is directly change in the magnitude of the velocity
proportional to the deflection of the coil. of the particle is produced by this force.
29. A moving coil galvanometer consists of (iii) Hence define the unit of magnetic field.
a rectangular coil of N turns, each of 33. Three long straight parallel wires are
area A, suspended in a radial magnetic kept as shown in Fig. 4.171. The wire
field of flux density B. Derive the (3) carries a current I
expression for the torque on the coil,
when current I passes through it. Draw
suitable labelled diagram.
[CBSE D 93C]
30. A moving coil galvanometer of
resistance G gives a full scale
deflection for a current I. Use the
suitable circuit diagram to convert it into
an ammeter of range 0 to 𝐼(𝐼 > 𝐼𝑔 ) . Fig. 4.171
Deduce the expression for the shunt
requried for this conversion. Hence (i) The direction of flow of current I in wire
write the expression for the resistance (3) is such that the net force, on wire
of the ammeter thus obtained. (1), due to the other two wires, is zero.
[Punjab 2000 ; CBSE D 09C] (ii) By reversing the direction of I, the net
31. Explain how will you convert a force, on wire (2), due to the other two
galvanometer into a voltmeter to read a wires, becomes zero. What will be the
maximum potential difference of V directions of current I, in the two cases?
volts. Can one use a voltmeter to Also obtain the relation between the
measure the emf of a cell? Justify your magnitudes of currents I1, I2 and I.
answer. [CBSE OD 97C, F 98] [CBSE D 16C]
32. (i) Write the expression for the
Answers
1. Refer answer to Q. 2 on page 4.1. page 4.26.
2. Refer answer to Q. 3 on page 4.2. 8. Refer answer to Q. 10 on page 4.23.
3. Refer to solution of Example 17 on 9. Refer answer to Q. 11 on page 4.24.
page 4.16.
10. (a) ∮ 𝐵 ⃗⃗⃗ = 𝜇0 𝐼
⃗ ⋅ 𝑑𝑙
Here 𝐿 = 𝜋𝑟 or 𝑟 = 𝐿/𝜋
(b) (i) Magnetic field inside solenoid
𝜇0 𝐼 𝜇0 𝐼 𝜋 𝜋 𝜇0 𝐼 𝑆1 , 𝐵1 = 𝜇0 𝜂1 𝐼 Magnetic field inside
𝐵= = ⋅ =
4𝑟 4 𝐿 4 𝐿 solenoid 𝑆2 , 𝐵2 = 𝜇0 𝑛2 𝐼 As the currents in
4. Refer answer to Q. 7 on page 4.12. the two solenoids are oppositely directed,
See Fig. 4.25. so direction of is opposite to that of Bl. The
net magnetic field at any inside point along
5. Refer answer to Q. 8 on page 4.13. the axis,
See Fig. 4.25 on page 4.14.
B= B1- B2=μ0(n1-n2)I
6. Refer answer to Q. 9 on page 4.22.
(ii) Outside the combined system, net
7. Refer to solution of Example 33 on
135
magnetic field = 0. such that 𝐹𝐸 = 𝐹𝐵 or 𝑞𝐸 = 𝑞𝑣𝐵 or 𝑣=
𝐸
11. Refer to the solution of Example 33(f) 𝐵
on page 4.26.
17. (i) Refer answer to Q. 17 (Theory part)
12. Force, 𝐹 = 𝑞(𝑣 × 𝐵 ⃗ ). The speed of the on page 4.40. (ii) Cyclotron frequency,
𝑞𝐵
charge is not affected in the magnetic 𝑓𝑐 = 2𝜋𝑚
field. Force 𝐹 will be zero if 𝑣 = 0 or if 𝑣
⃗ No. The masses of proton and deuteron
is parallel or antiparallel to 𝐵 .
are different but charge is same. As fc
⃗)
13. (a) 𝐹 = 𝑞(𝑣 × 𝐵 depends inversely on mass m, protons and
deuterons cannot be accelerated by the
(i) When the charged particle moves same oscillator frequency.
perpendicular to the magnetic field, it
will move in a circular path. 18. In the uniform magnetic field, the
magnetic force on the hydrogen ion
(ii) When charged particle moves obliquely acts perpendicular to both v and B.
(neither parallel nor perpendicular) to
the magnetic field, it will move in a ∴ F = evB sin 90° = evB
helical path. Magnetic force on the hydrogen ion =
(b) The magnetic force acts on the charged Centripetal force
particle perpendicular to its instantaneous 𝑚𝑣 2 𝑚𝑣
velocity v, at all instants. It cannot change 𝑒𝑣𝐵 = or 𝑟=
the speed of the particle. As speed remains 𝑟 𝑒𝐵
constant, the kinetic energy also remains Time period of hydrogen ion,
constant. 2𝜋𝑟 2𝜋 𝑚𝑣 2𝜋𝑚
𝑇= = ⋅ =
14. (a) Magnetic force acts on the charged 𝑣 𝑣 𝑒𝐵 𝑒𝐵
particle in a direction perpendicular to 19. Refer answer to Q. 15(2) on page 4.33.
both 𝑣 and B and provides centripetal
𝑞𝐵
force. We obtain, 𝑓𝑐 = 2𝜋𝑚
∴ Magnetic force, 𝑞𝑣𝐵sin90° A cyclotron makes use of this fact in which
𝑚𝑣 2 𝑚𝑣 alternating electric field of frequency f is
= Centripetal force, or 𝑟 = applied.
𝑟 𝑞𝐵
𝑣 𝑞𝐵 20. Refer answer to Q. 17 on page 4.40.
𝜔= =
𝑟 𝑚 21. Refer answer to Q. 17 on page 4.40.
(b) Refer answer to Q. 15(3) on page 4.34. 22. Refer answer to Q. 19 on page 4.44.
15. Refer answer to Q. 15(3) on page 4.34. 23. Refer answer to Q. 21 on page 4.49.
16. The directions of 𝐸⃗ and 𝐵 ⃗ must be 24. Refer answer to Q. 21 on page 4.49.
perpendicular to each other and also 25. Refer answer to Q. 22 on page 4.53.
perpendicular to the direction of v so that
electric and magnetic forces are in 26. Magnetic moment associated with the
opposite directions. current carrying coil is 𝑚
⃗⃗ = 𝐼𝐴𝑛̂ = 𝐼𝑏𝑛̂
where 𝑛̂ is a unit vector perpendicular to
the plane of the coil.
⃗ = 𝐼𝑙𝑏𝑛̂ × 𝐵
⃗⃗⃗ × 𝐵
Torque, 𝜏 = 𝑚 ⃗ = ⃗0
⃗ = 𝐼𝑙𝑏0
This is because n and B are either parallel
or antiparallel vectors.
27. Refer answer to Q. 22 on page 4.53.
The loop will be in stable equilibrium
The magnitudes of 𝐸⃗ and 𝐵
⃗ should be when m is parallel to B .
136
28. Refer answer to Q. 23 on page 4.57. 𝐹
(iii) Magnitude, 𝐹 = 𝑞𝑣𝐵sin𝜃 ⇒ 𝐵 = 𝑞𝑣sin𝜃
29. Refer answer to Q. 23 on page 4.57.
1N 1N
30. Refer answer to Q. 26 on page 4.63. SI unit of 𝐵 = =
1C ⋅ 1ms−1 ⋅ sin90° 1A. 1m
31. Refer answer to Q. 28 on page 4.64. = 1NA-Im-1 =1 tesla
No, a voltmeter cannot be used to
measure the emf a cell. A voltmeter So one tesla is the magnetic field in which
requires a small current for its a charge of 1 C moving with a velocity of
operation. It measures p.d. in a closed lms-1 at right angles to the field experiences
circuit, which is less than the emf of the a force of one newton.
cell. 33. (i) The net force on wire (1) will be
⃗) zero only when the forces exerted by wires
32. (i) 𝐹 = 𝑞(𝑣 × 𝐵
(2) and (3) on wire (1) are equal and
(ii) The magnetic force 𝐹 acts, at all opposite. For this, the current in wire (3)
instants, perpendicular to the must flow in a direction opposite to I2 i.e.,
instantaneous direction of 𝑣 , which is downwards or along -Y direction.
also the instantaneous direction of 𝜇0 𝐼1 𝐼 𝜇0 𝐼1 𝐼2
displacement ( 𝑑𝑠 ). So 𝐹 acts = ∴ 𝐼1 = 2𝐼2
2𝜋(2𝑎) 2𝜋(𝑎)
perpendicular to 𝑑𝑠 at all instants.
(ii) When the current in wire (3) is reversed,
𝑊 = 𝐹 𝑑𝑠 = 0 I flows upwards or along +Y direction.
The net force on wire (2) will become
As no work is done by the magnetic force,
zero when the forces exerted by wires
there can be no change in the K.E. and
(1) and (3) are equal and opposite.
hence no change in the magnitude of the
velocity of the charged particle by this 𝜇0 𝐼1 𝐼2 𝜇0 𝐼2 𝐼
= ∴ 𝐼 = 𝐼1 = 2𝐼2
force. 2𝜋𝑎 2𝜋𝑎
137
⃗⃗⃗ carrying current I at a distance 𝑟 from it
𝑑𝑙 solenoid specifying its polarity and
in a vector form. show that it acts as a bar magnet of
magnetic moment m = NIA.
Hence derive the expression for the [CBSE OD 13, D 15]
magnetic field due to a current carrying
loop of radius R at a point P distant x from
its centre along the axis of the loop. 8. (a) Using Ampere's circuital law, obtain
(b) Explain how Biot-Savart law enables the expression for the magnetic field
one to express the Ampere's circuital due to a long solenoid at a point inside
law in the integral form, viz., ∮ 𝐵 ⃗⃗⃗ =
⃗ ⋅ 𝑑𝑙 the solenoid on its axis.
𝜇0 𝐼, where I is the total current passing (b) In what respect is a toroid different from
through the surface. a solenoid ? Draw and compare the
pattern of the magnetic field lines in two
[CBSE OD 15]
cases.
5. (a) State Ampere's circuital law
⃗ over a (c) How is the magnetic field inside a given
connecting the line integral of 𝐵 solenoid made strong ?
closed path to the net current crossing the
area bounded by the path. [CBSE OD 11]
(b) Use Ampere's law to derive the formula 9. Derive a mathematical expression for
for the magnetic field due to an infinitely the force acting on a current carrying
long straight current carrying wire. straight conductor kept in a magnetic
field. State the rule used to determine
(c) Explain carefully why the derivation as the direction of this force. Under what
in (b) is not valid for magnetic field in a conditions is this force (i) zero and (ii)
plane normal to a current-carrying maximum ? [CBSE D 97C, 98]
straight wire of finite length and passing
through the midpoint of the axis. 10. Draw a schematic sketch of a
[CBSE OD 16] cyclotron. Explain briefly how it works and
how it is used to accelerate the charged
6. (a) Show how Biot-Savart law can be particles. [CBSE D 08 ; OD 09]
alternatively expressed in the form of
Ampere's circuital law. Use this law to (i) Show that time period of ions in a
obtain the expression for the magnetic field cyclotron is independent of both the
inside a solenoid of length '/', speed and radius of circular path.
cross-sectional area 'A' having 'N' closely (ii) What is resonance condition ? How is it
wound turns and carrying a steady current' used to accelerate the charged
/'. (b) Sketch the magnetic field lines for a particles ?
finite solenoid. Explain why the field at the
exterior midpoint is weak while at the 11. With the help of a labelled diagram,
interior it is uniform and strong. state the underlying principle of a
[CBSE D 06C, 15C] cyclotron. Explain clearly how it works
to accelerate the charged particles.
7. (a) State Ampere's circuital law. Use Show that cyclotron frequency is
this law to obtain the expression for the independent of energy of the particle. Is
magnetic field inside an air cored toroid of there an upper limit on the energy
average radius ‘r', having 'n' turns per unit acquired by the particle ? Give reason.
length and carrying a steady current I.
Show that the magnetic field in the open [CBSE Dll, 14, 14C]
space inside and exterior to the toroid is 12. (i) Derive an expression for the force
zero. between two long parallel current carrying
(b) An observer to the left of a solenoid of N conductors.
turns each of cross-section area 'A' (b) Use this expression to define SI unit of
observes that a steady current / in it current.
flows in the clockwise direction. Depict
the magnetic field lines due to the (c) A long straight wire AB carries a current
138
I. A proton P travels with a speed v, (b) A rectangular current carrying loop
parallel to the wire, at a distance d from EFGH is kept in a uniform magnetic
it in a direction opposite to the current field as shown in Fig. 4.175.
as shown in Fig. 4.174.
(i) What is the direction of the magnetic
What is the force experienced by the moment of the current loop ?
proton and what is its direction?
(ii) When is the torque acting on the loop
[CBSE D 06 ; OD 10] (A) maximum, (B) zero ? [CBSE OD 05,
09]
Fig. 4.174
13. Derive an expression for the torque
acting on a loop of N turns, area A, Fig. 4.175
carrying current 1, when held in a 15. (a) With the help of a diagram, explain
magnetic field B. With the help of a the principle and working of a moving
circuit diagram, show how a moving coil galvanometer.
coil galvanometer can be converted
into an ammeter of given range. Write (b) What is the importance of a radial
the necessary mathematical formula. magnetic field and how is it produced?
[CBSE D 04] (c) Why is it necessary to introduce a
14. (a) Two straight long parallel cylindrical soft iron core inside the coil
conductors carry currents l1 and l2 in of a galvanometer?
the same direction. Deduce the (d) "Increasing the current sensitivity of a
expression for the force per unit length galvanometer may not necessarily
between them. increase its voltage sensitivity". Justify
Depict the pattern of magnetic field lines this statement.
around them. [CBSE D 06, 13C ; OD 14, 14C, 15,16]
Answers
1. (a) Refer answer to Q. 4 on page 4.3. into two parts : I1 along the smaller part and
I2 along the larger part of the loop.
(b) Refer answer to Q. 7 on page 4.12.
Field due to I1 at O,
2. Refer answer to Q. 8 on page 4.13.
⃗ 1 = 1 ⋅ 𝜇0 𝐼1 normally into the paper.
𝐵
𝜇0 𝐼𝑎 2 4 2𝑅
𝐵 axial =
2(𝑟 2 + 𝑎 2 )3⁄2 Field due to I2 at O,
𝜇0 𝐼𝑎 2 𝜇0 𝐼 ⃗ 2 = 3 ⋅ 𝜇0 𝐼2 normally out of the paper.
𝐵
𝐵𝑟=√3𝑎 = 2 2 3 ⁄2
= 4 2𝑅
2(3𝑎 + 𝑎 ) 16𝑎
0 𝜇 𝐼𝑎 2 𝜇0𝐼 [Put r= 0]
𝐵 centre = 2(02+𝑎 2 )3⁄2 = 2𝑎
𝐵 16𝑎
∴ 𝐵 centre = =8
𝑟=√3𝑎 2𝑎
139
⃗ =𝐵
Net field at 𝑂, 𝐵 ⃗1 +𝐵
⃗2 so the net magnetic moment of the
solenoid is NIA.
1 𝜇0 𝐼1 3 𝜇0 𝐼2
⃗|=
|𝐵 −
4 2𝑅 4 2𝑅 8. (a) Refer answer to Q. 10 on page 4.23.
As the resistance of the larger part is 3 (b) A solenoid bent into the form of a closed
times the resistance of the smaller part, so ring is called a toroidal solenoid. The field
I1 =3I2 pattern of solenoid is similar to that of a bar
magnet. The field lines inside a toroid are
⃗|=0
Hence, |𝐵 circular loops and the field is uniform
4. (a) Refer answer to Q. 8 on page 4.13. everywhere inside the toroid. See Fig. 4.48
on page 4.23 and Fig. 4.52 on page 4.24.
(b) Biot-Savart law can be expressed as
Ampere's circuital law by considering the (c) The field inside a solenoid can be
surface to be made up of a large number of increased by (i) inserting an iron core
loops. The sum of the tangential inside it (ii) increasing number of turns per
components of the magnetic field unit length, and (iii) increasing the current
multiplied by the length of all such through the solenoid.
elements, gives the result 9. Refer answer to Q. 19 on page 4.44.
∮𝐵 ⃗⃗⃗ = 𝜇0 𝐼
⃗ ⋅ 𝑑𝑙 10. Refer answer to Q. 17 on page 4.40.
5. (a) Refer answer to Q. 9 on page 4.22. 11. For cyclotron, refer answer to Q. 17 on
page 4.40. Cyclotron frequency,
(b) Refer to solution of Example 33 on
page 4.26. 𝑞𝐵
𝑓𝑐 =
(c) A straight conductor of finite length 2𝜋𝑚
cannot by itself form a complete steady As fc is independent of velocity v, so f is
current circuit. Additional conductors independent of the kinetic energy of the
are necessary to close the circuit. particle.
These will spoil the symmetry of the
According to Einstein's special theory of
problem. The difficulty disappears if the
relativity, the mass of a particle increases
conductor is infinitely long.
with its velocity. At high velocities the
6. (a) Refer answer to Q.9 on page 4.22 cyclotron frequency will decrease due to
and Q. 10 on page 4.23. increase in mass. This will throw the
particle out of resonance with the
(b) The magnetic field due to the
oscillatory field. Hence the particles are not
neighbouring turns add up along the
accelerated further.
axis of the solenoid and tend to cancel
out in the perpendicular direction. Thus 12. (a), (b) Refer answer to Q. 21 on page
the field at the exterior mid-point is 4.49.
weak and at the interior, it is uniform
(c) The field due to current I at point P is,
and strong. 𝜇0 𝐼
𝐵 = 2𝜋𝑑
7. (a) Refer answer to Q. 9. on page 4.22
and Q. 11 on page 4.24. This field acts normally into the plane of
paper. According to Fleming's left hand
rule, a force acts on the proton in a
direction away from wire AB.
𝜇0 𝐼 𝜇0 𝐼𝑒𝑣
𝐹 = 𝑒𝑣𝐵sin90° = 𝑒𝑣 =
2𝜋𝑑 2𝜋𝑑
Fig. 4.177
13. Refer answer to Q. 22 on page 4.53.
The solenoid consists of N loops, each of
area A and carrying a current /. Each loop 14. (a) Refer answer to Q. 21 on page
acts as a magnetic dipole of dipole moment 4.49.
m = IA. As the magnetic moments of all (b) (i) According to right hand thumb rule,
loops are aligned along the same direction, the direction of the magnetic moment of
140
the current loop will be normally into the (b) Refer to the solution of Problem 29 on
plane of the paper. page 4.85.
(ii) Torque acting on the loop is maximum (c) A soft iron core makes the field radial. It
when its plane is parallel to the also increases the strength of the
magnetic field. Torque acting on the magnetic field and hence increases the
loop is zero when its plane is sensitivity of the galvanometer.
perpendicular to the magnetic field.
(d) Refer to the solution of Problem 33 on
15. (a) Refer answer to Q. 23 on page page 4.86.
4.57.
141
teacher for the explanation. (a) What values did Deepak have ?
(b) As domestic appliances carry electric
S.No. Description Magnitude of
current of the order of few amperes,
Current
write one safety precaution we should
take while working with them.
1. Domestic Few amperes
Appliance (c) An ammeter of resistance RA is
connected in series with a resistor R
2. Lightning Ten thousand and a battery of emf B and internal
amperes resistance r. The current flowing
through this circuit is IA. What will be
3. Nervous system Microamperes the current flowing through the circuit if
the given ammeter is replaced by an
4. Galvanometer Few milliamperes ideal ammeter and find the percentage
error in measuring the current through
5. Semiconductors Few milliamperes an ammeter ? [CBSE SP 18]
Answers
⃗)
1. (a) 𝐹 = 𝐼(𝑙 × 𝐵 ⃗
F is minimum when 𝑣‖𝐵
The direction of the force is given by 𝐹min = 𝑞𝑣𝐵sin0° = 0
Fleming's left hand rule. For statement,
⃗
F is maximum when 𝑣 ⊥ 𝐵
refer to point 11 of Glimpses on page
4.120. 𝐹max = 𝑞𝑣𝐵sin90°
(b) (i) Adaptation to different situations and = 1.6 × 10−19 × 104
flexible and adjustable attitude. × 0.1 × 1
(ii) Sharing excitement in classroom 3. (a) Critical thinking, hard working, quest
learning with family members. for knowledge.
(c) Avoiding unnecessary arguments in (b) One should not touch electrical
conflicting situations in everyday life. appliances with wet hands, proper
earthing should be used.
2. (a) Values displayed by Seema :
𝜉
Helpfulness, considerate Family : (c) 𝐼𝐴 = 𝑟+𝑅+𝑅
𝐴
Concerned, affectionate Doctor :
𝜀
Human nature, social responsibility For an ideal ammeter, 𝑅𝐴 = 0 ∴𝐼=
𝑟+𝑅
(b) The machinery/technique used in MRI Percentage error in measuring the current
is highly expensive.
𝐼 − 𝐼𝐴 𝑅𝐴
(c) F = qvBsin θ =
× 100 = × 100
𝐼 𝑟 + 𝑅 + 𝑅𝐴
COMPETITION SECTION
Magnetic Effect of Current
GLIMPSES
1. Oersted observation. A compass magnetic effect of current.
needle suffers a deflection when it is
2. Biot-Savart law. According to this law,
placed near a wire carrying an electric
the magnetic field due to a current
current. When the direction of current is
carrying element carrying current I at a
reversed, the direction of deflection of
point P at distance r from it is given by
the needle also reverses. This 𝜇 𝐼𝑑𝑙sin𝜃
conclusively proves that a current 𝑑𝐵 = 0 ⋅ 2
4𝜋 𝑟
carrying conductor produces a
magnetic field around it. This is called
142
magnetic field.
5. Magnetic field of a circular current loop.
The magnetic field of a circular current
loop of radius a carrying current I is
𝜇0 𝐼
(i) At the centre of the loop : 𝐵 = 2𝑎
143
solenoid bent into the form of a closed 1 tesla (T) = 1 N A-1m-1,
ring is called a toroidal solenoid. The
1 gauss (G) = 10-4 T.
magnetic field inside the toroidal
solenoid has a constant magnitude and 14. Lorentz force. The total force, called
tangential direction. It is given by B = μ0 Lorentz force, acting on a charge q
n I where I is the current in the windings moving with velocity v in an electric
and n is the number of turns per unit field £ and magnetic field B is
length. The field lines are concentric
𝐹 = 𝑞(𝐸⃗ + 𝑣 × 𝐵
⃗)
circles.
10. Force on a charge moving in a 15. Motion of charge inside an electric field.
magnetic field. A charge q moving with If a potential difference V is applied
velocity 𝑣 at an angle θ with the between two parallel plates separated
⃗ experiences the by distance d, then electric field set up
magnetic field 𝐵 between the plates is
magnetic Lorentz force,
𝑉
F = qvB sin θ 𝐸=
𝑑
⃗)
In vector notation, 𝐹 = 𝑞(𝑣 × 𝐵 The charge q of mass m experiences the
The direction of this force is perpendicular electric force, F𝑒 = q E
to both v and B, and work done by it is zero. Acceleration produced in the charge, 𝑎 =
This force is maximum when the charged 𝑞𝐸
particle moves perpendicular to the 𝑚
direction of the field (θ =90°) and minimum The moving charge follows a parabolic
when the charged particle moves along the path inside the electric field.
field (θ =0°).
16. Motion of charge inside a magnetic
11. Rules for finding the direction of force field.
on a charge moving perpendicular to a
magnetic field. ⃗ , the magnetic force on the
(i) When 𝑣 ⊥ 𝐵
charge makes it move along a circular path
(i) Fleming's left hand rule. Stretch the of radius,
thumb and the first two fingers of the
𝑚𝑣
left hand mutually perpendicular to 𝑟=
each other. If the forefinger points in 𝑞𝐵
the direction of the magnetic field, (ii) When v makes angle θ with B, the
central finger in the direction of current, perpendicular component : 𝑣⊥ = 𝑣sin𝜃 of
then the thumb gives the direction of the initial velocity' makes the charge move
force on the charged particle. 𝑚𝑣sin𝜃
along a circular path of radius, 𝑟 = 𝑞𝐵
(ii) Right hand palm rule. Open the right
hand and place it so that tips of the The parallel component : 𝑣‖ = 𝑣cos𝜃 of the
fingers point in the direction of the field initial velocity makes it move along the
⃗ and thumb in the direction of velocity
𝐵 direction of the magnetic field. TTie
𝐹 of positive charge, then the palm resultant of the two components makes the
faces towards the force 𝐹 . charge move along a helical path of pitch,
2𝜋𝑚𝑣cos𝜃
12. Definition of magnetic field. The ℎ=
magnetic field at a point may be 𝑞𝐵
defined as the force acting on a unit 17. Cyclotron. It is a device used to
charge moving with a unit velocity at accelerate charged particles like
right angles to the direction of the field. protons, deutrons, α -particles, etc., to
13. SI unit of magnetic field is tesla. One very high energies. Here charged
tesla is that magnetic field in which a particles move along a spiral path
charge of 1 coulomb moving with a under the action of a perpendicular
speed of 1 ms-1 at right angles to the magnetic field and gain energy as they
field experiences a force of 1 newton. cross an alternating electric field again
144
and again. The force is attractive when the currents
are in the same direction and repulsive
18- Cyclotron frequency. In a cyclotron, the
when the currents are in opposite
frequency of the applied alternating
directions.
electric field is equal to the frequency of
revolution of the charged particle. This 22. SI unit of current is ampere. One
frequency is called cyclotron ampere is that strength of current
frequency. which, when flowing in two parallel
𝑞𝐵 infinitely long conductors of negligible
It is given by, 𝑓𝑐 = 2𝜋𝑚 cross-section placed in vacuum at a
distance of 1 m from each other,
where m is the mass and q is the charge of
the positive ion. The cyclotron frequency is produces between them a force of 2 ×
independent of both the velocity of the 10-7 newton per metre length.
particle and the radius of its orbit. 23. Force between two current elements.
19. Maximum energy gained by positive The force between two parallel current
elements dl1 and dl2 carrying currents I1
ions. If v0 and r0 are the maximum
velocity and maximum radius of the and I2 and separated by distance r
much greater than their length is given
circular path of the positive ions in a
by
cyclotron, then
𝜇0 𝐼1 𝐼2 𝑑𝑙1 𝑑𝑙2
𝑚𝑣02 𝑞𝐵𝑟0 𝑑𝐹 = ⋅
= 𝑞𝑣0 𝐵 or 𝑣0 = 4𝜋 𝑟2
𝑟0 𝑚
1
24. Relative sizes of electric and magnetic
Maximum kinetic energy = = 𝑚𝑣02 = fields. Under similar conditions, the
2
𝑞2 𝐵 2𝑟02 magnetic forces are much smaller than
2𝑚 the electric forces.
If V is the accelerating potential of the high 𝐹𝑚 𝑣1 𝑣2
frequency oscillator and the charged = 𝑣1 𝑣2 (𝜇0 𝜀0 ) = 2
𝐹𝑒 𝑐
particle completes n revolutions before
leaving the dees, then Maximum kinetic 10−5 × 10−5
= ≃ 10−27
energy = 2nqV. (3 × 108 )2
20. Force on a current carrying conductor Here v1 and v2 are drift speeds of the
in a magnetic field. A conductor of electrons in the two conductors.
length / carrying current I held in a
25. Torque on current carrying coil in a
magnetic field B at an angle θ with it,
magnetic field. A rectangular coil of
experiences a force given by 𝐹 =
area A, carrying current I and capable
𝐼𝑙𝐵sin𝜃
of rotation about an axis perpendicular
⃗)
In vector notation, 𝐹 = 𝐼(𝑙 × 𝐵 to the field B experiences a torque,
The direction of F is perpendicular to both 𝜏 = NIB A sin θ = m B sin θ
and B and is given by Fleming's left hand where N = number of turns in the coil, m
rule. The force is maximum when θ=90° - NIA - magnetic dipole moment, θ = angle
and zero when θ =0° or 180°. which the normal to the plane of the coil
𝐹max = 𝐼𝐼𝐵 makes with the field B.
21. Force between two parallel infinitely In vector rotation, 𝜏 = 𝑚 ⃗
⃗⃗ × 𝐵
long current carrying conductors. The
Torque is minimum when the plane of the
force per unit length between two long
coil is perpendicular to the magnetic field (θ
parallel conductors carrying currents 1↑
=0°). Torque is maximum when the plane
and I2 and separated by distance r is
of the coil is parallel to the magnetic field (θ
given by
=90°). x =NIBA.
𝜇0 𝐼1 𝐼2
𝑓= ⋅ 𝜏max = 𝑁𝐼𝐵𝐴
2𝜋 𝑟
145
26. Moving coil galvanometer. It is a device 30. Conversion of a galvanometer into an
used to detect current in a circuit. It is ammeter. An ammeter is an instrument
based on the principle that a current used to measure electric current in a
carrying coil placed in a magnetic field circuit. A galvanometer of resistance G
experiences a current dependent can be converted into an ammeter of
torque, which tends to rotate the coil range 0 - I by connecting a small
and produces angular deflection. It resistance S in parallel with it. The
consists of a coil of wire of area A and value of small resistance, called shunt,
N turns carrying current I to be is given by
measured. It is suspended in a radial 𝐼𝑔
magnetic field so that its plane always 𝑆= ×𝐺
⃗ (sin𝜃 = 1) by a 𝐼 − 𝐼𝑔
remains parallel to 𝐵
suspension fibre of torsion constant k. where Ig is the current with which
In equilibrium position, Restoring galvanometer gives full-scale deflection.
torque = Deflecting torque
Total resistance of an ammeter,
𝑁𝐵𝐴
or 𝑘𝛼 = 𝑁𝐼𝐵𝐴 or 𝛼= .𝐼 𝐺𝑆
𝑘
𝑅𝐴 =
i.e. Deflection of coil ∝ Current in the coil. 𝐺 +𝑆
An ammeter is a low resistance device and
27. Figure of merit of a galvanometer. It is
is connected in series in a circuit.
the current which produces a deflection
of one scale division in the 31. Conversion of galvanometer into a
galvanometer. It is given by voltmeter. A voltmeter is used to
𝐼 𝑘 measure potential difference between
𝐺= = any two points of a circuit. A
𝛼 𝑁𝐵𝐴 galvanometer of resistance G can be
28. Current sensitivity of a galvanometer. It converted into a volttneter of range 0 -
is the deflection produced in a V by connecting a large resistance R in
galvanometer when a unit current flows series with it. The value of R is given by
through it. 𝑉
𝛼 𝑁𝐵𝐴 𝑅= −𝐺
Current sensitivity = = 𝐼𝑔
𝐼 𝑘
146
𝜇0 𝐼 1 1 𝜇0 𝐼 1 1
(𝑐) ( −𝑅 ) (𝑑) ( −𝑅 )
2 𝑅1 2 4 𝑅1 2
[IIT2002]
6. An infinitely long conductor PQR is
bent to form a right angle as shown. A
current I flows through PQR.
147
Y-axis. The electron exits from the region
after some time with the speed v at
ordinate y, then
(i) v > u, y < 0 (b) v = u, y > 0
(c) v > u, y > 0 (d) v = u, y < 0
[IIT 2004]
12. For a positively charged particle
moving in a x-y plane initially along the
x-axis, there is a sudden change in its path
due to the presence of electric and/or
148
(a) straight line (b) circle (a) contract (b) expand
(c) helix (d) cycloid (c) move towards +ve x-axis
[IIT 1999] (d) move towards -ve x-axis.
15. An ionized gas contains both [IIT 2003]
positive and negative ions. If it is subjected
18. A conducting circular loop of radius r
simultaneously to an electric field along the
carries a constant current i. It is placed
+x direction and a magnetic field along the
in a uniform magnetic field B0 such that
+z direction, then
B0 is perpendicular to the plane of the
(a) positive ions deflect towards +y loop. The magnetic force acting on the
direction and negative ions towards -y loop is
direction
(a) 𝑖𝑟𝐵𝑜 (b) 2𝜋𝑖𝑟𝐵0
(b) all ions deflect towards + y direction
(c) zero (d)πirBo
(c) all ions deflect towards -y direction
[IIT 1983]
(d) positive ions deflect towards -y
19. A uniform magnetic field with a slit
direction and negative ions towards + y
system as shown in figure is to be used as
direction.
a momentum filter for high-energy charged
[IIT 2000] particles. With a field B tesla, it is found that
the filter transmits α-particles each of
16. Two particles A and B of masses
mA and mB respectively and having the
same charge are moving in a plane. A
uniform magnetic field exists perpendicular
149
given by
............................................................................which results in the lowering of the potential of the face
............................................................................Assume the speed of the carriers to be v.
(a) evB, face ABCD (b) evB, face EFGH
(c)2evB, face ABCD (d)2evB, EFGH [IIT
1996]
21.A thin flexible wire of length L is
connected to two adjacent fixed points and
carries a current 1 in the clockwise
direction, as shown in the figure. When the
system is put in a uniform magnetic field of
strength B going into the plane of the
paper, the wire takes the shape of a circle.
Multiple Choice Questions with one or
more than one Correct Answers
24. A current I flows along the length of an
infinity long, straight, thin-walled pipe.
The tension in the wire is Then
𝐼𝐵𝐿 (a) The magnetic field at all points inside
(a) 𝐼𝐵𝐿 (b) the pipe is the same, but not zero
𝜋
𝐼𝐵𝐿 𝐼𝐵𝐿 (b) The magnetic field at any point inside
(𝑐) (𝑑) the pipe is zero
2𝜋 4𝜋
[IIT2010] (c) The magnetic field is zero only on the
axis of the pipe
22. Which of the field patterns given
below is valid for electric field as well as for (d) The magnetic field is different at
magnetic field ? different points inside the pipe. [1IT
1993]
25.A steady current I flows along an
infinitely long hollow cylindrical conductor
of radius R. This cylinder is placed
coaxially inside an infinite solenoid of
radius 2 R. The solenoid has n turns per
unit length and carries a steady current 1.
Consider a point P at a distance r from the
common axis.
The correct statement(s) is (are)
(a) In the region 0 < r < R, the magnetic
field is non-zero.
23. A infinitely long hollow conducting (b) In the region R<r<2R, the magnetic
cylinder with inner radius R/2 and outer field is along the common axis.
radius R carries a uniform current density
(c) In the region R <r <2 R, the magnetic
along its length. The magnitude of
field is tangential to the circle of radius
magnetic field, | B | as a function of the
r, centered on the axis.
radial distance r from the axis is best
represented by (d) In the region r>2R, the magnetic field is
non-zero. [JEE Adv. 13J
26. Two particles X and Y having equal
150
charges, after being accelerated through normal to the boundary as shown in the
the same potential difference enter a figure. Region II has a uniform
region of uniform magnetic field and magnetic
describe circular paths of radii ^ and r2
respectively. The ratio of the mass of X to
that of Y is
𝑟 1 ⁄2 𝑟
(𝑎) ( 1 ) (𝑏) ( 2 )
𝑟2 𝑟1
𝑟 2 𝑟 field B perpendicular to the plane of the
(𝑐) (𝑟1 ) (𝑑) ( 1 ) paper. The length of the Region II is /.
2 𝑟2
Choose the correct choice(s).
[IIT 1988]
(a) The particle enters Region III only if its
27. A proton moving with a constant 𝑞𝑙𝐵
velocity 𝑣 > 𝑚
velocity passes through a region of space
without any change in its velocity. If E and (b) The particle enters Region III only if its
B represent the electric and magnetic fields 𝑞𝑙𝐵
velocity 𝑣 < 𝑚
respectively, this region of space may have
(c) Path length of the particle in Region II is
(a) E = 0, B = 0 (b) E = 0, B≠O 𝑞𝑙𝐵
maximum when velocity 𝑣 = 𝑚
(c) E≠0, B = 0 (d) E≠0, B≠ 0.
[IIT 1985] (d) Time spent in Region II is same for any
28. An electron and a proton are v as long as the particle returns to
moving on straight parallel paths with same Region I.
velocity. They enter a semiinfinite region of 31. A particle of charge +q and mass m
uniform magnetic field perpendicular to the moving under the influence of a uniform
velocity. Which of the following(s) is/are electric field 𝐸 𝚤̂ and uniform magnetic
true ? field 𝐵𝑘̂ follows a trajectory from P
(a) they will never come out of the
magnetic field region
(b) they will come out travelling along
parallel paths
(c) they will come out at the same time.
(d) they will come out at different times, [in
2011]
to Q as shown in the figure. The velocities
29. H+ , He+ and O2+ all having the at P and Q are 𝑣𝑖̂ and − 2𝑣𝑗̂ . Which of the
same kinetic energy pass through a region following statement(s) is/are correct ?
in which there is a uniform magnetic field
3 𝑚𝑣 2
perpendicular to their velocity. The masses (a) 𝐸 = [ ]
2+ 4 𝑞𝑎
of H+ , He+ and O are 1 amu, 4 amu and
16 amu respectively. The (b) Rate of work done by the electric field
3 𝑚𝑣 3
4- at P is 4 [ ]
(a) H will be deflected most 𝑎
(b) O2+ will be deflected most (c) Rate of work done by the electric field
at P is zero.
(c) He+ and O2+ will be deflected
equally (d) Rate of work done by both the fields at
Q is zero. [IIT 1991]
(d) All will be deflected equally.
[IIT 1994] 32. Consider the motion of a positive point
charge in a region where there are
30. A particle of mass m and charge q, simultaneous uniform electric and
moving with velocity v enters Region II
151
magnetic fields 𝐸⃗ = 𝐸0 𝑗̂ and 𝐵⃗ = 𝐵0 𝑗̂ . 𝜇𝑜 𝐼𝑞𝑣 𝜇𝑜 𝐼𝑞𝑣
(𝑎) (𝑏)
At time t = 0, this charge has velocity v 2𝜋𝑟 𝜋𝑟
in the x-y plane, making an angle θ with 2𝜇0𝐼𝑞𝑣
(𝑐) (d) 0
x-axis. Which of the following option(s) 𝜋𝑟
is (are) correct for time t > 0 ?
(a) If θ =0°, the charge moves in a circular [IIT1998]
path in the x-z plane.
35. A wire ABCDEF (with each side of
(b) If θ = 0°, the charge undergoes helical length L) bent as shown in the figure and
motion with constant pitch along the carrying a current I is
y-axis.
(c) If θ = 10°, the charge undergoes helical
motion with its pitch increasing with
time, along the y-axis.
(d) If θ=90°, the charge undergoes linear
but accelerated motion along the
y-axis. [IIT 12]
33. A particle of mass M and positive
charge Q, moving with a constant
𝑢1 = 4𝑖̂ms−1 , enters a region of
velocity ⃗⃗⃗⃗
uniform static magnetic field, normal to placed in a uniform magnetic induction B
the x-y plane. The region of the parallel to the positive y-direction.
magnetic field extends from x = 0 to x -
(a) The force experienced by the wire is
E for all values of y. After passing
ILB
through this region, the particle
emerges on the other side after 10 (b) The force experienced by the wire is 3
milliseconds with a velocity 𝑢 ⃗⃗⃗⃗2 = ILB
−1
2(√3𝑖 + 𝑗̂)ms . The correct (c) The net force on the wire is in the
statement(s) is (are) negative z-direction
(a) The direction of the magnetic field is -z (d) The net force on the wire is in the
direction. positive z-direction.
(b) The direction of the magnetic field is +z 36. A rectangular loop carrying a
direction. current I is situated near a long straight
(c) The magnitude of the magnetic field is wire such that the wire is parallel to one of
50𝜋𝑀 the sides of the loop and is in the
units.
3𝑄
152
[IIT 1985] radius a with two long parallel wires
(numbered 1 and 2) all in the
37.A conductor (shown in the figure)
carrying current 1 is kept in the x-y plane in
a uniform magnetic
39.A microammeter has a resistance of (d) current in wire 1 and wire 2 is in the
100 Ω and a full scale range of 50 μA. It can direction PQ and RS, respectively and
be used as a voltmeter or as a higher range ℎ ≈ 1.2 𝑎
ammeter provided a resistance is added to [JEE Adv. 14]
it. Pick the correct range and resistance
combination(s). 41.Consider d » a, and the loop is
rotated about its diameter parallel to the
(a) 50 V range with 10 kΩ resistance in wires by 30° from the position shown in the
series figure. If the centres in the wires are in the
(b) 10 V range with 200 kΩ resistance opposite directions, the torque on the loop
in series at its new position will be (assume that the
net field due to the wires is constant over
(c) 5 mA range with 1 Ω resistance in the loop)
parallel
𝜇0 𝐼 2 𝑎 2 𝜇0 𝐼 2 𝑎2
(d) 10 mA range with 1 Ω resistance in (a) (b)
𝑑 2𝑑
parallel.
√3𝜇0 𝐼 2 𝑎2 √3𝜇0 𝐼 2 𝑎2
[IIT 1991] (𝑐) (𝑏)
𝑑 2𝑑
Linked Comprehension Type
[JEE Adv. 14]
Paragraph for Questions 40 and 41
Paragraph for Questions 42 and 43
The figure shows a circular loop of
153
In a thin rectangular metallic strip a between K and M in strips 1 and
constant current I flows along the positive 2, respectively. Assuming that the
x-direction, as shown in the figure. The current I is the same for both the strips, the
length, width and thickness of the strip are correct option(s) is (are) [JEE Adv.
I, w and d, respectively. A uniform 15]
magnetic field B is applied on the strip
(a) If 𝐵1 = 𝐵2 and 𝑛1 = 2𝑛2 , then 𝑉2 =
along the positive y-direction. Due to this
2𝑉1
the charge carriers experience a net
deflection along the z-direction. This (b) If 𝐵1 = 𝐵2 and 𝑛1 = 2𝑛2 , then 𝑉2 =
results in accumulation of charge carriers 𝑉1
on the surface PQRS and appearance of
(c) If 𝐵1 = 2𝐵2 and 𝑛1 = 𝑛2 , then 𝑉2 =
equal and opposite charges on the face
0.5𝑉1
opposite to PQRS. A potential difference
along the z-direction is thus developed. (d) If 𝐵1 = 2𝑅2 and 𝑛1 = 𝑛2 , then 𝑉2 =
Charge accumulation continues until the 𝑉1
magnetic force is balanced by the electric
Integer Answer Type
force. The current is assumed to be
uniformly distributed on the cross-section 44. A steady current I goes through a
of the strip and carried by electrons. wire loop PQR having shape of a right
angle triangle with PQ = 3x, PR = 4x and
QR = 5x. If the magnitude of the magnetic
𝜇0 𝐼
field at P due to this loop is 𝑘 (48𝜋𝑥 ) find the
value of k. [IIT 09]
154
47. A galvanometer gives full scale 2𝑛
𝛺
resistance, it becomes an
249
deflection with 0.006 A current. By
ammeter of range 0 -1.5 A The value of
connecting it to a 4990 Ω resistance, it
n is ____ . [JEE Adv. 14]
can be converted into a voltmeter of
range 0 -30 V. If In connected to a
Match - Matrix Type
48. Six point charges, each of the same magnitude q, are arranged in different manners as
shown in Column II.
In each case, a point M and a line PQ passing through M are shown. Let E be the electric field
and V be the electric potential at M (potential at infinity is zero) due to the given charge
distribution when it is at rest. Now, the whole system is set into rotation with a constant angular
velocity about the line PQ. Let B be the magnetic field at M and μ be the magnetic moment of
the system in this condition. Assume each rotating charge to be equivalent to a steady current.
[IIT 09]
Column I Column II
(q)
(r)
155
Charges are placed on two coplanar, identical
insulating rings at equal intervals. M is the
mid-point between the centres of the rings. PQ
is perpendicular to the line joining the centres
(t) and coplanar to the rings.
156
1
= = 2 𝐵1
Total field at M is
1 3
𝐵2 = 𝐵1 + 𝐵′ = 𝐵1 + 𝐵1 = 𝐵1
2 2
𝐵1 2
=
𝐵2 3
7. (d) The magnetic field at the point 11. (d) The magnetic force does not
P(a,0,a) is equal to the vector sum of the change the speed of electron. Hence v = u.
fields B1 and B2, produced by the loops
ΛBCDA and AFEBA respectively. The According to Fleming's left hand rule,
this force acts on the electron in the
negative Y-direction. As a result, the
electron follows a circular path in clockwise
direction. Hence when it exits from the
field, y < 0.
magnitudes of 𝐵 ⃗ 1 and 𝐵
⃗ 2 are equal. But 𝐵
⃗1
acts alone X-axis and B2 acts along Z-axis.
Hence the resultan field at point P will be in
1
the direction of (𝑖̂ + 𝑘̂)
√2
8. (b) At the centre C, net field = 0 12. (b) Electric field is given to be either
zero or acting along x-direction. It cannot
deflect the charged particle along -ve
y-direction. Hence only magnetic field is
responsible for the curved path.
In the region AX, fields of both wires
are in the downward direction. So net field Options (a) and (c) are not acceptable
is in the downward direction. because the path will be helix in these
cases (when the velocity vector makes an
Similarly, the net field in the region BX' angle other than 0°, 90° or 180° with the
is in the upward direction. magnetic field).
In the region AC, BA > BB and BA acts
upwards while BB acts downwards. The net
Now
𝐹 net = 𝑞𝐸⃗ + 𝑞(𝑣 × 𝐵
⃗)
field is upwards. Similarly, the net field is As initial velocity is along x-direction, so
downwards in region CB. let 𝑣 = 𝑣𝑙̂
Only graph (b) satisfies all the above In option (d), we then have
conditions.
𝐹net = 𝑞𝑎𝑖̂ + 𝑞[(𝑣𝑖̂) × (𝑐𝑘̂ + 𝑏𝑗̂)]
9. (a) Refer to the solution of Problem 19
on page 4.94. = 𝑞𝑎𝑖̂ − 𝑞𝑣𝑐𝑗̂ + 𝑞𝑢𝑏𝑘̂
10. (b) The required minimum velocity v is Thus the option (d) is not acceptable
given by because Fnet has a component along
z-direction while the motion of the particle
𝑚𝑣 2 is in the x - y plane.
= 𝑞𝑣𝐵
𝑟
𝑚𝑣 In option (b),
or = 𝑞𝐵
𝑏−𝑎
𝐹net = 𝑞𝑎𝑖̂ + 𝑞[(𝑣𝑖̂) × (𝑐𝑘̂ + 𝑎𝑖̂)]
𝑞(𝑏−𝑎)𝐵
or 𝑣 = = 𝑞𝑎𝑖̂ − 𝑞𝑣𝑐𝑗̂
𝑚
157
Thus Fnet acts in the x-y plane. system,
Hence only option (b) is the correct Centripetal force = Force exerted by
option. magnetic field
13. (d) Refer to the solution of Problem 20 𝑚𝑣 2
or = 𝑞𝑣𝐵 or 𝑝 = 𝑚𝑣 = 𝑞𝐵𝑟
on page 4.94. 𝑟
17. (b) Applying Fleming's left hand rule, (i) a current carrying toroid produces
we find that a force acts in the radially magnetic field lines of such pattern.
outward direction on every section of (ii) a changing magnetic field with respect
the conducting loop. to time in a region perpendicular to the
18. (c) As in the above problem, every paper produces induced electric field
section of the loop experiences a lines of such pattern.
radially outward force. But forces acting 23. (d)
on diametrically opposite sections
cancel out in pairs. Hence the net force (i) For r < R/2, B = 0
on the loop is zero. (ii) For R/2 ≤ r < R,
19. (b) For particles transmitted by filter 𝐵 × 2𝜋𝑟 = 𝜇0 𝐼 = 𝜇0 [𝜋𝑟 2 − 𝜋(𝑅/2)2 ]𝑗
158
𝜇0 𝑗 𝑅2 (d) 𝑟 > 2𝑅, 𝐵𝑆 = 0, so 𝐵 = 𝐵𝐶 > 0
𝐵= [𝑟 − ]
2 4𝑟 1 2𝑞𝑉
26. (c) 2 𝑚𝑣 2 = 𝑞𝑉 or 𝑣=√
(iii) For 𝑟 ≥ 𝑅 𝑚
𝜇0 𝐼
𝐵= 𝑚𝑣 𝑚 2𝑞𝑉 √2𝑚𝑞𝑉
2𝜋𝑟 𝑟= = √ =
𝑞𝐵 𝑞𝐵 𝑚 𝑞𝐵
𝜇0 𝑗 𝑅 2
= [𝜋𝑅2 − 𝜋 ( ) ]
2𝜋𝑟 2 For same q, v and B,
𝑟 ∝ √𝑚
or m ∝ r2
𝑚 𝑟
∴𝑚𝑋 = (𝑟1 )2
𝑌 2
159
𝑟H+ ∝ 1; 𝑟He+ ∝ 2
rO2+ ∝ 2
Thus He+ and 02+ will be deflected
equally.
But H+ will be deflected the most
because of its smallest radius.
30. (a), (c) and (d)
Time spent in region II [Fig. (iii)] before
the particle returns to region I,
1 2𝜋𝑚
𝑇=
2 𝑞𝐵
Radius of the circular path, This time is independent of v. Option (d) is
𝑚𝑣 correct.
𝑅=
𝑞𝐵 31. (a), (b) and (d) In going from P to Q
The particle will enter region III Increase in K.E. = Work done by electric
[Fig. (i)], when field = Electric force × distance
𝑚𝑣 1 1
or 𝑚(2𝑣)2 − 𝑚𝑣2 = 𝑞𝐸 × 2𝑎
𝑅 > 𝑙 or >𝑙 2 2
𝑞𝐵
3 𝑚𝑣 2
or 𝑣 >
𝑞𝑙𝐵 or 𝐸 = 4 𝑞𝑎
𝑚
∴ Option (a) is correct and option (b) is Rate of work done by electric field at P =
wrong. Electric force × velocity
160
If θ=90°, then Fβ =0 and due to qEQ Hence the net force on the wire is ILB in
motion is accelerated along y-axis and so the positive z-direction.
option (d) is correct.
36. (c) Refer to solution of Problem 27 on
33. (a), (c) page 4.84.
The component 2 j of the final velocity 37. (a),(b),(c)
u2 indicates that the magnetic force acts on
⃗ is uniform, the conductor can be
As 𝐵
the particle along +y axis and centre of the
circle lies on this axis. By Fleming's left treated as a straight conductor of length
hand rule, field Facts along -z direction. 2(L+ R).
⃗ ) = 𝐼[2(𝐿 + 𝑅)𝑖̂ × 𝐵
𝐹 = 𝐼(𝑙 × 𝐵 ⃗]
⃗ is along x,
If 𝐵
𝐹 = 2𝐼(𝐿 + 𝑅)𝐵𝑖̂ × 𝑖̂ = ⃗0
⃗ is along y,
If 𝐵
𝐹 = 2𝐼(𝐿 + 𝑅)𝐵𝑖̂ × 𝑗̂ = 2𝐼(𝐿 + 𝑅)𝐵𝑘̂
𝑢 (𝑦) 2 1 𝐹 ∝ (𝐿 + 𝑅)
Clearly, tan𝜃 = 𝑢2 (𝑥) = 2√3 = ⇒𝜃=
2 √3
𝜋 ⃗ is along z,
If 𝐵
6
𝜃 𝑄𝐵 𝐹 = 2𝐼(𝐿 + 𝑅)𝐵𝑖̂ × 𝑘̂ = 2𝐼(𝐿 + 𝑅)𝐵(−𝑗̂)
Now, 𝜔= =
𝑡 𝑀 => T ∝ (L+ R).
𝑀𝜃 𝑀×𝜋 𝐺+𝑆 100+0.1
𝐵= = 38. (a) 𝐼 = ⋅ 𝐼𝑔 = × 100𝜇𝐴
𝑡𝑄 10 × 10−3 × 6 × 𝑄 𝑆 0.1
50𝜋𝑀 = 100100μA = 100.1 mA.
= units.
3𝑄
39. (b) and (c)
34. (d) Magnetic field due to both the wires
is directed normally into the plane of For S = 200 kΩ,
paper at point P. Also, the direction of V=Ig(G+S)
motion of charge q is normal to the
= 50 × 10-6 (100 + 200000) = 10 V.
plane of paper.
For R = 1Ω,
That is,
𝐺+𝑆 100 + 1
⃗
𝑣‖𝐵 𝐼=( )𝐼𝑔 = ( )50 × 10−6 A
𝑆 1
⃗)=0
𝐹𝑀 = 𝑞(𝑣 × 𝐵 = 5 mA.
40. (c) The direction of magnetic field at
the given point due to the loop is normally
out of the plane. Therefore, the net
magnetic field due the both wires should be
into the plane. For this current in wire I
should be along PQ and that in wire RS
35. (a) and (d) Force on AB an EF is should be along SR.
zero because angle between 𝐿 ⃗ and 𝐵
⃗ is Field due to each wire at point
zero. 𝜇0 𝑙 𝜇0 𝑙
𝐵1 = 𝐵2 = =
⃗ ×𝐵
𝐹𝑚 = 𝐼(𝐿 ⃗)=0 2𝜋𝑟 2𝜋√𝑎 2 + ℎ2
Forces on BC and DE are equal and
opposite. According to Fleming's left hand
rule, the force on CD is ILB in the positive
z-direction.
161
𝐼
=> 𝑤𝑣 =
𝑒𝑛𝑑
𝐼
𝑉𝑀 − 𝑉𝐾 = 𝐵
𝑒𝑛𝑑
If 𝑤1 = 𝑤2 and 𝑑1 = 2𝑑2 , then 𝑉2 = 2𝑉1
=> option (a) is correct
If 𝑤1 = 2𝑤2 and 𝑑1 = 𝑑2 , 𝑡ℎ𝑒𝑛𝑉1 = 𝑉2
=> option (d) is correct
43. (a), (c)
𝑉 𝑛 𝐵
Here 𝑉1 = 𝑛2𝐵1
2 1 2
𝑉1
If 𝐵1 = 𝐵2 and 𝑛1 = 2𝑛2 then then =
𝑉2
𝑛2 𝐵 1
Resultant of 𝐵1 and 𝐵2 = 2𝐵1 cos𝜃 2𝑛2
⋅
𝐵
=
2
𝜇0 𝐼 𝑎 𝜇0 𝑙𝑎 ⇒ V2 =2 V1
=2 ⋅ =
2𝜋√𝑎 2 + ℎ2 √𝑎 2 + ℎ2 𝜋(𝑎 2 + ℎ2 )
∴Option (a) is correct.
Field due to the loop at the point A
𝜇0𝐼𝑎 2 If 𝐵1 = 2𝐵2 and 𝑛1 = 𝑛2
2(𝑎 2+ℎ 2)3⁄2 𝑉 𝑛 2𝐵2
Then 𝑉1 = 𝑛2 ⋅ =2
𝐵2
The net magnetic field at the given 2 2
5.87𝑎 2
𝑟 3
⇒ ℎ2 = 1.47𝑎 2 ⇒ ℎ = 1.2𝑎 sin𝛼 = =
4𝑥 5
41. (b) 12𝑥
or 𝑟 =
𝜇0 𝐼 5
𝜏 = 𝑚𝐵sin𝜃 = 𝐼𝜋𝑎 2 × 2 × sin30°
2𝜋𝑑
𝜇0 𝐼 2 𝑎 2
=
2𝑑
42. (a), (d)
FB = F E
𝑒(𝑉𝑀 − 𝑉𝐾 )
𝑒𝑣𝐵 =
𝑤
[u = Drift velocity of electrons]
V𝑀 - V𝐾 = w v B 𝜇0 𝐼
𝐵 = 𝐵𝑄𝑅 = (SIN𝛼 + SIN𝛽)
But I = e n A v = e n ( w d ) v 4𝜋𝑟
162
𝜇0 𝐼 3 𝑡
= ⋅ ( + )
4𝜋 12𝑥 5 5
5
𝜇0 𝐼 𝜇0 𝐼
𝑘( ) = 7( ) ∴ 𝑘=7
48𝜋𝑥 48𝜋𝑥
45.
0005
𝜇 𝐼 𝜇0 𝑗𝜋𝑅 2 𝜇𝑄 𝑗 ′
For r ≤ R, 𝐵 = 2𝜋𝑅
0
2 ⋅𝑟 = ⋅𝑟 = When the wires carry currents in the
2𝜋𝑅 2 2
opposite directions,
0 𝜇 𝐼 𝜇0 𝑗𝜋𝑅 2 𝜇0𝑗𝑅 2
For r> R, 𝐵 = 2𝜋𝑟 = = 𝜇0 𝐼 1 1
2𝜋𝑟 2𝑟
𝐵2 = [ + ]
2𝜋 𝑥1 𝑥0 − 𝑥1
𝜇0 𝐼 3 3 9𝜇0 𝐼
= [ + ]=
2𝜋 𝑥0 2𝑥0 4𝜋𝑥0
𝑚𝑣 1
𝑅= ⇒ 𝑅∝
𝑞𝐵 𝐵
𝑅1 𝑅 9
∴ = 𝐵2 = 3 = 3
𝑅2 1
47.
Magnetic field due to the cylinder at P is
𝜇0 𝑗𝑎 0005
𝐵1 =
2 For voltmeter : Ig = 0.006 A, R = 4990 Ω, V
=30 V
Magnetic field due to the cavity at P is
𝑎
𝜇0 𝑗( 2)2 𝜇0 𝑗𝑎
𝐵2 = =
3𝑎 12
2( )2 𝑉 = 𝐼𝑔 (𝑅𝑔 + 𝑅)
2
Net magnetic field at point P is 30 = 0.006 (Rg +4990)
𝜇0 𝑗𝑎 1 30 × 1000
𝐵 = 𝐵1 − 𝐵2 = (1 − ) 𝑅8 + 4990 = = 5000
2 6 6
5𝜇0 𝑗𝑎 𝑁𝜇0 𝑗𝑎 Rg = 10 Ω
= =
12 12 For ammeter : I = 1.5 A, Rg = 10 Ω
Hence, N = 5.
46.
0003
When the wires carry currents in the same
direction,
𝜇0 𝐼 1 1 𝐼𝑔 𝑅𝑔 = (𝐼 − 𝐼𝑔 )𝑅𝑠
𝐵1 = [ − ]
2𝜋 𝑥1 𝑥0 − 𝑥1 2𝑛
0.006×10 = (1.5-0.006)× 249
𝜇0 𝐼 3 3 3𝜇0 𝐼
= [ − ]= 0.06 × 249
2𝜋 𝑥0 2𝑥0 4𝜋𝑥0 𝑛= =5
1.494 × 2
48. A → p, r, s; B→ r, s ; C → p, q, t and D → r, s.
163
(p) From vector addition, electric field at M will be zero. On
rotating net current is zero, hence B = 0, μ =0
JEE Main
1. If in a circular coil A of radius R, the centre is 54 μT. What will be its value at
current I is flowing and in another coil B of the centre of the loop?
radius 2 R, a current 21 is flowing ; then the
(a) 250 μT (b) 150 μT
ratio of the magnetic fields BA and BB
produced by them will be (c) 125 μT (d)75 μT
[AIEEE 2004]
(a) 1 (b) 2 (c) 1/2 (d) 4
3. Two concentric coils each of radius
[AIEEE 2002]
equal to 2π cm are placed at right angle to
2. The magnetic field due to a current each other. 3 A and 4 A are the currents
carrying circular loop of radius 3 cm at a flowing in each coil respectively. The
point on the axis at a distance of 4 cm from magnetic induction (in Wb m -2) at the
164
centre of the coils (μ0 =4π×10-7 Wb A-1 m-1) the pipe is zero. [AIEEE 2007]
will be
8. Two identical conducting wires AOB
(a) 12 × 10-5 (b) 10-5 and COD are placed at right angles to each
other. The wire AOB carries an electric
(c) 5× 10-5 (d) 7 × 10-5
current Iv and COD carries a current Ir The
[AIEEE 2005] magnetic field on a point lying at a distance
'd' from O, in a direction perpendicular to
4. A long wire carries a steady current.
It is bent into a circle of one turn and the the plane of the wires AOB and COD, will
magnetic field at the centre of the coil is B. be given by
It is then bent into a circular loop of n turns. 𝜇𝑜 2 𝜇 𝐼 + 𝐼2 2
(𝑎) (𝐼1 + 𝐼22 ) (𝑏) 𝑜 ( 1 )
The magnetic field at the centre of the 2𝜋 2𝜋 𝑑
coil will be 𝜇 𝜇
(𝑐) 𝑜 (𝐼12 + 𝐼22 ) 1⁄2 (𝑑) 𝑜 (𝐼1 + 𝐼2 )
2𝜋𝑑 2𝜋𝑑
(a) nB (b)n2B
9. A long straight wire of radius 'a'
(c) 2nB (d) 2n2 B carries a steady current 1. The current is
[AIEEE 2004] uniformly distributed across its
cross-section. The ratio of the magnetic
5. A long solenoid has 200 turns per cm 𝑎
field at 2 and 2 a is
and carries a current I. The magnetic field
at its centre is 6.28 × 10 -2 Wb m-2. Another 1
(a) 2 (b) 1
long solenoid has 100 turns per cm and it
carries a current - The value of the (c) 2 (d)
1
magnetic field at its centre is 4
165
12. A charge Q is uniformly distributed 15. A charged particle moves through a
over the surface of non-conducting disc of magnetic field perpendicular to its
radius R. The disc rotates about an axis direction. Then
perpendicular to its plane and passing
(a) kinetic energy changes but the
through its centre with an angular velocity
momentum is constant
ω. As a result of this rotation a magnetic
field of induction B is obtained at the centre (b) the momentum changes but the kinetic
of the disc. If we keep both the amount of energy is constant
charge placed on the disc and its angular (c) both momentum and kinetic energy of
velocity to be constant and vary the radius the particle are not constant
of the disc, then the variation of the
magnetic induction at the centre of the disc (d) both momentum and kinetic energy of
will be represented by the figure the particle are constant. [AIEEE
2Q07]
[AIEEE 12]
16. A charged particle with charge q enters
a region of constant uniform and
mutually orthogonal fields 𝐸⃗ and 𝐵⃗ with
a velocity v perpendicular to both 𝐸⃗ and
⃗ and comes out without any change in
𝐵
magnitude or direction of if. Then
⃗ × 𝐸⃗
𝐵 𝐸⃗ × 𝐵 ⃗
(𝑎)𝑣 = 2 (𝑏)𝑣 = 2
𝐸 𝐵
⃗ × 𝐸⃗
𝐵 𝐸⃗ × 𝐵⃗
(𝑐)𝑣 = (𝑑)𝑣 =
𝐵2 𝐸2
[AIEEE 2007]
17. If an electron and a proton having
same momenta enter perpendicularly to a
13. A uniform electric field and a magnetic field, then
uniform magnetic field are acting along the (a) curved path of electron and proton will
same direction in a certain region. If an be same (ignoring the sense of
electron is projected along the direction of revolution)
the fields with a certain velocity, then
(b) they will move undeflected
(a) its velocity will decrease
(c) curved path of electron is more curved
(b) its velocity will increase than that of proton
(c) it will turn towards right of direction (d) path of proton is more curved.
of motion
[AIEEE 2002]
(d) it will turn towards left of direction of
motion. 18. A particle of charge -16 × 10-18
coulomb moving with velocity 10 ms-1
[AIEEE 2005 ; CBSE PMT 2011] along the X-axis enters a region, where a
14. In a region, steady and uniform magnetic field of induction B is along the
electric and magnetic fields are present. Y-axis and an electric field of magnitude
These two fields are parallel to each other. 104 Vm-1 is along the negative Z-axis. If the
A charged particle is released from rest in charged particle continues moving along
the region. The path of the particle will be the X-axis, the magnitude of B is
(a) ellipse (b) circle (a) 103 Wb m-2 (b) 105 Wb m-2
166
19. A particle of mass M and charge Q
moving with velocity v describes a
circular path of radius R, when
subjected to a uniform transverse
magnetic field of induction B. The work
done by the field, when the particle
completes one full circle is
by width 'd'. If 'a' be the angle of deviation
𝑀𝑣 2 of proton from initial direction of motion
(a) ( 𝑅 )× 2𝜋𝑅 (b) zero
(see figure), the value of sin α will be
(c) BQ(2πR) (d) BQv(2πR) 𝐵 𝑞 𝑞
[AIEEE2003] (a) √ (b) 𝐵𝑑√
𝑑 2𝑚𝑉 2𝑚𝑉
20. A charged particle of mass m and
charge q moves along a circular path of 𝐵 𝑞𝑑 𝐵𝑑
(c) √ (d) 𝑞𝑉 √
radius r that is perpendicular to a magnetic 2 𝑚𝑉 2𝑚
field B. The time taken by the particle to
complete one revolution is
2𝜋𝑚𝑞 2𝜋𝑞2 𝐵 [JEE Main 15 Online]
(𝑎) (𝑏) 24. Proton, deutron and alpha particle of
𝐵 𝑚
2𝜋𝑞𝐵 2𝜋𝑚 same kinetic energy are moving in
(𝑐) (𝑑) circular trajectories in a constant
𝑚 𝑞𝐵 magnetic field. The radii of proton,
[AIEEE 2005] deuteron and alpha particle are
respectively rp,rd and ra. Which one of
21. The time period of a charged
the following relations is correct ?
particle undergoing a circular motion in a
uniform magnetic field is independent of its (𝑎)𝑟𝛼 = 𝑟𝑝 = 𝑟𝑑 (𝑏)𝑟𝛼 = 𝑟𝑝 < 𝑟𝑑
(a) speed (b) mass (𝑐)𝑟𝛼 > 𝑟𝑑 > 𝑟𝑝 (𝑏)𝑟𝛼 = 𝑟𝑑 < 𝑟𝑝
(c) charge (d) magnetic [AIEEE 12]
induction
25. A wire carrying current 7 is tied
[AIEEE 2002] between points P and Q and is in the
22. At a specific instant, emission of shape of a circular arch of radius R due
radioactive compound is deflected in a to a uniform magnetic field B
magnetic field. The compound can emit (perpendicular to
167
26. A conductor lies along the z-axis at (c) remain same (d) none of these.
−1.5 ≤ 𝑧 < 1.5m and carries a fixed
[AIEEE 2002 ; VMMC 2011]
current of 10.0 A in −𝑎̂𝑧 direction (see
figure). 30. Two long straight parallel wires,
carrying (adjustable) currents and I2, are
kept at a distance d apart. If the force 'F'
between the two wires is taken as 'positive'
when the wires repel each other and
'negative' when the wires attract each
other, the graph showing the dependence
of 'F', on the product I1I2, would be
𝑜𝜇 𝐼2
(d) repel each other with a force of 2𝜋𝑑 2
[AIEEE 05]
28. Two long conductors, separated by
a distance d carry currents I1 and I2 in the
same direction. They exert a force F on
each other. Now the current in one of them
threads making an angle θ with the vertical.
is increased to two times and its direction is
If wires have mass 𝜆 per unit length, then
reversed. The distance is also increased to
the value of (g = gravitational acceleration)
3d. The new value of force between them
I is
is
𝐹 𝜋𝜆𝑔𝐿 𝜋𝜆𝑔𝐿
(a) -2 F (b) (𝑎)sin𝜃√ (𝑏)2sin𝜃√
3
𝜇0 cos𝜃 𝜇0 cos𝜃
−2𝐹 −𝐹
(𝑐) (𝑑)
3 3
𝜋𝑔𝐿 𝜋𝜆𝑔𝐿
[AIEEE 2004] (c) 2√ tan𝜃 (b) √ tan𝜃
𝜇0 𝜇0
29. If a current is passed through a
spring, then the spring will [JEE Main 15]
(a) expand (b) compress 32. A rectangular loop of sides 10 cm and 5
cm carrying a current J of 12 A is
168
placed in different orientations as reads 1 V, the resistance (in ohm) needed
shown in the figures below : to be connected in series with the coil will
be
[JEE Main 15]
(a) 103 (b) 105
(c) 99,995 (d) 9,995
[AIEEE 2005]
36. This question has Statement I and
Statement II. Of the four choices given
after the Statements, choose the one
that best describes the two Statements.
Statement I. Higher the range, greater
is the resistance of ammeter.
Statement II. To increase the range of
ammeter, additional shunt needs to be
used across it.
(a) Statement I is true, Statement IIis true, Stateme
If there is a uniform magnetic field of (b) Statement I is true, Statement IIis true, Stateme
0.3 T in the positive z-direction, in which
(c) Statement I is true, Statement II is
orientations the loop would be in (z) stable
false.
equilibrium and (it) unstable equilibrium ?
(d) Statement I is false, Statement II is
(a) (A) and (B), respectively
true.
(b) (A) and (C), respectively
[JEE Main 13]
(c) (B) and (D), respectively
Paragraph Type
(d) (B) and (C), respectively
Directions. Question numbers 37 and
[JEE Main 15] 38 are based on the following paragraph. A
current loop ABCD is held fixed on the
33. If an ammeter is to be used in place of a
plane of the paper as shown in the figure.
voltmeter, then we must connect with
The arcs BC (radius = b) and DA (radius =
the ammeter
a) of
(a) a low resistance in parallel
(b) a high resistance in parallel
(c) a high resistance in series
(d) a low resistance in series.
[AIEEE 02 ; TPUEE 11]
34. An ammeter reads upto 1 A. Its internal the loop are joined by two straight wires AB
resistance is 0.81 Ω. To increase the and CD. A steady current I is flowing in the
range to 10 A, the value of the required loop. Angle made by AB and CD at the
shunt is origin O is 30°. Another straight thin wire
with steady current I1 flowing out of the
(a) 0.03 Ω (b) 0.3 Ω
plane of the paper is kept at the origin.
(c) 0.9 Ω (d) 0.09 Ω
37. The magnitude of the magnetic field (B)
[AIEEE 2003]
due to loop ABCD at the origin (O) is
35. A moving coil galvanometer has
150 equal divisions. Its current sensitivity is
[AIEEE 2009]
10 divisions mA and voltage sensitivity is 2
divisions mV-1. In order that each division (a) zero (b)
𝜇0 𝐼(𝑏−𝑎)
24𝑎𝑏
169
𝜇0 𝐼 𝑏−𝑎 𝜇0 𝐼 𝜇0𝐼𝐼1 𝜋
( C) [ ] (d) [2(𝑏 − loop is given by [2(𝑏 − 𝑎) + 3 (𝑎 −
4𝜋 𝑎𝑏 4𝜋 4𝜋
𝜋
𝑎) + 3 (𝑎 + 𝑏)] 𝑏)]
(d) The magnitude of the net force on the
38. Due to the presence of the current f at 𝜇0 𝐼𝐼1
the origin loop is24𝑎𝑏 (𝑏 − 𝑎)
(a) The forces on AB and DC are zero
(b) The forces on AD and BC are zero [AIEEE 2009]
(c) The magnitude of the net force on the
𝜇𝑜 𝐼1 4𝜋 × 10−7 × 3
𝐵1 = =
2𝑟 2 × 2𝜋 × 10−2
= 3 × 10−5 Wbm−2
𝜇0 𝐼2 4𝜋 × 10−7 × 4
𝐵2 = =
2𝑟 2 × 2𝜋 × 10−2 11. (a) Net force on each of them would be
= 4 × 10−5 Wbm−2 zero.
170
The two forces will oppose each other if
v is along 𝐸⃗ × 𝐵
⃗
𝐸⃗ × 𝐵 ⃗
𝑣= 2
𝐵
𝐸⃗ and 𝐵
⃗ being perpendicular,
𝐸⃗ × 𝐵⃗ 𝐸𝐵sin90° 𝐸
Current I associated with the rotation of = =
𝐵2 𝐵2 𝐵
the charge dq is
𝑑𝑞 2𝑄𝑟 𝜔 𝑄𝑟𝜔𝑑𝑟
𝐼= = 2 𝑑𝑟 ⋅ =
𝑇 𝑅 2𝜋 𝜋𝑅2
Magnetic field at the centre due to the
elementary ring is
𝜇0 𝐼 𝜇0 𝑄𝑟𝑜𝑑𝑟 𝜇0 𝑄𝜔
𝑑𝐵 = = = 𝑑𝑟
2𝑟 2𝜋𝑅2 𝑟 2𝜋𝑅2 17. (a) Radius of the circular path of a
Magnetic field at the centre due to the charged particle in a perpendicular
entire disc is magnetic field.
𝑚𝑣
𝜇0 𝑄𝜔 𝑅 𝜇0 𝑄𝜔 𝑟=
𝐵 = ∫ 𝑑𝐵 = 2 ∫ 𝑑𝑟 = 𝑞𝐵
2𝜋𝑅 0 2𝜋𝑅
1 For both electron and proton, quantities
For constant Q and ω, 𝐵 ∝ 𝑅 mv, q and B are all same. Hence radius r
will be same.
Hence, the variation of B with R should
be represented by a rectangular hyperbola 18. (a) Given q = -16 × 10-18 C, v =10 ms-1
as in option (a).
⃗ = 𝐵𝑗̂𝑊𝑏, 𝐸⃗ = −104 𝑘̂Vm−1
𝐵
13. (a) When projected along the direction
of the magnetic field, the electron 𝐹𝑒 = 𝑞𝐸⃗ = −16 × 10−18 × (−104 𝑘̂ )
experiences no force. = 16 × 10−14 𝑘̂N
As the electron is negatively charged, ⃗ ) = −16 × 1018 (10𝑖̂ × 𝐵^)
𝐹𝑚 = 𝑞(𝑣 × 𝐵
its motion gets retarded when projected
along the direction of the electric field. = −16 × 10−17 𝐵𝑘̂N
Hence its velocity will decrease.
As the particle continues to move along
14. (d) As the directions of electric and the same direction,
magnetic fields are parallel to the
direction in which the charged particle 𝐹𝑚 = 𝐹𝑒
is released, the particle will move along 16 × 10-17 B = 16 × 10-14
a straight line.
B=103 Wb m-2.
15. (b) The magnetic field exerts a force
perpendicular to the direction of motion 19. (b) As the magnetic force acts
of the charged particle. It continuously perpendicular to the direction of motion
deflects the particle from its path but of the charged particle, the work done
does no work on it. Hence momentum by the field is zero.
𝑚𝑣
of the particle changes but kinetic 20. (d) As 𝑟 = 𝑞𝐵
energy remains same.
2𝜋𝑟 2𝜋 𝑚𝑣 2𝜋𝑚
16. As E and B are mutually perpendicular ∴ 𝑇= = ⋅ =
and v' does not change, so 𝑣 𝑣 𝑞𝐵 𝑞𝐵
𝐸 21. (a) As proved in the above problem, T
𝑞𝐸 = 𝑞𝑣𝐵 or 𝑣= is independent of speed v.
𝐵
171
22. (a) Only charged particles like 2
2𝑚𝐾 √𝑚 But I1 = I2 = I
𝑟=√ ⇒𝑟∝
𝑞𝐵 𝑞 𝜇0 𝐼 2
𝐹= (attractive)
2𝜋𝑑
172
10
== 5𝛺
2
Number of divisions on the galvanometer
scale,
n = 150
Current required for full scale deflection,
𝑛 150
𝐼𝑔 = =
Current sensitivity 10
𝑇cos𝜃 = 𝜆lg
= 15 mA =15× 10-3 A
𝜇0 𝐼×𝐼
𝑇sin𝜃 = ⋅ 𝑙 Required range of voltmeter = 150 × 1
2𝜋 (2𝐿sin𝜃)
=150 V
𝑇sin𝜃 𝜇0 𝐼 2 𝑙 1
= ⋅ Required series resistance,
𝑇cos𝜃 4𝜋𝐿sin𝜃 𝜆𝑙𝑔
𝑉 150
𝑅= −𝐺 = − 5 = 9995𝛺
𝜋𝜆𝑔𝐿 𝐼𝑔 15 × 10−3
𝐼 = 2sin𝜃√
𝜇0 cos𝜃 36. (d) Shunt resistance of an ammeter,
32. (i) Stable equilibrium when 𝑚 ⃗
⃗⃗ ‖𝐵 𝐼𝑔 𝐺
𝐼=
𝐼 − 𝐼𝑔
For increasing range I, S should
decrease. For this, an additional shunt S is
connected across it. But then net
resistance decreases. Hence, statement I
is false and statement II is true.
37. (b) Net magnetic field due to loop
(ii) Unstable equilibrium when 𝑚 ⃗)
⃗⃗ ‖(−𝐵 ABCD at O is
B=B +B +B +B
𝐴𝐵 𝐵𝐶 𝐶𝐷 𝐷𝐴
𝜇0 𝐼 𝜋 𝜇0 𝐼 𝜋
=0+ × +0− ×
4𝜋𝑎 6 4𝜋𝑏 6
𝜇0 𝐼 𝜇0 𝐼 𝜇0 𝐼
= − = (𝑏 − 𝑎)
24𝑎 24𝑏 24𝑎𝑏
173
2. In a circular coil of radius r, the solenoid is
magnetic field at the centre is proportional
(a) 2π× 10-3 T (b) 2π×10-5T
to
(c) 4π× 10-2 T (d) 4π× 10-3 T
(a) r2 (b) r
[IPUEE 2007]
(c) 1 / r (c) 1/r2
[DCE 1997] 7. Two long straight wires are set parallel
to each other. Each carries a current i
3. A circular current carrying coil has a
in the same direction and separation
radius R. The distance from the centre of
between them is 2 r. Intensity of
the coil on the axis of the coil, where the
magnetic field midway between them is
magnetic induction is l/8th of its value at
the centre of the coil is 𝜇0 𝑖
(a) (𝑏) zero
𝑅 𝑟
(𝑎)√3𝑅 (b)
√3 4𝜇𝑜 𝑖 𝜇0 𝑖
(𝑐) (𝑑)
2 𝑅 𝑟 4𝑟
(𝑐)( )𝑅 (d)
√3 2√3 [DCE 2002]
[DCE 2005 ; VMMC09] 8. A very high magnetic field is applied
4.A current i is flowing in a hexagonal to a stationary charge. Then, the charge
coil of side I (figure). The magnetic experiences
induction at the centre of the coil will be (a) a force in the direction of magnetic
field
(b) a force perpendicular to the
magnetic field
(c) a force in an arbitrary direction
(d) no force.
√3𝜇 𝑖 𝜇0 𝑖
(𝑎) 𝜋𝑙 0 (𝑏) [DCE 2004]
√3𝜋𝑙
174
(a) an upward force (b) a downward force [IPUEE 2006]
(c) an accelerative force 17. A proton and an α-particle are
projected normally into a magnetic field
(d) no force
with the same speed. What will be the ratio
[IPUEE 15] of radii of the trajectories of the proton and
12. Cathode rays enter a magnetic field α-particle ?
making oblique angle with the lines of (a) 2 : 1 (b) 1 : 2
magnetic induction. What will be the nature
(c) 4 : 1 (d) 1:4
of the path followed ?
[DCE 2001,2003]
(a) Parabola (b) Helix
18. Which particles will have minimum
(c) Circle (d) Straight line
frequency of revolution when projected
[IPUEE 09] with the same velocity perpendicular to
13. The radius of motion of a charged magnetic field ?
particle orbiting in a magnetic field is (a) Li+ (b) Electron
𝑚𝐵 𝑚𝑣 (c) Proton (d) He+
(a) (b)
𝑞𝑣 𝑞𝐵 [IPUEE 2004]
𝑚𝑞 𝑞𝑣
(c) (𝑑) 𝑚𝐵 + 2+
19. Mixed He and 0 ions (mass of
𝑣𝐵
He+ = 4 amu and that of 02+ =16 amu) beam
[DCE 2000]
passes a region of constant perpendicular
14. An electron having mass (9.1×10 31 magnetic field. If kinetic energy of all the
kg) and charge (1.6 × 10-19 C) moves in a ions is same, then
circular path of radius 0.5 m with a velocity
(a) He+ ions Will be deflected more
106 m/s in a magnetic field. Strength of
than those of o2+
magnetic field is
(b) He+ ions will be deflected less than
(a) 1.13× 10-5 T (b) 5.6 × 10-5 T
those of 02+
(c) 2.8 × 10-6 T (d) none of these.
(c) all the ions will be deflected equally
[DCE 2005]
(d) no ions will be deflected.
15. One proton beam enters a
magnetic field of 10 -4 Wb/m2 normally. If [IPUEE 2004]
specific charge is 10-11C /kg and velocity of
20. An electron is moving in a region of
proton is 109 m/s, then the radius of circle
electric field and magnetic field, it will gain
described will be
energy from
(a) 0.1 (b) 10 m
(a) electric field (b) magnetic field
(c) 100 m (d) none of these.
(c) both of these (d) none of these.
[DCE 1999]
[DCE 1998]
16. Electron of mass m and charge q is
21. When deuterium and helium are
travelling with a speed v along a
subjected to an accelerating field
circular path of radius r at right angles
simultaneously, then
to a uniform magnetic field of intensity
B. If the speed of the electron is (a) both acquire same energy
doubled and the magnetic field is (b) deuterium accelerates faster
halved, the resulting path would have a
radius (c) helium accelerates faster
(a) 2 r (b) 4r (d) neither of them is accelerated.
𝑟 𝑟 22. An electric field of 1500 V/m and a
(c) (d)
4 2 magnetic field of 0.40 Wb/m 2 act on a
175
moving electron. The minimum uniform respectively. In which of the following
speed along a straight line the electron cases, the charge will go undeflected ?
could have is
(a) 𝐸⃗ and 𝐵
⃗ become 2𝐸⃗ and 2𝐵
⃗
(a) 1.6× 10-15 m/s (b) 6× 10-16 m/s respectively
(c) 3.75× 103 m/s (d) 3.75×102 m/s. (b) 𝐸⃗ ⊥ 𝐵
⃗ (c) 𝐸⃗ ‖𝐵
⃗ ⊥𝑣
[IPUEE 2006] (d) none of the above.
23. An electron is projected along the [DCE 2002]
axis of a circular conductor carrying some
current. Electron will experience force 28 A force acting on a conductor of
length 5 m carrying a current of 8 A kept
(a) along the axis perpendicular to the magnetic field of 1.5 T
(b) perpendicular to the axis is
(c) at an angle of 4° with axis (a) 100 N (b) 60 N
(d) no force experienced. (c) 50 N (d) 75 N
24. Maximum kinetic energy of the positive [DCE 1998]
ion in the cyclotron is 29. A straight wire of mass 200 g and
𝑞2 𝐵𝑟0 𝑞𝐵2 𝑟0 length 1.5 m carries a current of 2 A. It is
(𝑎) (b) suspended in mid air by a uniform
2𝑚 2𝑚
horizontal magnetic field R The magnitude
𝑞2 𝐵2 𝑟02 𝑞𝐵𝑟0 of B (in tesla) is (assume that g =9.8 ms-2)
(𝑐) (d)
2𝑚 2𝑚2
(a) 2 (b) 1.5
[IPUEE 2010]
(c) 0.55 (d) 0.65 .
25. A cyclotron is operating at a frequency
of 12 MHz. Mass and charge of [IPUEE 2002]
deuteron are 3.3 × 10-27 kg and 30. A wirePQR is bent as shown in the
1.6×10-19C TO accelerate deutron, the figure and is placed in a region of uniform
necessary magnetic field is magnetic field R The
(a) 0.16 T (b) 1.6 T
(c) 0.016 T (d) 16 T
[IPUEE 2015]
26. A circular coil of 20 turns and
radius 10 cm is placed in an uniform length of PQ = QR = I. A current I ampere
magnetic field of 0.1 T normal to the plane flows through the wire as shown. The
of the coil. The coil carries a current of 5 A. magnitude of the force on PQ and QR will
The coil is made up of copper wire of be
cross-sectional are 10−5 m2 and the
(a) BIl, 0 (b) 2 BIl, 0
number of free electrons per unit volume of
copper is 1029. The average force (c) 0, BIl (d) 0, 0
experienced by an electron in the coil due [IPUEE 2007]
to magnetic field is
31. When two parallel wires carry currents
(a) 5 × 10-25N (b) zero in the same direction,
(c) 8 × 10-24 (d) none of these (a) they attract each other
[IPUEE 2010] (b) they repel each other
27. A charged particle is fired with (c) magnetic forces on two wires are
velocity v' through a space having both perpendicular to each other
electric and magnetic fields E and B
(d) they do not experience any magnetic
176
force. current of 20 ampere and weighs 0.04
newton per metre. The distance between
[IPUEE 2005]
the two wires for which the upper wire is
32. If two wires carry currents in opposite just supported by magnetic repulsion is
directions,
(a) 10-2mm (b) 10-2cm
(a) the wires attract each other
(c) 10-2m (d) 10-2km
(b) the wires repel each other [IPUEE 14]
(c) the wires experience neither attraction 37. If the beams of electrons and
nor repulsion protons move parallel to each other in the
(d) the forces of attraction or repulsion do same direction, then they
not depend on current direction. (a) attract each other (b) repel each
[IPUEE 2005 ; DCE 2009] other
33. A steady electric current is flowing (c) no relation (d) neither attract
through a cylindrical conductor. not repel.
177
40. A current loop placed in a non-uniform (a) number of turns of the coil
magnetic field experiences
(b) magnetic field
(a) a force of repulsion
(c) area of the coil
(b) a force of attraction
(d) restoring couple per unit twist of
(c) a torque but not force suspension.
(d) a force and a torque. [IPUEE 11]
[DCE 1997]
45. A galvanometer coil has a resistance of
41. What is the net force on the rectangular 15 Ω and gives full scale deflection for a
coil ? current of 4 mA. To convert it to an
ammeter of range 0 to 6 A
(a) 10 m Ω resistance is to be connected in
parallel to the galvanometer.
(b) 10 mΩ resistance is to be connected in
series with the galvanometer.
(c) 0.1 Ω resistance is to be connected in
parallel to the galvanometer.
(a) 25 × 10-7 N towards wire (d) 0.1 Ω resistance is to be connected in
-7 series with the galvanometer.
(b) 25 × 10 N away from wire
[IPUEE 2007]
(c) 35 × 10-7 N towards wire
46. By mistake a voltmeter is connected in
(d) 35 × 10-7 N away from wire.
series and an ammeter is connected in
[DCE 2000]
parallel with a resistance in an
42. A rigid circular loop of radius r and electrical circuit. What will happen to
mass m lies in the x-y plane of a flat table the instruments ?
and has a current i flowing in it. At this
(a) Voltmeter is damaged
particular place the earth's magnetic field is
⃗ = 𝐵𝑥 𝑖̂ + 𝐵𝑧 𝑘̂
𝐵 (b) Ammeter is damaged
The value of i so that the loop starts tilting (c) Both are damaged
is (d) None is damaged.
𝑚𝑔 𝑚𝑔
(𝑎) (b) [DCE 2007]
2
𝜋𝑟√𝐵𝑥 + 𝐵𝑧 2 𝜋𝑟𝐵𝑥
47. In an ammeter, 10% of main current
𝑚𝑔 𝑚𝑔 is passing through the galvanometer. If the
(𝑐) (𝑑)
𝜋𝑟𝐵𝑧 𝜋𝑟√𝐵𝑥 𝐵𝑧 resistance of the galvanometer is G, then
the shunt resistance, in ohm is
[DCE 2005]
(a) 9G (b) G/9
43. A moving coil galvanometer has a coil
of effective area A and number of turns (c) 90 G (d) G / 90.
N. The magnetic field B is radial. If a [IPUEE 2006]
current I is passed through the coil, the
torque acting on the coil is 48. A voltmeter of range 2 V and
resistance 300 Ω cannot be converted into
(a) NA2B2I (b) NABI2 ammeter of range
(C) N2ABI (c)NABI (a) 1 A (b) 1 mA
[IPUEE 11] (c) 100 mA (d) 10 mA
44. The sensitivity of moving coil [IPUEE 08]
galvanometer can be increased by
decreasing 49. In case of Hall effect for a strip
178
having charge Q and area of cross-section (c) inversely proportional to A
A, the Lorentz force is
(d) directly proportional to A.
(a) directly proportional to Q
[DCE 2004]
(b) inversely proportional to Q
2 2 3 ⁄ 2
= 16. (b) 𝑟 =
𝑚𝑣
2(𝑅 + 𝑟 ) 8 2𝑅 𝑞𝐵
𝑅2 1
or = When the speed is doubled and magnetic
(𝑅 2 +𝑟 2 )3⁄2 8𝑅
field is halved,
or (𝑅2 + 𝑟 2 )3⁄2 = 8𝑅3 𝑚 × 2𝑣
𝑟′ = = 4𝑟
or 𝑅2 + 𝑟2 = 4𝑅2 or 𝑟 = √3𝑅 𝑞(𝐵/2)
𝑚𝑝 𝑣 𝑚𝑝 𝑣
4. (a) Refer to the answer of Exercise 12 17. (b) 𝑟𝑝 = =
𝑞𝐵 𝑒𝐵
on page 4.12.
𝑚𝛼 𝑣 4𝑚𝑝 𝑣
𝜇𝑜 𝐼 4𝜋×10−7×10 𝑟𝛼 = = = 2𝑟𝑝
5. (b) 𝐵 = = T 𝑞𝛼 𝐵 2𝑒𝐵
4𝑅 4×0.20
179
energy. 𝑚𝑔 200 × 10−3 × 9.8
𝐵= = = 0.65T
21. (d) Both deuterium and helium are 𝐼 2 × 1.5
neutral particles. They cannot be 30. (c) FPQ = IIBsin00 = 0
accelerated by electric or magnetic
FQR = IIBsin90° = IIB
field.
31. (a) Two wires carrying currents in the
22. (c)For the electron moving along the
same direction attract each other.
straight line,
32. (b) Two wires carrying currents in the
Fm = Fg or evB = eE
opposite directions repel each other.
𝐸 1500
or 𝑣=𝐵= = 3.75 × 103 m/s 33. (c) Refer to the solution of Example 33
0.40
180
= 30 × 10-7 N (attractive) 4 × 10−3 × 15 60 × 10−3
= = 𝛺
2×1 6 − 4 × 10−3 5.996
𝐹𝐶𝐷 = 2 × 10−7 × × 15
12 × 10−2 ≈ 10 × 10−3 𝛺 = 10m𝛺
−2
× 10 N
46. (d) Due to the high resistance voltmeter
= 5×10-7 N (repulsive) connected in series, the net resistance
Fnet = FAB - FCD =25 × 10-7 N (towards wire) increases and so current in the circuit
decreases. Consequently, neither
⃗⃗ = 𝑖𝐴 = 𝑖𝜋𝑟 2 𝑘̂
42. (b) 𝑀 ammeter nor voltmeter is damaged.
⃗ = 𝐵𝑥 𝑖̂ + 𝐵𝑧 𝑘̂
𝐵 47. (b) Ig = 10% of I = 0.1 I
⃗ = (𝑖𝜋𝑟 2 𝑘̂) × (𝐵𝑥 𝑖̂ + 𝐵𝑧 𝑘̂)
⃗⃗ × 𝐵
𝜏=𝑀 𝐼𝑔 𝐺 0.1𝐼𝐺 𝐺
𝑆= = =
𝐼 − 𝐼𝑔 𝐼 − 0.1𝐼 9
= 𝑖𝜋𝑟 2 𝐵𝑥 𝑘̂ × 𝑖̂ = 𝑖𝜋𝑟 2 𝐵𝑥 ^
2V 2000
Torque due to the weight of the loop = 48. (b) 𝐼𝑔 = 300𝛺 = mA = 6.67mA
300
mgr The range of an ammeter cannot be
𝑚𝑔
∴𝑖𝜋𝑟 2 𝐵𝑥 = 𝑚𝑔𝑟 or 𝑖= decreased but can be increased only. So
𝜋𝑟𝐵𝑥
the instrument cannot be converted into
43. (d) 𝜏 = NIBAsm90°= NIBA ammeter of range 1 mA.
44. (d) By decreasing restoring couple per 49. (A) Lorentz force, 𝐹 = 𝑄(𝐸⃗ +
unit twist (k), sensitivity can be ⃗ ) i.e. 𝐹 ∝ 𝑄
𝑣×𝐵
increased.
𝑔 𝐼 𝐺
45. (a) 𝑆 = 𝐼−𝐼
𝑔
AIIMS
1. A wire of certain length is bent to straight wire carry currents Ic and Ie
form a circular coil of a single turn. If the respectively as shown in the figure.
same wire is bent into a coil of smaller
Assuming that these are placed in the
radius so as to have two turns, then
same plane, the magnetic
magnetic field produced at the centre by
the same value of current is
(a) one quarter of its value in first case
(b) one half of its value in first case
(c) two times its value in first case
(d) four times its value in first case. [AIIMS
1990]
2. A circular coil of radius R carries an
electric current. The magnetic field due fields will be zero at the centre O of the
to the coil at a point on the axis of the loop, when separation H is
coil located at a distance r from the
centre of the coil, such that r » R, varies 𝐼𝑒 𝑅 𝐼𝑐 𝑅
(𝑎) (𝑏)
as 𝐼𝑐 𝜋 𝐼𝑒 𝜋
1 1 𝐼𝑐𝜋 𝐼𝑒𝜋
(a) (b) (𝑐) (d)
𝑟 𝑟 3 ⁄2 𝐼𝑒 𝑅 𝐼𝑐 𝑅
1 1 [AIIMS
(c) 2 (𝑑)
𝑟 𝑟3 2006]
[AIIMS 2004] 4. What is the magnetic field at a distance
3. Circular loop of a wire and a long R from a coil of radius r carrying current
181
I? current i and with central point O. The
magnetic field at the central point O is
𝜇𝑜 𝐼𝑅2 𝜇0 𝐼𝑟 2
(𝑎) (𝑏)
2(𝑅2 + 𝑟 2 )3⁄2 2(𝑅2 + 𝑟 2 )3⁄2
𝜇0 𝐼 𝜇0 𝐼
(𝑐) (𝑑)
2𝑟 2𝑅
[AIIMS 2007]
5. The magnetic field due to a
straight conductor of uniform cross-section
of radius a and carrying a steady current is
2𝜇0𝑖
represented by (a) acting downward 3 πR
3𝜋𝑅
5𝜇0𝑖
(b) , acting downward 12 R 6
12𝑅
6𝜇0𝑖
(c) , acting downward HR 6
11𝑅
3𝜇0𝑖
(d) acting upward
7𝑅
[AIM
MS 2015]
9. An uncharged particle is moving with
a velocity of 𝑣 in non-uniform magnetic
field as shown.
6. The magnetic field at P on the axis
of a solenoid having 100 turns/m and
carrying a current of 5 A is
Velocity 𝑣 would be
(a) maximum at A and B (b) minimum
at A and B
(c) maximum at M
(a) 250μQ (b) 500√2𝜇0 (d) same at all points
(c) 500μQ (d) 250 √2𝜇0 [AIIMS 2010]
[AIIMS 11] 10. An electron is travelling along the
7. The wire shown in the figure X-direction. It encounters a magnetic field
carries a current of 32 A. If r =3.14 cm, the in the Y-direction. Its subsequent motion
magnetic field at point P will be will be
(a) straight line along the X-direction
(b) a circle in the XZ-plane
(c) a circle in the YZ-plane
(d) a circle in the XY-plane.
[AIIMS 2003]
(a) 1.6× 10-4T (b) 3.2 × 10-4T
11. A charged particle enters a
(c) 4.8 × 10-4T (d) 6.4× 10-4T magnetic field H with its initial velocity
[AIIMS 13] making angle of 45° with H. The path of the
particle will be
8. Consider the circular loop having
182
(a) a straight line (b) a circle [AIIMS 12]
(c) an ellipse (d) a helical. 17. In the following diagram which
particle has the highest el m value ?
[AIIMS 1999]
12. A proton and an α-particle
moving with the same velocity enter into a
uniform magnetic field, acting normal to the
plane of their motion. The ratio of the radii
of the circular paths described by the
proton and α-particle is
(a) 1:2 (b) 1 : 4
(a) A (b) B
(c) 1 : 16 (d) 4:l
(c) C (d) D
[AIIMS2004]
[AIIMS 2007]
13. A proton is about 1840 times
18. Three particles having charges in
heavier than an electron. When it is
the ratio of 2:3:5 produce the same point
accelerated by a potential difference of 1
on the photographic film in Thomson
kV, its kinetic energy will be
experiment. Their masses are in the ratio
1
(a) 1,840 keV (b) 1840 keV of
183
21. A rectangular loop carrying a 25. Two identical galvanometers are
current I: is situated near a long straight converted into an ammeter and a
wire carrying a steady current I2. The wire milliammeter. As compared to the
is parallel to one of the sides of the loop shunt resistance of ammeter, the shunt
and is in the plane of the loop as shown in resistance of milliammeter will be
the figure.
(a) less (b) equal
(c) more (d) zero.
[AIIMS 1999]
26. A moving coil galvanometer has a
resistance of 900 Ω. In order to send only
10% of the main current through this
Then the current loop will galvanometer, the resistance of the
required shunt is
(a) rotate about an axis parallel to the wire
(a) 0.9 Ω (b) 100 Ω
(b) move towards the wire
(c) 405 Ω (d) 90 Ω
(c) move away from the wire
[AIIMS 2009]
(d) remain stationary.
27. A galvanometer has a current
[AIIMS 2003]
sensitivity of 1 mA per division. A variable
22. A closely wound solenoid of 2000 shunt is connected across the
turns and area of cross-section 1.5 × galvanometer and the combination is put in
10−4 m2 carries a current of 2.0 A. It is series with a resistance of 500 Ω and cell of
suspended through its centre and internal resistance 1 Ω. It gives a deflection
perpendicular to its length, allowing it to of 5 divisions for shunt of 5 ohm and 20
turn in a horizontal plane in a uniform divisions for shunt of 25 ohm. The emf of
magnetic field 5 × 10−2 T, making an angle cell is
of 30° with the axis of the solenoid. The (a) 47.1 V (b) 57.1 V
torque on the solenoid will be
(c) 67.1 V (d)77.1 V
(a) 3× 10-3Nm (b) 1.5 × 10-3Nm
[AIIMS 2010]
(c) 1.5×10-2Nm (d) 3× 10-2Nm
28. Consider the diagram shown below :
[VMMC 12 ; AIIMS 14]
23. A galvanometer can be changed
into ammeter by providing
(a) low resistance in series
(b) low resistance in parallel
(c) high resistance in series
A voltmeter of resistance 150 Ω is
(d) high resistance in parallel. connected across A and B. The potential
[AIIMS 1996] drop across B and C measured by
voltmeter is
24. The resistance of a galvanometer
is 50 Ω and the current required to give full (a) 29 V (b) 27 V
scale deflection is 100 μA. In order to (c) 31 V (d) 30 V
convert it into an ammeter for reading upto
10 A, it is necessary to put a resistance of [AIIMS 2015]
(a) 5× 10-3Ω (b) 5× 10-4Ω Assertions and Reasons
(c) 5× 10-5Ω (d) 5× 10-2Ω Directions : In the following questions,
a statement of assertion (A) is followed by
[AIIMS 1999]
184
a statement of reason (R). Mark the correct actual potential difference across the
choice as : terminals.
(a) If both assertion and reason are true [AIIMS 11]
and reason is the correct explanation of
34. Assertion. Magnetic field
assertion
interacts with a moving charge and not with
(b) If both assertion and reason are true a stationary charge.
but reason is not the correct
Reason. A moving charge produces a
explanation of assertion
magnetic field.
(c) If assertion is true but reason is false
[AIIMS 2014]
(d) If both assertion and reason are false.
35. Assertion. A wire bent into an
29. Assertion. A charge, whether irregular shape with the points P and Q
stationary or in motion produces a fixed. If a current I is passed through the
magnetic field around it. wire, then the area enclosed by the
irregular portion of the wire increases.
Reason. Moving charges produce only
electric field in the surrounding space.
[AIIMS 2009]
30. Assertion. An electric field is preferred
in comparison to magnetic field for
detecting the electron beam in a Reason. Opposite currents carrying
television picture tube. wires repel each other.
185
1
5. (a) For 𝑟 < 𝑎, 𝐵 ∝ 𝑟 and for 𝑟 > 𝑎, 𝐵 ∝ 𝑟 14. (c) A cyclotron is used to accelerate
positive ions.
𝜇0 𝑛𝐼
6. (d) 𝐵 = (cos45° + cos45° ) 𝑞𝐵 1.6×10−19 ×1
2 15. 𝑓𝑐 = = Hz
2𝜋𝑚 2×3.14×9.1×10−31
𝜇02 × 100 × 5 2
= × T = 28 × 109 Hz = 28 GHz.
2 √2
16. (b) When 𝐸⃗ ⊥ 𝐵
⃗,
= 250√2𝜇0 T
qE = qvB
7. (c) Straight portions do not contribute to
B because point P lies along these 𝐸 |𝐸⃗ |
portions. 𝑣= =
𝐵 |𝐵⃗|
3
𝐵= × Field of the loop |𝐸⃗ | = 𝑣|𝐵
⃗|
4
𝑚𝑣 2 𝑒 𝑣
3 𝜇0 𝐼 3 4𝜋 × 10−7 × 32 17. (d) Here 𝑒𝑣𝐵 = or = 𝑟𝐵
= × = × 𝑟 𝑚
4 2𝑟 4 2 × 3.14 × 10−2 𝑒 1
For same v and B, ∝
= 4.8 × 10-4T. 𝑚 𝑟
8. (b) Net field at the centre O, The radius of curvature is minimum for
D. Hence its el mis highest.
𝐵 = 𝐵arcl + 𝐵arclI 𝑒 1
18. (a) As seen in the above problem 𝑚 ∝ 𝑟
𝜇0 𝑖 3𝜋 𝜇0 𝐼 𝜋
= ⋅ + ⋅
4𝜋𝑅 2 4𝜋(3𝑅) 2 The three particles will strike the
𝜇0 𝑖 3 1 photographic film at the same point (same
= ( + ) r) if their masses are also in the ratio 2:3:5.
𝑅 8 24
5𝜇0𝑖 19. (c) Two parallel beams of
== acting downward positrons moving in the same direction set
12𝑅
9. (d) No force is exerted by the magnetic up two parallel currents flowing in the same
field on the moving uncharged particle. direction. Hence they attract each other.
186
into an ammeter by providing a low
resistance in parallel.
𝐼𝑔 𝐺 100×10−6×50
24. (b) 𝑅𝑠 = = ≈
𝐼−𝐼𝑔 10−0.0001
5 × 10−4 𝛺
𝑔𝐼 𝐺
25. (c) 𝑅𝑠 = 𝐼−𝐼
𝑔
Clearly, the shunt needed to convert a 28. (c) When voltmeter of resistance 150 Ω
galvanometer into a milliammeter has a is connected across AB,
higher value than that required to convert 100 × 150
into an ammeter. 𝑅𝑒𝑞 = + 100 = 160𝛺
100 + 150
𝑔 𝐼 𝐺 0.10𝐼×900
26. (b) 𝑆 = 𝐼−𝐼 = = 100𝛺 𝑉 50 5
𝑔 𝐼−0.10𝐼 𝐼= = = A
𝑅𝑒𝑞 160 16
27. (a) Here
5
𝜀 𝑉𝐵𝐶 = 𝐼𝑅𝐵𝐶 = × 100 = 31.25V ≃ 31V
𝐼= 16
𝐺𝑆
𝑅+𝑟+𝐺 +𝑆 29. (d) A stationary charge produces
𝐼𝑆 only an electric field. A moving charge is
𝐼𝑔 = associated both with electric and magnetic
𝐺+𝑆
fields.
𝜀 𝑆
= × 30. (d) If electric field is applied for
𝐺𝑆 𝐺+𝑆
(𝑅 + 𝑟) + 𝐺 + 𝑆 detecting the electron beam, then very high
voltage have to be used or very long tube is
𝜀𝑆 needed.
=
(𝑅 + 𝑟)(𝐺 + 𝑆) + 𝐺𝑆 2𝜋𝑚 𝑚
31. 𝑇 = or 𝑇 ∝
When S =5Ω, 𝑞𝐵 𝑞
𝐼𝑔 = 5 × 10−3 A 𝑇𝑝 𝑚𝑝 𝑞𝛼 𝑚𝑝 2𝑒 1
= ⋅ = ⋅ = or 𝑇𝛼 = 2𝑇𝑝
𝑇𝛼 𝑚𝛼 𝑞𝑝 4𝑚𝑝 𝑒 2
𝜀×5
∴5 × 10−3 = 501(𝐺+5)+5𝐺 𝑚𝑣 𝑝
32. (d) 𝑟 = = 𝑞𝐵
𝑞𝐵
When S =25Ω,
Both electron and proton have same p
𝐼𝑔 = 20 × 10−3 A and q, so r is same for them. Both assertion
𝜀 × 25 and reason are false.
20 × 10−3 =
501(𝐺 + 25) + 25𝐺 33. (a) Both assertion and reason are
true and the reason is the correct
On solving (i) and (it), we get G = 88.2 Ω
explanation of the assertion.
From (i),
34. (a) Both the assertion and reason
𝜀 = 10−3 [501(88.2 + 5) + 5 × 88.2]
are true and reason is the correct
= 47.1𝛺 explanation of the assertion.
35. (a) Opposite pairs of current
elements on the lower and the upper
halves of the current loop carry currents in
opposite directions. Repulsion between
opposite pairs increases area enclosed by
the irregular loop.
AIPMT
1 Tesla is the unit of (c) electric field (d) magnetic
induction.
(a) electric flux (b) magnetic flux
187
[CBSE 1997]
2. The magnetic field dB due to a small
current element dl at a distance r* and
element carrying current / is
𝜇 ⃗⃗⃗⃗ ×𝑟
𝐼𝑑𝑙
⃗⃗⃗⃗⃗ = 0 ×
(a) 𝑑𝐵 4𝜋 𝑟
𝜇 ⃗⃗ ×𝑟
𝐼𝑑𝑙𝑙 and CD are perpendicular to each other
(b) ⃗⃗⃗⃗⃗
𝑑𝐵 = 4𝜋𝑜 × and symmetrically placed w.r.t. the
𝑟2
currents. Where do we expect the resultant
𝜇0 𝐼𝑑𝑙×𝑟
⃗⃗⃗⃗ =
(c) 𝑑𝐵 × magnetic field to be zero ?
4𝜋 𝑟3
𝜇 ⃗⃗⃗⃗ ×𝑟
𝐼𝑑𝑙 (a) on AB (b) on CD
⃗⃗⃗⃗⃗ = 𝑜 ×
(d) 𝑑𝐵 4𝜋 𝑟4 (c) on both AB and CD (d) on both OD and
[CBSE 19961 BO.
3. The magnetic field at a distance r from [CBSE PMT 1996]
a long wire carrying current I is 0.4 7. Magnetic field intensity at the
tesla. The magnetic field at a distance 2 centre of a coil of 50 turns, radius 0.05 m
r is and carrying a current of 2 A is
(a) 0.1 tesla (b) 0.2 tesla (a) 0.5× 10-5 T (b) 1.26× 10-4 T
(c) 0.8 tesla (d) 1.6 tesla. (c) 3 × 10-5 T (d) 4 × 10-5 T
[CBSE PMT 1992] [CBSE PMT 1999]
4. A straight wire of diameter 0.5 mm 8. A circular coil A has a radius a and the
carrying a current of 1 A is replaced by current flowing in it is I. Another circular
another wire of 1 mm diameter carrying the coil B has a radius 2 a and if 2 I is the
same current. The strength of the magnetic current flowing through it, then the
field far away is magnetic fields at the centre of the
(a) twice the earlier value circular coils are in the ratio of
(b) one half of the earlier value (a) 1:1 (b) 2:1
(c) one quarter of the earlier value (c) 3 : 1 (d) 4:1
(d) same as the earlier value. [CBSE PMT 1993]
[CBSE 97, 99 ; DPMT 99] 9. Two circular coils 1 and 2 are made
from the same wire but the radius of
5. Two long parallel wires P and Q are both the first coil is twice that of the second
perpendicular to the plane of the paper with coil. What ratio of the potential
distance of 5 m between them. If P and Q difference (in volt) should be applied
carry currents of 2.5 A and 5 A respectively across them, so that the magnetic field
in the same direction, then the magnetic at their centres is the same ?
field at a point half-way between the wires
is (a) 2 (b) 3
√3𝜇0 𝜇0 (c) 4 (d) 6
(A) T (B) T
2𝜋 𝜋 [CBSE PMT 2006]
3𝜇𝑜 𝜇𝑎 10. The magnetic field of a given length of
(𝑐) T (d) T
2𝜋 2𝜋 wire carrying a current for a single turn
[CBSE PMT 2000,01] circular coil at centre is B. Then its
value for two turns coil of the same
6. Two equal electric currents are wire, when the same current passes
flowing perpendicular to each other as through it, is
shown in the figure. AB
188
(a)
𝐵 (b) 𝐵 centre of the ring is
4 2 𝜇0 𝑞𝑓 𝜇0 𝑞
(c) 2 B (d) 4 B (𝑎) (b)
2𝜋𝑅 2𝜋𝑓𝑅
[CBSE PMT 2002] 𝜇0 𝑞 𝜇0 𝑞𝑓
(𝑐) (d)
11. A wire of certain length is bent to 2𝑓𝑅 2𝑅
form a circular coil of smaller radius so as [CBSE Pre 2011]
to have two turns, then magnetic field
produced at the centre by the same value 16. Two similar coils of radius R are
of current is lying concentrically with their planes at
right angles to each other. The currents
(a) one quarter of its value in first case flowing in them are I an 2 7, respectively.
(b) one half of its value in first case The resultant magnetic field induction at
the centre will be [AIPMT Pre 12]
(c) two times its value in first case
(d) four times its value in first case. √5𝜇0 𝐼 3𝜇0 𝐼
(𝑎) (𝑏)
2𝑅 2𝑅
[CBSE PMT ; AIIMS 1990]
𝜇0 𝐼 𝜇0 𝐼
12. If a long hollow copper pipe carries a (𝑐) (𝑑)
2𝑅 𝑅
current, the magnetic field produced
17. Two identical long conducting wires
will be
AOB and COD are placed at right angle
(a) inside the pipe only to each other, with one above other
(b) outside the pipe only such that O is their common point for
the two. The wires carry I and /9
(c) neither inside nor outside the pipe currents, respectively. Point P is lying
(d) both inside and outside the pipe. at distance d from O along a direction
perpendicular to the plane containing
[CBSE 1999] the wires. The magnetic field at the
13. A long solenoid carrying a current point P will be
produces a magnetic field B along its (a) 𝜇0 𝐼 𝜇
( 1) 0
(𝑏) 2𝜋𝑑 (𝐼1 + 𝐼2 )
axis. If the current is doubled and the 2𝜋𝑑 𝐼2
number of turns per cm is halved, then 𝜇0 2 𝜇
new value of the magnetic field is (𝑐) (𝐼1 − 𝐼22 ) (𝑑) 0 (𝐼12 + 𝐼22 )1⁄2
2𝜋𝑑 2𝜋𝑑
(a) B (b) 2 B 18.A wire carrying current I has the
(c) 4'B (d) B/2 shape as shown in the given figure. Linear
parts of the wire are
[CBSE 2003]
14. An electron moving in a circular
orbit of radius r makes n rotations per
second. The magnetic field produced at the
centre has magnitude
𝜇0 𝑛 2 𝑒
(𝑎) zero (b)
𝑟
𝜇0 𝑛𝑒 𝜇0 𝑛𝑒 very long and parallel to X-axis while
(𝑐) (d) semicircular portion of radius R is lying in
2𝑟 2𝜋𝑟
Y-Z plane. Magnetic field at point O is
[AIPMT15]
[AIPMT 15]
15. A thin ring of radius R metre has
charge q coulomb uniformly spread on it. ⃗ = − 𝜇0 𝐼 (𝜋𝑖̂ − 2𝑘̂)
(a) 𝐵 4𝜋 𝑅
The ring rotates about its axis with a
constant frequency of f revolutions/s. The ⃗ = − 𝜇0 𝐼 (𝜋𝑖̂ + 2𝑘̂)
(b) 𝐵 4𝜋 𝑅
value of magnetic induction in Wb/m2 at the
189
⃗ = 𝜇0 𝐼 (𝜋𝑖̂ − 2𝑘̂ )
(c) 𝐵 (d) angle between them can have any
4𝜋 𝑅
value other than zero and 180°.
⃗ = 𝜇0 𝐼 (𝜋𝑖̂ + 2𝑘̂)
(d) 𝐵 4𝜋 𝑅 [CBSE 2006]
19. When a charged particle moves 23. A charge q moves in a region, where
perpendicular to a magnetic field, then ⃗ and magnetic field 𝐸⃗
electric field 𝐵
(a) speed of the particle is changed both exist. Then the force on it is
(b) speed of the particle remains ⃗)
(a) 𝑞(𝑣 × 𝐵 (b) 𝑞𝐸⃗ + 𝑞(𝑣 × 𝐵
⃗)
unchanged
(c) 𝑞𝐸⃗ + 𝑞(𝐵
⃗ × 𝑣) (d) 𝑞𝐵
⃗ + 𝑞(𝐸⃗ × 𝑣)
(c) direction of the particle remains
[CBSE PMT 2002]
unchanged
24. In a certain region of space, electric
(d) acceleration of the particle remains
unchanged. field 𝐸⃗ and magnetic field 𝐵⃗ are
perpendicular to each other and an
[CBSE 2003] electron enters in region perpendicular
20. A positively charged particle moving to the direction of 𝐸⃗ and 𝐵
⃗ both and
due east enters a region of uniform moves undeflected. Then velocity of
magnetic field directed vertically electron is
upward. The particle will |𝐸⃗ | ⃗|
|𝐵
(𝑎) |𝐵⃗| (𝑏) ⃗⃗⃗
(a) get deflected in vertically upward |𝐸 |
direction (𝑐)𝐸⃗ × 𝐵
⃗ (d) 𝐸⃗ ⋅ 𝐵
⃗
(b) move in circular path with an increased
speed [CBSE PMT 2001]
(c) move in circular path with a decreased 25 The magnetic force acting on a
speed charged particle of charge -2 μC in a
(d) move in a circular path with a uniform magnetic field of 2 T acting in y-direction,
speed. when the particle velocity is (2𝑖̂ +
3𝑗̂) × 106 ms−1 , is
[CBSE 1997]
(a) 4 N in z direction (b) 8 N is y
21. A charged particle moves with velocity
direction
𝑣 in a uniform magnetic field ⃗ . The
𝐵
magnetic force experienced by the (c) 8 N is z direction (d) 8 N in -z
particle is direction.
(a) always zero [CBSE PMT 2009]
(b) never zero 26. Under the influence of a uniform
magnetic field, a charged particle is
⃗ and 𝑣 are perpendicular
(c) zero, if 𝐵 moving in a circle of radius R with
⃗ and 𝑣 are parallel. constant speed v. The time period of
(d) zero, if 𝐵
the motion
22. When a charged particle moving with
(a) depends on both R and v
velocity 𝑣 is subjected to a magnetic
⃗ , the force on it is
field of induction 𝐵 (b) is independent of both R and v
non-zero. This implies that (c) depends on R and not on v
(a) angle between them is either zero or (d) depends on v and not on R.
180°
[CBSE PMT 2007, 09]
(b) angle between them is necessarily 90°
27. A uniform magnetic field acts at right
(c) angle between them can have any angles to the direction of motion of
value other than 90° electrons. As a result, the electron
190
moves in a circular path of radius 2 cm. magnetic induction in negative X-direction.
If the speed of the electrons is doubled, As a result, the charge will
then the radius of the circular path will
(a) remain unaffected
be
(b) start moving in a circular path in Y-Z
(a) 2.0 cm (b) 0.5 cm
plane
(c) 4.0 cm (d) 1.0 cm
(c) retard along X-axis
[CBSE PMT 1991]
(d) moving along a helical path around
28. An electron moves with a velocity of 10 3 X-axis.
ms-1 in a magnetic field of induction 0.3
[CBSE PMT 1993]
T at an angle 30°. If el m of electron is
1.76 × 1011 C kg-1, the radius of the path 33. A very long straight wire carries a
is nearly current I. At the instant when a charge
+ Q at point P has velocity v as shown,
(a) 10-8m (b) 2 × 10-8 m
the force on the charge is
(c) 10-6 m (d) 10-10 m
[CBSE PMT 2000]
29. An electron having mass m, charge q
and kinetic energy E enters a uniform
magnetic field B perpendicularly. Then
its frequency of rotation will be
𝑞𝐵 𝑞𝐵
(a) (b)
𝜋𝑚 2𝜋𝑚
(a) along OY (b) opposite to OY
𝑞𝐵𝐸 𝑞𝐵
(𝑐) (𝑑) (c) along OX (d) opposite to OX.
2𝜋𝑚 2𝜋𝐸
[CBSE PMT 2001] [CBSE PMT 2005]
191
𝑚𝑎0 2𝑚𝑎0
(a) west, up
𝑒 𝑒𝑣0
𝑚𝑎0 2𝑚𝑎0
(b) west, down
𝑒 𝑒𝑣0
𝑚𝑎0 3𝑚𝑎0
(c) east, up
𝑒 𝑒𝑣0
𝑚𝑎0 3𝑚𝑎0 uniform magnetic field acting along AB. If
(d) east, down
𝑒 𝑒𝑣0 the magnetic force on the arm BC is F, the
[NEET13] force on the arm AC is
192
43. A current loop in a magnetic field [CBSE PMT 2002 ; AFMC 1993, 95]
(a) Experiences a torque whether the field (a) low resistance in series
is uniform or non-uniform in all
(b) low resistance in parallel
orientations.
(c) high resistance in series
(b) Can be in equilibrium in one
orientation. (d) high resistance in parallel.
(c) Can be in equilibrium in two 48. To convert a galvanometer into a
orientations, both the equilibrium states voltmeter,
are unstable. (a) a high resistance is connected in
(d) Can be in equilibrium in two parallel
orientations, one stable while the other (b) a low resistance is connected in
is unstable. series
[NEET 13] (c) a low resistance is connected in
44. A current loop of area 0.01 m 2 and parallel
carrying a current of 10 ampere is held (d) a high resistance is connected in
perpendicular to a magnetic field of series.
intensity of 0.1 tesla. The torque (in N m)
acting on the loop is [CBSE PMT 2002, 04]
(a) 0 (b) 0.001 49. A galvanometer having a
resistance of 8 Ω is shunted by a wire of
(c) 0.01 (d) 1.1 resistance 2Ω. If the total current is 1 A, the
[CBSE PMT 1994] part of it passing through the shunt will be
45. A rectangular coil of length 0.12 m (a) 1.2 A (b) 0.8 A
and width 0.1 m having 50 turns of wire is (c) 0.5 A (d) 0.3 A.
suspended vertically in a uniform magnetic
field of strength 0.2 weber/m2. The coil [CBSE PMT 1998]
carries a current of 2 A. If the plane of the 50. A galvanometer of 50 ohm
coil is inclined at an angle of 30° with the resistance has 25 divisions. A current of 4×
direction of the field, the torque required to 10-4 A gives a deflection of 1 division. To
keep the coil in stable equilibrium will be convert this galvanometer into a voltmeter
(a) 0.12 Nm (b) 0.15 Nm having a range of 25 V, it should be
connected with a resistance of
(c) 0.20 Nm (d) 0.24 Nm
(a) 2,500 Ω as a shunt (b) 2,450 Ω as a
[AIPMT Re 15] shunt
46. A coil in the shape of an equilateral (c) 2,550 Ω in series
triangle of side / is suspended between the
pole pieces of a permanent magnet, such (d) 2,450 Ω in series.
that B is in plane of the coil. If due to a [CBSE PMT 2004]
current I in the triangle, a torque 𝜏 acts on
it, the side l of the triangle is 51. The resistance of an ammeter is
2 𝜏
13Ω and its scale is graduated for a current
𝜏
(a) (𝑎) ( ) (𝑏)2( )1⁄2 upto 100 A. After an additional shunt has
√3 𝐵𝐼 √3𝐵𝐼
been connected to this ammeter, it
2 𝜏 1⁄2 1 𝜏
(𝑐) ( ) (𝑑) ( ) becomes possible to measure currents
√3 𝐵𝐼 √3 𝐵𝐼
upto 750 A by this meter. The value of
shunt resistance is
[CBSE PMT 2005] (a) 2Ω (b) 0.2 Ω
47. To convert a galvanometer into an (c) 2 kΩ (d) 20 Ω
ammeter, we connect
[CBSE PMT 2007]
193
52. A galvanometer having a coil 𝑆2 𝑆𝐺
resistance of 60 Ω shows full scale (𝑐) (𝑑)
(𝑆 + 𝐺) (𝑆 + 𝐺)
deflection when a current of 1.0 amp
passes through it. It can be converted into
an ammeter to read currents upto 5.0 amp [CBSE Final 2011]
by
55. A millivoltmeter of 25 millivolt range
(a) putting in series a resistance of 15 Ω is to be converted into an ammeter of 25
(b) putting in series a resistance of 240 Ω ampere range. Tire value (in ohm) of
necessary shunt will be
(c) putting in parallel a resistance of 15 Ω
(a) 0.001 (b) 0.01
(d) putting in parallel a resistance of 240 Ω.
(c) 1 (d) 0.05
[CBS
E PMT 2009] [AIPMT Pro 12]
53. A galvanometer has a coil of 56. A circuit contains an ammeter, a
resistance 100 ohm and gives a full-scale battery of 30 V and a resistance of 40.8
deflection for 30 mA current. If it is to work ohm all connected in series. If the ammeter
as a voltmeter of 30 volt range, the has a coil of resistance 480 ohm and a
resistance required to be added will be shunt of 20 ohm, the reading in the
ammeter will be
(a) 900 Ω (b) 1800 Ω
(a) 1 A (b) 0.5 A
(c) 500 Ω (d) 1000 Ω
(c) 0.25 A (d) 2 A
[AIPMT Re 15]
[CBSE Pre 2010 ; VMMC 12]
57. In an ammeter 0.2% of main current
54. A galvanometer of resistance G, is passes through the galvanometer. If
shunted by a resistance S ohm. To keep resistance of galvanometer is G, the
the main current in the circuit unchanged, resistance of ammeter will be
the resistance to be put in series with the
galvanometer is 1 499
(a) 𝐺 (b) G
499 500
𝐺2 𝐺
(𝑎) (𝑏) 1 500
(𝑆 + 𝐺) (𝑆 + 𝐺) (c) 𝐺 (d) 𝐺
500 499
[AIPMT 14]
194
𝜇0 𝑁𝐼 4𝜋×10−7×50×2 𝜇0𝐼1 𝜇0𝐼2
7. (b) 𝐵 = = T 17. (d) 𝐵1 = and 𝐵2 =
2𝑎 2×0.05 2𝜋𝑑 2𝜋𝑑
= 1.26 × 10-4 T. 𝐵1 ⊥ 𝑅2
𝜇𝑜 𝐼 𝜇
8. (a) 𝐵 = ∴
𝐵 = √𝐵12 + 𝐵22 = 2𝜋𝑑
0
(𝐼12 + 𝐼22 )1⁄2
2𝑎
𝜇𝑜 × 2𝐼 18. (d) Field at O due to each linear part,
𝐵′ = =𝐵
2 × 2𝑎 𝜇0 𝐼
⃗1 = 𝐵
𝐵 ⃗3 = (sin90° + sin0° )(−𝑘̂)
9. (c) Let the radii of the two coils be 2 a 4𝜋𝑅
and a, then their resistances will be 2 R 𝜇0 𝐼
and R respectively. = (−𝑘̂ )
4𝜋𝑅
Given B1 = B2
𝜇 𝐼
0 1 𝜇0𝐼2
or 2×2𝑎 = 2𝑎
𝜇0 𝑉1 𝜇0 𝑉2
⋅ = ⋅
4𝑎 2𝑅 2𝑎 𝑅
𝑉
Or 𝑉1 = 4
2
10. (d) Refer to the solution of Problem 3 Field at O due to the semicircular part,
on page 4.79. 𝜇0 𝐼
⃗⃗⃗⃗2 =
𝐵 (−𝑖̂)
11. (d) Refer to the solution of Problem 3 4𝑅
on page 4.79. 𝜇 𝐼
⃗⃗⃗⃗0 = 𝐵
𝐵 ⃗1 +𝐵 ⃗ 2 = − 0 (𝜋𝑖̂ + 2𝑘̂ )
⃗2+𝐵
12. (b) Refer answer to Q. 24 on page 4𝜋𝑅
4.129. 19. The magnetic force acts perpendicular
𝐵′ 𝜇0(𝑛/2)2𝐼 to the motion of the charged particle.
13. (a) = =1
𝐵 𝜇0𝑛𝐼 So speed of the particle remains
or B'= B. unchanged.
14. (c) Equivalent current is 20. (d) The perpendicular magnetic force
continuously deflect^ the charge from
𝑒 𝑒 its path making it move along a circular
𝐼= = = 𝑛𝑒
𝑇 1⁄𝑛 path with a uniform speed.
𝜇0 𝐼 𝜇0 𝑛𝑒 ⃗ are parallel, the
𝐵= = 21. (d) When 𝑣 𝑎𝑛𝑑 𝐵
2𝑟 2𝑟 magnetic force on the charged particle
𝑞
15. (d) Current, 𝐼 = 𝑇 = 𝑞𝑓 is zero.
𝜇0 𝐼 𝜇0 𝑞𝑓 𝐹𝑚 = 𝑞(𝑣 × 𝐵 ⃗)=𝑣
⃗ ) = 𝑞(0
𝐵= =
2𝑅 2𝑅 22. (d) When the angle between 𝑣 and 𝐵 ⃗
𝜇0 𝐼 2𝜇0(2𝐼) has any value other than zero and 180°
16. (a) 𝐵1 = and 𝐵2 =
2𝑅 2𝑅 (so that sinθ is non-zero), the magnetic
force on the charged particle will be
non-zero.
23. (b) Lorentz force on a charge is
𝐹 = 𝐹𝑒 + 𝐹𝑚 = 𝑞𝐸⃗ + 𝑞(𝑣 × 𝐵
⃗)
24. (a) For undeflected electron,
The resultant magnetic induction at the 𝐸 |𝐸⃗ |
centre is 𝑒𝐸 = 𝑒𝑣𝐵 or 𝑣 = =
𝐵 |𝐵⃗|
𝜇0 𝐼 √5𝜇0 𝐼 25. ⃗)
(d) 𝐹 = 𝑞(𝑣 × 𝐵
𝐵 = √𝐵12 + 𝐵2 = √1 + 4 =
2𝑅 2𝑅
195
= −2 × 10−6 C[(2𝑖̂ + 3𝑗̂) × 106 ms−1 × 2𝑗̂T] When protons attain maximum speed,
= -8 k 𝑚𝑣02 𝑒𝐵𝑅
= 𝑒𝑣0 𝐵 or 𝑣0 =
i.e., 8 N in the -ve z direction. 𝑅 𝑚
𝑚𝑣 1 1 𝑒 2 𝐵 2𝑅 1 𝑒2𝑅 4𝜋2𝑚𝑣 2
26. (b) 𝑅 = ∴𝐾 = 2 𝑚𝑣02 = 2 × 𝑚 =2 ×
𝑞𝐵 𝑚2 𝑚 𝑒2
2𝜋𝑅 2𝜋 𝑚𝑣 2𝜋𝑚 = 2𝜋 2 𝑚𝑣 2 𝑅2
𝑇= = ⋅ =
𝑣 𝑣 𝑞𝐵 𝑞𝐵 𝐹
35. (b) 𝑎0 = 𝑚0 =
𝑒𝐸
⇒𝐸=
𝑚𝑎0
( west )
𝑚 𝑒
𝑚𝑣
27. (c) As 𝑟 = i.e., 𝑟 ∝ 𝑣 𝐹𝑒 + 𝐹𝑚 𝑒𝐸 + 𝑒𝑣0 𝐵
𝑞𝐵
= = 3𝑎0
When v is doubled, r also gets doubled. 𝑚 𝑚
𝑚𝑣sin𝜃 𝑣sin𝜃 e v0 B = 3 m a0 - e E = 3 m a0 - m a0 = 2 m a0
28. (a) 𝑟 = = 𝑒
𝑒𝐵 2𝑚𝑎0
𝑚 ∴𝐵 = 𝑒𝑣0
103 × sin30°
= The field B must act downwards in
1.76 × 1011 × 0.3
accordance with Fleming's left hand rule.
= 9.46 × 10−9 m ≈ 10−8 m
𝑚𝑣 √2𝑚𝐾 𝑞 2 𝐵 2𝑅 2
36. (a) 𝑅 = = ⇒𝐾=
29.(b) From Q. 26 above, frequency is v = 𝑞𝐵 𝑞𝐵 2𝑚
1 𝑞𝐵
= 2𝜋𝑚 𝑒 2 𝐵2 𝑅2
𝑇
𝐾𝑝 =
2𝜋𝑚 2𝑚𝑝
30. (b) As 𝑇 = 𝑞𝐵
(2𝑒)2 𝐵2 𝑅2
∴𝑇𝑝 =
2𝜋𝑚𝑝 𝐾𝛼 = = 𝐾𝑝 = 1MeV
𝑒𝐵 2 × 4𝑚𝑝
2𝜋 × 4𝑚𝑝 37. (b) Refer to the solution of
𝑇𝛼 = = 2𝑇𝑝
2𝑒𝐵 Problem 27 on page 4.84.
𝑚𝑣 2 𝑞𝑟𝐵 38. (b) Component of AC
31. (d) Here = 𝑞𝑣𝐵 or 𝑣=
𝑟 𝑚 perpendicular to the magnetic field is just
1 𝑞2 𝑟 2 𝐵2 equal in magnitude and opposite in
𝐾= 𝑚𝑣 2 = direction to BC.
2 2𝑚
𝐾𝑝 𝑒 2 𝑟 2 𝐵2 2 × 2𝑚𝑝 So the force on AC is - 𝐹 .
= ⋅ 2 2 2 =2
𝐾𝑑 2𝑚𝑝 𝑒 𝑟 𝐵 39. (b) F𝐴𝐷 = F𝐵𝐶 = 0
𝐾𝑝 = 2𝐾𝑑 = 2 × 50 = 100keV 𝐹𝐴𝐵 = −𝐹𝐶𝐷 = −𝐹
⃗ is
32. (a) Here angle between 𝑣 and 𝐵
180°.
𝐹 = 𝑞𝑣𝐵sin180° = 0
33. (a) The field B due to current I acts
normally into the plane of paper i.e.,
along -ve Z-direction.
𝜇0 2𝐼1𝐼2
⃗ ) = 𝑄[𝑣𝑖̂ × (−𝐵𝑘̂ )] 40. (a) 𝐹 = ⋅ 𝑟
𝐹 = 𝑄(𝑣 × 𝐵 4𝜋
196
mBsin0°=0. Required series resistance,
43. (d) 𝜏 = mBsinθ When 𝑚 ⃗ ,𝜃 =
⃗⃗ ↑↑ 𝐵 𝑉 25
°
𝑅= −𝐺 = − 50 = 2450𝛺
0 ,𝜏 = 0 𝐼𝑔 0.01
loop is in stable equilibrium. 51. (a) The original ammeter can be
⃗ , θ = 180°, x = 0, loop is in treated as the galvanometer.
When 𝑚⃗⃗ ↑↓ 𝐵
unstable equilibrium. 𝐼𝑔 𝐺 100 × 13 1300
𝑆= = = = 2𝛺
𝐼 − 𝐼𝑔 750 − 100 650
44. (a) Normal to the loop area will be
parallel to the magnetic field, i.e., θ =0°. 𝐼𝑔 𝐺 1×60
52. (c) 𝑆 = = = 15𝛺, in parallel.
𝐼−𝐼𝑔 5−1
𝜏 = IBA sin0° = 10 × 0.1 × 0.01 × 0 = 0.
𝑉 30
45. (c) 𝜏 = NIMBsinθ 53. (a) 𝑅 = 𝐼 − 𝑅𝑔 = 30×10−3 − 100
𝑔
or GS+GR + SR = G2 + GS
46. (b) Normal to the plane of the coil will
𝐺2
be perpendicular to the field B. or 𝑅 = (𝑆+𝐺)
∴ 𝜏 = 𝐼𝐵𝐴sin90° = 𝐼𝐵𝐴 𝐺𝑆 𝑉𝐺 25×10−3
55. (a) = =
Area of equilateral triangle, 𝐺+𝑆 𝐼 25
1 𝐺𝑆
𝐴 = × Basex Height = 0.001𝛺
2 𝐺+𝑆
1 S = 0.001 Ω.
= × 𝑙 × 𝑙sin60°
2
√3 2
= 𝑙
4
√3 2 𝜏
∴𝜏 = 𝐼𝐵 × 𝑙 or 𝑙 = 2( )1⁄2
4 √3𝐵𝐼
56. (b)
47. (b) To convert a galvanometer into an
480 × 20
ammeter, we connect a low resistance 𝑅𝑒𝑞 = 40.8 +
in parallel with it. 480 + 20
197
𝐺 𝐺
𝑅𝐴 = =
𝑛 500
VMMC PMT and other Medical Entrance Exams
1. For the magnetic field to be particle is
maximum due to a small element of current
(a) 10-3μo (b) 10-11μo
carrying conductor at a point, the angle
between the element and the line joining (c) 10-7μo (d)10-17 μ0
the element to the given point must be [DPMT 92,98]
(a) 0°A (b) 90° 7. The magnetic field intensity due to a
(c) 180° A (d) 45° thin wire carrying current I in the figure
shown is
[VMMC 10]
2. Magnetic field at the centre of a
current carrying circular coil is
𝜇𝑜 𝐼𝑁 2 𝜇0 10𝐼
(a) 𝐵 = × (𝑏)𝐵 = ×
2 𝑎 2𝜋 𝑎
𝜇𝑜 𝐼𝑁 𝜇0 𝐼𝑁 𝜇𝑜 𝐼 𝜇𝑜 𝐼
(𝑐)𝐵 = × (d) 𝐵 = × (𝑎) (𝜋 − 𝛼 + tan𝛼)(𝑏) (𝜋 − 𝛼)
4 𝑎 2 𝑎 2𝜋𝑅 2𝜋𝑅
𝜇𝑜 𝐼 𝜇0
(𝑐) (𝜋 + 𝛼) (𝑑) (𝜋 + 𝛼 − tan𝛼)
[DPMT1991] 2𝜋𝑅 2𝜋𝑅
3. Magnetic field at the centre of a current [DPMT 2006]
carrying circular loop having 1 A 7. 20 amp current is flowing in a long
current and number of turns one will be straight wire. The intensity of magnetic
(radius of the loop is 1 m) field at a distance of 10 cm from the
(𝑎)
𝜇0
(𝑏)2𝜇0 (𝑐)
𝜇𝑜
(d) 4𝜇0 wire will be
2 4
(a) 4 × 10-5 Wb/m2 (b) 2 × 10-5 Wb/m2
[DPMT 1993]
(c) 4 × 10-5 Wb/m2 (d) 8× 10-5 Wb/m2
4. To produce a magnetic field of π
tesla at the centre of circular loop of [VMMC 2005]
diameter 1 m, the current flowing through 9. The magnetic field at a distance r from a
loop is long wire carrying current I is 0.4 tesla.
(a) 5× 106 A (b) 107 A The magnetic field at a distance 2 r is
(c) 2.5 × 106 A (d) 2 × 106 A (a) 0.2 tesla (b) 0.8 tesla
[DPMI 2002] (c) 0.1 tesla (d) 1.6 tesla
5. Two circular coils of diameter 10 and [DPMT 2004]
20 cm have same number of turns. The 10. A vertical straight conductor carries a
ratio of magnetic field inductions produced current vertically upwards. A point P
at the centre of coils when connected in lies to the east of it at a small distance
series is and another point Q lies to the west at
(a) 1:2 (b) 3:2 the same direction.
(c) 2:1 (d) 2:3 The magnetic field at P is
[DPMT 1994] (a) greater than at Q
6. A particle having charge 100 times (b) same as at Q
that of an electron is revolving in a circular (c) less than at Q
path of radius 0.8 m with one rotation per
sec. Magnetic field produced at a centre of (d) greater or less that at Q, depending
upon the strength of current.
198
[DPMT 2004] [DPMI 2004]
11. A long straight wire of circular 14. The force on a charge due to a
cross-section (radius a) carries a steady magnetic field can act
current I and the current I is uniformly
(a) on a charge which is at rest
distributed across this cross-section.
Which of the following plots represents the (b) which is moving in the direction of the
variation of magnitude of magnetic field B magnetic field
with distance r from the centre of the wire? (c) moving in the opposite direction of the
magnetic field
[DPMT 2009] (d) moving in the perpendicular direction.
[DPMT 2006]
15. The magnetic force on a point charge is
⃗)
𝐹 = 𝑞(𝑣 × 𝐵
Here q = electric charge
𝑣 = velocity of point charge —⇒
⃗ = magnetic field
𝐵
The dimensions of B are
(a) [MLT-1A] (b) [M2LT-2A-1]
12. A coil having N turns carry a current (c) [MT-2A_1] (d) none of these.
as shown in the figure. The magnetic field
intensity at point P is [VMMC 06]
16. A positive charge is moving upward in a
magnetic field which is towards north.
The particle will be deflected towards
(a) east (b) west
(c) north (d) south
[DPMT 2002]
𝜇0 𝑁𝑖𝑅2 𝜇0 𝑁𝑖
(a) (b) 17. A positively charged particle
2(𝑅2 + 𝑥 2 )3⁄2 2𝑅 moving due east enters a region of uniform
𝜇0 𝑁𝑖𝑅2 magnetic field directed vertically upwards.
(𝑐) (𝑑) zero
(𝑅 + 𝑥)2 [DPMT 1997]
This particle will
[VMMC13] (a) get deflected in vertically upward
direction
13. A magnetic field
(b) move in a circular path with a
(a) always exerts a force on charged decreased speed
particle
(c) move in a circular path with an
(b) never exerts a force on charged increased speed
particle
(d) move in a circular path with a
(c) exerts a force, if the charged particle is uniform speed.
moving across the magnetic field line
18. If a particle is moving in a uniform
(d) exerts a force, if the charged particle is magnetic field, then
moving along the magnetic field line.
(a) both momentum and total energy will
199
change action of an electric field 𝐸⃗ = 𝐸0 𝑖̂ and
(b) its momentum changes but total energy ⃗ = 𝐵0 𝑖̂ with a velocity 𝑣 = 𝑣0 𝑗̂ .The
𝐵
remains the same √5
speed of the particle will become 𝑣
2 0
(c) both momentum and total energy after a time
remain the same 𝑚𝑣0 𝑚𝑣0
(𝑎) (𝑏)
(d) its total energy changes but momentum 𝑞𝐸 2𝑞𝐸
remains the same
√3𝑚𝑣0 √5𝑚𝑣0
[DPMT 1992, 93] (c) (d)
2𝑞𝐸 2𝑞𝐸
19. The radius of the circular path of an [VMMC 2015]
electron moving in magnetic field
perpendicular to its path is equal to 24. An electron having charge
1.6×10“19 C and mass 9 × 10-19 kg is
𝐵𝑒 𝑚𝑒
(a) (b) moving with 4 × 106 m/s speed in a
𝑚𝑣 𝐵 magnetic field of 2 × 10-1 tesla in a circular
𝑚𝐸 𝑚𝑣 orbit. The force acting on an electron and
(𝑐) (d)
𝐵 𝐵𝑒 the radius of circular orbit will be
[DPMT 1996, 97, 2001 ; VMMC 2002] (a) 1.28× 10-14 N, 1.1× 10-3 m
20. A beam of ions enters normally into a (b) 1.28 × 1015 N, 1.2 × 10-12 m
uniform magnetic field of 4× 10 -2 tesla
(c) 1.28×10-13 N, 1.1× 10-4 m
with velocity of 2 × 10 5 m/s. If the
specific charge of the ion is 5 × 10 7 (d) none of these.
C/kg, then the radius of the circular
[VMMC 2005]
path described will be
25. A charged particle is moving in a
(a) 0.10 m (b) 0.06 m
uniform magnetic field in a circular path
(c) 0.20 m (d) 0.25 m. with a radius R. When energy of the
particle is doubled, then the new radius
[VMMC 2005]
will be
21 A proton travelling at 23° w.r.t the 𝑅
direction of a magnetic field of a strength (𝑎)𝑅√2 (b)
2.6 mT experiences, a magnetic force of 2
6.5 × 10-17N. What is the speed of the 𝑅
(𝑐)2𝑅 (d)
proton ? √2
5 5
(a) 2 × 10 m/second (b) 4× 10
m/second
[DPMT 1998]
(c) 6 × 105 m/second (d) 8 × 10D
26. In a cyclotron, if a deuteron can gain an
m/second
energy of 40 MeV, then a proton can
[DPMT 2011] gain an energy of
22. What uniform magnetic field applied (a) 40 MeV (b) 80 MeV
perpendicular to a beam of electrons
(c) 20 MeV (d) 60 MeV
moving at 1.3 × 106 m/s, is required to
make the electrons travel in a circular [DPMT 2005]
arc of radius 0.35 m ?
27. A proton and an α-particle follow the
(a)2.1×10-5G (b) 6× 10-5T same circular path in a transverse
magnetic field. Their kinetic energies are in
(c) 2.1× 10-5T (d) 6×10-5G
the ratio
[DPMT 2011]
(a) 1:4 (b) 1 : √2
23. A particle having charge q and mass m
starts moving from the origin under the (c) 1 :2 (d) 1:1
200
[VMMC 2003] 2𝑉𝐸 2 𝐸2
(𝑐) (𝑑)
28. The ratio of time period of α particle 2𝐵2 2𝑉𝐵2
to that of proton circulating with same where V is the potential difference between
speed in the same uniform magnetic field is cathode and anode.
(a) √2 : 1 (b) 1 : √2 [VMMC 12]
(c) 1 : 2 (d) 2:1 34. A certain electric motor wire that carries
a current of 4 amp is perpendicular to a
[DPMT 2002]
magnetic field of 0.8 T. What is the
29. A proton and an α-particle enter a force on each cm of the wire ?
uniform magnetic field perpendicular to
(a) 0.08 N (b) 0.06 N
y-axis with the same speed. If proton takes
25μ sec to make 5 revolutions, then time (c) 0.04 N (d) 0.02 N
period for the α-particle would be [VMMC 2002]
(a) 5μ sec (b) 10 μ sec 35. A wire of length l carries a current i
(c) 25μ sec (d) 50μ sec along x-axis. A magnetic field exists
given by 𝐵 = 𝐵0 (𝑖̂ + 𝑗̂ + 𝑘̂ )T . The
[VMMC 2003]
magnitude of the magnetic force acting
30 Mass of proton is 1.6 ×10“27 kg. The on the wire is
proton enters the magnetic field of 2 T at an
angle 30° with direction of field. The (a) ilBQ (b) √3𝑖𝑙𝐵0
velocity of proton is 2 × 10 7 m/s. The radius (c) 2 i l B0 (d) √2𝑖𝑙𝐵0
of the path described by proton is
[DPMI 2006J
(a) 3 cm (b) 4 cm (c) 5 cm (d) 6 cm.
36. What is the magnitude of
[DPMT 07] magnetic force per until length of a wire
31. Electron is moving perpendicular to carrying a current of 5 A and making an
z-axis ; the magnetic field BQ is present angle of 30° with direction of a uniform
along the z-axis; the radius of circular path magnetic field of 0.1 T?
is a. Angular momentum is (a) 0.25 Nm (b) 0.45 Nm
(a) e BQa2 (b) 0 (c) 0.35 Nm (d) 0.55 Nm.
(c) 𝑒 2 𝐵02 𝑎 2 (d) e B0 a [DPMT 1999]
[DPMT 2007] 37. Two parallel wires of length 9 m each
32. An α-particle crosses a space are separated by a distance 0.15 m. If
without any deflection. If electric field E =8 they carry equal currents in the same
× 106 V/m and magnetic field is B = 1.6 T, direction and exert a total force of 30 ×
the velocity of particle is 10“7 N on each other, then the value of
current must be
(a) 2.5 × 106 m/s (b) 5 ×106m/s
(a) 2.5 amp (b) 3.5 amp
(c) 8 × 106 m/s (d) 5 × 107 m/s.
(c) 1.5 amp (d) 0.5 amp.
[DPMT 2002]
[DPMT 98 ; VMMC 2004]
33. A beam of cathode rays is
subjected to crossed electric (E) and 38. In the given figure, the loop is fixed but
magnetic fields (B). The fields are adjusted straight wire can move.
such that the beam is not deflected. The The straight wire will
specific charge of the cathode rays is given
by (a) remain stationary
201
(d) rotate about the axis. of 2 A. Then required shunt resistance to
convert it into an ammeter reading up to 6
A, will be
[VMMC 2006]
(a) 2 Ω (b) 4Ω (c) 3 Ω (d) 5 Ω.
[DPMT 1997]
43. Two identical galvanometers are
converted into an ammeter and a
milliammeter. Resistance of the shunt of
milliammeter through which the current
passes through will be
39 A circular loop of area 0.01 m 2
(a) more (b) equal
carrying a current of 10 A, is held
perpendicular to a magnetic field of (c) less (d) zero.
intensity 0.1 T. The torque acting on the [DPMT 1997]
loop is
44 The resistance of an ideal voltmeter is
(a) zero (b) 0.001 Nm
(a) zero (b) 100 Ω
(c) 0.01 Nm (d) 0.8 Nm.
(c) infinity (d) 500 Ω.
[DPMT 1996]
[DPMT 1999]
40. A sensitive galvanometer like a
moving coil galvanometer can be 45. A voltmeter has resistance of G ohm
converted into an ammeter or a voltmeter and range of V volt. The value of
by connecting a proper resistance to it. resistance used in series to convert it
Which of the following statements is true ? into a voltmeter of range nV volt is
(a) a voltmeter is connected in parallel and (a) nG (b) (n-l)G
current through it is negligible 𝐺 𝐺
(𝑐) (𝑑)
(b) an ammeter is connected in parallel 𝑛 𝑛−1
and potential difference across it is [DPMT 2004]
small
46. If number of turns in moving coil
(c) a voltmeter is connected in series and galvanometer becomes half, then the
potential difference across it is small deflection for the same current will become
(d) an ammeter is connected in series in a (a) same (b)half
circuit and the current through it is
negligible. (c) double (d) four times
[DPMT 1998] [VMMC 11]
41. A galvanometer of resistance 25 Ω 47. A galvanometer coil has a
is shunted by a 2.5 Ω wire. The part of total resistance of 10 Ω and the meter shows full
current that flows through the scale deflection for a current of 1 mA. The
galvanometer is given as shunt resistance required to convert the
galvanometer into an ammeter of range 0 -
I 1 I 3 100 mA is about
(a) = (b) =
I0 11 I0 11
(a) 10 Ω (b) 1 Ω
I 2 I 4
(c) = (d) = (c) 0.1 Ω (d) 0.01 Ω
I0 11 I0 11
[DPMT 2009]
[DPMT 1999] 48. The scale of a galvanometer of
resistance 100 Ω contains 25 divisions. It
42. If resistance of a galvanometer is 6
gives a deflection of one division on
Ω and it can measure a maximum current
passing a current of 4× 10 -4 A. The
202
resistance in ohm to be added to it, so that (d) Both assertion and reason are
it may become a voltmeter of range 2.5 V is false.
(a) 150 (b) 170 (c) 110 (d) 220 51. Assertion. Magnetic field is not
associated with a stationary charge.
[VMMC 14]
Reason. A magnetic field can be
49. A galvanometer of resistance 20 Ω
detected by means of its effect on a current
shows a deflection of 10 divisions when
carrying conductor.
current of 1 mA is passed through it. If a
shunt of 4 Ω is connected and there are 50 [VMMC 2007]
divisions on the scale, the range
52.Assertion. Magnetic field interacts
galvanometer is
with a moving charge and not with a
(a) 35 mA (b) 40 mA stationary charge.
(c) 30 mA (d) 25 mA Reason. A moving charge produces
magnetic field.
[VMMC 15]
[VMMC 12]
50.A voltmeter of resistance 2000 Ω,
0.5 V/div is ! be converted into a voltmeter 53. Assertion. A proton and an alpha
to make it to read 2 V/div. The value of high particle having the same kinetic energy are
resistance to be connected in series with it moving in circular paths in a uniform
is magnetic field. The radii of their circular
paths will be equal.
(a) 6000 Ω (b) 4000 Ω
Reason. Any two charged particles
(c) 5000 Ω (d) 1000 Ω
having equal kinetic energies and entering
[VMMC 11] a region of uniform magnetic field 𝐵 ⃗ in a
Directions (Q. No. 51 to 54) In each of ⃗
direction perpendicular to 𝐵, will describe
the following questions, two statements are circular trajectories of equal radii.
given. One is assertion and the other is [VMMC 09]
reason. Examine the statement carefully
and mark the correct answer as 54. Assertion. The torque on the coil is
maximum, when coil is suspended in a
(a) Assertion is true and reason is its radial magnetic field.
correct explanation
Reason. The torque tends to rotate the
(b) Both assertion and reason are true but coil about its own axis.
reason is not correct explanation of
assertion [VMMC 08]
(c) Assertion is true but reason is false
Answers and Explanations
1. (b) 𝑑𝐵 = 4𝜋0
𝜇 𝐼𝑑𝑙sin𝜃 𝐼 = 2.5 × 106 A
𝑟2
5. (c) For the coils connected in series,
dB will be maximum when θ =90°.
current I is same. Also N is given to be
2. (d) Field at the centre of current same.
carrying circular coil, 1 𝐵1 𝑎2 20
𝜇𝑜 𝑁𝐼 𝐵∝ or = = = 2: 1
𝐵= 𝑎 𝐵2 𝑎1 10
2𝑎
6. (d) A revolving charge is equivalent to
𝜇𝑜 𝑁𝐼 𝜇0×1×1 𝜇𝑜
3. (a) 𝐵 = = = current loop. Here
2𝑎 2×1 2
𝜇0𝑁𝐼 I = frequency × charge = 100 e × 1 =100 e
4. (c) As 𝐵 = 2𝑎
4𝜋 × 10−7 × 1 × 𝐼
𝜋=
2 × 0.5
203
𝜇𝑜 𝐼 𝜇0 × 100 × 1.6 × 10−19 6.5 × 10−17
𝐵= = = ms−1
2𝑎 2 × 0.8 1.6 × 10−19 × 2.6 × 10−3 × sin23°
= 10−17 𝜇𝑜
6.5 × 105
7. (a) Refer to the solution of Example 24 = ms−1 ≃ 4 × 105 ms−1
1.6 × 2.6 × 0.39
on page 4.18. 𝑚𝑣 9.11×10−31×1.3×106
2⋅×10−7×20
22. (a) 𝐵 = =
0 𝜇 𝐼 𝑞𝑟 1.6×10−19 ×0.35
8. (c) 𝐵 = 2𝜋𝑟 = =4×
0.10
10−5 Wb/m2 911 × 13 × 10−6
= = 2.1 × 10−5 T
𝐵2 𝑟1 𝑟 1 16 × 35
9.(a) = = =
𝐵1 𝑟2 2𝑟 2 𝑞𝐸
23. (b) 𝑣 = √𝑣𝑦2 + 𝑣𝑥2 = √𝑣02 + ( 𝑚 𝑡)2
1 1
∴𝐵2 = 𝐵 = × 0.4 = 0.2T
2 1 2
𝜇 𝐼 √5 𝑞𝐸
0
10. (b) 𝐵 = 2𝜋𝑟 ( 𝑣0 )2 = 𝑣02 + ( 𝑡)2
2 𝑚
1 𝑚𝑣0
11. (a) 𝐵 inside ∝ 𝑟 and 𝐵 outside ∝ 𝑟 =𝑡= 2𝑞𝐸
𝜇 𝑁𝑖𝑅 2 24. (c) 𝐹 = 𝑒𝑣𝐵 = 1.6 × 10−19 × 4 × 106 ×
12. (a) 𝐵 AXIAL = 2(𝑅20+𝑥 2)3⁄2
2 × 10−1
13. (c) A magnetic field exerts force on a = 1.28x10-13 N.
charged particle if F = qvBsinθ is
non-zero. 𝑚𝑣 9 × 10−31 × 4 × 106
𝑟= =
14. (d) Force, F =qvBsin.90°=qvB. In all 𝑒𝐵 1.6 × 10−19 × 2 × 10−1
other cases, F=0 = 1.1 × 10−4 m
𝐹 √2𝑚𝐾
15. (c) 𝐹 = 𝑞𝑣𝐵sin𝜃 or 𝐵 = 𝑞𝑣sin𝜃 25. (a) 𝑅 = i.e., 𝑅 ∝ √𝐾
𝑞𝐵
204
From the above problem, while its right position will exert repulsion.
The net force is attractive. The wire will
𝑇𝛼 = 2𝑇𝑝 = 2 × 5 = 10𝜇s
move towards the loop.
𝑚𝑣sin𝜃
30. (c) 𝑟 = 39. (c) 𝜏 = NIB Asm Q
𝑞𝐵
205
10−3 × 10 10
= −3 = ≃ 0.1𝛺
(100 − 1) × 10 99
48. (a) 𝐼𝑔 = 4 × 10−4 A × 25 = 10−2 A
Required series resistor,
𝑉 2.5
𝑅= − 𝐺 = −2 − 100
𝐼𝑔 10
= 250 − 100 = 150𝛺
1
49. (c) 𝐼𝑔 = 10 × 50mA = 5mA = 0.005A
𝐼𝑔 𝐺 or
0.005 × 20
𝑆= 4=
𝐼 − 𝐼𝑔 𝐼 − 0.005
7-0.005 = 0.025
7 = 0.030 A = 30 mA.
50. (a) Let n be the total number of
divisions on the scale.
0.5𝑛
In first case, 𝐼𝑔 =
2000
2𝑛
In second case, 𝐼𝑔 = 2000+𝑅
0.5𝑛 2𝑛
=
2000 2000 + 𝑅
On solving, R = 6000 Ω.
51. Assertion is true, the reason is true but
not correct explanation of assertion.
52. Both the assertion and reason are true
and reason is the correct explanation of
the assertion.
53. Assertion is true but reason isfalse.
𝑚𝑣 √2𝑚𝐾
𝑟= =
𝑞𝐵 𝑞𝐵
√𝑚
𝑟∝
𝑞
√ 𝑚𝑝
𝑟𝑝 =
𝑒
√4𝑚𝑝
𝑟𝛼 = = 𝑟𝑝
2𝑒
√4𝑚𝑝
𝑟𝛼 = = 𝑟𝑝
2𝑒
54. Assertion is true but reason is false.
In a radial magnetic field,
τ = NIBAsin90° = NIBA = τmax
206